Skip to main content

Full text of "Mathematical questions and solutions"

See other formats


This is a digital copy of a book that was preserved for generations on library shelves before it was carefully scanned by Google as part of a project 
to make the world's books discoverable online. 

It has survived long enough for the copyright to expire and the book to enter the public domain. A public domain book is one that was never subject 
to copyright or whose legal copyright term has expired. Whether a book is in the public domain may vary country to country. Public domain books 
are our gateways to the past, representing a wealth of history, culture and knowledge that's often difficult to discover. 

Marks, notations and other marginalia present in the original volume will appear in this file - a reminder of this book's long journey from the 
publisher to a library and finally to you. 

Usage guidelines 

Google is proud to partner with libraries to digitize public domain materials and make them widely accessible. Public domain books belong to the 
public and we are merely their custodians. Nevertheless, this work is expensive, so in order to keep providing this resource, we have taken steps to 
prevent abuse by commercial parties, including placing technical restrictions on automated querying. 

We also ask that you: 

+ Make non-commercial use of the files We designed Google Book Search for use by individuals, and we request that you use these files for 
personal, non-commercial purposes. 

+ Refrain from automated querying Do not send automated queries of any sort to Google's system: If you are conducting research on machine 
translation, optical character recognition or other areas where access to a large amount of text is helpful, please contact us. We encourage the 
use of public domain materials for these purposes and may be able to help. 

+ Maintain attribution The Google "watermark" you see on each file is essential for informing people about this project and helping them find 
additional materials through Google Book Search. Please do not remove it. 

+ Keep it legal Whatever your use, remember that you are responsible for ensuring that what you are doing is legal. Do not assume that just 
because we believe a book is in the public domain for users in the United States, that the work is also in the public domain for users in other 
countries. Whether a book is still in copyright varies from country to country, and we can't offer guidance on whether any specific use of 
any specific book is allowed. Please do not assume that a book's appearance in Google Book Search means it can be used in any manner 
anywhere in the world. Copyright infringement liability can be quite severe. 

About Google Book Search 

Google's mission is to organize the world's information and to make it universally accessible and useful. Google Book Search helps readers 
discover the world's books while helping authors and publishers reach new audiences. You can search through the full text of this book on the web 



at |http : //books . google . com/ 



MATHEMATICAL QUESTIONS, 

WITH THKIH 

SOLUTIONS. 

FROM THE "EDIJOATIONAL TTHES." 
VOU XLVII. 



\MoJ^2^'6%.%^ 




SCIENCE CENTER LIBRARY 




Digitized by VjOOQIC 



Digitized by VjOOQIC 



Digitized by 



Google 



MATHEMATICAL il^:^ 

QUESTIONS AND SOLUTIONS, 

FEOM THE "EDUCATIONAL TIMES/' 

WITH MAVT 

PAPEES AKD SOLUTIONS 

nr ABDinov to thobb 
PUBLISHED IN THE "EDUCATIONAL TIMES,*' 

AND FOUR 

APPENDICES. 



BDITBD BY 

W. J. C. MILLER, B.A., 

BBGIBTBAB 

OV THB 

QBNBEAL MBDICAL COUNCIL. 




VOL. XLVIL 



c 

LONDON: 



FRANCIS HODGSON, 89 PARRINGDON STREET, B.C. 

1887. Digitized by GOOglC 










\* Of this series there have now been published foriy-seyen Volumes, 
each of which contains, in addition to the papers and solutions that 
have appeared in the Educational Times, an equal quantity 
of new articles, and comprises contributions, in all branches of 
Mathematics, from most of the leading Mathematicians in this and 
other countries. 

New Subscribers may have any of these Volumes at Subscription-prices. 



Digitized by VjOOQIC 



LIST OP CONTEIBUTOES. 



AlTAB, Prof. SWAMIVATHA: Pros. Coll., Madras. 

Allbk, Rev. A. J. C;;, M.A. ; College, Chester. 

Allman, Professor Geo. J., LL.D. ; Galway. 

AvDEBSov, Alex., B.A. ; Queen's Coll., Galway. 

Anthoht, Edwyn, M.A.: The Elms, Hereford. 

Abmevakte, Professor; PeSaro. 

BALL,Sir Eobt.Stawbll^LL.D.,P.R.S. ; Dublin. 

Bastov, W. J., M.A. ; HiKhgftte, London. 

Basu, Satish Chandba ; Presid. Coll.,CaIcutta. 

Battaglini, Prof. Giuseppe ; Univ. di Soma. 

Batliss, Geoboe. M.A. ; Kenilworth. 

Bebvob, B. J., B.A. ; B«gent's Park, London. 

Bbltbami, Professor ; University of Pisa. 

Bebg, Professor F. J. van dew ; Delft. 

Besakt, W. H., D.Sc., P.E.S. : Cambridge. 

Bbyevs, Professor Igvacio, M.A. ; Cadiz. 

Bhattachabta, Prof. Mubibdbakath, M.A. ; 
Delhi. 

Bhtjt, Prof. Ath Bijah, M.A.; Dacca. 

Biddlb, D. ; Gough Ho., Kingston-on-Thames. 

Bibch, B«v. J. G., M JL. ; London. 

Blackwood, Elizabeth, B.Sc; Boulogne. 

Bltthe, W. H., BJl. ; B^ham. 

BoBCHABDT, Dr. C. W. ; Victoria Strasse, Berlin. 

BOBDAOB, Pirof. Edmovd ; Coll. de Nantua. 

Boult, S. H^M.A. ; Liverpool. 

BouBNE, C. W., M.A.; College. Inverness. 

Bbill, J., B.A.; St. John's Coll., Camb. 

Bbocabd, Capitaine H. ; Montpellier. 

Bbooks, Professor E.; Millersville, Pennsylvania. 

Bbown. a. Cbum, D.Sc. : Edinburgh. 

BucHHEiM,A.,M.A.,Ph.D.: GrOv.Sch.,M'chester. 

Buck, Edwabd, M.A. ; Univ. Coll., Bristol. 

BUBNSIDE, Professor W. S., M.A. ; Univ., Dublin. 

Capel, H. N., LL.B. ; Bedford Square, London. 

Cabmodt, W. p., B.A. ; Clonmel Gram. School. 

Cabb, G. S.. M.A. ; 3 Eudsleigh Gardens, N.W. 

Casey, Prof.,LL.D.,F.R.S. ; Cath. Univ., Dublin. 

Catalan, Professor ; University of Lidge. 

Ca VALLIK, Prof., M.A. ; University of Upsala. 

Cave, A. W., B.A.; Magdalen College, Oxford. 

Catlby, a., F.R.S. ; Sadlerian Professor of Ma- 
thematics in the University of Cambridge, 
Member of the Institute of France, &c. 

Chakbayabti, Prof. Byom., M.A. ; Calcutta. 

Chase, Prof., LL.D. ; Haverford College. 

Chbistib, E. W. D^ B.A. ; Newmarket. 

Clabee, Colonel A. £., C.B., F.B.S.; Bed Hill. 

CocHBZ, Professor; Paris. 

Cockle, Sir James, M JL., F.B.S. ; London. 

CoHEN, Abthub, M.A., Q.C., M.P. ; Holland Pk. 

CoLSON, C. G., MJL.'j university of St. Andrews. 

Constable, S. ; Swinford Kectory, Mayo. 

COTTEBILL, J. H., M.A. ; E. N. Coll., Greenwich. 

Cbbmona, Prof. LuiGi : Bome. 

Cbofton, Prof. M. W., F.B.S. ; Dublin. 

Cboeb, J. O'Bybnb, M.A. ; Dublin. 

ClTLLBY, Prof«MJL. ; St. David's Coll.,Lampeter, 

ClTBTis, B., M.A., S.J. : Univ. Coll., Dublin. 

Dabboux, Professor; Paris. 

Data, Prof. Peomathanath ; Calcutta. 

Dayibs, D. O. S., M.A. : Univ. Coll., Bangor. 

Dayis, B. F., M.A. ; Endsleigh Gardens. 

Dawson, H. G., B.A. ; Christ Coll., Camb. 

Day, Bev.H. G.,M.A.; BichmondTerr., Brighton. 

De Longchamps, Professor ; Paris. 

Db Wachtbb, Prof., M.A. ; Schaarbeck. 

Dby, Prof. Nabendba Lal, M.A.; Calcutta. 

DlCK,G. B., M.A. ; Victoria University. 

D'OCAGNE, Maubice ; Bochefort-sur-Mer. 

Dobson, T., B.A. ; Hexham Grammar School. 

Dboz, Rof. Abnold, M.A. ; Porrentruy, Berne. 

DUPAIN, J. C. ; Professeur au Lyc6e d'Aiigoul6me. 

Eastwood, G.. M.A.; Saxonville, Massachusetts. 

Easton, Belle, B.Sc. ; Lockport, New York. 

Edwabd, J., M.A. ; Head Mast., Aberdeen Coll. 

Edwabdes, Dayid; Erith Villas, Erith, Kent. 

Elliott, E. B.. M.A. ; Fell. Queen's Coll., Oxon. 



Ellis, A lexandeb J., F.EJ.^ Keinaington. 

Ethta^u], W. T. a.; Pmnbroke CclL, Oifopi. 

EssENNiiLL, Em HA; Coventrj. 

Ev ,iNs, Hrofmssor, M.A.; Look port. New York. 

EvFRKTt, Pmr. J. D., D.O.L.: Qu. Coll.. Bt^lfast 

Fi L K LIN, Jose Pff; Prtaf. in Univ, of Migaouri* 

FofijiiY. H.^ M.A.; BollaTV, Madrjia Prt-sidfitic:f 

FcKs TFia^F. W„ B, A , ; OllRlf^ea. 

FosTiLii. l*rDf.G,CAa£v\ P,R,S.; Ur»iv.Coll.,Lond, 

FBA.NH;LI?f^Cllllf3TINELA.DD;M.A.;?Ptjf.ofNat. 

^i^^. and Math., Union Spriuga, New York. 
FUGHTSS. E. ; Uiiiverisity of ^n.^Jen. 
Ga LitHAiTiT,R^vJ..M.A. . Fell,Trin.Coll.,Dub!in. 
GALE.Kitir; Wrtrc*(?j5t^r Piirk, Surrey. 
Gallatt.t, W., B.A.J Earl's Court, Lon<loii. 
Gailiebs, flin,T.,>LA.; Ei.-Foll.C.GolUC^m, 
Galton, Filancie, M.A., F.R.S.; Loudon. 
Geni;se, 9mL, M,A.; Univ. Coll.. Aberyafcwith, 
GKHRAN'a. II. T., B.A. s Stud, of Ch.Ch., Oxford. 
Gla ISHES, J. W.L.p F,a.S.; FeU.Tnu.Coll.. Ciunh. 
GoLJtEJfBERa, Pfofe^Hrsr* M.A.^ Moacow, 
GoRDoy, Alice, B.Sc.; Gloucts-ster. 
GouDiE, W. P., Irvine Houses Durby. 
GiiAMAM, E, A.* M.A.- Triiiity Coikjje, Dublin. 
Ghheptfteld, Rev. W. J., M.A. t Duiwich College. 
GKEiE.-fSTREET. W. J., B.A.; HulL 
Gee r^^ WOOD. Jami^s M.; Kirksvilk, MisaourL 
GEir FJTEr:!. G. J.. M.A. ; Poll. Cli. ColL, Catiib. 
GKiFFiTtta, J.,, M.A. i FfjUrnvrif JtisujiColUOiOn, 
GRavE. W. B.. B,A.; Perry Hur. HirmmEliam. 
Hadamaru, ProfcHHDr, MA., Paris. 
Ha T'lU, E.. B.A.. B.Sc. i Kltii^s Sdi.. ^""arwiok. 
Hall. Pm3V*sor ASAPir, UA.i \Vnatuu|SftQU, 
H A 3J M OS II . J ., M.A,; Bu L;kh ui-at H i 1 1 . E iisei. 
Hariteu^^ Q.; Uiiiveraity of ^t. Petersburg. 
HA.BKEB, ALPKBD, M.A. ; Cheltenham. 
Haekis, H. W., B.A. ; Trinity College, Dublin. 
Habt, Dr. David S. ; Stoninfcton, Connecticut. 
Habt, H. ; R. M. Academy, Woolwich. 
Hau&hton, Rev. Dr., F.R.S. ; Trin.Coll., Dubl. 
Hendricks, J. E., M.A.; Des Moines, Iowa. 
Heppel, G., M.A.; The Grove, Hammersmith. 
Hebmav, R. a., M.A. ; Trin. Coll., Cambridge. 
Hebmite, Ch.; Membre de I'Institut, Paris. 
Hebvby, F. R. J., M.A.; Worthing. 
Hill, Rev. E., M.A. ; St. John's College, Camb. 
HiNTON, C. H., M.A. ; Cheltenham College. 
Hiest, Dr. T. A., F.R.S.; London. 
Holt, J. R., M.A. ; Trinity College, Dublin. 
Hopkins, Rev.G. H., M.A.; Stratton, Cornwall. 
HoPKiNsoN, J., D.Sc, B.A. ; Kensington. 
Hudson, C. T., LL.D. ; Manilla Hall, Clifton. 
Hudson. W.H.H.,M.A.;Prof.inKing*sColl.,Lond. 
Inglbby, C. M., M.Ay LL.D. ; London. 
Jenkins, Mobgan, M..A. ; London. 
Johnson, Profo M.A.; Annapolis, Maryland. 
JOHNSTONjjW. J., M.A.; Univ.Coll.,Aberystwith 
Jones, L. W., B.A. ; Merton College, Oxford. 
Kennedy, D., M.A. ; Catholic Univ.. Dublin. 
KiBKMAN, Rev. T. P., M JL., F.R.S. ; Croft Rect 
KiTCHiN, Rev. J. L., M.A. ; Heavitree, Exeter. 
KiTTUDGE, Lizzie A. ; Boston, United Statea 
Knisely, Eev. U. J. ; Newcomerstown, Ohio 
Knowlbs, R., B.A., L.C.P. ; Tottenham. 
KoEHLEB, J. ; Rue St. Jacques, Paris. 
Lachlan, R., B.A. ; Lewisham. 
LAVBBTY,W.H.,MJ^..:lateExam.inUniv.Oxforu 
Lawbence, B. J. ; Ex-Fell. Trin. Coll., Camb. 
Leidhold, R., M.A. ; Finsbury Park. 
Leudesdobp.C, M JL.; Fel.PembrokeColl.,Oxon. 
Leyett. R.. M J^. : King Edw. Sch.. Birmingham. 
London, Rev. H.,MJl.; Pockiington. 
LowRY, W. H., M.A. ; Blackrock, Dublin. 
Macdonald, W. J., M.A. ; Edinburgh. 
Macpablane, Prof. A., D.Sc. ; Univ. of Texas. 
Mackenzie, J. L., B.A. ; Gymnasium, Aberdeen. 
MacMahon, Capt. P. A. ; R. M. Academy. 
MacMubchy, a., B.A.; Univ. Ci^U.. Toronto. 



Digitized by VjOOQIC 



IV 



McALiSTEa, Boviut, M.A., J>M.i C^rohrEdge. 
McCat, W. S., M.A. 5 Fell. Trin. ColL. DiibUa. 
Mc Clella )?p, W . J„ B. a . ; Prin . of Santry School. 
McGOLL. H,, E.A. 1 73, RLie-SiljIiqumH BcmloNTHe. 
McIntosh^ AlK3t,,B.A,; BodfurdR^JW, Londqn. 
McLEOn, j.,M.A.i II,Mh Atjulpoiy. Woolwich. 
Mapisoit.Ibabml, B.A.J Cardiir. 
Malbt, Prof.. MA,; Queen 'a Colh, Cork. 
Ha'NN, M. p. J., M,A.; Kel^Hi^^J^t-on. 
MAy^irrni, M : Vr.-.f, ri, ^'|^v,-.i,.i Poly tech,, Paris. 

Martin, ASTEMAB, M.A., Ph.D.; Washinffton. 
Mathbwb, G. B., B.A. ; Univ. Coll., N. \^es. 
Matz. Prof.. M JL. ; Kind's Mountain, Carolina. 
Mebrifield, J., LL.D.,P.B^A.8. ; Plymouth. 
Mbbbimait, Maitsfield, M.A.; Tale College. 
Mbtbb, Mabt B. : Girton College, Cambridge. 

MlLLBB, W. J. C, BX, (BdITOB); 

The Paragon. Bichmond-on-Thames. 
MiircHiir, G J(.,M JLj Prof, in Cooper's Hill Coll. 
Mitohbbon, TMB.Aa<.C.P. ; City of London Sch. 
MoircK, Prof. H. St., M.A. ; Trin. Coll., Dublin. 
MoNCOUBT, Professor ; Paris. 
MooK, RoBBBT Jf JL. ; Ex-Pell. Qu. Coll., Camb. 
MooBB, U. K., B.A^ Trin. Coll., Dublin. 
MoBBL, Professor; Paris. 
MoBOAV, C, B.A.; Salisbury School. 
MoBLBT, Pbahk, B.A. ; Bath CoU., Bath. 
MoBBiOB, G. G., B.A. ; Weymouth. 
MniB, Thomas, M.A., F.B.S.B. ; BothweU. 
MnKHOPADHYAT,AsnTOSH,M.A.;Bhowanipore. 
Nash, Prof. A. M., M.A. ; Calcutta. 
Nbubbbo, Professor; Univ. of Li^. 
Nbwcomb, Prof. Simon, M.A. ; Washington. 
O'CONNELL, Major-General P. ; Bath. 
Openshaw, Iti'v.T. W,. MA,' ClilicuL 
O'liKDiN, John; ^'l'W Street, Limorick, 
Ohcitarti* H. L., M.A.,L.C.P. ; HamfKitoad. 
OyfMis, J. A., B»iSc. ; Temiyaoti Bt., Llverpt*ol. 
PA5T0N. A. W., M,A. ! Fell, of Trin. ColL^Dablin. 
PaMULKBPST, C, M.A. ; Ijondoii* 
FBRRiar, E!!j:h,t: Gtrton Callep, Cambridj^. 
PHILLIPB. F. B. W. J Ballinl Collegp, OjJurd. 
PiLLAi, C, K,, M;A. ; THohy, Madras. 
PiBlE. A., M.A. \ Univeraity of St. Andniwa. 
Pl^AMBBBWBKI, H., M.A.; Diipihtiitim. 

Pooui, GisiBtJtJK, B,A. i Clif^ltenham. 
FocBiiNGTOB-j H. C.,M.A. ; York« Coll., Leecis. 
Polio N AC, Pnnire CAMrLLB de i Paris. 
POLLBltFEN. H., HA.} Wiiidennerc College. 
PoTT^B, J B.A.; Ki chill ond-ou-Tharoeji. 
Pbujjde.s, FfiANCEB E.; LookpoFt, Niiw York. 
Ptt^eh. Prof. F. , M.A, ; Queen's College. Belfast. 
PuTNASi, K. S.. MA.; Eome.New York. 
H.A1?, Prof. B. IlAJfL^MANTA. B.A.; Madraa. 
Eawbok, Robert ; Hnvwit. Hants* 
Rait, Pfot M. Nath. M.A., LL.B. ; Howrah, 
Hat, Prof. SabaiiaranjaxV, M,A. ; Dneoa. 
Hbad, H. J., B.A* ; Brajericiae C<3lJ,, Oiford. 
Bektks, G. M„ M,A. i Lee, Kent. 
EETNOLue, B.J M.A.; Nottinp: Hill* London. 
RiCHAkHB, LiAViDj B.A.; Aberystvrith. 
EjcnASimoN, Kev. G., M.A.^ WincKester. 
EoESET5>R, A„ M.A.; SchoLof 'f Hii,Coll,,Dublin. 
RoBEETB, S., M.A., F.R,S. ; London. 
Roberts, W. IL, M^A. ; Fell, of Trin. OoU., Dub. 
Eon&ON, B.C., B.A.; Sidney JSusaex Coll., Cam. 
ItOQKES. L. J., B.A, ; Oxford. 
Rosenthal, L. H. ; J^lcholnrof Trin. Coll., Dublin. 
Boy, Prt>I, KAUPEAHA?iNA, M.A. ; Agiu. 
RirGGBIlO^^lMONKLLI; Univi^rsitA di B^ma. 
EU85EU„ J. W., M.A, ; Mertoii Coll., Oxford. 
RtfBBSLL, R„ B.A.i Trinity Uoilogti, Dublin, 
EUTTEB. l^Di^AED ; Suiiderknd. 
SAiaioaf, Rev. G.. D.D., F.R.S.; RcKJtis Professor 

tif Divinity Id the University of Dublin. 
Sakpbbs, J. B.; BioomLni^ton, Indiana^ 



Baitdbbsov, Ber. T. J., MX ; BoyBtoxi, Cambt* 

Sabkab, Prof. NiLEAKTHA, MJL; Oalcutta. 

ScHBVVBB, Professor ; Meroersbury Coll., Pa. 

ScHOUTB, Prof. P. H. ; University, Groningen. 

ScoTT, A. W., M.A. ; St. David's Coll., Lampeter. 

SooTT, Charlottb a., B.Sc.: Professor in 
Bryn Mawr College, Philadelphia. 

Scott, B. P., M.A.; Fell. St. John'sColl., Camb. 

Serrbt, Professor ; Paris. 

Shabp, W.J. C. M.A.: Greenwich. 

Shabpb, J. W., MJL ; The Charterhouse. 

Shabpb, Bev. H. T.. M.A. : Cherry Marham. 

SHBPHBBixBev. A. J.P.,B.A.; Fell, Q.Coll.,Oxf. 

Simmons, Bev. T.C., M.A.; Christ's Coll.,Breoon. 

SiBCOM, Prof., MJL.: Stonyhurst Colle(|;e. 

SiYBBLY, Waltbb; Oil City, Pennsylvania. 

SKBIM8HIBB, Bcv. B., MJL ; Llandaff. 

Smith, C, M..A. j Sidney Sussex Coll., Camb. 

Stabbnow, H., M.A. ; New York. 

Stbgoall, Prof. J. B. A., MJL.; Dundee. 

Stbin, A.; Venice. 

Stephen, St. John, B.A.; Caius Coll., Cambridge. 

Stbwabt, H., MJL. ; Framlingham, Suffolk. 

Storb, G. G., B.A. ; Clerk of the Medical Council. 

SwiBT, C. A., B.A. ; Grammar Sch., We/bridge. 

Stlvebtbb, J.J., D.C.L., P.B.S.; Professor of 
Mathematics in the University of Oxford, 
Member of the Institute of France, &c. 

Symons, B. W., M.A.; Fell. St. John's ColUOxon. 

Tait, Prof. P. G., M.A.; Univ.^dinburgh. 

TANNBB,Prof, H.W.L.,M.A.: S.Wales Univ. Coll. 

Tablbton, P. a., M.A. ; Pell. Trin. Coll., Dub. 

Taylob, Bev. C, D.D. ; Master of St. John's 
College, Cambridge. 

TaylobTh. ml M.A7Fell. Trin. Coll., Camb. 

Taylob, W. W., M.A. ; Bipon Grammar Schooi. 

Tebay, Septimus, BJL. : Famworth, Bolton. 

Tbbbt, Bev. T. B.,M.A.. Fell. Ma«i. ColL.Oxon. 

Theodobius, a. F., M.A. ; Bath College. 

Thomas, Bev.D., M.A.: Garsington Bect.Oxford. 

THOM80N,Bev.FJ).,M.A.;Ex-Fel.St.J.Coll.,Cam. 

TiRBLLi, Dr. Francesco ; Univ. di Boma. 

ToBELLi, Gabbibl; University of Naples. 

TOBBY, Bev. A. F., MJL. ; St. John's Coll., Camb. 

Tbaill, Anthony, M.A., M.D.; Fellow and 
Tutor of Trinity College, Dublin. , 

TucKBB, B., MA. ; Mathematical Master in Uni- 
versity College School, London. 

TuBBiFF, Geobob, M.A. ; Aberdeen. 

ViGABiA, Bmilb ; Hotel Sue«, Paris. 

YiNCBNzo, Jacobini; UniversitIL di Boma. 

VosE, Professor G. B.; WashhijKton. 

Walenn, W. H. ; Mem. Phys. Society. London 

Walkbb, J. J., MA., F.B.S. ; Hampstead. 

Walmslby, J^.A. ; Eccles, Manchester. 

Wabbubton-Whitb, B., B.A., Salisburv. 

Wabben, B., ma. ; Trinity College, Dublin. 

Wathbbston, Bev. A. L^M.A. ; Bowdon. 

Watson, Bev. H. W.; Ex-Fell. Trin. ColL, Camb. 

Wbbtsch, Fbanz ; Weimar. 

White, J. B., B.A. ; Worcester Coll.. Oxford. 

Whitb, Bev. J., MA ; Boyal Naval School 

Whiteside, G., M.A. ; Bccleston, Lancashire. 

Whitwobth, Bev. W. A, M.A. ; London. 

Williams, C. £., M.A. ; Wellington College. 

Williamson, B., M.A.: Fellow and Tutor of 
Trinity Coll^, Dublin. 

Wilson, j. M., M.A. ; Head-master, Clifton Coll. 

Wilson, Bev. J., M A^ Beet. Bannockbum Acad. 

Wilson, Bev. J. B., MA. ; Boyston, Cambs. 

Woodcock, T.,B.A.; Twickenham. 

WoLSTENHOLME, Bcv. J., M.A, Sc.D. : ProfcssoT 
of Mathematics in Cooper's Hill College. 



WooLHOirsB, W. S. B., F.B.A.S.,&c.; London. 
Wright, Dr. S. H., M.A.; Penn Yan, New York. 
Wright, W. B., B.A.: Heme Hill. 
Young, John, BJL; Academy, Londondeny, 



Digitized by VjOOQIC 



CONTENTS. 



£Mf)tmntit&l ^apersf^ ^t. 



Pa«e 
Note on Queetion 8376 (Vol. xlv., p. 98). Professor Nash, M.A. 97 
Note on Anallagmatic Curves. (Professor Wolstenholme, Sc.D.). 61 

Note on a Probability Question. (The Editor.) 72 

Note on the Brocardal Ellipse. (B. F. Davis, M.A.) 134 

Geometrical Construction for the Brocard Angle, &c. (R. F. 
Davis, M.A.) 135 

Note on the use of Common Logarithms in the Numerical Solution 
of Equations of the Higher Orders. (Major-General P. O'Connell). 166 



(Bm&tion& ^oVat^. 



1866. (Professor Sylvester, F.R.S.) — Prove that the Jacobian of the 
time of a planet's describing any arc, the chord of the arc, and the sum of 
the two extreme distances from the sun, in respect to the eccentricity and 
two extreme eccentric anomalies, is zero ; and hence deduce the time for 
a planet or comet in terms of the said chord and sum 163 

2391. (Professor Sylvester, P.R.S.) — Let fi points be given on a 
cubic curve. Through them draw any curve (simple or compound) of 
degree y; the remaining Zy—fi (say f/) points may be termed a first 
residuum to the given ones. Threugh these f/ points draw any curve of 
degree t/ ; the remaining Zv'—iif points may be termed a residuum of the 
second order to the given ones ; and in this way we may form at pleasure 
a series of residua of the third, fourth, and of any higher order. If /i is 
of the form 3»— 1, a residuum of the first or any odd order, and if /x is 
of the form 3i + 1, a residuum of the second or any even order in such 
series, may be made to consist of a single point, which I call the residual 
of the original /x points. Prove that any such residual is dependent 
wholly and solely on the original /i points, beiu^ independent of the 
number, degrees, and forms of the successive auxiliary curves employed 
to axriye at it 137 

a 



Digitized by VjOOQIC 



VI CONTENTS. 

2810. (Professor Sylvester, F.R.S.) — Let Si be used as the symbol 
of the sum of »-ary products ; required to prove that, if « is <n, 

SS, (O^ 0,, ... , On) (^-^>| (^-^) "• (^-°") 

— S,+i (a,, Oj, ... , an)Si^i (oi, oj, ... , On). 
[For example, let » » 3, » » 2, then the theorem becomes 
^, («-«)(«-iB)(a-7) ,,^ (^~«)(^-/3)(^-7) ,^fr (g-a)(^-3)Cg-7) 

a abe—afiy, which is obviously true.] 21 

2832. (Professor Sylvester, F.R.S.) — ^Prove that the curve of inter- 
section of two right cones with parallel axes is a spherical curve, and that 
a third right cone may be drawn through it. Prove also that a plane 
circular cubic may be found such that the distances of any two fixed points 
on it from everi/ point in the curve of double curvature above-mentioned 
shall be in a fixed linear relation 90 

2866. (Professor Sylvester, F.R.S.)— Given the simultaneous equa- 
tions f>«+i = f>«+(«2— fl>)rx_i, w«+i = w«+(a?'— a?)tt«-i + (2« + l)r,*i, prove 
that the general solution is of the form v« — Ac^ + fi/3«, 

w. = iX {a;« + 2a; + i (-)«} o. + iX {a:3 + 2a;-i (-)«} /3, + w« + »/3„ 
where A, /x, y, «- are arbitrary constants, and detennine the values of 
Ox, fix 37 

2934. (Professor Sylvester, F.R.S.) — If «i, 8^ «8, s^ «5, a^ repre- 
sent the sum of xyztuVf and of their binary, ternary, quaternary, 
quinary, and sextic combinations respectively, and if E stands for the 

symbol of Emanation a--+*--+c-- +rf--. +tf^+/4-, prove that 
dx dy ds dt du dv 

the resultant of «|, E«2, s^ E44, s^, E«e is the product of 

(a-*)»(a + i-tf-rf)»(a + i + tf-rf-tf-/)W, 

and of the similarly formed powers of products of the analogous linear 

functions oidbedef, 101 

2936. (Professor Sylvester, F.R.S.)— If /(a?) be a rational integral 
function of a; of a higher order than the second, prove that it is impossible 
for (fxY+ (f^Y to be a perfect square unless /(ic) contains at l^st two 
groups of equal factors , 85 

3427. (Professor Sylvester, F.R.S.)— If ^, i(^ are quantics in x, y, 
each of degree /x ; F a quantic in ^, i|^ of degree m, and consequently in 
Xf y of degree mit, ; and if J denote the Jacobian of ^, i|^, that is, 

^ ^_ ^ ^. 

dx dy dy dx* 
D^,^ F the discriminant of F treated as a quantic in ^, i|^ ; Dx, y, F the dis- 
cnniinant of F, treated as a quantic in x, y ; and if R be used as the 
symbol of ** resultant in regard of or, y " ; prove that 

D„, F = 2 [R {tp, rp)]-*'-^ R (F, J) (D^,^F)^. 
As a particular case of the foregoing theorem, show that the discri- 
minant of F, any symmetrical quantic of an even degree inx, y, is of the 
form F (1, 1) F (1, — 1) Q,^, where A is a rational integral function of the 
ooefEidents in F. 



Digitized by VjOOQIC 



CONTENTS, VU 

Find also what the general formula becomes when <l>, ^ are taken linear 
functiona of a?, y 146 

3636. (Professor Sylvester, F.R.S.)— 1. Euleb has shown that the 
number of modes of composing n with i distinct numbers is equal to the 
denumerant (that is, the number of solutions in positive integers, zeros 
included) of the equation 

x + 2f/ + Zz+ ... +t«=sn-i(t2+f). 

(1) Show more generally that the number of modes of composing n 
with i numbers, all distinct except the largest, which is to be always 
taken/ times, is the denumerant of the equation 

>x+C; + l)y+ ... +t« = n-J(t-y+l)(t+y). 

(2) Show also that, if all the partitions of n into i parts are distinct ex- 
cept the least, which is to be taken/ times, then the number of such par- 
titions is the denumerant of the equation 

x + 2p+ ... +(i-»^ + iV» = n-J{(t-y)8 + 3f-y} 138 

3661. (Prof essor Sylvester, F.R.S.) — ^If through 3« + l given points 
on a cubic curve a curve of the order N + « be drawn, and through the 
remaining 3K— 1 intersections of the two curves a third one be drawn of 
the order K ; prove that this will intersect the cubic at a fixed point. 

[This point may be called the opposite of the 3n + 1 given points ; it 
becomes Dr. Salmon's opx)Osite, as defined by him in tibe Fhilosophieal 
Transaetiont for 1858, when n » 1, K » 1, and is independent of the 
value of N.l 137 

6271. (Professor Cayley, F.R.S.) — If w be an imaginary cube root 
of unity, show that, if 

^" l-«2(«,-«2)ar»' (l-y2)»(l+«,y2)» (1 - ic?)* (1 + -^^ ' 
and explain the general theory 141 

5305. (Professor Sylvester, F.R.S.) — Dtf/^m^ww. — Six right lines 
along which six forces can be xnade to equilibrate are said to be in Statical 
Involution. 

Prove that any six right lines lying on a ruled cubic surface are in 
statical involution ; and, vice versd, S. six right lines are in statical 
involution, a ruled cubic surfiEtce can be made to pass through them... 166 

6420. (Professor Sylvester, F.R.S.) — From the expansion of 
{log (1 +a;)}* , in a series according to powers of », prove that S<,> [the 
coefficient oft^ is the developed product of (1 + ^(1 + 2^) ...... (l+t<)] 

is divisible by every prime number gpreater than j'+l contained in any 
term in the series t + 1, i, »— 1, , i—J+l 144 

5640. fW. J. Curran Sharp, M. A.) — Salmon says [Higher Flane 
Curves, p. 98] that the curve parallel to a given curve may be obtained (1) 
as the envelope of a circle of given radius whose centre moves on the given 
curve ; (2) as the envelope of the parallel to the tangent to the given curve 
drawn at a constant distance. Prove that these processes are equivalent. 

146 

5643. (W. S. B. Woolhouse, F.R.A.S.) — Any two triangles being 
given, the first may always be orthogonally projected into a triangle 
similar to the second ; determine the magnitude of the projected triangle 
geometrically by an easy construction^with the ruler and compasses... 26 



Digitized by 



Google 



YIU OOKTENTS. 

5765. (W. J. 0. Sharp, MJL) — H, tfaioii«^ any point of infleotfon 

O on an n-io, there he drawn three straight lines meeting the onrye in 
Ai, A4...A».i ; Bi, Bs...B».i ; C,, C,...C»-i, respectively ; prove that eyery 
curve of the n^ degree through the 3ji-2 points O, Ai, Aj...A«-i; 

Bi...B»-i ; Ci...C«-i will have for a point of inflexion. ..".'.. 146 

6809. (D. Edwardes.)— If O be any point within a triangle ABO 
prove that OA«8in2A + OB«8in2B + OC»sin2C is least when O is the 
centre of the oircnmscribedcircle , 14g 

6828. (Professor Darbonx.) — On coupe une pyramide triangulaire 
SABOparun plan parallMek la base ; ceplan rencontre les ar^t^sl^rales 
SA, SB, SO en A', B', 0' ; on m^e ensuite les plans OA'B', AB'O* BCA' 
Soit P leur point commun. Determiner le lieu d^crit par le point P lorsl 
que le plan A'B'O' se d^place en demeurant parall^e k la base 148 

5966. (Pwfessor Sylvester, P.B.S.)— By a Oartesian Oval in spaca 
let^us understand a curve the distances of whose points from three fixed 
points are linear functions of each other, or, which comes to the same 
thing, is the intersection of two surfcices of revolution, described by two 
plane Cartesians having a focus in common. Conversely, when two points 
can be found whose distances from any point in a space-Oartesiiui are 
linear fimctions of one another, let them be termed focL Required to 
prove, that the locus ol such foci is a plane curve of the 3rd degree. It 
will be observed that this curve for Cartesians of double curvature is the 
exact analogue of the three foci in a straight line for plane Cartesians. . . 69 

6686. (W. J. 0. Sharp, M.A.)— If a bar naturally curved be strained 
the bending moment at a point, whose natural curvature is r-^ and 
strained curvature p-*, isE(p-i-r-i) {l + Ar-> + Br-* + ic} 143 

6827. (Prof. Cayley, F.R.S.)— Consider a triangle ABC, and a point 
P; and let AP meet BO in M, and BP meet AC in N (if , to fix the ideas 
P is within the triangle, then M, N are in the sides BO, AC, respectively' 
and the triangles APN, BPM are r^^arded as positive) ; find (1) the locus 
of the point P, such that the ratio ( AAPN+ aBPM) : a ABC may have 
a given value ; (2) drawing from each point P, at right angles to the 
plane of the triangle, an ordinate PQ of a length proportional to the fore- 
going ratio ( A APN + aBPM) : A ABC,trace the surface which is the locus 
of the point Q, a surface which has the loci in (1) for its contour lines • 
(3) find the volume of the portion standing on the triangle ABC as base • 
and (4) deduce the solution of the following case of the four-point prob- 
lem, viz., taking the points P, F at random within the triangle ABO 
what is the chance that A, B, P, P' may forma convex quadrangle P. . . 149 

7060. (D. Edwardes.)— If P be any point in the plane of a triangle 
ABO, and d its distance from the circumscribed centre, show that 
PA*sin2A+PB2sin2B + PC»8in2C = 4(E2+<P)sinAsinBBinC... 147 

7069. (D. Edwardes.)— If a?, y, « be the distances of a point P from 
the angular points of a triangle, prove that the mean value of jb* sin 2A 
+y«sin2B+«*sin2C, as P ranges over the circle about ABC, is three 
times the area of the triangle 147 

7169. (W. J. Greenstreet, B.A.)— The sum of the three sides of a 
right-angled spherical triangle is a quadrant : im>ve that (1) the minimum 
value of the hypotenuse is cos-^f, and (2) in this case the spherical excess 
issin-i^ , , 5j 



Digitized by VjOOQIC 



OONTENTS« IX 



7270. (W. J. 0. Sharp, M.A.)— Eliminate a, b, t, a', *', e' from 



a+b+e = d 
ab + be+ea » 8 
abe = 8 



a' + b' + i/^df 



aa' + W + c*/ — ^, 
a'be + aye+abe' :^ e, 
ab'e' + afbc' + a'b'e^e',,,. 162 



7801. (Professor Cayley, F.R.S.) — If^ the function ?5!Lt^ « ^ 

is periodic of the third order {<ffiu = u) : given that the cubic equation 

(ft, b. e, dSx, 1)8 = has two roots w, v such that v =■ **"'*'^ , find u as 
^ "^ ' 7M + 8 

a rational function of a, d, 0, <^, a, /3, 7, 8 ; and examine the case in which 

u is not thus expressible 163 

7366. (Professor Wolstenholme, M.A., Sc.D. Suggested by Ques- 
tion 7286.) — At each point P of a given curve is drawn a straight line 
U, making a given angle with the tangent at P, and a straight line Y, 
such that U, V are equally inclined to the ordinate at P ; prove that the 
point of contact of U with its envelope is the projection upon U of the 
centre of curvature at P, and that the point of contact of Y with its en- 
velope is the projection upon Y of the image with respect to the tangent 
at P of the centre of curvature at P. [That is, if be the centre of cur- 
vature at P, and OPO' be a straight line bisected in P, then, if OL, O'M 
be let fall perpendicular to U, Y respectively, L, M will be the points of 
contact of U, v with their envelopes.] 64 

7469. (Professor Wolstenholme, M.A., Sc.D.) — Prove that if , in a 
tetrahedron, any one or two of the equations a±a: = ^±y = c±«be 
true, tlien will also the corresponding one or two equations of the set 
AiX«B±Y = C±Z also be true 86 

7661. (Rev. T. 0. Simmons, M.A.)— If 

8in(a + iB) + 8in(/3 + 7) + 8in(7 + a) -0, 

show that cos ^ (o — jB) cos i (jS - 7) cos i- (7 — o) cannot be greater than J. 

124 

7707. (Professor Malet, F.E.S.)— Prove that 
fi'log sin B (log tan 6-log A (6)} 
Jo AM ^— ^^-iK{log*log*' + iir«}+i,K'log^', 

)*«log cos e {log cot 6- log A (e) } 
A^6) ^ ^ *'' ^ 
fl«log A (a) (log A (e) -log sin 6 cos e\ 
Jo AW ^rf6 = iKlog^log^ + i,K'log^, 

where A(e)s(l-A;2 8in2e)*, F + A;^=l, and Z and K' are complete 
elliptic integrals of the first kind with moduli respectively k and hf... 68 

7987. (Rev. T. R. Terry, M.A.) — Four spheres, whose radii are a, ft, 
tf, d respectively, are such that each touches the other three externally. 
Li the space between these four, another sphere of radius r is described 
touching all four externally. Show that 

^-7'(l)"(ir)-(i)-« «•)■ 



Digitized by VjOOQIC 



X • COKOTNTS. 

8036. (B. Lachlan, B.A.)— If four spheres; radii a, b^ e, d, touch 
^ one another externally ; and r be the radius of the sphere whLoh cuts them 

orthogonaUy,theni— 2a/JL\ -2 f-^) 46 

8117. (Professor "Wolstenholme, M.A., Sc.D.)— Two coincoids S, S' 
have two common plane sections, and the poles of these planes with 
respect to S are the points P, P' ; prove that (1) if S' pass through P, it 
will also pass through R ; (2) also, in this case, the following relations 
must hold (02-3#A) (ee'-6AA0 + S^aa'^ 0, e» = 27a« (ee'-6AA0. 
ThediscriminJantof A?S + S' is A)fe* + e/fc5 + #A;3 + e'^ + A'.] 79 

8135. (Hev. T. G. Simmons, M.A.)— If G be the centroid, I the in- 
entre, of a phme triangle,^ prove that 

IGP = 4R2(i + cosAco8BcosC)-jRr + fr« 86 

8204. (A. Gordon.) — ^If A, B, G, D, E are five points on a sphere, 
Vi2 the volume ACDE, V,3 the volume ABDE, &c. : prove that, with a 
certain convention as to sign, 

Vm(AB)2 + Vi3(AG)«+Vi4(AD)« + Vi5(AE)2- 165 

8207. (W. J. G. Sharp, M.A.)— -If A be the angle contained in the 
half of a small Gircle of a sphere, angular radius a, by arcs of lengths 
b and e ; i.e., if ABG be a spherical tnangle having the angle A » B + G ; 
show that (1) cos ^ + cos c = 1 + cos a ; (2) cos A = —tan ^* tan ^c ; and (3) 
hence deduce Euclid I. 47 and III. 31 62 

8236. (Rev. T. R. Terry, M.A.)— A uniform lamina bounded by the 
arc of a parabola and its latus rectum is revolving with angular ^locity 
» about the latus rectum : suddenly the latue rectum becomes free and the 
vertex becomes fixed. Show that the angular velocity about the tangent 
to the vertex is |« 116 

8237. (W. J. G. Sharp, M.A.)— If nOr denote the number of combi- 
nations r together which ean be formed out of n things ; show, from 
d priori considerations, that, 

nCr - nG„-r, nCr = ^n-lGr-i, nOr = n-l^r + n-lOr-l (1, 2, 3), 

or, more generally, nOr — ii-j»Gr + pGj n-pGr-i +p02 »-pGr-2 + &c. ... 166 

8242. (Professor Sylvester, F.R.S.) — If by a Simplicissimum of the 
nth. order be understood a figure in space of n dimensions formed by 
the indefinite protraction of the series of which a linear segment, a tri- 
angle, and a pyramid are the three first terms, prove that (1), when each 

edge is unity, the squared content is .^ ^^"^ ' -, and hence (2) de- 
duce that the Gayleyan Persymmetrical Invertebrate Determinant of the 
squared edges by which such squared content is imaged must be 
diminished in the ratio of negative unity to ( -2)» (1 . 2 . 3 ... «)2, in order 
that it may represent its absolute value. JSx, gr,^ the determinant 

(abY {ae)^ 1 

{bay . (bcY 1 

{caf {eby . 1 

11 1 . 



Digitized by VjOOQ IC 



CONTENTS. XI 

images the squared content of a triangle whose edges are^ {ab)^ (ae), {be)f 
for the triangle vanishes when this determinant yanishes, hut the actual 
value of the squared content is this determinant diminished in the ratio 
of negative unity to 2^ (1 . 2)', i.e., multiplied hy — ^ = (2) 53 

8298. (E. Knowles, B.A.)— If Pr denote the coefficient of of, in the 
expansion of (1 +^)'*, where n is a positive Integer, prove that 

119. 

8302. (A. Gordon.) — Prove that the surface a:' +y« +«' = <?« will 

represent a surface of revolution if oo^xy « cos*y« « cos' 2^, where xy 
denotes the angle hetween the axes of x and y, 117 

'8311. (Prof. R. Swaminatha Aiyar, B.A.) — Find how many numhers 
can he formed, having n for the sum of the digits (zero not heing used 
as a digit) 64 

8336. (Asparagus.) — From a point are drawn two chords OPF, 
OQQ' of a given circle, and two tangents OA, OB ; a conic is drawn 
touching the straight lines PQ, PQ', P'Q, P'Q' and passing through A 
or B ; prove that this conic will toudi the circle in A or B 117 

8380. (P. C. Ward, M.A.)— Prove that (1), 

a + *— c, 4a, 6fl, 4a 

4i, a + *— c, 4a, 6a 

6i, 4i, a + b—Cf 4a 

4kby 6d, 4d, a-^-b-c 

s (a + i+c)4-8 (a + i + c)2(ic+ (?a+ a3) - 128 a*<j(a +i + <?) + 16 (*<? + ca + a*)» 

= result of rationalizing a^ + 6^ + e;^ = ; and hence (2) the ahove deter- 
minant is symmetrical with respect to a, d, 107 

8386. (J. Brill, M.A.) — Three paraholas are drawn having a common 
focus ; from a point T, external to all three, tangents TP and Tp are 
drawn to the first parabol^, TQ and Tq to the second, and TE and Tr to 
the third ; prove that Qr . Ej? . P^ == ^E . rP ,pQ, 95 

8406. (F. 0. "Wace, M.A.)— If aj + aj + «8 + . • . + «n «= «, prove that 



(^')"(t-)'' (i-'r<<-)' 



66 



8420. (Emily Perrin, B.Sc.) — Prove that the axis of perspective of a 
triangle and its pedal triangle is the common radical axis of the circum- 
circle, nine-point circle, and self-conjugate circle 67 

8423. p. Edwardes.)— Prove that 



(1) f*'sina;(logsina;)aefo? = 2 + (log2)2-21og2— ^ir2, 

(2) (*'sin2a;(logsinar)2c?a;=: Jir{2(log2)2-21og2 + ^ir2-l}. ... 119 
Jo 

8452. (Satis Chandra Esly, B.A.)^Two rods, of lengths a and 3, are 
jointed hy a smootii hinge and rest on the convex side of a parabolic 



Digitized by 



Google 



XU OOKTBNTS. 

arc whose axis is vertical ; if, in the position of equilibrium, tbe rods in- 
clude a right angle, show that Ihe angle which the chord of contact 
makes with the axis ifl given by tan iO B a^/ ^ 62 

8473. (Professor Booth, M.A.) — If a, 6, ^ be the sides of a plane 
triangle, prove that the diameter of the circumscribing circle is a root of 
the equation 

••■• o9 

8498. (AsfLtosh Mukhopddhyay, M.A., F.B.A.S.)— The equation 

\dxl dfi dx'dt'dxdt \dtldx^ \dx I 

is intepfrable when (1) Q— 0, (2) Q «» dx/dt. Hence, obtain the complete 
primitive 61 

8501. (R. Knowles, B.A.) — ^In any triangle, show that 
icos^Acos'B— cos(A— B)(3co8Aco8B— sinAsinB) scos^c... 75 

8602. (B. Hanumanta Rau, B.A.) — Through any point K are drawn 
the straight lines B"KC', C"KA', and A"KB' respectively parallel to 
the sides BC, CA, AB of a triangle ; prove that (1) A A'B'C'= A A"B"C" ; 

(2) parallels through A, B, C to A'B', B'C, C'A' meet at a point 0'; 

(3) parallels through A, B, C to A"C", A"B", 0"B" also meet at a point 
(y ; (4) if the coordinates of and 0' are (a, fi, y) and (a', jS', y') then 
aa : ey'^ bfi : oaf— cy i bfi' \ (5) if K is the Symmeoian point, O and (Y 
become the Brocard points 49 

8621. (Professor Wolstenholme, M.A., Sc.D.) — The circle of curva- 
ture is drawn at a point P {am^, 2am) of the parabola i/^ = iax, RE' is the 
common tangent of the puabola and circle, and meets PQ their common 
chord in T ; prove that 

(1) TR : TR' = 1 +4m3 : 1 ; (2) RR' = 16anfi (1 +m2)t « PQ8/ 32aa; 
(3) TQ : TR' = TR' : TP « «»2 : l + m2; 

and (4) the locus of T is a seztic having no real rectilinear asymptotes. 

114 

8537. (For Enunciation see Question 7169.) 61 

8643. (R. Curtis, M.A.)— Prove (1) the following formula of trans- 
formation for the equation of a conic to a triangle of reference 
the sides of which are the polars of the vertices of the former tri- 
angle (a, b, c, /, ffy h) (ar, y, «)« - A- 1 (A, B, C, F, G, H) (X, Y, Z)2 ; and 
hence (2) show that the equations of a conic referred to an inscribed tri- 
angle and to one circumscribed at the points of contact will be 

fyz^-gzx^hxy = and (/X)* ± (^Y)* ± (AZ)* = Ill 

8648. (Asparagus.) — ^Two straight lines turn about two fixed points 
0, O' with angular velocities which are as 1 : 3, both straight Hues coin- 
ciding with 00' initially ; if P be their point of intersection, prove that 
the envelope of a straight line drawn tiirough P at right angles to OP 
will be a parabola whose directrix passes through 0' and is at right angles 
to 00' and whose latus rectum is twice 00' 65 



Digitized by VjOOQIC 



• •• 



CONTENTS. XIU 

8557. (Frofeesor Mathews, M.A.)— If 

^ + i^ = l and ^+i^^l 

are two confocal eUipses, such that polygons of r sides can be simnl- 
taneously inscribed in the second ellipse and circumscribed to the first, 

prove that a - a'sn tl^^^, b - *'cn ?5, 

r r 

where the modulus of the elliptic functions is equal to the eccentricity of 
the inner ellipse. Verify the above when r = 3, 4, 6 respectively.... 66 

8659. (Professor Wolstenholme, M.A., Sc.D.) — ^From a fixed point 
are drawn OP, OQ tangents to one of a system of confocal conies (foci 
S, S^ centre C), and from C are let fall perpendiculars on the normals 
at P, Q ; prove that the envelope of the straight line joining the feet of 
these perpendiculars is the conic (parabola) 

{X(a:coso-hysina)+Y (a:sina— y cosoj-fiScosa}' 

— 4XY(a:cosa + ysino)(«sina— ycosa), 
where is origin, OS axis of x, (X, Y) the point 0, SS'= 2c, and a is 
the sum of the angles which SO, S'O make with the axis of x» 

[If X«-Y3 « c«, the straight line is fixed.] 128 

8671. (Asatosh Mukhopadhyay, M.A., F.R.A.S.) — Show that the 
reciprocal polar of the evolute of the reciprocal polar of the evolute to 

the conic -^ + -^ = 1, with respect to the circle described on the line 
joining the foci as diameter, is the curve 

mHmm'(i)'-(i)'mHf-ir-'> 

8686. (B. F. Davis, M.A.)— If TP, TQ be tangents to a parabola, and 
PQ meet the directrix in Z, prove that ZT will be a mean proportional 
between ZP and ZQ 60 

8600. (Professor Mahendra Nath Ray, M.A., LL.B.)— Through an 
indefinite point of a given hyperbola straight lines are drawn to meet the 
asymptotes ; show that the hyperbola itself is the envelope of the locus 
of the middle points of the straight line 50 

8607. (R. Curtis, M.A.) — Between two curves, f{x, y) — and 
^ (^f y) — 0) <>f degrees m and n, show that there can be drawn mn straight 
lines of a given length I, parallel to a given straight Une, from the curve 
/{xy) « to the curve ^ {xp) = 0, and mn othera from tp (i^y) = to /(ay) 
= 0. [The lines must be drawn in a given direction, say from left to 
right.] 66 

8623. (N'Importe.) — From the centre of similitude S, common radii 
vectores SPP', SQQ' are drawn to similar curves PQ, P'Q'. Having 
given the centre of gravity of the area SPQ, find that of PQQT' ; and 
its limiting position when PQ, P'Q' ultimately coincide 60 

8631 . (Professor Sylvester, F.R.S.)— Find the discriminant of 

a^ + y^ + sfi + Ze^^ + 6exyz 118, 166 

8637. (Professor Hudson, M.A.) — The tangent and normal to 
y/e =• ^ (€*/« + €-*/«) cut the axis of « in T, O, respectively ; find tiie mini- 
mum VBlue of GKT. ...► 161 

b 



Digitized by VjOOQIC 



XIV CONTENTS. 

8639. (Frofeflsor Wolstenholme, M. A., So.D.) — The distances of a 
point O (within a giv^n triangle ABC) from the angular points are 
«, y, z respectively; prove that the volume of a tetrahedron in which 
a, bf e are the lengths of three conterminous edges, and x, y, z the lengths 
of the edges respectively opposite, is -^ {{(^-sc') (*'— y^) (c*-**)}*... 30 

8643. (Professor Gtenese, M.A.)— Four complanar forces Pj, Pj, &o. 
are in equilibrium. Their lines of action (omitting one at a time) deter- 
mine four triangles whose areas are A^, A3, &c., and circum-radii Bi, B3, 

&c. Prove that Pi : Pj, &c. - ^ : ^, &c 35 

Ki B, 

8646. (Emile Yigarie.) — On donne deux points A et B sur une drcon- 
fi^rence de centre O ; trouver sur cette circonference un troisi^me point 
P, tel que les droites PA, PB coupent un diam^tre fixe CC en des points 
M, N, tels que I'on ait OM - ON 36 

8647. (R. W. D. Christie, M.A.)-— Prove that 2 + S » 2«», if 

« = 18+28+... +n», S- 16 + 2« + ...n6, 2 - l7 + 27+...n7. ... 33 

8650 (E. M. Davys, M.A.) — ^Prove that, in any plane triangle, 
cos"A + 4 cosM> A sin«A+6 cos* Asia* A-5 cos* A sin8 A-4 oos» A sinw» A 
-sin«A = cos2A 46 

8656. (W. J. Greenstreet, B.A.) — ^In a spherical triangle, prove that 

8in*R= (coso-co8»R)* + sin«asin3Rsin«(S-A) 118 

8668. (W. J. C. Sharp, M.A.)— Boolb obtains the result 

[[ ... X<fe»s is^-^ f X(te--(n-l) «*-« {Xxdx 

^ (n--l)(n-2) ^^,f.^^^^ ^f^^.1^7 

by a symbolical method ; show that it may also be obtained by ordinary 
methods 71 

8660. (B. Hanumanta Bau, M.A.) — Investigate the singular sdn- 
tion of the equation 4t Ix + p J^\ +25 [ y^—xf/ ^ j « 0, and show that 

it is the envelope of a series of circles described on the subnormal of an 
ellipse as diameter 83 

8661. (A. Q-ordon.) — ^Any curve of the 4th degree intersects a conic 
in 8 points. These are joined, forming an octagon A^As... A«. Show 
that the 8 intersections of Aj with At and A«, As with A^ and Ag, &c., 
&c., will lie on a conic 118 

8671. (D. Edwardes.) — Prove that the laiua rectum of a parabola 
is half the harmonic mean between any two focal chords at right angles 
to one another 123 

8676. (Professor Bordage.) — The three sides of a triangle forming an 
arithmetical progression, (a) being the shortest, (a') the longest ; if the 
distance of the centres of the inscribed and circumscribed circles is desig- 
nated by t, and the diameter of the nine-point circle by D, prove that 
tfa'- 3(D2-f3) 107 



Digitized by VjOOQIC 



00NTBNT8. XV 

8678. (Profeflsor (^enese, M.A.) — AB is a obord of a. conic, BP, BQ 
are parallels to the asymptotes, APQ a variable transversal meetiDg the 
curve at E. Prove that the ratio PB : EQ is constant 46 

8681. (Professor Hudson, M.A.) — In the epicycloid of two cusps, 
if P be the describiug point when Q is the point of contact, prove that ' 
the tangents at P, Q to the epicycloid and the fixed circle respectively 
meet on the line joining the cusps 46 

8683. (Professor Ignacio Beyens, M.A.)— Si dans un triangle la pro- 
jection du c6t^ BC sur BA qui est BH, et la projection du BA sur ACTqui 
est AH', sont egales ; la bissectrice du A, la hauteur du et la m6diane 
de B se rencon^ent en le mdme point 96 

8685. (The Editor.) — Given an angle at the base of a triangle, the 
sum of the two sides, and the distance between the given angular point 
and the point of contact of the escribed circle touching the base ; con- 
struct the triangle 75 

8692. (B. F. Davis, M.A. Suggested by Question 8559.) — If, from 
any point in a given fixed straight line passing through the focus of a pa- 
rabola, tangents be drawn to the curve, prove that the envelope of the Hne 
joining the feet of the perpendiculars on these tangents from a given 
fixed point on the directrix is another parabola 129 

8696. (Emile Vigari6.) — Dans un cercle donn6, par un point P pris 
sur la circonf6rence on m^ne trois cordes PA, PB, PC ; sur chacune 
d'elles comme diam^tre on d6crit une circonf6rence ; d^montrer que les 
trois points de rencontre sont en ligne droit 32 

8697. (Captain H. Brocard.)— Les droites qui joignent le centre du 
cercle circonscrit aux milieux des segments interceptis par les perpen- 
diculaires aux milieux des cdt6s sur les hauteurs du trian^e sont perpen- 
diculaires aux m^dianes correspondantes 49 

8700. (R. W. D. Christie, M.A.) — Given the sum of the expres- 
sion 1*" + 2*" + S*" + 4*' +«•', to find a method of writing down at once 

the sum of l»'+i + 2^+1 + 3*'+^ + 4*'+ * +!•••♦*, where r is even or vice 

verad 96 

8708. (Maurice D'Ocagne.) — Si, dans le quadrilat^re ABCD, les 
angles opposes B et D sont droits, la droite qui joint les pieds des perpen- 
diculaires abaiss^es du sommet C sur les bissectrices int^rieure et ext^rieure 
de Tangle A passe par le milieu de la diagonale BD 34 

8713. (Professor Steggall, M.A.) — Show that the solution in ra- 
tional quantities of the equation z^+y^-^-z^ =^u^, is x ^k{a^-¥b'^—i^, 
y^2kae, z ^ 2kbe, w « A; (a2 + 62 + c2) 31 

8714. (Professor Genese, M.A.) — S is a focus of a conic, PN a 
fixed ordinate to the diameter through 8, PQP' a circle with centre S ; 
a variable radius SQ meets PN at L and the conic at B. Prove that the 
cross ratio {SLQB} is constant 107 

8715. (Professor Hudson, M.A.) — Prove that 

tan37i°--v/6+ V3-V2-2 88 

8716. (Professor Mathews, M.A.) — Prove that the real common 
tangents of the circles ic^+y^— 2aa: = 0, afi-¥y^ — 2by = are represented 
by 2a*(af'+y2— 2<M?) « (*y— fla? + a*)', or, which is the same thing, by 

2ai(»« + y»-2iy) =-(iy-a«-a4)2 6X, 83 



Digitized by VjOOQIC 



XVI oonxRB. 

8718. (PwfiMnr VmSbm^.}-J/mm^ BAG ^^mat ^n^ aatfere, et 
(1) le emtn de gnTit^ (2) Fortkocaotie, (3) le eoatee dm ooRfe cnoon- 
icnt pMComi BBe dlioite do niife . Troowl'eBvdoppedmoM BC... 116 

8719. (ProfiBMor Wolrtenbofane, ILA., ScJ).)— Tlie kngtiis of the 
edge* OA, OB, OC of a tetnhednm OABC an denoted bj a» *, r, thofle 
of tltere^ecthreljoppoaiteedg«8BC,CA«ABb7x,jr,s; and tiie dihedral 
aii£^ i;p>poM^ to then hj A, B, C ; X, T, Z ; prove that, if Y be the 

Tolmne expraMed in terms ofa,^^, ^ 9$ *» ^* |«jpoot A, &c. ; 

and tiienoe tiiat (1) when a, s ; ^, jr, and c+s are /»•«», Y will he a 
niaTimom when C » Z ; (2) when a» s ; I, jr, and «~s are girai, Y will 
heaniazimiimiHMnC-i-Z s 180°; (3) iHbena, s; ^-fjf; c-fs are given, 
Y will be a TnaTJimim irilien B » X and C » Z (i.«., iri^ ^ ««, « « i) ; 
(4) iHien a, x ; b+f, e—z are giren, Y will be a maTJirnim when B » X 
C+Z » 180*" ; and (5) inyeiti^te if Y can have a mazimnm when a, s ; 
3— y, e^s are given, or iritien a^a, ^— 9f» «— s are given; the given 
qnandlxties being sappoeedahrays real and finite. 

In case (4), prove also that 180** is the mazimnm valoe ol C-l-Z for 
Tariations inb,p (sabject to the conditions stated), and that aero is the 
mininram value of B— Y for variations in ^ s 39 

8724. (B. Tucker, M.A.)— If a constant line AB moves with ends 
on OX, OT, two rectangular axes, and on it a semicircle is described, 
then locos of mid-point <^ arc is one of two straight lines. Find locus of 
any other fixed point on the arc. [If G is Ihe mid-point of AB, and P the 
mid-point of the arc, then the question may be enunciated for a bar rigidly 
connected with the bar AB.] 36 

8725. (8. Tebay, B.A.)— If the sines ol opposites dihedral angles of a 
tetrahedron be ren)ectively proportional to the edges formed by these 
angles, the areas of the four &oes are eqnaL 48 

8726. (8. Boberts, M.A. Analogous to Quest. 3068.)— Show that, if 
some four of the roots of a quintic form an harmonic system, then 

J»-27.3«.JK+ 2B.3».L = 0, 
where J, K, L are the three fundamental invariants of the orders 4, 
8, 12 [see Salmon's Higher Algebra, 3rd ed., p. 211] 33 

8727. (B. Hanumanta Bau, B.A.)— The sides BC, GA, AB of a tri- 
angle ABU are produced to a, b, e, such that Ga, Ab, "Re are respectively 
equal to BG, GA, AB. Prove that the centroid of the triangle abe coin- 
cides with that of ABG 84 

8728. (Gaptain H. Brocard.) — De cheque sommet d'un triangle on 
m^ne des perpendiculaires aux c6t6s adjacents, jusqu*^ leurs rencontres 
aveo le c6t6 oppKOs^. Les centres A', B', G' des cercles circonscrits aux 
trois triangles ainsi formes sont les sommets d'un second triangle homo- 
logique avec le propose. Le centre d'homologie est le centre O du cercle 
circonscrit au triangle donn6. En d'autres termes, les nouveaux cercles 
iont tangents au cercle circonscrit, aux points A, B, G 121 

8729. (B. W. D. Ghristie, M. A.)— Of the series 

1.1 + 3.6 + 6.13 + 7.26 + &c. 
to n terms, show that, (1) the sum is a square ; (2) the square root is 
m 1 + 3 + 6 + 7 + &C. to n terms; (3) each term is the product of two num- 



Digitized by 



Google 



contents; xvu 

bers which may be taken to represent the shorter side and hypothenuse of 
a right-angled triangle ; (4) if unity be added to the nth term, it is 
divisible by 2», and H unity be subtracted it is divisible by tt— 1 ; and (6) 
all terms are odd numbers 106 

8737. (W. J. Greenstreet, B.A.) — ABC is a spherical triangle, the 
mid-points of its sides are the angular points of a triangle DEF ; prove 
that (1) cosEF / cos |a = cosFD / cos ^b » cosDE / cos ^c ; 
and (2) if cos^ ia » coB^{b + e) coa^(b^e), the angle D is a right 
angle. ;. 108 

8746. (W. W. Taylor, M.A. Suggested by Question 7938.) — 
Prove that (1) the areas of the TAYLOR-circles of the four triangles 
ABC, PBC, PCA, PAB are together equal to the area of the circum- 
scribed circle, P being the orthocentre of the triangle ABC ; also (2) the 
lines joining their centres are the SiMSON-lines of the middle points of the 
sides or of AP, BP, CP with respect to the pedal triangle 100 

8749. (F. Purser, M.A.) — Find (1) the cubic locus of the centre 
of a coDic passing through the three vertices A, B, C of a tri- 
angle, and such that the three normals at these vertices are concurrent ; 
and prove that (2) this cubic passes through the in-centre I, the three ex- 
centres la, Ifr, Ic, the circum-centre O, the orthocentre H, tiie centroid G, 
and the symmedian G', and cuts the three sides normally at their middle 
points L, M, N ; also (3) tangents at (I, !«, Ij, Ic), (A, B, C, G), (L, M, 
N, G') are respectively concurrent in G, G', ; and (4) the lines joining 
L, M, N meet the curve again in their intersections with the respective 
perpendiculars from the vertices 35 

8764. (Professor Mahendra Nath Eay, M.A., LL.B.)— Prove that 

(1) r^^^=^(i,-l), (2) ri«lB!£^ = ,(^-|) 123 

^ ' Jo l + cos^a: ^* '» V / J^ i + cos2a; ^ '^ 

8766. (Professor Neuberg.) — On prolonge les hauteurs du triangle 
ABC au deU, des sommets des quantit^s AA'=BC, BB'^CA, CC'^AB ; 
d^montrer : (1) les triangles ABC, A'B'C ont m^me centre de gravite ; 

(2) si a, a' sent les angles de Brocard de ces triangles, on a 

A'B'C=2ABC(2 + cota), (A'B')2+(B'C')3+(C'A')« = 8ABC (3 + 2cota), 
cota'= (2coto + 3)/(cota + 2) ; 

(3) les points A, B, C sont les centres des carr^s construits, int^rieurement, 
sur les c6t^ du triangle A'B'C ; (4) les milieux des c6t^s de A'B'C sont 
les centres des carr^ construits, ext^rieurement, sur les c6t68 de ABC. . . 56 

8767. (Professor Edmund Bordage.) — If two fractions aJa'f bjb' are 
such that ab' ^baf = I, prove that the simplest fraction included between 
the two given fractions is {a + b) j (a* + b*) 46 

8763. (H. Stewart, M.C.P.)— Prove that 

(l)if w(l-a;2)* + a;(i-w»)* = fl2, then ^ + tlr:!^* = 0; 

^ (1-a^* 

(2) if (l-«2)i + (i.^)i = a («-«), then ^==(ir:^*.... 100 

^ ' ^ ' ^ (!-«»)* 



Digitized by 



Google 



XVIU OONTBNTS. 

8765. (L. J. Bogers, M.A.) — If :r|-(-02 + ^-l-ai + as + as« 0, prove 
that (1) tan (^i + a^) , tan {±^ + aj) , tan {x^ + oii) 

tan (a?! + oj, tan (arj + oj), tan (a:, + a,) 

tan (a?! + 03), tan(a;3 + 08), tan(a>8 + aa) 
and (2) the same is true if tangents are replaced by sines 88 

8768. (R. F. Davis, M.A.) — Required an Analytical Proof of Feuer- 
bach's Theorem. 

[An analytical proof is given by the Rev. J. J. Milnb, M.A., in the 
Introduction to his "Weekly Problem Papers, but this proof may be modi- 
fied and improved.] 131 

8769. (J. Brill, M.A.) — An ellipse is inscribed in the triangle PQR 
so as to touch the sides QR, RP, PQ at P', Q', R' respectively ; prove 
that, if C be the centre of the ellipse, 

aQOR : aQ'CR' - aRCP : aR'CF - aPCQ : aP'CQ'.... 34 

8778. fW. J. 0. Sharp, M.A.) — If wy—vz, uz^wx, and vx—uy be 
cogredient to x, y, and z, the rectangular Cartesian coordinates of a point 
respectively, «, v, And to are so 44 

8783. (Captain H. Brocard.) — ^Les droites qui joignent I'orthocentre 
aux milieux des segments intercept's par les hauteurs du triangle sur les 
rayons du cercle circonscrit aboutissant aux sommets du triangle sent 
perpendiculaires aux sym'dianes correspondantes 87 

8784. (R. W. D. Christie, MA..)— Prove that, if 

« = 1+2 + 3 + .. .+n, S = 13+28 + 3«+...+n2, S' = 18 + 28 + 3»+. ..+»», 
2 = l* + 2* + 3*+. ..+«*, <r = l« + 26+3«+... + fi«, 

then '-^-1^ 99 

8786. (R. Tucker, M.A.)~Pp, Qq, Rr are focal chords of a para- 
bola ; if P, Q, R have a conormal point, then the centres of curvature 
for j3, qt r are collinear. (This is another way of putting the first part of 
Question 8693.) Find the equation to this Ime. Prove that this central 
line envelopes (1) a hyperbola when tangents atjt?, q meet on the directrix, 
(2) an ellipse when the tangents meet on the latus rectum 61 

8789. (Professor CuUey, M.A.) — Find by a geometrical construc- 
tion the equation 8S' = P* of the pair of tangents from T (a:', y') to a 
circle S s a?*+,v2— «« =s 0, whose centre is ; and show that, if perpen- 
diculars RL, RM, RK be drawn to the tangents and their chord of con- 
tact from a point R not on the circumference, RL . RM differs from RK* 
by TO*^ cos* ^ cos* 0, where 2e0 and 20 are the angles subtended by the 
circle at T and R respectively 37 

8793. (Professor Neuberg.) — Soient AB, CD deux droites, qui se 
coupent en O. Si Ton fait toumer AB autour de O, le point double 
de deux figures semblables construites sur AB et CD d^crit une circon- 
f^rence de cercle 30 

8797. (Professor S§irad§iranjan Rly, M.A.) — GAP and OBQ are 
two fixed straight lines intersecting at 0, and C is a circle touching them 
both at P and Q ; prove that the perimeter of the triangle OAB, circum- 
scribed by ani/ circle passing through O and touching C externally, is 
constant , , 89 



Digitized by VjOOQIC 



CONTENTS. XIX 

8799. (Professor Hudson, M.A.) — ^Find the envelope of the straight 
line flw =s/(o)cos(e— o)+/(o)8in(e-ai) 41 

8801. (Rev. T. P. Kirkman, F.K.S. Suggested by Quest. 8326.) 
— Upon the most sacred of the undiscoyered Egyptian pyramids, 
whose vertex P is at unequal distances from the angles of its scalene 
base ABC, is cut in the face PAG an ascending flight of steps from A to 
b, in PC ; from b another flight mounts to in PB ; and a third leads up 
from c to a, At a. given altitude h in PA. These three flights are equi- 
gradient. A second path from A mounts to ^' in PC, and thence pro- 
ceeding to (/ in PB, ascends from (/ to the above point a in P A . These 
three flights also are equigradient. By Abca Pharoah and his Grandees 
of Churdi and State were wont to mount to a platform at a for solemn 
religious rites ; the admitted vulgar used only the path A&Va. Required, 
a plane projection of the pyramid and its six flights of stairs, and proof 
from that prelection that the first three are equigradient, and the second 
three equigradient, up to the given altitude h 24 

8804. (S. Tebay, B.A.) — If (a, ft, c) be conterminous edges of a 
tetrahedron, {x, y, z) the respective opposites, (A, X ; B, Y ; C, Z) cor- 
responding dihedral angles, and Y the volume ; show that (1) 
ax by ^ cz . 

sinAsinX sinBsinY sinCsinZ' 
also (2), if the areas of the four faces are equal, then 

a - a:, ft - y, <j - «, A = X, B - Y, = Z. 
V = tV {2(ft2+(^-a?) (c2 + a2-fta) (a^^b^^(^)}\ 

(ft-<>)sinA+(<j-a)sinB+(a-ft)sinC — 31 

8807. (For Enunciation see Question 8746.) 100 

8808. (F. R. J. Hervey.) — ^Find (1) in how many ways n lines of 
verse can be rhymed, so as to have r different rhymes, and no line un- 
rhymed ; and (2) show that, in the case of the sonnet, the numbers of ways 
with 2, 3, ... 7 rhymes are, respectively, 8177, 731731, 6914908, 12122110, 
4099096, and 136136 76 

8810. (Rev. T. R. Terry, M.A.) — Find the equation to the straight 
lines through the origin and the intersection of the conies 

«»-3a;y— 4a: + y-l « 0, 2a;3 + a;y + 4y3 + 2a;+13y+8 =* 0. ... 68 

8813. (J. Griffiths, M.A.)— If we have 
f{x) -Ao+2A3a;3 + 2A4r* ■•-... adinf,, ^{x) ^\-2.B^-2^^x^-^,..adinf, 

where Ao - 1 + 22 (-)*cn2 ?5, Bj = 2 (-)* cn^*^ -i- sns*-^, 

n n n 

A, « ifesi :i(-)' cn2 !5sn2?5, B4 « 2(-)' cn«^ + sn* *-?, 
n n n n 

A^^k^% (-)• cn^^sn*^, Be» 2(-)*cn2?5 + Bn«?l, 



and « is a nvmber from 1 to n— 1, show that/(a;) ,^(x)^ A«. 78 



Digitized by VjOOQIC 



XX CONTENTS. 

8814. (The Editor.) — Prove that the triangle formed by joii 

the mid-points of the altitudes of a triangle is one-fourth of the pe 

triangle, and that the theorem holds good for any other three concurrent 
lines drawn through the vertices of the triangle 59 

8816. (Captain H. Brocard.) — n, Of d6signant les points de Brocard, 
S le milieu de Xlfi', S' le point correspondant k S par droites sym6- 
triques, It le point de Steiner ; d^montrer que ce point It se troave sur la 
droite SS'. (R est une des intersections du cercle ABC avec le diam^tre per- 
pendiculaire k I'axe d*homologie du triangle ABC et du premier triangle de 
Brocard. Ce point est aussi I'intersection commune des paralldles auz 
c6tes du premier triangle de Brocard, menses par les sommets corres- 
pondants du triangle ABC.) 22 

8820. (Charles F. Lodge.) — A mirror, measuring 33 inches by 22 
inches, is to have a frame of uniform width, whose area is to equal that 
of the glass ; show that the width of the frame is 6\ inches 78 

8823. (Professor Crofton, F.R.S.)— Two discs of any form are move- 
able in a plane round two fixed points A, B, respectively. Show that, 
when they are in such positions that the lengfth of an endless band en- 
veloping both is a maximum or a minimum, the portions of the band 
which form the common tangents will meet on AB if produced, and are 
equally inclined to AB 26 

8830. (Professor Bordage.) — Given the equation 

sin 2a . 0?^ + 2 (sin a + cos o) a; + 2 — 0, 
find, and solve, the equation of the second degree that has as roots the 
squares of the roots of the given equation 110 

8833. (Professor Neuberg.) — Un angle de grandeur constante toume 
autour de son sommet A, ses c6tes rencoutrent une droite donn^ XY aux 
points B et C. Trouver (1) les enveloppes de la m^diane BB' et de la 
sym^diane BH ; (2) les points oil ces droites touchent leurs enve- 
loppes 38 

8836. (Professor de Longchamps.) — On consid^e un triangle ABC 
et le cercle circonscrit A ; soit /a une transversale rencontrant les cdt^s 
de ABC aux points A', B', C. Par A' on m^ne h. A une tangente que 
Ton rabat sur BC de telle sorte que le point de contact vienne occuper sur 
BC une certaine position A". Demontrer que la droite AA" et les deux 
droites analogues BB", CC" vont passer par un mSme point M 94 

8837. (The Editor.)--OD the sides of a regular «-gon, n points are 
taken at random, one on each, forming the apices of inscribed M-gon ; 
again, inside the n triangles that He outside this last polygon, n points 
are taken at random, one in each, forming, when joined, another n-gon ; 
find the general average area of this last n-gon, and show therefrom that, 
in the cases of a hexagon and a square, the averages are fj- and -|g- of the 
original figure respectively 41 

8840. (W. S. M'Cay, M.A.)— Prove that (1) the locus of the mean 
centre of the four points, in which a line of given direction meets the 
faces of a tetrahedron, is a plane (diametral plane) ; and (2) if A, B, C, 
D be the areas of the faces of the tetrahedron, aU the diametral planes 
envelope the quartic (Aj?)*+(By)* + (Ca)* + (Dio)* = 40 



Digitized by VjOOQIC 



•OOKTllKT£r. XXI 

8843. (Bey; T. P. KirlnnaTi, F.R.S.) — The angles nabc ... /m of any 



formed: first, nai>aia; and next, if ^«i/i be any three consecutive 
points of it, the distances in the directions /j^i and eidi of ei from the lines 
fe and ed in N are equal. The line nitni meets the edge nm of N in r. 
Prove that, if from s in an produced we draw snj — r»i, «»i is parallel to 
the first portion ««! of L 68 

8847. (W. Mills, B. A.)— Given the focus of a parabola, one tangent, 
and a point on it through which another tangent passes ; prove that the 
locus of the point of intersection of the variable tangent with a diameter 
through the point of contact of the fixed tangent is a circle which touches 
the fixed tangent at the given point on it 105 

8861. (J. J. Walker, P.R.S.)— Show that either segment of a focal 
chord of a conic section is a mean proportional between its excess over 
the semi-latus rectum (or viee vertd) and the whole chord 41 

8864. (Rev. T. R. Terry, M.A.)— Solve (1) the equation 
w«+2= (2ic + 6) u>,+i— (or^ + 4a: + 3) Wg I 
and show (2) that, if «« and v« both satisfy this equation, and if Mi » 2, 
u^ — lO, while 1^1 — 1, i>3 = 2, then 2ux = x{x + d)Vm 44 

8856. (B. Hanumanta Rau, B.A.) — AB is the diameter of a semi- 
circle; AG and BF tangents. CF cuts the semicircle in D. DOt is 
drawn at right angles to CF, meeting AB in G-. Prove that 

AC.BF = AG.GB 112 

8868. (Professor Gochez.) — ^R^soudre les Equations 

(ay + 1) (a; + y) = axy, («V+ 1) (^^ + 1/^ = *'^'- 
Montrer que ce syst^me est quadratique. Appliquer les formules au cas 
particulier suivant: a = J^, J = -»^ 60 

8862. (R. Curtis, 8. J., M.A.)— Find (1) the locus of a point in a 
material lamina, such that if the lamina were moving without rotation in 
its own plane, and that point were suddenly fixed in space, the ensuing 
rotation of the lamina would be given ; (2) where the point should be if 
the rotation produced is a maximnm ; and (3) where the point should be 
in order that the loss of kinetic energy would be given. 67 

8863. (Ch. Hermite, Membre de I'lnstitut.) — Determiner les int4- 
gralesd^finiee T^-^n' f'*'-^— n' 

en supposant que p soit une quantity imaginaire quelconque 53 

8866. (Amiral de Jonquils.) — Soient trois nombres entiers, a, b, c, 
premiers entre eux, deux h, deux, et v6rifiant T^uation o^ + ^^s^. D^- 
montrer que Q'exposant n premier et sup^eur k 3) : (1) a et b ne 
peuvent dtre, simultan^ment, premiers ; (2) si a, suppose inf^rieur k b, 
est premier, ^ *- d + 1 56 

8866. (Professor Catalan.) — ^Dtoontrer les contributions au th^or^e 
de Fbbmat :—« suppose premier, (1) «-l-aR(»); (2) «*- 1 = 3R (n*) ; 
(3) tout diviseur premier, de 0— a, divise^ a— I; (4) a + b et 0— a sont 

C 



Digitized by VjOOQIC 



ZZU OONTBKTS* 

premiers entire enx; (6) 2a— 1 et 2^ + 1 sont premiers entrt enx ; (6) le 
nombre premier, a (s'll existe) est oompris entre 

!7) a et b surpassent n; ^8) le nombre b, qui satisfcdt k Filiation 
4+ 1)*— i* « a*, est compris entre 

a (a/nf*"^ et - 1 + a (a/n)^*"^ ; 
(9) soit b un nombre entier, sup&ienr au nombre entier n, Entre 

ilya, tout an pins, un nombre entier; (10) aucun des nombres a-k-b, 
e^a, e—b, n*est premier; (11) chacun d'euz a la forme N, on la forme 
{I In) N, N ^tant un nombre entier ; (12) soient, s*il est possible, 

a+b^ tf"», e-a = A"», c— * « a'* ; 
alors tf = 3R (n) ; (13) (ic + y)''-^*-^** - fwry (a;+y) P, 

P = Hia:'*-a + H5a^-*y+ ... +Hiy*-», 
les coeflBcients sont donn^ par la formule Hp « (1/n) [0»-i^ ± 1], 
le signe + repondant au cas oii /? est pair, et le polyn6me P est ivisible 
par a;2 +xp + y\ et mSme par («» + ary + y^\ si « = 5K (6) + 1 ; (14) la 
difference des puissances n^*°^ de deux nombres entiers consecutifs, 
a, a+1, ^tant diminu^ de 1, est divisible par na(a+l) (A^ + a+l) ; 
les facteurs a, a + 1, a^ + a + 1 sont premiers entre euz, deux It deux ; en 
outre, le troisi^me 6gale le produit des deux autres, augmente de 1 ; 
(15) si, dans T^quation de Ferhat, le nombre a est premier, on a, par le 
th^or^me 8865, de M. de Jonqui^res, a»»-l - QR [ni(A + l) (A^ + J + l)]; 
et (16) e est compris entre a + ^et^(a + d) 56 

8869. (Professor Steggall, M.A.) — A shot of mass m is fired in vacuo 
from an air gun of length /, with a charge of air that at norma] pres- 
sure p would occupy a volume v ; this air initially occupies a length b of 
the barrel of the gun. Show that the time of passage along the barrel 
and the velocity with which the shot leaves the gun are given by the 

equations T = (/+f 1**1) + (5iH;/m)*, V = (1 -b\l2ll)(fipvlm)\ 

where b is small compared with /, and the ratio of the specific heats of 

air is taken as 1*4 , SO 

8873. (Professor Edmund Bordage.) — Given two relations, 

(2S = a + ft + c), 
dp + ycosg-i-gcos* __ y + gcosg-i-a?cosg __ g + a? cos 3 -i- y cos a ^ « 
cos(S-a) "" cos(S-A) cos(S-g) '^ ' 

deduce therefrom -£- = _y_ « _£_ = ^ 91 

sina Bind sing sinS 

8875. (Professor Nash, M.A.) — Professor Casey's Quest. 7839 may 
be enunciated as follows : — DD', EE', FP' are the intersections of the 
sides of a triangle ABC with the cosine circle, the order of the letters being 
such that E'F, FD, IXE are diameters. The circle round AE'F cuts the 
circles on AB, AC as diameters in the points a, a' ; and b, b', e, if are 
similarly determined upon the circles BFD, CB^E. To show that the 
circles roimd ACa, BA*, CBc pass through « the positive Brocard-point 
of ABO, and the circles ABa', BO^', GAg' through the negative ^rocaxd- 



Digitized by 



Google 



CONTENTS. XXlU 

point. ProTO the following additional propexties : — (1) The circleB ACn, 
ABaf touch at A the rides AB, AC, and intersect agun in a vertex of 
Brocard'B second triangle ; (2) the taogents to these circles at aaf bisect 
AB, AC ; (3) the points B, C, a, of are concyclic, and the circle through 
them touches OB, OC ; (4) AaD and A<i1)' are collinear ; (6) Ca, Ba' in- 
tersect upon the symmedian AP, and Ca', Ba upon the perpendicular AL; 
(6) aE', aT meet BC at the foot of the symmedian AP ; (7) ««', E'P, 
BO, and the radical axis of the circles ABO, AE'F, are concurrent; 
(8) the three points of coucurrence of the three sets of lines in (7) lie upon 
a line parallel to »<u\ and therefore the triangle formed by the lines «ia% 
bb\ ctf is in perspective vdth ABC ; (9) the p<«le of aa' with respect to the 
circle AE'F lies upou the median of ADD'; (10) the pairs of points ae', 
ba\ eV are isogonal, and the infcribed conic whose foci are atf touches 
CA at the foot of the perpendicular from B ; (11) one directrix of this 
conic is the line joining to the intersection of FD' and DE', and the 
other the line joining A to the intersection of EF', FD' 131 

8876. (Professor Hudson, M.A.) — Prove that the normal chord which 
subtends a right angle at the focus of a parabola is divided by the axis 
in the ratio 2 ; 3 108 

8886. (Rev. T. P. Kirkman, M.A., P.R.S.) — Two 28-edra have 
the following triangles, the summits being marked ahe „, npq. Required, 
all the symmetry of both. 

abfff aic, acd^ ade^ aef^ afg, bj'e, bih, bj\ big, ^7, elk^ ekd, dkm, dfm, drf, 
fmq, fgq, ghq, ghi, hpj\ hpq, jlk, jkp, mnq, mnp, mpk, pqn, 

abg, afg, afe^ aed^ abc, acd, bhg, bhi, bet, qji, edj\ c{jk, dkl, dfl, def, fkl, 
fknif jfgtn, ghm, ifq, ihpf ipn, inq, j'kn, jnq, hnfif hmrtf hpn 126 

8887. (W. J. Greenstreet, B.A.)— In the "ambiguous case" of tri- 
angles, ^ven a, b, and A ; r,r' being the radii of the in-circles, p, p' of 
the escribed circles to a ; prove that 

rf^{b + a) ^pp'{b^a), rp(b^a) ^ t'p'e 87 

8888. fS. Tebay, B.A.) — Dj, D,, D, are the shortest distances of 
opposite edges (a, a ; b, b\ c, e) of a tetrahedron, Y the volume, and A 
the area of one of the equal faces ; show that 

V « i (D1D3D3), a2 - ^ (Di2a2+D3«ft« + Da'cS), 

and Di» + D2«+D32-i(a2 + «2 + c2)-Di« + a« = &c 161 

8903. (Professor Oenese, M.A.)— If /(r, 0) be the equation to a conic 
referred to any pole 0, the value of (df)l(dr) at any point P of the plane 
varies as (PQ + PR) / OQ . OR, Q, R being the intersections of OP with the 
conic ; thus, for a point Q of the conic, (df)l{(fr) « QR/OQ . OR; and 
in particular, if O be a focus, {df) / {dr) is constant over the curve. [In 
this case /=(/-<T cos e)2-rs.] 93 

8905. fProfessor Neuberg.) — D6montrer qu'i tout point / de la dis- 
tance des foyers F, F d'une ellipse E correspond une droite D ext^rieure 
an plan de E et parallMe an petit axe, telle qu'il existe le rapport con- 
stant e : a entre les distances d*un point quelconque de E It / et D. 
Lorsque/occupe toutes les poritions sur FF', D se d^place sur un cylindre 
qui a pour base ime ellipse semblable k E 113 

8908. (Enunciation included in Question 8866.) 66 



Digitized by VjOOQIC 



Xnv CONTENTS. 

B912. (Professor Bordage.)-— A square ABOD and a fltraigl^ line A 
in its plane are given, and through the points A, B, 0, D perpendiculars 
AA', BB\ CC, DB' are drawn on A, A and G heing two opposite sum- 
mits; prove that (1) (BB')«+(DD')''-2AA'.BB' is a constant quantity 
for every position of A ; and (2) deduce therefrom an application for the 
envelope of the straight lines, such that the sum of the squares of the dis- 
tances of one of them from itwo given points is constant 109 

8916. (Bev. T. 0. Simmons, M.A.)— A point P being given in the 
plane of a triangle ABC, it is Imown that, with P as focus, five oouics 
can be drawn, four of them circumscribing the triangle, and one inscribed 
in it. Show that for certain positions of P a sixth conic can be drawn, 
also having P for focus, and touching the other five ; and find the locus 
of P when this is possible. fThe Proposer suggests that conies with one 
focus common should be called eofoeal, reserving the term confocal for 
conies having both foci common.] 96 

8920. (B. Enowles, B.A.) — From a point O {x, y) tangents are drawn 
to meet the ellipse a V + *'^ = ^^^ ^ P ^^^ Q» * tangent parallel to the 
chord PQ meets OP, OQ in^? and q respectively ; prove that 

pqlTQ^ (aV+*^')* : aft + (flV + ^«^* 116 

8924. (Captain H. Brocard.) — Former r^quation des paraboles tan- 
gentes aux deux bissectrices de chaque angle du triangle et aux per- 
pendiculaires aux cdt^s de cet angle en leurs milieux. Ces coniques 
admettent, comme on sait, pour directrices les m^dianes et pour /oyers les 
sommets A", B", C" du second triangle de Brocard 70 

8927. (S. Tebay, B.A.) — If A^, A,, A3, A4 be the areas of the 
faces of a tetrahedron, B the radius of the circumscribing sphere, and 
Bj, B,, Bg, B4 the radii of spheres passing through the centre of B and 
the angles of Ai, A3, A3, A4 ; the volume 

^"*^'(l+i:*l^l) "» 

8928. (Bev. T. B. Terry, M.A.) ~ Find the value of the definite 

rdx 
— sini?ic(acos2a: + dsin'rr), whenjE? > 2 157 

8929. (B. Tucker, M.A.)— Find (1) the equation of the circle through 
the images of the centroid of a triangle wil^ respect to the bisectors of 
angles ; and show (2) that the simi of the squares of tangents to it from 
the angles (taken once) is \{c^ + 6^ + c*) 88 

8931. (Emile Vigari6.) — On donne deux points O et A et on con- 
sid^ toutes les paraboles ayant O pour sommet et qui passent par A. 
Trouver g^om^triquement le lieu (1) du point de concours des tangentes 
en A et O ; (2) du point d'intersection de la normale en et de la tan- 
gente en A (ces deux lieux sont tangents en O) ; (3) du point d'inter- 
section de la tangente en O et de la normale en A ; (4) du point de con- 
cours des normales en et A (ces deux lieux sont tangents en A). ... 81 

8933. (B. Hanumanta Bau, M.A.) — If a body describe an ellipse 
about a centre of force in the focus, the time of describing the arc between 
the extremities of the latera recta on the same side of the major-axis is to 
tlie|>erio<}|ctinieassin~V(tf) : *..,......? ,.•:•••••••••?• vv m H^ 



Digitized by 



Google 



CONTENTS. 



XXV 



8946. (W. J. 0. Sharp, M.A.)— Show that 



-a, -a, 
a?, -a, 



—a, — fl 
— a, — « 



—a 
b 



=■ *(« + «)», 



122 



^y d) bf ... .Ty 

ft, A, b, ... ft, 

where the determinant has it+ 1 rows 

8948. (W. J. C. Sharp, M.A.)— Show that 

fi— llsm^eV d$J ^ ' 

-(srn'e;^) (sinecose) 123 



and 



cosnO : 



f»— 1! sin^j 



del 



8977. (Ptofessor Haughton, P.R.S.) — The form of the Terrestrial 
Badiation Function has been proved to be A (0—60)** » 0, where A, 69, 
n are unknown parameters, and 0, a are given by observation. The mean 
monthly observations at Greenwich, ext^ding over thirty-six years, give 

January, A (38-9-eo)" = 21-4 ; February, A (40-4 -60)* = 36-6 ; 

March, A(42-8-eo)» - 66-9. Find e^, A, and n 69 

8978. (Professor Sylvester, F.R.S.) — Show algebraically that, if 
41, ft, e are the three sides of a triangle of reference, and A, B, C, the l^ree 
perpendiculars on a variable line from the angles of that triangle, are 
regarded as its inverse coordinates, then the equation to the two circular 
points at infinity is 

a«(A-B) (A-.C)+ft«(B-A)(B-0)+<?«(C-A) (C-B) - 0.... 167 

8979. (Professor Brunei.) — Soient ABC un triangle, A,BiCi un autre 
triangle d^uit du premier en menant par les sommets A, 6, C des droites 
faisant avec les c5t4s du triangle et dans le mSme sens un angle ^. Du 
triangle AiB^Ci Ton d^duit, de m§me, un triangle A3B2C2 et ainsi de 
suite, toujours avec le mime angle ^. D6montrer que les points 
A, A|, A3 ... sent sur trois groupes de spirales logarithmiques ayant pour 
pdles les points de Brocard. Pour queUes valours de ^ le triangle d6riv^ 
A|B|C] est-il ^gal au triangle propose ? 109 

8980. (Professor Steggall, M.A.) — In a rectangular hyperbola OY 
is drawn at right angles to the tangent at P ; prove that, if YX produced 
cuts SP in R, and Y be joined to X', then XR = YX', where X, X' are 
the feet of the directrices 92 

8981. (Professor Schoute.) — Given two conies; find (1) the locus of 
the vertex of a right angle circumscribed to these curves ; and (2) con- 
sider the particuls^ case of two homo-focal conies. [The problem in its 
general form may be otherwise stated thus : — ^Find the locus of points 
such that one of the tangents from it to a conic (2), together with one of 
the tangents to a second conic (3), form with the two tengents to a third 
eonic (1) a harmonic pencil.] 102 

8985. (Professor Byomakesa Chakrayarti, M.A.) — ^A cylinder, weight 
W, radius r, is placed on a rough horizontal plane ; a uniform plank, 
weight P, len^h 2Q, is inclined at an angle 9 to |hd horizon, and res^ 



Digitized by VjOOQIC 



XXVI CONTENTS. 

with one end on the ground, the other on the cylinder (the pUnk being 
at right angles to the axis of the cylinder) ; if ifr be the angle made with 
the verticaf by the re-aotion of the ground on the cylinder, prove that 

cot4^-oot4- + - .?-Beca 104 

8994. (Charlotte A. Scott, B.Sc.) — A rectangular sheet of stiff paper, 
whose length is to its breadth aa V2 is to 1, lies on a horizontal table 
with its longer sides perpendicular to the edge and projecting over it. 
The comers on the table are then doubled over symmetrically so that the 
creases pass throuffh the middle point of the side joining the comers, and 
make angles of 45 with it. The paper is then on the point of falling 
over ; show that it had originally J| of its length on the table 112 

8996. (Hahendra Nath Bay, M.A., LL.B.) — If x, y, z denote the 
req>ective distances of any point in the plane of a given triangle ABC 
from the angular poiots ; show that the following relation subsists among 
them, («« + y« + 2* + a* + A«+c»)(a*a?« + iV + <j»a») ^2a*x^{ifi + a^ 

+ 2ftV(A34-y«) + 2ca«a(c2+«8)+ay«3 + Wt«*a+c««V+«'^<^. ••• 1^8 

8998. (Rev. T. E. Terry, M.A.)— Solve the equation 

^-.6a;^+(12«2-6)-^4(2a^-3)a:y = 108 

ax* dx^ ax 

9002. (E. M. Langley, M.A.) — If a circle and a Simson-line of one 
of its points be both inverted with regard to that point, the two inverses 
will have the same relation to each and to the given point that the 
originals have 109 

9003. (R. F. 'Davis, M.A.) — If upon each side of a triangle a pair of 
points be taken so that the pairs on any two sides are concyclic, prove 
that all three pairs are concyclic 93, 158 

9008. (S. Roberts, M.A.) — Q-iven three circles Cj, Cj, Cs, determine 
a circle cutting C| orthogonally, bisecting C^y and bisected by Cs ; and 
show that in general there are two such circles which may coincide or be- 
come imaginary 159 

9009. (Rev. T. P. Kirkman, M.A., F.R.S.)— P and Q are a regular 
20-edron and 8-edron. KLMN are 4-edray each on a base that covers a 
face of P or of Q. Required the number of polyedra, of which none is 
either the repetition or the reflected image of another, that can be made 
by laying one or more of KLMN on as many faces of P or of Q, with an 
account of the summits and symmetry of the constructed solids 125 

9010. (J. J. Walker, F.R.S.) — Prove that the area contained by two 
tangents to a central conic and the semi-diameters to points of contact is 
equal to b'^x^+aY-a'^bK [See Quest. 3099, Vol. xiv., pp. 74, 76.]... 92 

9011. (R. Lachlan, B.A.) — Show that the product of the three 
normals drawn from any point on a conic is equal to the product of the 
perpendiculars from the point on the asymptotes and the diameter of 
curvature at the point 106 

9013. (Emile Vigarie.)— Les projections orthogonales de denx points 
inverses Mi, Mj {xf, y^, «') sur les trois cdt^s d'un triangle ABC, sent six 
points d'une m^e circonf^rence dont I'^uation en coordonn^es nonnales 



Digitized by 



Google 



CONTENTS. XXVll 

est: (y£iinA-i-«a;smB + «y8mC) (d;'sinA+^BinB+j{'8inO) 

C BinA 

, yy" (a?' -fg" COB B)(g^ + a^ COB B) ^ gg'(a/-t>y^C08C) (y' + a/coflC) ') 
sin B sin G 5* 

[M. YioABi^ remarks that " on appelle pointi inverses en France ce 
qae M. Casbt appelle uogonal eo^yugate points (Sequel to Euelid^ 1886, 
p. 166)."] 1 93 

9017. (A. Russell, B.A.)— If a?i, iCj, ..., a?„ «- ai, Oj, ..., fln, prove that 

«! (ai-oj) ... (oi-aj «i(«j-«3)-(«j-«i) 

9020. (F. Morley, B.A.>-^ABDO is a parallelogram ; is any point 
on the line bisecting the angle A ; CO, BO meet BD, CD in E, F ; prove 
that BE - OF 104 

9024. (Professor Sylvester, F.R.S.) — For greater distinctness, the 
name of Hyper-eartesianijioi to be confounded with a hyper-eartesie) being 
given to that particular form of the bicircular quartic in which four con- 
oyclic fod become collinear ; prove that, if four points are given in a 
plane, the locus of the curve in space whose distances from any three of 
them are subject to a given homogeneous linear relation is a curve of the 
4th order. (This space curve may be termed a Hyper-cartesic.) 160 

9026. (Professor Catalan)— Soit 

fl(a+l)(fl + 2)...(a + c)±3(3 + l)(* + 2)...(3 + <j)-(a + 3 + tf)^(«, *). 

(Le signe +, si est pair). (1) ^ (a, h) est un polyndme entier, k coeffi- 
cients entiers ; (2) si a, 5 sent remplac^ par des nombres entiers, ^ (a, h) 
devient un nombre entier ; (3) pour ces valours de a, by 

(« + * + <?) ^ (fl, 3) = S» [1 . 2 . 3 ... (c + 1)] ; 
(4) si, en outre, a + 3 + est premier, ^ (a, ^) i- 9)t [l . 2 . 3 ... (0 + 1)]. 101 

9032. (For Enunciation see Question 8876.) 108 

9060. (Maurice D*Ocagne.)— AB et MN 6tant deux diam^tres d'un 
mdme cerole, si ime parallMe quelconque k AB coupe la corde NA en B', 
la corde NB en A', et que les droites MA^ MB' rencontrent le cercle 
respectivement auz points A'' et B", les droites AA", BB'' se coupent au 
pied H de la perpendiculaire abaiss^ du point N sur la droite A'B'... 113 

9063. (Mahendra Nath Bay, M.A., LL.B.)— If Oi, a^ az'"fhn~\ be 
2»~1 positive numbers connected by the relation aia^,.,a2n-\^l\ 
show, by elementary algebra only, that the minimum value of 

(l+fl,)(l+iij)(l + aJ(l + a4)...(l + ««H-i)ifl22«-i 161 



Digitized by VjOOQIC 



APPENDIX I. 
Solationa of Qaestions 8886 and 9009, by Rev. T. P. Kirkman, 
M.A., F.R.8 126 

APPENDIX n. 

MiflceUaneoos i^lutions and QuestionB, by R. F. Davis, M.A 128 

APPENDIX ni. 
Solations of some Old Questions, by W. J. Curran Shaip, M.A. ... 137 

APPENDIX rV. 

Note on tbe nse of Oommon Logarithms in the Numerical Solation 
of Equations of the Higher Order, by Major-General P. O'ConneU 166 



CORRIGENDA. 



S 8' 

p. 99, line 6 from bottom, for ~ read -^. 

p. 161, line 7 from bottom, for ej (...) read ie (...). 
p. 162, last line should be thus : — 

,VD,«D,»D,»x4«(iDiD2D,)». 
p. 163, line 10, readr »...»» a(l—tfcosu). 

„ „ 2Z, for Tread J, 

„ „ 24, readi.^anu^y, 

„ „ 31, read 2e (sin Ui— sin u^) times, 
p. 164, line 3, read (. . . cos m^), (. . . + sin u^) . 

„ „ i, for T read J, 

„ „ ll,n^...(...) )-'^("»-" | ) . 

'l + C0S(tl2-lli) 



Digitized by VjOOQIC 



MATHEMATICS 

PBOM 

THE EDUCATIONAL TIMES. 
WITH ADDITIONAL PAPERS AND SOLUTIONS. 



2810. (By Professor Sylvester, F.R.S.)— Let Si be used as the 
symbol of the sum of i-ary products ; required to prove that, if t is < >», 

(«i-«2)(«i-«a) •..(«!-«») 

« Si + i («i, 02, ... , «n) — Si+i (Oi, 02, ... , a«). 

[For example, let » » 3, t == 2, then the theorem becomes 
. Aa-a)(a-&)(a-y) . (b-'a)(b^0)(b-y) . . {e^a) (e-$) (e^y) 
(a-^)(a_c) {b-a){b-c) ^ {c^a){e-b) 

= abe—afiy, which is obviously true.] 



Solution by W. J. Curran Sharp, M.A. 
Assume (a?— ai)(j?— «2) ... («—«n) — (^p— aj) (a:— oj) ... (a?— On) 

sPi(a;-«2) (a;-ffj) ... (a;-«n) + P2(^-«3)(^-«4) — (a?-an)(ir-«i) 
+ ... + P„ {x-a{) (ar-fla) ... (ic-^n-i) ; 
then, by putting x = «<, it appears that 

-(fl<-ai)(a<-a2) ... (a<-o«) = Pi (»<-«<♦ i) ... («i--«i). 
Hence we have P,=.-^ (^>-ai)(«»-^aV. (^>-^n) 

and equating the coefficients of «— t— 1, 

(-.l)<+i |Si+i (a„ ^2 ••• «»)-S»>i (ai, 02 ... On)} 

= (-l)»2?Pi8.(«i,a2...«,-i,a<^i, a«) ; 

therefore 2Si (a,, a, ... «„) (^i-«i^ (^i-o.) ... K^o. ) 
(ai-Ojj) («i-«a) ••• («2-«J 

= Si+i («!, fla ... a»)-Si+i (oi, os ... a»). 

VOL. XLVII. 



Digitized by VjOOQIC 



22 



8816. 

aurd, 8l6 
triqoet, R 
droiteSS'. 

brocard. 
cAUm da I 
poiuUntoS 



(Bj Captain H. Brocard.) — n, Of d^gnant lea points de Bro* 
nuliea de CiGfy 8' le point correspondant k 8 par droitee sym^- 
le point de Steiner ; d^montrer que ce point R se trouve snr la 
(B est nne des intersections du cercle ABO avec le diam^tre per- 
'e k Paxe d'homolo^e du triangle ABC et du premier triangle de 
Ce point est aussi Tintersection commune des parallMes aux 
remier triangle de Brocard, men^ par lea sommets corres- 
u triangle ABC.) 



BoUUiom by (1) D. Biddlb ; (2) Professor Db Lonoohamps. 




1. In the triangle ABC, let n, ti' be the Brocard-points ; AECB, the 
circumscribed circle, with centre H ; Hnn', the Brocard-circle ; AiBiGi, 
the first triangle of Brocard ; GGj, the axis of homology' of the triangles 
ABC and AiB^Ci ; EN, the diameter of the circumscribed circle perpen- 
dicular to GG] ; 8, the mid-point of AA', and 8' its corresponding point. 
It is required to prove that 8, 8', E are in one straight line. This will be 
thecaseif (Ra-8i) : (8l-8a)=(R6-8i) : (Si-86), in which E«, Eft... are 
the distances of the points from BC, AC respectively. Let A — area of ABC ; 
a3+i« + «j2-m2; aJ^ + aV+i^cS - n* ; a< + A* + «* « i7<. Now 8., St, 8^, 
S^ are well known, and are as follows : — 



- 2A(g^ + ^«)(ag4-ga) 



s* — • 



3(«2+c«), 



8' 



Si' 






Digitized by VjOOQIC 



23 

In order to find B^ and E^, we have H, the circum-oentre of ABO, with 
H^ and H^ known, HE the radios of the circam-circle, and D, the centre 
of homology of ABC and AjBiOi, with D^ and D^, known. 

HB=^; H =-4^, «K-a2); H. = --?—:- M*""- 2*2); 

Now B. = H,.(D,.HJHB/HD= ?M^^^^^zi^ 
B,-H,.(D,.HJHB,HD=-?-I^^=^^ 
therefore (Ra-^'J/CS^-Sa) 

and (R6-Sa/(Si-S^ 

^ 2w< {33(j?<~n<) (gg + ^g) (^ + c3)~ [gg^^c^-^^ (m»-2y»)] (3n<+i?^) *} 

On the first reduction, (R^-S;) / (S^- S^ : (B,-S;)/ (SJ- S^) 
_ Pc8-g2(w8-2g8)](3w<+p^)--(jp<-n^) (g3+^ (g2+g8) . 

2«4 (a2 + ^i) («2 + c5) « (3„4 +jp4) «2 (62 + ^2) 
2f»4 (a2 + *2) (^2 + ^2). (3n4 + p*) ft3 (a2 + <j2) 

^ 2»iM2g^~g2y~g«c24-2*V)-2p<(ggcg + a^yO . 

f»4(2a4-a2^2_«2^ + 2*?c3).jp4(flf2^2 + a2^2) ' 

2n4 (2^~ gg^'- ^gg + 2g2g2) - 2jp< (y^g^ -f g^^^ ) ^0.2 

«4(2*4«a2^S«^c2+'2g2c2)-i?4(*^g2 + fl52^) *" ' * 

Wherefore, B, S', and S are in one straight line. Moreover, since 
2»^ / (,1^— M*} was the common factor left out on the first reduction, 
BS':S'S = 4«*:i;4«„4. 

2. Othenoiae : — Le point S, milieu de la droite qui joint les points de 
Brocabd, est repr^ente en coordonnees barycentriques par 

a^(b^ + c^, ^(g2 + a2), <j8(g2 + *2). 
Le point S', le point inverse, suivant la d^omination du Colonel Mathibu, 
a done pour coordonnees (b^+(^-^, (c* + g^) - ^ (g^ + ^2) -1 . 
Enfin, le point de Steiner B a pour coordonnees, comme I'on saity 

(i2-<j2)-l, (g2-g2)-l, (g2-^2)-l. 

On conclut de Ik que 1* Equation de BS' est , 

o fi y \ 

(i2_^-i^ {di^a^y\ (g«-^-> = 0, 

(62 + C2)-1, (c2 + «2) -1, (g2 + J2) -1 | 

ou, apr^ developpement, 2a (g^— ^c^j (^-^4) = (1), 



Digitized by VjOOQIC 



24 

D'apr^ cda, on Toit que la droite A qui correspond k cette ^nation 
pawe (1) par le point S, parce qae Ton a 

(2) par le point J dont lee coordonn^ sont : 

Cette demi^ propri^t^ a 6U signal^ d^j^ par H. Brocard an Gongr^ de 
Bouen. [Voyez VAnnuairt de CAnociatUm Franfoise pour Vanancement de» 
SeieneeSf 1883, p. 104.] 

Un cinqui^me point, sitn^ snr A, apparait imm^diatement d'apr^ 
r Equation (i) ; nous voulons parler d*nn point U dont lee coordonneea 
■ont (a*-W;-)-S (A<-«»a«)-i, (<?*-a«6«)-i. 

Ce point IT se rencontre fr^nemment dans la gi6om4trie triangnlaire e^ 
il est certainement remarqu6 de tons cenx qui la connaissent un pen. 

Iiorsqu^on cherche T^uation de la droite G, axe d'homologie du triangle 
dereference ABC et du triangle de Bbocakd AjB|C|, on trouve 

a/(a*-AV)+/3/(A*-c«aS) + 'y/(«*-a«*2)= 0. 

Cette droite est harmoniquemmt tusoeUe au point XJo dont les coordonneea 

■ont («*-*V), (d*-c»a2), (c«-a5*2); 

TJo eet done, dans le sens que nous avons donn^ autrefois 2k ce mot, sens 
qui est aujourd*hui gen^ralement adopte, croyons-nous, le rieiproqtie de TJ. 

[Le terme harmoniquement assoeii a ete propose par nous pour repre- 
•enter une droite fiy associee k un point donne M, dans les conditions 
suivantes : — La droite AM rencontre BC en un certain point A' ; soit A'^ 
le point qui forme avec BA'C une ponctuelle harmonique ; les trois points 
A'', B'', C sont Bitu6s sur une droite fi que nous appelons V harmonique 
mtaooiie de MJ] 

L' Equation (1) donne ainsi 5 points en ligne droite: le milieu de la 
droite qui joint les points de Broca&d, le point inverse de celui-ci, le 
point de Steinbr, le point de concours de la droite <^ui joint le centre du 
cercle circonscrit k 1 orthocentre avec la droite qui joint le reciproque 
du point de Lemoine k son inverse (le point J), et enfin le point Uq, ci- 
desBus defini. 



8801. (By Rev. T. P. Kirkman, P.R.S. Suggested by Quest. 
8326.) — Upon the most sacred of the undiscovered Egyptian pyra- 
mids, whose vertex F is at imequal distances from the angles of its scalene 
base ABC, is cut in the face PAC an ascending flight of steps from A to 
by in PC ; form h another flight mounts to in PB ; and a third leads up 
from c to a, at a given altitude h in PA. These three flights are equi- 
gradient. A second path from A mounts to A' in PC, and thence pro- 
ceeding to c'in PB, ascends from c' to the above point a in PA. These 
three flights also are equigradient. By khca Pharoah and his Grandees 
of Church and State were wont to moimt to a platform at a for solemn 
religious rites ; the admitted vulgar used only the path Kh'c'a. Required, 
a plane projection of the pyramid and its six flights of stairs, and proof 
from that projection that tiie first three are equigradient, and the second 
three equigradient, up to the given altitude A. 



Digitized by 



Google 




25 

Solution by the Profobeb. 

Let the distance *A of every point 
h of PC in the projection from A he 
greater than *B, and also than bG, 
"We can with the ruler find through 
any point b in PC the equigradient 
path Ahca^ so that if Va — x and 
rb=^ y^ X is given with y, one 
value of X for every value of y, and 
that X and y vanish together ; for, if 
we take P for our A, the lines bD and 
bO in the projection coincide, and P 
is also our point c, t.^., x = Q follows 
from y = 0. And evidently there 
cannot he two equigradient descend- 
ing paths from a to A, ahout the 
pyramid, for no two lines ac and etc' 
across the triangle APB can be equi- 
gradient with the same Ab, That is, for every x there is one y, and one 
only. To find x when y is given is very easy. To find y when x is given 
is Mr. Biddlb' 8 problem in Quest. 8325, and to work out his solution of it 
is not easy. We must content ourselves with the method of interpolation. 
We can write y = ^a; = Ar + Bx^+ ... + Na;". 

Next, taking any n points ^i, dj ... ^n on PC in our projection, we get with 
the ruler the corresponding Oj, a^-.^an on PA, and write (P^i = yi, 
Pill = iTi), yi = ipXij yg = ipX2 ..., yn =* ^Xn ; from which n equations we 
obtain AB ... N in <px. Then, if Pa = X be our given distance from P, 
Y = <^X is our required distance Pi, and when the point b is given, the 
path Ahca is easily found by the ruler, and this solves our problem with 
accuracy increasing without limit with n, the order of the parabola so 
found in the face CPA, if PA and PC are our axes of x and y. This 
parabola passes through (0, 0) and {x = PA, y = PC). 

The second path ac'b'A. cannot be all through a descending path ; but 
nothing prevents its being equigradient, if c'b' is an ascent, while a& and 
b'K are downwards. Let b' in our projection be any point of PC. In CB 
produced, at G make b^Qc « A'A, cutting PB in e'. In AB produced at 
F make c'¥ = <j'G, and produce it to a' in AP. This a' may or not be the 
a before found ; although in the figure, by a few trials, a and a* are made 
to coincide. It is evident that for every b' in PC we can find an a' 
in PA. 

The lines on the pyramid whose projections are iV and b'K are the 
sides of an isosceles tnangle drawn to the base in the planes CPB and 
OPA from b' in both, and are therefore equigradient. For a like reason 
e^a' and </b' in the planes BPA and CPB are equigradient, and the up and 
down path Kb' (/a' is equigradient to the altitude of a'. Putting Pa'= u 
and P^' = Vf we have the right, as above shown, to write 

r = l|^ = A'M + B'tt2+...+N'M»». 

Taking any n points rfj, d^.-.d^'in PC. we get easily the corresponding 
points aiya2..,an in PA; and by n equations {Fd^ — v-^y P^i = Mi), 
t?i = t|rt<i, Vg = 4^, &c., we obtain the coefficients of if/ij, so that, if our 
given distance Pa = X, we get V = if/X, our required distance 
Prf = Pd' :a V on PC, which gives by the ruler the requisite up and 



Digitized by 



Google 



26 

down path A&Va to the given altitude of a in PA. The two flights he 
and bV most have a common step at some point p ; on which, before or 
after the descent of the vulgar, a fitting carpet would be laid for the foot 
of ascending majesty. 

If the descent of the « vulgar " took place on the third fisice instead of 
on the second, the two paths would not cross each other, and the gradient 
of the ** conmion " steps might be lessened. 

From the solution of Quest. 8843, it is evident that, with the concession 
first above asked, the solution of this problem of two equigradient paths 
about the pyramid from A to a in FA, is equally in our power, whatever 
be the polvgonal base of it, and whether the vertex P is vertically over a 
point inside or outside the polygon. 



8823. (By Professor Cropton, P.R.S.) — ^Two discs of any form are 
moveable in a plane round two fixed points A, B, respectively. Show that, 
when they are in such positions that the length of an endless band en- 
veloping both is a maximum or a minimum, the portions of the band 
which form the common tangents will meet on AB if produced, and are 
equally inclined to AB. 

Solutions by (1) Professor Stbooall, M.A. ; (2) Professor Sibcom, M.A. 

1. Let the discs be rotated through infinitesimal angles dd, d<f> about 
A, B ; let Pu p^ ; qi, q^ be the perpendiculars from A, B on the straight 
parts of the string. Then the amount of slack is easily seen by a diagram 
to be (Pi—Pj) d$ + (qi—q^ dip. This must vanish for a minimum length, 
and therefore Pi = pg, qi = jj, since de, dip axe independent, whence the 
result immediately follows. 

2. Otherwise : —Let the discs be smooth, and the band elastic. In the 
position of equilibrium the length of the band will be a maximum or a 
minimum. But then each disc is acted on by equal forces at its rim ; the 
common direction of the resultants will therefore pass through A, B, and 
be equally inclined to the directions of the forces. Hence the theorem. 



5643. (By W. S . B. Woolhouse, F.R. A. S.)— Any two triangles being 
given, the first may always be orthogonally projected into a triangle 
similar to the second ; determine the magnitude of the projected triangle 
geometrically by an easy construction with the ruler and compasses. 



Solutions by the Profosbs. 

1 . Let a triangle ABC, having the sides a, b, c, be orthogonally projected 
vertically into the triangle A"B"0", having the sides a", 0", c", in a hori- 



Digitized by 



Google 



27 

Bontal plane. Then, considering the neutralizing result of changes of alti- 
tude in passing first from B to C, then from C to A, and thence from A to 
the starting point B, there arises the following relation, 

^/(a2-«t"2)+ -v/(i2_y/2)+ ^(CS-^'S) =. (1). 

This relation can suhsist only when the three terms of the expression are 
either all real or all imaginary. In the latter case the imaginary char- 
acter is removed hy writing it thus, 

A/(a"2-aa^+ A/(y'2-^+ ^((j"s-ct) = (2). 

But the form (1) implies that ABO is projected into A"B"C" ; and (2) im- 
plies that ABC is projected /row A"B"C' 

Hence, when (1) and (2) are cleared of radicals, the involved algehraical 
relation amongst the sides a, b, c, and a", l/\ c", will he symmetrical and 
identically the same whether the triangle ABO be projected into or pro- 
jected from the triangle A"B"C". 

According to the problem, A"B"0" is recnured q 

to be similar to another given triangle. Upon ^^J^ 

AB construct a triangle ABC similar to this 
given triangle ; and let the sides of the triangle 
80 constructed be a', b\ e. The sought triangle > 

A"B"C" being similar to this, we can assume /^ 

that a"-A;«', b"^kb% e'^kc, \ 

in which the common factor k remains to be 
determined. The relation (1) now becomes 

a/(«2-A;2^'2)+ ^/(^^A;2y2)+^(tfa-.A;8(j2) «0 (3). 

If A, A' denote the respective areas of the two triangles ABO, ABO' ; 
then 16A2 =2a2i2 + 2«»V + 2*2<j2-a<-M-<j* ") ... 

16A'2 = 2a'2y'' + 2«'V + 2i'V-a'*-y*-tf*3 ^ ^* 

To free (3) from surds, conceive the three terms to be the sides of a sup- 
posed triangle ; then, according to the stated condition, the area of such 
triangle is obviously zero. Hence, substituting, in (4), 

= 2 (a»-.Aj2a'2)(*»-A;2d'2) + 2 («2- kH"^){i?--k^(^ + 2 {)f^-m'^){,^'kH^) 

-(«2-;fc2^'2)2-.(^-A;2*'2)2- {^^^k^^\ 

which, arranged according to powers of A;, is, by (4), 

-16A'2;fc4«2{(*2 + <j3-a2)a'2+(a3+c2-i2)^'2+(«S + ^.^c2}^2+16A2, 

or i^. (^ + ^~a2)^/2^(^s^,2^^2)^/2^(a2^^2,^)e^ ^^^ -^!,-0-(f^). 

But, joining 00' and denoting this line by 7, then 

2cV ^ 2c2 {*2 + ^'2_2W'cos (A=fA')}, 
according as 0, 0' are on the same or on opposite sides of the base AB. 
That is, 2^272 « 2tf2 (^2 ^ y^ _ 4^2^^' (cos A cos A' ± sin A sin A') 

= 2c2 (i2 + i'2)_(i2 + c2-a2) (i'2+^_a'2) :p 16aA' 

. (^2+c2_rt2)«/2+(a2 + c2-^)i'2+(flp2 + i2_^g2:p 16^A'. 

Denoting OE, 00', the two values of 7, by 71, 72 respectively, we have 
therefore 62(712 + 722) « (^ + c2-a2)a'2+ (a2+(,2_^2) y2 + («2 + i2_c2)c2, 

c^M-yi) - 16AA'. 



Digitized by VjOOQIC 



28 



Hence (6) may be stated thus, 
the solution of which is 



8A'3 



266A'^ 



.0, 



therefore 



**-'^^-^'=^<-^=i^^(-=^-^'= 

* = 4l. (---«) -'-^' 




and the perpendicular of the sought triangle iap" = kp' = ^ (Ti^Ts)- 
We have therefore the following 
Construction. — Let ABC be the 

first of the given triangles ; and on the 

base AB describe ABC' similar to the 

other given triangle. Then the pro- 
jection of ABC is required to be similar 

to ABC, and is constricted as follows : 
From G* demit the perpendicular 

CD', and in continuation of it make 

D'E equal to CD'. Draw a straight 

line through the vertices C and C, 

and on it take CF equal to CE ; then on CE take CH equal to CF ; and 

lastly bisect CH by the perpendicular line ab, thereby cutting off from 

ABC the required projected triangle. 

It may be observed that C and £ need not be joined, and also that the 

problem admits of three essentially different projections, for BCA, CAB 

may each in like manner be projected into tnangles respectively similar 

to ABC. 

2. OtherwisAy Oeometrieally : — Take the 
plane of the paper for the plane of pro- 
jection. Let ab be the projection of 
AB, the triangle being turned round the 
perpendicular CD as an axis so as to 
bring the point A vertically above a and 
B vertically below b. Then, denoting by 
c the angle of inclination of the side AB 
with the plane of projection, we shall 
have Da = DA cos e, D* = DB cos €, 
ah = AB cos c. 
To completely generalize the arrangement, now conceive the triangle 
ABC in its new position to turn about the side AB as an axis. The 
vertex C will describe a circle having its plane inclined to the plane of 
projection at an angle equal to the complement of €. This circle will 
therefore project into an ellipse the semi -diameters of which are a — CD, 
j3 = CD sin € ; and the excentricity is hence e — cos c. 

The periphery of the ellipse is the locus of the projection of C. Let e 
be the actual projection of C, and abc will then be the resulting projection 
of the triangle ABC from the position it has finally acquired. As this 
projection is required to be similar to the triangle ABC, the projected 
vertex c must fall on the straight line DC, making Dc = DC cos € =« . DC. 
Let/,/ be the foci of the ellipse ; draw C'H perpendicular to AB, making 




Digitized by VjOOQ IC 



29 



D'H = CD' ; ed perpendicular to CE ; and other lines joining various 
points as represented in the diagram. Then D/ = « . DC, D/*= e . DE ; 
therefore cf, cf are respectively parallel to C'C, CE. 

By Conies, De?(- x) = ^£^ ^ \ (CE-C'C) - \ (CE'-CCO, 

which is the perpendicular of the sought triangle abe, and determines its 
magnitude by a simple and easy construction. 
3. From the preceding article we have 



Drf 
'' CD'' 



cosE = 



a be 



Bd . ab Bd 



ABC CD . AB CD 



= «^ cos € = 



Drf« 



CD . CD'' 




which determine the respective inclinations of the side AB and of the 
plane of the tiiangle ABC with the plane of projection. 

4. Another Geometrical Solution 
may be obtained thus: — Let ABC 
be the triangle to be projected 
from, and or the base A B let ABC 
be constructed similar to the tii- 
an^le into which ABC is to be pro> 
jected. The plane of the paper 
and of the triangle ABC being 
taken as the plane of projection, 
the position of the triangle in space 
may be assumed to be suitably 
located so as to cause the projection 
of AB to fall on the line AB, and 
such that the perpendicular of the 
projected triangle shall fall on C'jy, 

In effect, as regards the projected triangle, the point D is removed to D', 
the segments DA, DB are proportionally shortened by projection, and 
the perpendicular is the same as CD' when diminished in a like proportion. 
Now, if we retain the various positions indicated and in either case con- 
ceive AB to be an indefinite line, and assume in it any two points A' and 
B', the distances DA', DB' will obviously be reduced by projection in 
exactly the same ratio. It hence follows that, wherever the points A', B' 
are taken in AB, the triangle A'B'C will be projected into a triangle 
similar to A'B'C. Taking advantage of this circumstance, such points 
may be conveniently chosen so as to make the angles at C and C both right 
angles ; thus, join CC, bisect it with the perpendicular MO meeting AB 
in O, and with O as a centre describe the circle CC'A'B'. 

A consideration of what has been advanced establishes the general pro- 
position that any projection of the figure which makes A'B'C similar to 
A'B'C will, at the same time, make the triangle ABC similar to ABC as 
required. As the angle C of the triangle A'B'C after projection must 
remain a right angle, the most simple and direct mode of projection is 
arrived at by drawing B'A" meeting A'C, and making the angle A"B'G 
equal to A'B'C. For, if A'B'C merely turn round B'C as a horizontal 
axis until A" becomes the projection of A', then the projection of A'B'O 
will be A"B'C, which is obviously similar to A'B'C. 

Produce the perpendiculars CD, CD' to meet the circle again in E, E', 
and join B'E' ; also join CE' intersecting A"B' in D" and bisect it in 
m. The angle E'CB' is equal to E'C'B' or the complement of A'B'C or 
the complement A"B'C. Therefore CE' is perpendicular to A"B' and 

VOL. XLVII. D 



Digitized by VjOOQIC 



30 

CD" 18 fhe peipendiciilar of the projected triaagle. By the georoetiy 
ofth diagnun, 

C«i-iCE', mir-icxr; 
therefore CD"- i (CE'-CCr), EOX'^ i{CE'+C(y). 

These are wholly independent of the position of the points A, B in the 
line AB. The former is the perpendicular of the projected triaogle ; the 
latter is the perpendicnlar of the triangle similar to ABC from which 
ABO may he projected. 

Note$. — If E denote the inclination of the plane of the triangle with 
the horizontal plane, then 

cosE A"c ^ ciy^ ^ CE'-CCy 

"a'C ED" CE' + CC 

The line CA' bisects the angle CCE' ; the line CB' is perpendicular to it, 
and therefore bisects the exterior angle G of the triangle CCE'. It has 
been shown that the latter of these lines determines the direction of the 
intersection of the projecting plane with the horizontal plane of pro- 
jection. 

[An attempt at a solution, the only one heretofore leceiTed, is g^ven on 
p. 24 of Vol. XXX.] 



8793. (By Professor Neubbbo.)— Soient AB, CD deux droites, qui 
Be coupent en 0. Si Ton fait toumer AB autour de 0, le point double 
de deux figures semblables construites sur AB et CD decrit une circon<- 
f^nce de cerele. 

Solution hy Professors Schoutb, Beyens, and others. 

For any position of AB the centre of similitude of the two similar 
fig^es on AB and Gi) is evidently the second point of intersection P of 
the two circles A OB, COD. Therefore transformation of the problem by 
reciprocal radios with the centre O brings it in the following form : — 
Two lines A'B', CD' intersect at O ; to find the locus of the point P' 
common to A'C' and B'D', when A'B' rotates about 0. Considering 
A'C'P' as a transversal of the triangle OBO)', we find 

(A'B' / A'O) . (CO / C'DO . (Fiy / P'BO = 1, 
and as A'B' / A'O and CO / CD' are constant, the same may be said 
of P'D'/ P'B'. Thus the locus of P' is a circle, for which D' is a centre 
of similitude with respect to the circle described by B', and the locus of P 
is also a cirele. 



8639. (By Professor Wolstenholme, M. A., Sc.D.) — The distances of 
a point O (within a given triangle ABC) from the angular points are 
a?, y, t respectively; prove that the volume of a tetrahedron in which 
a, bf e are the lengths of three conterminous edges, and Xy y, z the lengths 
of the edges respectively opposite, is ti {(<**— ^) (*'— y') (<^— «^}*- 



Digitized by 



Google 



31 

Solution by Professor Nbubb&o ; Professor Gbnbsb, M. A. ; and others, 
Soit y le volume du t^tra^dre en question ; on salt que 

Soit y le volume d'un t^tra^dre dont x, y, z sont les ardtes d*un angle 
Bolide, et a, b, e les ardtes respectivement oppos^ ; on aura 

144V'3 - 2a?«8(A2 + y' + tf2+aS«a^--a:2)-a2^JVj3--aV82-W5^2_^>p3yi, 
D*oii 144 {V»- V*) = (««-«») (y»-y2) (c2-,s). 

Dans la question propose, on a V » 0. 



8713. (By Professor Steooall, M.A.) — Show that the solution in 
rational quantities of the equation j^^+y^' +2^ — u', is ^ » ^ (a^ + d^— ^), 
y » 2kae, s -> 2kbc, u ^ k (a^ + b^ + d^). 



Solution by W. J. Babton, M.A. ; Sa&ah Marks ; and others. 
Let y/z =s afb ; substituting for y, we get (a^+i') «2 _ ^2(,,i_j,jj^ 
This equation is satisfied by u—x — k<P, u + x '^ k{a^ + b^ ; which g^vo 
u =» iAr(a2 + ^ + cS), x = iA;{a2+^-(j«), z = kbe, y - W. 



8804. (By S. Tbbay, B.A.) — If (a, b, e) be conterminous edges of a 
tetrahedron, {x, y, z) the respective opposites, (A, X ; B, Y ; C, Z) cor- 
responding dihechtd angles, and Y the volume ; show that (1) 

_h 



sin A sin X sin B sin Y sin C sin Z ' 
also (2), if the areas of the four faces are equal, then 

a-af, d-y, <?-«, A = X, B-Y, C = Z. 

V = A {2(ft2+c«-a^ (c3 + a2-.^ («2 + J8-.«jS)}*, 

(J— <j)sinA+(o— a)sinB + (a— i)sinO «■ 0. 



Solution by Professor Wolstbnholme, M.A., Sc.D. 
(1) If ai, jSj, 7i be the plane angles at the point where a, b, c meet, X 
the dihedral angle opposite «, cosX = cos a,- cos ^, cos 71 

Bmi8iSm7i 
, a* X » (^ ~ CQ"^ «i •" cos^ i^i — cos^i + 2 cos ai cos jSi cos 7t)* 

sin jSi sin 71 
but volume (V) = J abe (1 - cos^ ai — cos^ fi^ - cos^ 71 + 2 cos oi cos jSj cos 71)* ; 



Digitized by VjOOQIC 



32 

also, if E), B,, B3, R4 be the radii of the circles circumscribing the tri- 
angular faces, sin /3i - ^, sin 71 = ^, &c., 

whence sinX - ?iYKs?a • and -^ ^*"^^ ; 

and, similarly, -^ = ^^""^^ - ; 



or 



hy __ cz _ (abcxyzy 



sin A sin X sin B sin Y sin C sin Z 676V2R1R2RJR4 
If Ai, A3, A^ A4 denote the areas of the faces, 

T» ryz T» xbc t» ayo -p abz . 
^=4T.' ^-4^' ^-4A,' ^^'TI,' 

by et _4 Aj^AgA^^ 



sin A sin X sin B sin T sinCsinZ 9 V^ 

(2) In an equifacial tetrahedron, a ^ Xj b ^ y, c==g; A»X, B — Y, 
C n Z ; and oi + i8| + 71 = ir, so that the volume 

« J abe (1 — cos' ai - cos' jSi—cos' 71 + 2 cos ai cos fii cos 71)* 

B ^ a^0 (cos ai cos jSi cos 71)^ 

The equations of (1) become -r^r = -^Ku " -^"Tri * 4 ^^ 
^ ' sin' A sin'B sin'C 9 V^ 

* 

where A is the area of any face ; whence 

a h ^ _c_ ^ 2_A^ 

sinA ^sinB "" sinC 3 V* 
and (*—<') sin A + {e—a) sin B + (a— A) sin C — 0. 

In such a tetrahedron, the radius of the circumscribed sphere is 
4[2(a' + *' + c')]*, a result I sent to the Educational Times long ago. I 
cannot discover any single length connected with the tetrahedron whose 
square is equal to o^ / sin A sin X, and I do not know that we have any 
right to expect it. I have much admiration of this question. 

The equations 

( ax ^ by _^ ez _ _4^ A|AgA3A4 \ 

sin A sin X sinBsinY sinCsinZ 9 V / 

are strictly analogous to the equations .^ , = -r— :;r^ = -r^ ** ^, 
•^ * ^ sinA sinB sinO 4S 

and, writing the numerical coefficients (|)', (|)', it would suggest that the 

general form in geometry of n dimensions is {(»— 1) /»}'. This I leave 

to those who are experienced in such high matters. 



8696. (By E. ViGARife.) — ^Dans un cercle donn6, par un point P pris 
BUT la drconference on m^ne trois cordes PA, PB, PC ; sur chacune 



Digitized by VjOOQIC 



33 

d'elles comme diam^tre on decrit une circonf6rence ; d^montrer que les 
trois points de rencontre sont en ligne droit. 



Solution by H. F. Davis, M.A. ; H. Brakspeab ; and others. 

For, if a, b, c be the feet of the perpendiculars from P upon the sides of 
the triangle ABC, the circle described on FA as diameter passes through 
b, Q ; while a, b, e are collinear. 



8647. (By R. W. D. Christie, M.A.)— Prove that 2 + S = 2«', if 
«= 13+23 + ... +n3, S» P + 26 + ...W*, 2= l7 + 27+...f|7. 



Solution by Rev. D. Thomas, M.A. ; Professor Beyens ; and others. 
Assuming the relation to be true for n, it will be true f orn + 1 if 
5 + S + («+ 1)'^+(« + 1)7 = 2 [«3 + 2« (»+ 1)3+ («+ 1)«], 

or if 4« = (»+ 1)4+ (« + 1)2-2 (« + 1)8 = [n (n + l)p, 

and this is the case, therefore, &c. 



8726. (By S. Roberts, M.A. Analogous to Quest. 3068.)— Show that^ 
if some four of the roots of a quintic form an harmonic system, then 

J»-27.32.JK+ 2»8.38.L = 0, 

where J, K, L are the three fundamental invariants of the orders 4, 
8, 12 [see Salmon's Migher Alyebra, 3rd ed., p. 211]. 



Solution by the Proposer. 

In the Proceedings of the London Mathematical Society, Vol. xiv., Prof. 
M. J. M. Hill gave the equation determining the anharmonic ratios of 
the roots of a quintic. This is of the order 24 in the coefficients, and, sub- 
stituting therein any value for the variable, we get the condition that 
some four of the roots of the quintic may form an anharmonic ratio having 
that value. 

But in our particular case the condition is of the reduced order 12, and 
is therefore of the form J« + A;JK + /L «= 0. Taking the form 

z {ax* + lOftr^ + be) 

(which we can do, since it only implies the evanescence of Hermitb's 
Skew Invariant of the order 18), we have for the T of the quartic factor 



Digiti 



ized by Google 



34 



e (M~{^t And, sabstitatiiig {«* for «# in 
J - 16e(M« + 3«*), 

giving (9-512)> -i- ib 2432 -i- 2/ - 0. Bat abo, if a » 0, the roots of 

form an harmonic system, so that 4d» +* 18 . 48+4/ = 0. The required 
invariant condition is therefore J» - i (48)« JK + 2 . 48» L = 0. 



^f"". 



8708. (By Mauricb D'Ocaokb.)— 
Si, dans le quadrilat^re ABCD, les angles 
opix)8e8 B et D sent droits, la droite qui 
joint les pieds des perpendicolaires 
abaisseesdn sommet sur les bissectrices 
int^rieore et exterieure de I'angle A 
passe par le milieu de la diagonale BD. 



SoUUion hy R. F. Davis, M.A. ; 
G. G. Mon&iCB, M.A. ; and others. 

Bisect AC, BD in O and E respectively ; 
then O is the circnmcentre of the quadri- 
lateral. If H, K be the extremities of 
the diameter OE of this circle, AH and 
AK are the bisectors of the angle A, and 
the angles CHA, GKA are right angles. 




8769. (By J. Brill, M.A.) — An ellipse is inscribed in the triangle 
PQE so as to touch the sides QR, RP, PQ at P', Q', R' respectively ; 
prove that, if G be the centre of the ellipse, 

AQCR : aQ'CR' - aRCP : aR'CP' - aPCQ : aP'CQ'. 



Solution hy Isabel Maddison ; Belle Easton ; and others. 

Let x^/a^-i-y* I i^ = I be the equation of the ellipse; (ar^y^), {x^y^, 
and (x^ yj the coordinates of P', Q', R'. Then the coordinates of P are 
«^ (^3-^2) / Xi» ^ (^2-^3) / Xi> where Xi == x^^—x^^, Xg = x^y-x^y^ 
X3 « Xii/2—^\i <^<^ ^6 coordinates of Q and R are found by substituting 
in cyclic order. After reducing, we find 

AQCR : aQ'CR' = (Xi + X2 + X3) / X1X2X3, which is symmetrical ; 
hence AQCR : AQ'CR' = aRCP : aR'CF = aPCQ : aFCQ'. 



t 



Digitized by VjOOQ IC 



36 



8749. (By F. PuBSBB, M. A.)— Find (1) the cubic locus of the centre 
of a conic passing through lie three vertices A, B, C of a tri- 
angle, and such that the three normals at these vertices are concurrent ; 
and prove that (2) this cubic passes through the in-eentre I, the three ex- 
centres la, Ibi Ic, the circum-centre O, the orthocentre H, the centroid Or, 
and the symmedian G\ and cuts the three sides normallv at their middle 
points L, M, N ; also (3) tangents at (I, !«, 1*, Ic), (A, B, C, G), (L, M, 
N", G') are respectively concurrent in G, G', ; and (4) the lines joining 
L, M, N meet the curve again in their intersections with the respective 
perpendiculars from the vertices. 



Solution by W. J. Greenstebet, B.A. ; Professor Beyens ; and others. 

(1) Let the chord joining the points a, /3 make an angle with diameter 

which bisects it, then we find cotfl — ——r (sina + /3), therefore normals 

*2ao 

are concurrent if cot 0i + cot $2 + cot ^3 — 0, where sides of A are at angles $1, 

62, 63 to diameters bisecting them. Now let a, /3, 7 be perpendiculars 

from a point on the sides of the triangle, then 

^j _ fl8inB-yrinC + arin(B-0) ^ . g^ ^ ^^ 

2o Sin B Sin C 

/BsinB— 78inC4-asin(B— C) ^ a 

• • r : ;;r — z — ^z — + ^ v, 

2a Bin B sin 

*" ri^ ('*'->') +S^('^-"^ + -<*' 

Bin A sin x> 

the cubic locus required. 

(2) This obviously passes through the centroid, the ex-centres, the in- 
and circum-centres, and symmedian point. Put a=fi — y - 0, we find it 
cuts the sides at the same points as the lines a sin A = /3 sin B * 7 sin C, 
viz., the mid-points. [The other parts of Quest, not solved.] 



8643. (By Professor Genese, M.A.) — Four complanar forces 
P], P2, &c. are in equilibrium. Their lines of action (omitting one at a 
time) determine four triangles whose areas are Ai, A3, &c., and circum- 

Ai . A^^ 



radii Bi, R2, &o. Prove that Pi : Pj, &c. — 



&c. 



Solution by Aspahaous ; D. Edwardbs ; and others. 

Let the forces P], P2, P3 act along the sides 
BC, CA, AB of the triangle ABO, and P4 
along the straight line A'B'C^ The con- 
ditions of equilibrium will be satisfied if the 
moments of all the forces about A, B, C 
vanish. Taking moments about A, we get 



AARC 
BO 



"^1- 



AAB^' 
• B'O' • 




Digitized by VjOOQIC 



36 



But AAnC = A4, BC = 2R4 8in A, AAB'C= A„ and B'C'= 2B, sin A, 
hence P, : P, = ^ : ^^ 



Similarly, by taking moments about B and 0, 



wegetP,:P,:P,-|2:^:^. 

Jtij rvj £(4 



8724. (By R. TucKBR, M.A.) — If a constant line AB moves with ends 
on OX, OY, two rectangular axes, and on it a semicircle is d« scribtd, 
then locus of mid-point of arc is one of two straight lines. Find locus of 
any other fixed point on the arc. [If C is the mid-poiut of AB, and P the 
mid-point of the arc, then the question may be enunciated ior a bar rigidly 
connected with the bar AB.] 



Solution by C. E. Williams, M.A. ; Professor 
Mathews, M.A. ; and others, 

A circle passes round AOBP, therefore angle 
POA = PBA = ^iTf therefore P must lie on one 
of the bisectors of OX, OY. If P be any fixed 
point on the arc, we still have 

POA =s PBA = constant, 
and therefore P lies on oHe of two straight lines. 




8646. (By E. VioARife.) — On donne deux points A et B sur une 
circonf^renie de centre ; trouver sur cette circonference un troisidme 
point P, tel que les droites PA, PB coupent un diam^tre fixe CC en des 
points M, N, tels qiie I'on ait OM = ON. 



Solution by Professor Mathews, M.A. ; Professor Schoijtb ; and others. 

Produce AO to meet given 
circle in A' ; through B, A' 
draw a circle BNA'N' so that 
th« segment BNA' may be 
similar to BA'A; produce NB 
to P, etc. , as in figure. Then 
Z A'NB = supplement of BPA, 
hence NA' is parallel to PAM ; 
and, since OA'=OA, ON = OM. 
Similarly, if BN' be produced 
to P' and P'A meet CC in M', M'= ON'. 




Digitized by 



Google 



37 

2866. (By Professor Sylvestbb, F.R.S.)— Given the simultanebiui 
equationa Vxf i = v,+ (x^—x) fx-i, ««+i = Ug+{x^—sc) tt,_i + (2a: + 1) r,+i, 
prove that the general solution is of the form Vg =» \ax -t- ii$x, 

where A, ft, u, x are arbitrary constants, and determine the values of 

Solution by Professor Sebastian Sibcoic, M.A. 

The values of a*, fix are given in Quest. 2845 [Vol. xiii., p. 60], and 
may be written «, = n {a;-i-(-)*i}, /S, = n{a;-i+ (-)*^}. 

The solution of the equation in Ux is easily verified. 

Writing a? (a; +1) in the form {a;-i-(-l)*i} {a?-l + i-(-)«-i i}, 
and assuming «« = \yx a*, the equation for 7, will be 

= (2x + l){a:+l-i-(-)«*ii}, 
whence yx must be of the form a* + ( — I)* c. Substituting, and equating 
the terms that do, and those that do not, contain ( — )* separately, 

(» + i)«r,.i-tf,-(a;~i)a,., = (2a:+l)(a; + i) (1), 

««+i— «;c-l — 4c « 2x-¥ 1 (2). 

EMminating a^^i, we have iix-dx-i — 4c (a? + i) and ax^,l~ax = 4c(» + f), 
whence an*. 1— «» = 8c« + 8c, which agrees with (2) if c « J ; then 
««-«x-i=-a; + i, and a;p = it-*^ + 2a;)+y, 7, - i {«*+2j?+ (-)*i} +y. 
Similarly for «, = /ia5, /B*. 



8789. (By Professor Gullet, M.A.)— Find by a geometrical construc- 
tion the equation SS' = P* of the pair of tangents from T {x\ y') to a 
circle S s x'^ + i^^-z^ = 0, whose centre is O ; and show that, if perpen- 
diculars £L, KM, RK be drawn to the tangents and their chord of con- 
tact from a point R not on the circumference, RL . RM differs from RK* 
by TO- cos2 » cos* 0, where 2« and 20 are tiie angles subtended by the 
circle at T and R respectively. 

Solution by Professors Wolstbnholme, Betens, and others. 

If TP, TQ be tangents from the point (XY) to the circle x^ + y^^ r*, 
and (x, y) be any point P on either tangent (say TQ), 
PQ« = 2:2 + y2_r2, TQ2 = X2+ Y^-j-s, and PQ2 ; TQ2 = PM^ : TN2, 
where PM, TN are perpendiculars on QQ'. But the equation of QQ' 
being xX.-\-yY^r\ PM : QN « a:X + yY-r2 : X- + Y^-gS^ 
or «2 + y2_^2 : x^i- Y2-r2 = (a-X + yY-r»)2 ; (X^ + Y^-r*), 
giving the required equation 

(a,-2 + y2_,.2)(X2 + Y«-r2) = (j-X + yY-T^'. 

[Almost exactly the same proof applies to the central conies.] 

vol. xLvn. B 



Digitized by VjOOQIC 



38 



8833. (By Professor Nbvbbrg.) — ^Un angle de grandeur constante 
toume autour de son sommet A, ses cdtes rencontrent line droite donn^ 
XY aux points B et 0. Trouver (1) les enveloppes de la m^iane BB' et 
de la sym^diane BH ; (2) les points oil ces droites touchent leurs enve- 
loppes. 

Solutiont hy Professors (1) Schoute, (2) di Lonochamps. 

1. As the points B, 
B' (Fig. 1) determine 
on the parallels XY, xy 
two homographic divi- 
sions, the line BB' en- 
velopes a conic that 
touches the parallels 
and has A for a focus. 
When BiACi and 
B2AO2 are the positions 
of the angle BAG, in 
which ACi and BjA 
are parallel to XY, Bj 
and B2' are the points 
where XY and xy are 
touched hy this conic. 
And by applying the 
ry of Briancb 




theory 



x—B c y 

Fig. 2. 

hexagon to the triangle YBB'y circumscribed to the conic, the point E, 
where BB' touches it, is easily found. 

When BE (Fig. 2) is a symmedian of triangle BAC, the angles ABE and 
B'BC are equal, and the triangles A HE and ADB are similar. Then the 
corresponding medians AF and AB' of these triangles are isogonal with 
respect to AB and AE. Thus Z FAE ^ L BAC = constant. This proves 
that all the lines BE pass through the fixed point F. And as in the same 
manner the symmedians with respect to C pass through another fixed point 
G of xy^ and there is evidently a correspondence (1, 1) between these two 
pencils F and G, the locus of the point of Lemoine K of the triangle ABC 
will be a conic through F, G, touching AF, AG. Only when the rotat- 
ing angle BAC is right the points F and G coincide, and all the triangles 
BAC have the same point of Lemoine. (N.B. — Symediane is introduced 
by M. D'Ocagne for antim^diane.) 

2. Otherwise : — Gdneralisons la question en supposant quo les cot^s de 
Tangle A rencontrent : Tun, une droite fixe A, au point B ; I'autre, une 
seconde droite fixe A', au point C. Soit B'le milieu de AC ; le lieu d^crit 
par B' est une droite i parall^le a A', et les points B, B' d^crivent sur A et 
sur H deux divisions homographiques. On sait que, dans les conditions, 
BB' enveloppe une conique tangente li A et i 8 en de certains points 
M, «», que Ton sail determiner. 

Si Ton iniagine le point R commun ^ BB' et ^ Mw, la polaire de R coupe 
BB' au point cherche p. II est visible que p est le conjugue harmonique 
de R sur le segment BB'. 

Remarque,— &\, dans P^nonce donne, il fautsupposer que BH designe la 
hauteur du triangle ABC ; alors, en observant que BH fait avec BA un 
angle constant, on reconnalt que BH enveloppe une parabole. Le point 
de contact de BH avec son enveloppe se trouve, comme Ton sait, sur la 
circonference qui passe par A et par B tangentiellement k A. 



Digitized by 



Google 



39 

8719. (By Professor Wolstbnholmb, M.A., Sc.D.)— The lengths of 
the edg^s OA, 0B» OC of a tetrahedron OABC are denoted by a, b, e, those 
of the respectively opposite edges BO, CA, AB by x,y,s; and the dihedral 
angles opposite to these by A, B, ; X, Y, Z ; prove that, if Y be the 

8V 

volume expressed in terms of a, d, e, x, y, z, -— -• ^ax cot A, &c. ; 

oa 

and thence that (1) when a^ x; b, y, and e + z are ffiven, Y will be a 

maximum when C = Z ; (2) when a, x; b, y, and e—z are given, Y will 

be a maximum when C + Z = 180°; (3) when a, x ; d + y ; tf + ^are given, 

Y will be a maximum when B = X and Q ^2i (t.^., when i = a;, c ■■ «) ; 

(4) when a, « ; b + y^ e—z are given, Y wiU be a mairimum when B ■> X, 

C + Z = 180° ; and (6) investigate if Y can have a maximum when a, x ; 

b—y, e—z are given, or when a— », *— y, e—x are given; the given 

quantities being supposed always real and finite. 

In case (4), prove also that 180° is the maximum value of C+Z for 

variations in 6, y (subject to the conditions stated), and that zero is the 

minimum value of B—Y for variations in e, z. 




Solution by Professors Mathbws, Bbtens, and others. 

Draw AD perpendicular to BO, 
and AE perpendicular to OBC. 

Let the vectors DA, OB, 00 be 
represented by a, /3, 7. 

Then, if A be the volume OABO, 
6a = Sai87. 

In order to find ~ , turn the tri- 
oa 
angle ABO through a small angle 
about BO, so that A comes to A'; 
this changes a, and leaves d, «, x, y, s 
unaltered. AA' is ultimately per- 
pendicular to ABO, so that we may put 

da « vector AA' « h (Y/Sy + Y70 + Yo^) (1), 

where A is a small scalar quantity. 

Hptic«6?^ „ d^aBy ^ Bfiyda ^ ^ Sj3y (V0y + yya + Ya$) ,5. 

«a dTa Soda Safiy ^^' 

Ta 
by (1), observing that Sa (V$y + Y7a + Ya$) = 80/87. 
Now S/37 (Yi87 + Y70 + Yo/3) = S . Yi87 (Vfiy + Y7a + Ya0) 

= 40B0 . ABO . cos A = 20BO . BO .AD cos A 
» 20B0 . BO . AEcot A =. 6Aa;cot A, 
hence (2) becomes 

8A^^ 6A^cotA ^^^^^ therefore ?^-Jaa: cot A. 
8a 6A ' 9a ^ 

If A is to be made a maximum or a minimum by the variation of a, b, 



Digitized by VjOOQIC 



40 

or a« (oot A. itf + cot X.dir) + ... + ... =o (8). 

Case (1) gives da '^ dx ^ db ^ dy =: 0, de-\-dz » 0, so that (3) reduces 
to et (cot 0— cot Z)»0, e = 0, z^O make A « a minimum; hence 
oot B cotZ, or ■■ Z makes A a maximum; 

(2), (3), and (4) may be similarly treated. 



8840. (By W. 8. M'Oay, M.A.)— Prove that (1) the locus of the 
mean centre of the four points, in which a line of given direction meets 
the faces of a tetrahedron, is a plane (diametral plane) ; and (2) if A, B, 
C, D he the areas of the faces of the tetrahedron, all the diametral planes 
envelope the quartic (Aa?)* + (By/ + (Cz)* + (Du;)* = 0. 



Solution by Professor Schoxjtb. 

1. When a, iS, 7, 5 represent the four g^ven planes, L the point at in- 
finity indicating the direction of the lines, and ir, ^, if the planes that are 
separated harmonically from L by the couples of planes a and /3, y and S, 
IT and % respectively, then it is evident that in the determination of the 
locus in question the planes a and /3 may he replaced by ir counted twice, 
the planes y and 8 by ^ counted twice, and the planes « and ^ each of 
them counted twice by ^ counted four times. This proves that ^ is the 
locus. 

2. If F represents a vertex of the given tetrahedron, Q the point of in- 
tersection of the line LP with the opposite face of the tetrahedron, and R 
the point on PQ for which PR = iPQ, the locus of the mean centre 
passes through R. When a, &, c^ d are the coordinates of F, which, as P 
IS at infinity, satisfy the condition Aa + B^ + C(; + D<^ — 0, the locus 
contains the four points 

(—3a, bj Cf d)f (a, —3*, e, rf), (a, b, —Ze, d), {a, b, e, -3<Q. 

Therefore its equation is — + ^+— + — = 0. 
abed 

By differentiation the coordinates of the point, where this plane touches 

its envelope, are found to be proportional to 

Aa2, Bi^ Ccj2, Dd^; 

which, in due combination with the condition Aa + Bd + O + Dd ^ 0, 

proves that the equation of the envelope is 

(Ar)* + (By)* + (Cz)2 + (Dtr)« « 0. 
Thus this envelope is the Roman surface of Steiner, the quadric surface of 
which the three Hues, that join the centres of the opposite edges of the 
given tetrahedron, are double edges. 

[If a system of parallel lines pierce the faces of the tetradedron at 
angles a, /3, y, 8, the equation of the corresponding diametral plane 

is obviously -: — + -:^— + -: — + - — - = ; the projection of the faces 

sma sin/3 sm7 sin 8 
gives A8ina + B Bini3 + C8in7 + Dsin8s 0, whence the result.] 



Digiti 



ized by Google 



41 

8799. (By Professor Hudson, M. A.) — Find the envelope of the 
straight line au^^f (o) cos (6 — a) +/ (o) sin (0 — a) . 



Solution by Aspabaoub ; Professor Matz ; and others. 

The equation au —/(a) cos {0 — a) ■¥/* (a) sin (0— o) represents the tan- 
gent line to the curve au = f{d) at the point where = a, and of course 
the envelope of the straight line is the curve au =f(B). By the usual 
rule, we get = [/" (o) +/(o)] sin {B-a), Hence, unless /" (o) +/(a) = 0, 
we shall have for the point of contact with the envelope 6f = o, giving for 
the equation of the envelope au =f(d). In the particular case when 

/" (o) +/(a) = 0, /(o) = A cos a + B sin a, f (a) « — A sin o + B cos a, 
and the equation becomes aw ss A cos + B sin 0, or represents a definite 
straight Ime, and no envelope exists. 



8851. (By J. J. Walker, F.R.S.)— Show that either segment of a 
focal chord of a conic section is a mean proportional between its excess 
over the semi-latus rectum (or vice versd) and the whole chord. 



Solution by W. J. Grsbnst&bbt, B.A. ; Professor Betens; and others. 

The semi-latus rectum is an harmonic mean between the segments of 
a focal chord ; hence 

SP . 8F = RR' . PP', or SP (PF-SP) - RR' . PP', 
or SP»-PF(SP-RR'); 

similarly for the other segment SF. 



8837. (By the Editor.) — On the sides of a regular n-gon, n points 
are taken at random, one on each, forming the apices of inscribed »-gon ; 
again, inside the n triangles that lie outside this last polygon, n points 
are taken at random, one in each, forming, when joined, another n-gon ; 
find the general average area of this last n-gon, and show therefrom that, 
in the cases of a hexagon aud a square, the averages are f f and -^f of the 
original figure respectively. 

Solution by D. Biddle. 

If the mid-points of the sides of the original n-gon be first joined, and 
then the centroids of the resulting triangles, a polygon is formed whose 
area represents the area required. Accordingly, its comparative value 
may be thus defined (} + i cos of, where a is the angle subtending a side 
of the original n-gon. But the following solution gives an independent 



Digitized by VjOOQIC 



42 



Tiew of the case, and the concluding equation has an interesting bearing 
on the value of ir, when the n-gon is regarded as carried to its limit in the 
circle. 

Through the angle A draw AD, a diameter of the circumscribing circle, 
as a line of reference, and reckon perpendiculars to it as positive when to 
the right, negative when to the left of it ; also distances measured on it 
as positive when upwards, negative when downwards, from the several 
^t of the perpendiculars drawn from the angles of the original »-gon. 




Taking the case of the hexagon, let Pi, Pa, ... Pj be the points taken 
on the perimeter, and Q^, Qa, ... Q« be the points taken in the triangles. 
Then the w-gon formed by joining QiQg, Q^Qat &c. 

+ Q3?3 (H^3-H?4)-Q5y6 (H?6-H^4^ + Q4^4 (H^S-H^s) 

+ (02^2 + Q«?3) (H^a -H?3) -(05^6 + QUq,){B.q,-Kq^)] 

From this it is evident that the area of any n-gon so formed is 

i [Q'lqi (D^n-Dy-i) + ^2^2 (I>?l-I>?3) + .- + Q*»^» (Dqn-l-'Dqd] -(A), 
in which the y-terms within brackets are respectively the one before and 
the one after that corresponding to the factor outside. 

Now it is easy to see, Irom an examination of the shaded triangle in 
the diagram, that the average position of Q is the centroid, X, and this 
can be given in terms of P. For, it' we suppose Qj to be at X, then 

Q3y2-i{Pli?l + P2i^2)+i{Ba-HPl/'l + P2/'2)}-4(BG + P,i;i+P2i?2), 



Digitized by VjOOQIC 



43 

and D^, = i (Dp^ + Dpa) +i {DG-i (Dp^ + Dj^a)} = J (DG + Dpi + Dp,). 

Let ?i, ?2, ^3, &c. = DA, DG, DH, &c., and *i, k^, k^y &c. = (0), BG, 
CH, &c. Also, let a;, = APi / AB, r^ = BPg /BC, &c. ' Then 

and Dpi = ^i + (^— ^ a^i, T>Pi = ^ + (?3 - ^ a^j, &c. 

Consequently, 

Qafi-a = H*2 + [*i + (^2 - ^i) ^i] + [^2 + (^3-^2) ^2] }» &c., 

and 'Dq,^^{h+[ln + {li-ln)Xn] + [h + {l2-h)^l]}y 

Now, bearing in mind that Ar„ =s — Atj, A:„_i = — X-g, &c., and In = ^2» 
^n-i = ^3» &c., by symmetry, let k^ — ki^* mj, Atj— Atj = wtj, &c., and let 
h-h=fi^h-h^ U &c. Then, 
Qij-i =. A:i+|{mia;i-w„(l-ir„)}, %q<i^k^'^\{in^x^-m^{y-x^'\, &c., 

and D^i = ^i+i {/« (l-^»)-/i^i}» I>y2 = ^2 + i {/i (l-«i)-/2^2}, &c., 

where /« = -/i, /«. 1 = -/g, &c. 

Consequently, i [Qi^-i (Dj-n-D^g) + Q2$'2 (I^fi'i-I^^a) + — ] 

« i [[[{^i + i [♦«i^i--»»»(l-a:,,)]} {/» + i[/«-i(l-a:„.i)-/„i-n] 

-^2-i [/i(l-^i)-/2^2]}+&c.J 
= i||[[*i^n-A;i?2 + iV«-i(l-^n.i)-iAri/,.a:n 

- i^l/l (1 - ^1) + 1*1/2 ^2 + i^n »»i a?! - \l^ IHn {I - Xn) 

+ i*nifn.iXi {l-Xn.ii-imifnXiXn - 4m„/n.i (1 -a:„)(l-a;„_i) 

+ JWn/n Xn-imnfn x\ - \l^ m^ X^ + ^/^ »»„ (1 - a:„ ) 

-i^ifi ^1 + 4wi/i jri2 + ^,^^^2 a:i x^ + 4w„/i (1 -flTn ) (1 ~a;i) 

-i»Wn/2a*2 (l-ir„) I + &C. 

= i [[*l(^n-y + J*l(/„-l-/n-/i+/2)+J(/n-y K-mn) 

+ inr K-*»") (/«-l +/2) +7B K/l-'Wl/n) +Ti^y K/n-fWi/l)"! + &C."1 

= -siir r[(3^2^«-i) + (12^-1 ;„-! + 12A:2g + (ilk^ln + Vi + 3^„?n-i) 

L +{13W„-13V2)-(47A;i;2 + 3*2^ + *?i) •) -, 

^(Uk^h+nknh)- (3W3)j + &C-J- 

From this formula we are able [by joining a series, the several terms 
of which are made to correspond with the respective terms in (A), by 
being all raised one degree at a time, k^ to Atj, k^ to A-j, /„ to l^ l^ to l^t 
&c., until the n terms of (A) are satisfied] to calculate the numerator of 
a fraction of which the area of the original n-gon is the denominator ; and 
this fraction is the mean area required. 

The above formula is easily reducible to the following, in which the 



Digitized by VjOOQIC 



44 

Greek p at the foot of the several factors represents each figure horn 1 to 
n in succession, 

7V*'('.-8 + 8/,.2+l7/,.l-l7/,*l-8/,^j-/..3). 

In the case of the hexagon, taking the radius of the circumscrihing 
circle as unity, and the area of the original figure consequently as 
3v/3/2, A:i = 0, *, - ix/3, A', - i^/S, A:^ - 0, /tj - -iv/3, h'^-W^'. 
li =^2y Z, * 1|, ^s ' i) ^4 "" 0» ^ "== i) ^ = 1^ ; c^d the required area is 
Hv/3/W3-f#. 

In the. case of the square, taking the diagonal « 2, and the area also as 
2, k^ = 0, Xtj - 1, Atj = 0, A:4 - -1, ^ « 2, i, « 1, /j = 0, (4 - 1 ; and 
the required area » f^/2 « ^ « ^g. 



8854. (By Rev. T. R. Tbret, M.A.)— Solve (1) the equation 
w*+2 = (2:c + 6) Wa, ♦ 1 — (ar* + 4a: + 3) tr, ; 
and show (2) that, if u^ and Vg both satisfy this equation, and if Ux =» 2, 
«3 = 10, while Vi « 1, ^2 « 2, then 2w, « jc (j? f 3) c?,. 



Solution by Professors Sircom and Bbtbns. 

Dividing by (a; + 8j ! the equation may he written 

_i_ f .Jffiil. . _!^£±J_| « _J_ ( ^-1 - Jf? ^ = constant 5 
a; + 3t(a- + 2)! (a:+l)I) a? + 2t(a;+l)! x\ S 

whence iff - JfiLiL^ = C(a:+ 1), and m>. - a;! Ca^i (a? + 3)+ar!Ci; 
a:! (a;-l)! 

then, if «x = ar ! Cx J (a; + 3). r, = ar ! Ci, the given values of « and r make 
C » 1 «- Ci, then 2tf« » 4; (a; + 3) i;«. 



8778. (By W. J. C. Shabp, M.A.) — If wy^vz, uz-wx, and 
vx^uy be cogredient to a?, y, and z, the rectangular Cartesian coordinates 
of a point, respectively, «, r, and «; are so. 



Solution by D. Edwardes ; Sarah Marks ; and others. 

We have WY- VZ = l^ (wy-vz) + /j (wz-w^a;) + /g (f;a:-«y) 

= W (Wja; + Wo y f W3 c) - V (wj a; + «2y + fj3z), 
whence iWjW — «iV = /,p - l^w, m^W - WjV = liw — /jW, 

i«3W — WjV = /« « — /i» ; 
multiplying these by mj, Wj, w.,, and adding, we have 
W = (wi3^- W2/3) w + (wii^3 - W3/1) V + (?iW2~ ^2*«i) '''j or W - Wi« + «2t' + n^w. 
Similarly for U and V. 



Digitized by VjOOQIC 






45 



8650. (By E. M. Davys, M.A.)— Ptbve thit^ in atoy J)latie triangle, 

fcofii^ A + 4 cos^° A sin^ A f 6 cos^ A sin* A — 6 CDS'* A sin^ A — 4 cos^ A sin^® A 

-sini2A = C08 2A. 



Solution by Rev. T. R. Tebey, M.A. ; Hannah MotLBK ; and others. 

Quotient of left-hand side by rigM-hand side 

^ (cos^ A + cos* A sin* A + sin^ A) + 4 sin^ A cos^ A (cds* A + sin* A) 
+ o cos* A sin* A = (cob^ A + sin^ A)* =« 1. 



8678. (By Professor (^enese, M.A.) — AB is a chord of a conic, 
BJP, BQ are parallels to the asymptotes, A'^Q, a varitibl^ transversal 
meeting the curve at R. Prove that the ratio PR : RQ is constant. 



Solution by R. F. Davis, M*A. ; 
Professor Cha&ravarti ; and others. 

Let parallels through R to the asymp- 
^tes meet BP,' BQ in M and N respec* 
tively ; and let -or, W be the points at oo 
on the curve through which BP, BQ re- 
spectively pass. Then 

R {ABtr-'or'} i^ constant; 

dr, estimating upon BPiw, 

•[PBwM} is constant ; 

whence tflso so is PM : BM or PR : RQ. 




d757. (By Professor Edmund Bordaoe.) — If two fra'ctions ala\ 
bj b' are such that ab' — ba' = 1, prove thai the' sirnplesi fraction inclnd'ed 
between the two given fractions is (a + b) / {of + b') . 



Solution by H. L. Orchard,' B.Sc, M.A. ; Professcw^ B'fitBlTtf ; aitd others. 

If aV-ba^ = 0, then 4- -= ^ = 4±4. 
a b' a' + b' 

a+^a «+ — »' 



Kow 



a + b 



a' + b' 
Vol. XLVii. 



/ ■ ,, < -7-, and IS >. 
a' + 0' a' 



a' + lf ^ If • 



Digitized by VjOOQIC 



46 

8681. (By Professor Hudson, M.A.)— In the epicycloid of two cusps, 
if P be the describing point when Q is the point of contact, prove that 
the tangents at P, Q to the epicycloid and the fixed circle respectively 
meet on the line joining the cusps. 



. Solution hy R. F. Davis, M.A. } Rev. J. White, M.A. ; and others. 

Let O, O' be the centres of the fixed and moving circles respectively ; 
B the extremity of the diameter of the latter through the point of contact 




Q, so that PR, PQ are tangential and normal to the curve at P. Then, 
if A be the cusp, or point on fixed circle through which P was originally 
coincident, arc AQ — arc PQ, and OQ =» 20'Q ; hence 

Z AOQ = i Z PO'Q = Z PRQ. 

But OQ = OR, hence QT (perpendicular to OQ) meets PR on OA pro- 
duced. 



7987 & 8036. (By the Rev. T. R. Terry, M.A.)— (7987.)— Four 
spheres, whose radii ar« «, by e, d respectively, are such that each touches 
the other three externally. In the space between these four, another 
sphere of radius r is described touching all four externally. Show that 



^-f'(i)"(i)-(i)-«- 

R. Lachlan, B.A.) — If four sp] 
emally ; and r be the radius of tl 

.orthogonally, then -i- = 22 i^ - 2 f-^) . 



(8036.) — (By R. Lachlan, B.A.) — If four spheres, radii a, *, c, d, touch 
one another externally ; and r be the radius of the sphere which cuts them 



Solution by W. W. Taylor, M.A. 

Inverting with respect to the point of contact of one pair of the four 
spheres, this pair of spheres becomes two parallel planes (1, 2) ; the other 



Digitized by VjOOQIC 



47 

pair of spheres and the two spheres that touch the four original spheres 
become four equal spheres (3, 4, 6, 6), whose centres form two equilatertd 
triangles ; and the sphere of Quest. 8036 becomes the plane (7) passing 
through the four points of contact. This inverse system can be represented 
by the equations 

y ± A = 0, a;2 + y2 + «2 ±2Ax« (1, 2, 3, 4), 

ic2 + y2 + -5Si2>v/3Ai5 + 2A2«0, z =0 (6, 6, 7). 

Now, the radius of the inverse of the sphere /(a?, y, z) 

= a:2 + y2 + 2;2 + 2Aa; + 2By + 2C« + D = 0, 

with respect to the point h, k, I (constant of inversion «= 4), is 

Invert with reference to the point h, k, l, and call the radii corresponding 
to these seven equations a, b, c, rf, r^ r2, r^ ; then 

i. « i (A + ^), -i = i (A-A;) (8, 9), 

JL « 1 (A2 + A:2 + ^s + 2A;0, 4- = — (*' + 't* + ^-2AA) (10. 11), 

Q 4A a 4A 

_L« -L(A2+A;2 + /2 + 2^/3A^+2A2) (12), 

-1 = -L(A2 + ;fc2 + ^«2y3A?+2A2), Jl ^ i. (13, 14). 

r2 4A ^a 2 

From-(8), (9), A«i-+i-, Ar = ^ - i- (16, 16) ; 

a a 

from (10), (11), we have 

»-(7-i) '"■'«' 

from (10), (12), and (15), (18), 

WZ-h = 2 f i ^ ^ -A, /^/3 = — -2-i- (19) ; 

\r^ c I ri a 

also, by (16), (17), (18), 

(20) ; 

therefore, by (19), (20), ±^ - ± 5 1 +2^ +22^ = 65 ^ -S^l, 

therefore 1 - l-^ J_ ^.5 1 .^ 1 = q. 

ri2 Ti a a^ ao 

For fj substitute for I in (20), from (14), -t :*. 22 ^ - S ^. 

[These results are most readily obtained from the general formula given 
by the late Prof. CLirroiiD (on p. 335 of his Mathematical papers) for 



Digitized by VjOOQIC 



48 



finding the radloi of a sphere which cuts five given flpheres at givei^ 
angles: — via., if the spheres whose radii are r,, r, cut at the angl^ 
^M,p, then 



0. ~ 



1 
77' 



1 



1 






— 1, COSftrj 2) COSWjg, COSCtfj^^, COSWj g, 
C08(|f>2,j, —l, &C„ 

&C., ... -I, 



-1, 



-1, 



= 



By taking m,2 = m,i^ &c. = 0, a result equivalent to (7987) is £it once 
obtained ; and My taking ei^i,2= m,3 = «i,4 = w-i.s = <9i,4 = W3,4 = 9> 
and «i 6 = «2,6= «3,6 = «4.6= i^, a result equivalent to (8036) is 
()btainea. 

The above theorem is a particular case of a more general theorem given 
h^ the Boyal Society's Froceedinga for March 11th, 1886.] 



8725. (By S. Tebay, B.A.)— If the sines of opposite? dihedral angles 
of a tetrahedron be respectively proportional to the edges formed by these 
^gles, the areas of the four faces are equal. 



Solution by Pyofessprs Mathews, Beyens, and others. 

Denoting by (CD) the dihedral angle of /^ 

which the containing planes intersect in 
CD, we have 

8i»(CD)-AF/AE 

^ 3 vol. ABCD / 2 area ACD 
area BCD f pD 
Bin rCD) 3 vol. ABCD 
CD " 2 . ACD . BCD' 
60 also 

sin (AB) _ 3 vol- ABCD « 
AB " 2 CAB. DAB' * 

Hence, if ^^^^^ = ^^°/^^\ &c., ACD . BCD ^ CAB . DAB ; 
Cp AB 

■^ith two similar equations : whence BCD = CD^ = D^-B = ABC, 




Digitized by VjOOQIC 



49 



8697. (By Captain H. Brocard.)— Les droites qui joignent le centre 
du cercle circonscrit aux milieux des segments interceptes par les perpen- 
diculaires aux milieux des cotes sur les hauteurs du triangle sont perpen- 
diculaires aux medianes correspondantes. 



Solution by Professor Schoute ; R. F. Davis, M. A. ; and others. 

Let O l?e the circum-centre and D, E, P 
the middle points of the sides of the tri- 
angle. Let OE, OF meet the perpendicular 
AH on BC in the points Cj, i,. aud let dy be 
the middle point of biCi. Then the tri- 
angles OdiCi, ABO are obviously directly 
aimUar, corresponding lines being at 
right angles. Hence the median line 
Oil is perpendicular to the corresponding 
line AD. 




8502. (By B. Hanumanta Rau, B.A.) — Through any point K are 
drawn the straight lines B"KC, C"KA', and A"KB' respectively parallel 
to the sides BC, CA, AB of a triangle ; prove that (1) A A'B'C'= A A"B"C" ; 

(2) parallels through A, B, C to A'B', B'C, C'A' meet at a point O'; 

(3) parallels through 4., 5, C to A"C", 4"B", 0"B" also meet at a point 
D' ; (4) if the coordinates of O and O' are (o, i3, 7) and (o', /8', y') then 
aa ; cy'=b^\ aa! — cy : b$' ; (6) if K is the Symmedian point, O and Q' 
become the Brocard points. ' 



Solution by R. F. Davis, M.A. ; Professor Sab^ar, M.A. ; and others. 

Let Xy y, s be the trilinear coordi- 
|iates of K. Then it will be seen that 
each side of the triangle ABC is 
divided similarly into three parts 
^ ax '.by I cz, 

(1) AA'B'C=sum of As B'KC 
C'KA', A'KB'=s semi-sumof par^^llel- 
ogramsA'A^^B'B'^C'C"= AA^'B' a'. 

(2) Since BB' : B'C"= cz : rtr, a line 
through A parallel to A' IV divides 
BC in the same ratio, and its equation ia fi j y ^ ax I bz. Similarly the 
equations to lines through B, C parallel to B'C, C'A' respectively ar^ 
7 / o = by lex and aj fi = cz j ay. These meet in a point 0, 

o : i8 : 7 = czx : axy : byz, 

(3) Similarly for 0', a' : i8' : 7' « bxy : cyz : azx. 

(4) Whence aa : cy' = &c. 

(5) 11 X \ y : z =i a \b \ c then a : fi : y = / b : a j c : b / a, &c. 




Digitized by VjOOQIC 



50 



8585. (By E. F. Davis, M. A.)— If TP, TQ be tangents to a parabola, 
and PQ meet the directrix in Z, prove that ZT will be a mean proportional 
between ZP and ZQ. 

Solution hy Bev. T. R. Tebrt, M.A. ; Prof. Bbtbns, M.A. ; andothert. 

If the cooi'dinates of Z are 
(—a. A), and equation to ZQP be 

t^-^±i?«c (I), 

Bin«^ cos a 

then 

ZP.ZQ-(A2 + 4a')co8ec2a...(2). 

Also, from (1), coordinates of T 

are a + Acot0, 2a cot 0, 

therefore 

ZT2 = (A2 + 4a2) cosec''a...(3). 

[Since SZ bisects externally 
the angle PSQ, 

ZP.ZQ - SZ^ + SP . SQ « SZ« + ST3 « ZT».] 




8600. (By Professor Mahendra Nath Ray, M.A., LL.B.)— Through 
an indefinite point of a given hyperbola straight lines are drawn to meet 
the as3rmptotes ; show that the hyperbola itself is the envelope of the 
locua of the middle points of the straight line. 



Solution hy Professors Scott and Ignacio Bbyens. 

Let xy -» k^ be the hyperbola. 
The line is xa + y$= 2a$t where 
(o,i8) is the middle point ; it passes 
through a?', y', therefore 
x'a + y'fi = 2ai3, 

also y' « — , 




hence ax^- 2oj3a/ + /8A;2 = and o^jB* ' 
is the envelope. 



■■ a$k'^ or 0)3 = A;2 ; therefore xy = A* 



8623. (By N'Importe.)— From the centre of similitude S, common 
radii vectores SPP', SQQ' are drawn to similar curves PQ, P'Q'. Hav- 
ing given the centre of gravity of the area SPQ, find that of PQQ'P' ; 
and its limiting position when PU, V'Qf ultimately coincide. 



Digitized by VjOOQIC 



51 

Solution by Rev. H. London, M.A. ; Professor Ignacio Bbybns • and others. 

Let G be the centre of gravity of _ /-,' 

SPQ, and let ^^ ^^^ 

SP : SP' = SQ: SO/ = 1 : A:, 
/. area SPQ : areaSP'Q' = 1 : >fc2. 

Let G' be centre of gravity of SP'Q' 
and X that of PQQT', and m = mass of 
SPQ. Therefore, taking moments about S, 

k^m .[SG' = m . SG + (k^- 1) m . SX ; 
but SG' : SG = * : 1, SG {Jfi- 1) = SX (&2- 1), 

SX = %p^ . SG = ^!±A±1 . SG « fSG in the limit. 
k^ — \ k-k-l 




8716. (By Professor Mathews, M.A.) — Prove that the real common, 
tangents of the circles x^-^-y^—lax — 0, x' + y^-1by « are represented 
by 'lab (z^ -k- y'^—^ax) =» (by-^ax + abYy or, which is the same thing, by 
2ab (a;2 + y2_2*y) = {by-ax-aby. 



Solution by C. E. Williams, M.A. ; W. J. Grebnstrbet, B.A. ; and others. 
Two tangents from (a:', y') to the circle x^-^y^ — 2ax = are given by 
lxx''¥yy'''a{x + x')Y = (x^ + y^-2ax){x'^ + y'^-2ax'). 
But {x*, y') is in this case the external centre of similitude whose co- 
ordinates are easily found to be a3/(^— a), abl{a—b). Substituting these 
values for (a?', y'), we get (by^ax^ab)^ = 2ab {x^ + y'^^2ax)f and the other 
expression is obtained similarly from the other circle. 



7169 & 8537. (By W. J. Greenstreet, B.A.)— The sum of the 

three sides of a right-angled spherical triangle is a quadrant : prove 
minimum value of hypotenuse is cos-^ f , and that in this case the spherical 



Solution by Rev. T. R. Terry, M.A. ; J. Young, M.A. ; and others. 

Since cos c = cos a cos i, a.nd a + b + e =^ itr (1»2), 

therefore, for minimum value of c, 

tan a da + tan b db = 0, and da + db = 0, 
therefore a ^ b, whence, by (1) and (2), cos a = cos d = 2/^/6, cose =» |. 
This is obviously a minimum. Also 

sin E = sin (2A-iir) =» J, since cos A = f. 
[For another solution, see Vol. xxxviii., p. 89.] 



Digitized by VjOOQIC 



52 



8453. (By Satis Chandra Ray, B.A.) — Two rods, of lengths a and 5 
are jointed by a smooth hinge and rest on the convex side of a parabolic 
arc whose axis is vertical ; if, in the position of equilibrium, the rods in- 
clude a right angle, show that the angle 6 which the chord of contact 
makes with the axis is given by tan id — a^l IP, 



Solution by W. J. Barton, M.A. ; Sarah Marks, B.Sc. ; and others^ 

Let CA, CB be the rods, 
of lengths a and b ; Gj, Gj 
their middle pOints ; fia, fib 
their weights ; 8 the fociis, 
MOM' the directrix of the 
parabola; P, Q the points 
of contact of the rods ; ST 
the axis ; Ri, R> the reactions 
at P and Q. Then TSP= d ; 
and iiherefore MCA = id, 
M'CB = iir-^d. 

(1) Taking moments about. 
C for the equilibrium of 
each rod separately, we ge£ 

fia . iacok^a = R^^i, 

fitb . J* sin Jfl^ = R2^2» 

where CP = ^i, CQ = ^j. 




whence 



Ri _ tj 



<«fl2 



Resolviirg horizontally for the equilibrium of the system, 



"R 

Rj sin id = Rg cos id, whence ^ =* cot id 

Ri 



.(2). 

From (1) and (2) we get ^ / /j = a" / R 

If CB meet the axis in O, since SO = SQ and OSP = 0, the angle 
CQP = id J therefore ^1/^2 = tan id ; whence tan ^d == ^2/ b^. 

[The equations to EF, FD, DB are 

— acosA + /8cosB+7CosC = O-, acos A— )3cosB +7C0i?C = 0...(1, 2y, 

acosA + iScosB— 7COSC = (3). 

Hence the points of intersection of (1), (2), (3) with a = 0, jB = 0, 7 = 0, 
respectively, lie on the line 

ocos A + i8cosB + 7C0rfC = (4). 

If S = aBy + bya + ca$, then the equations of the circuniscribed, nine* 
point, and self-conjugate circles are, respectively, 

S = 0, 2S-(rto + ii8 + <?7)(acosA + i8co8B + 7COsC) = 0, 

S — (ao + *i8 + cy) (a cos A + i3 COS B + 7 cos C) = ; 

hence (4) is the common radical axis of these circles. } 



Digitized by VjOOQIC 



53 

(By Cb. Hermits, Membre de I'lnBtitut.)— Determiner les 

int6grale8d6finies f . f ^ . f**'-^^ r, 

jo sin (a? +p) J - J, sin (x +p) 

en Bupposant que p soit une quantity imaginaire quelconque. 



Solution hy the Proposer. 

La determination de ces integrales definies s'obtient comme conse- 
quence des r^sultats suivants auxquels conduisent faciloment les methodes 
eiementaires. 

Soit a une quantity r^elle, positive, et diff^rente de zero, on a : 

Jo sin (x - ia) Jo sin {x + ia) 1 - c-« ' 

I^- -=—7 ^=~" -^—f ^ ='4tarctg(f-«. 

Eemarquons ensuite qu'en rempla9ant a par une variable imaginaire 
o + tjS, dont la partie r^elle soit positive et diff6rente de zero, et par 
consequent dans toute la region au-dessus de I'ate des abscisses, les 
diverses integrales definies sent, des fonctions holomorph.es de cette vari- 
able. Observons maintenant que z etant k Tinterieur d*une circonference 
de rayon ^gal k 1' unite et dont le centre est S Torig^e, la fonction 
log(l +2/1 — 2) et arc tg 2 sont aussi'holomorphes. Or on obtient en 
posant z » e-^., des valours qui remplissent cette condition, et 1' extension 
des relations (I.) et (II.) k toutes les quantites a^ a.-h tjS, oil a est positif 
et different de z^ro, est la consequence immediate de la proposition bien 
connue de Biemann : Deux fonctions, uniformes, holomorphes ou n'ayant 
dans une aire donnee que des discontinuites en nombre fini, sont egales. 
en tons les points de cette aire, si elles coincident le long d'une ligne de 
grandeur finie. 



8242. (Professor Sylvester, F.R.S.) — ^If by a Simplicissimum <tf 
the f>th order be understood a figure in space of n dimensions formed by 
the indefinite protraction of the series of which a linear segment, a tri- 
angle, and a pyramid are the three first terms, prove that (1), when each 

edge is unity, the squared content is \!LL-1 _ and hence (2) de- 

^ ^* ^ 2»»(1.2.3...«)2' ^ ' 

duce that the Cayleyan Persymmetrical Invertebrate Determinant of the 

squared edges by which such squared content is imaged must be 

diminished in the ratio of negative unity to ( — 2)" (1 . 2 . 3 ... «)2, in order 

that it may represent its absolute value. £x, gr,^ the determinant 

{obY {acY 1 

{baY . {bo)^ 1 

((ja)2 (cby . 1 
11 1 . 

images the squared content of a triangle whose edges are (ab), (ac), (be), 
for the triangle vanishes when this determinant vanishes, but the actual 
value of the squared content is this determinant diminished in the ratio 
of negative unity to 22(1 . 2)'\ t.«., multiplied by — ^^ = (2). 

VOL. XLVII. o 



Digitized by VjOOQIC 



54 



Soluium by W. J. C. Sharp, M.A. 
If orthogonal ooordinates be employed, the contents of the successiye 
Simpliciflsima are 

a?i» Pu «i» 1 



i I ^1' M J_ 
l.'K, ir 2! 



^1, yi. 1 


1 


X2 y^y 1 


' 3! 


a?8» Viy 1 





and in n-dimension space 



«i» yu 
«2. yai 



•''•ij .'a* *2» ^ 

a-a* y8» «8» 1 

X^y Va, «4» 1 

.... 1 



, &c., 



V suppose. 



^»+b y»+ii 1 

Now, if the origin be taken at the centre of the hyper-sphere radius 
R whidi passes through the n + 1 vertices, and if the edges be denoted by 
(1.2), (2.3), &c., 

x^ +yi« + «R2, 2 (a:iarj + yiy2+ ) = 2R2_ (i . 2)2, 

x^ +ys«+ = R^ 2(a;jaJs + y,y8 + ) - 2R2-(2. 3)«, 



^i.i+yi.i -^^^ 



and 

v«2 



1, 0, 0, 
0, «i, Vu 

0, «!, y„ 



0, 



^ (^lyn^i 



2»*n! 



0, 0, 0, 

1, 2^1, 2^1, 

1, 2^, 2<„ 



0, «n+b yn+1 tn*h 1 

therefore y « (^:ili?^' 

2»*(«!)* 

X 0, 1, 1, 

1, 2(a;i«+yi« ... ^i«), 2 (x^x^-^y^yt ... ^i^,) 

If 2(ari«j + yiy,...<i^,), 2 (i-j^ + y,^... ^«) 



1, 2a?n+b 2<n*i, 



1, 2(a;iar„+i+yiyn+i...^i<«+i), 2 (araa?„+i + yiy»+i ... <i^„+i). 

0, 1, 1, 

1, 2R», 2R«- (1.2)2, ... 
1, 2R2- (1.2)2, 2R2, 



(«1J2«+1 

' 2»(fi!)2 



(~l)2H^l 

2»»(n!)2 



^ (-l)»»^g 
" 2»(n!)2 



1, 2R2-(l.n+l)2, 2R2-(2.n + l)2 

0, 1, 1, 

1, 0, -(1.2)2, .... 
1, -(1.2)2, 0, 



1, -(l.n + m -(2.n + l)2, 
0, 1, 1, 



(1.2)2, 
0, 



1, (1.2)«, 

1, (l.n+l)2, (2.n + l)», 



Digitized by VjOOQIC 



53 



which prorea (2) ; also, if (1 . 2)* - (2 . 3)> - &c »> I, the determinant is 
- (-1)"*> (»+ 1), and T^ = g^,, which proTes (1). 

[If the n+ 1 points, which detennine the Simplicissimum in space of n 
dimensions, lie on a linear locus in space of (m— 1) dimensions, the deter- 
minant V vanishes, and therefore the corresponding Cayleyan Persymme- 
trical Invertebrate Determinant ; and this equated to zero gives a relation 
between the mutual distances ofn + l points on a linear locus in space of 
n—l dimensions, as of three points in a straight line, four in a plane, five 
in a space, &c. 

Mr. Sharp considers this to be an epoch-making qmstion, as he believes 
that it will lead to the application to space of higher dimensions of methods 
analogous to those of tnUnear and quadriplanar coordinates. 



8755. (Professor Nbubero.) — On prolonge les hauteurs du triangle 
ABC au deli des sommets des quantitis AA'=BC, BB'=CA, CC'=«AB ; 
d^montrer : (1) les triangles ABO, A'B'C ont mdme centre de gravity ;" 

(2) si a, a' sent les angles de Brocard de ces triangles, on a 

A'B'C = 2ABC (2 + cot o), (A'B')2 + (B'0')2 + (C'A')« =- 8AB0 (3 + 2 cot a), 
coto'= (2coto + 3)/(cota + 2) ; 

(3) les points A, 6, sent les centres des carr^s construits, int^rieurement, 
sur les c6tes du triangle A'B'C ; (4) les milieux des c6t6s de A'B'O' sent 
les centres des carr^s construits, ext^eurement, sur les c6t68 de ABO. 



Solution by R. F. Davis, M.A. ; Prof. Aitab, M.A. ; and others. 

Let fall the perpendiculars A'm, 
B'n upon AB. Then, since the tri- 
angles AA'wi, CBF are equal in all 
respects, A'm =» BF and Aw =« OF. 
Similarly, B'n = AF, and B/» = OF. 
Hence <?, the middle point of AB, 
is also the middle point of mn ; and 
if ec' be drawn perpendicular to AB 
meeting A'B' in c', </ will be the 
middle point of A'B', and 

cc'= i (A'm + B'w) = Ac or Be, 
Thus (4) c' is the centre of the 
square described externally on AB. 

Since c</ is parallel to and one-half 
of CO' ; Ccy Co' intersect in a point 
G which divides each of them in the 
ratio 2:1, and therefore (1) the triangles ABC, A'B'C have the same 
centre of gravity G. 

Since BB', Be' are equal and perpendicular to AO, Atf', respectively, 
BV i& also equal and perpendicular to Cc', and (3) is the centre of the 
square doacribed internally on A'B'. 




Digitized by 



Google 



66 

(2) Since HA'- a + 2Rco8A « a(l + cot A), HB'= &(l + cotB), 
/A'HB'«ir-C, area A'HB'« A(l +cotA) (1 + cotJB). 
Hence area A'B'C= A {(1 + cotA)(l + cotB) + ... + ...} 

= A(4 + 2cota) = 2A(2 + coto), 
■mce cot B cote + ... + ... B ly cota»cot A + ... + .. .. 

Again, (A'BO' « (A'm-B'«)« + mn« » (BF-AF)2+4m« 

«4{<yF»+(Ac + CF)3}«4{Cc9 + Ac« + 2A}=-2(a- + ft») + 8A, 
hence (A'B')« + (B'CO' + (C'AO« - 24 A + 4 (a« + A^ + c^) = 8 A (3 + 2 cot a), 
whence cot a' as stated. 

It is to be observed, when ABC is equilateral, then so also is A'B'C, and 
oota- cota' - V3. 



8865. (Amiral db JowQUitRES.) — Soient trois nombres entiers, 
0, by e, premiers entre eux, deux h deux, et verifiant T^quation a" + ^=<?*. 
D6montrer que (I'exposant n premier et superieur k 3) : (\) a et b ne 
peuTent Stre, simultan^ment, premiers ; (2) si a, suppose inf erieur k b, 
est premier, e » b+l, 

8866 & 8908. (Professor Catalan.) — Demontrer les contributions au 
thlor^e de Fehmat:— a suppose premier, (1) a— I = 9JJ (,/) ; (2) «»— I 
=» 9}? (nb) ; (3) tout diviseur premier, de «— a, divipe a— I; (4) a + b et 
tf— a sent premiers entre eux; (5) 2a— 1 et 2b + I sent premiers entre 
eux ; (6) le nombre premier, a (s'll existe) est compris entre 

(nb^'^f"^ et {n (* + l^«-i}^^*; 

17) a et 6 surpassent n; (S) le nombre b, qui satisfait h Tequation 
^4. Ijn.^n ae a**, est compris entre 

aialn)^^*'^'^ et -l + «(fl/«)^^*"^ 

(9) soit b un nombre entier, sup^eur au nombre entier n. Entre 

{nb^-i)y» ei {n(* + l)*-^}^^*, 

11 7 a, tout au plus, un nombre entier; (10) aucun des nombres a + b, 
c—a, c^b, n'est premier; (11) chacun d'eux a la forme N, ou la forme 
(l/«) N, N ^tant un nombre entier; (12) soient, s'il est possible, 

a + i « <?'»», <?-a = b'\ e-b^ «'»' ; 
alors c « SK (») ; (13) (a; + y)»»~a;»»-y»» « nxy {x+y) P, 

P = Hia:»»-» + H3a;»»-*y+ — +^i!f*"\ 
les coefiBcients sent donnas par la formule Hp = (I/')) [Cn-i,p ± 1], 
le signe -I- r^pondant au cas oii p est pair, et le polyn6me P est divisilde 
par x^ + xy + y^f et mdme par {x^ + xy + y^)\ si n = 3)2 (6) + 1 ; (14) la 
difference des puissances ft**°^» de deux nombres entiers consecutifn, 
a, a+1, ^tant diminu^e de 1, est divisible par na{a+l) {a^ + a-\-\) ; 
les facteurs a, a + 1, a' + a + 1 sont premiers entre eux, deux k deux ; en 
outre, le troisi^me egale le produit des deux autres, augmente de 1 ; 
(16) si, dans 1' equation de Febiiat, le nombre a est premier, on a, par le 
th^r^me 8866, de M. de Jonqui^res, a»- 1 » 3R [w* (A + 1) (^ + i + 1)] ; 
et (16) ff est compris entre a + b et ^{a + b). 



Digitized by VjOOQIC 



57 

Solution by Samubl Roberts, F.R.8. 

It seems convenient to take these three questions together. The par- 
ticular results are so numerous, that brevity must be specially consulted 
in each instance. They are, however, in general, simple deductions from 
the considerations indicated by Baklow (Theory of Numbers, Cap. vi.) 
and by Abel ( Works, Holmboe's Edition, p. 264, Vol. n.). 

We cannot have a » 6 ; but H a, b are prime the conditions «— « 
e—b= 1 are necessary, and c cannot be prime, since c* =» (a + ^) ^, ^ > 1. 
If a (the least number) is prime, <?— 3 = 1. 

(1) By F£KMAT*8 theorem applied to ^— a*— i* — 0, we have a+b—e 
s-gK« = a-1. 

(2) a»-l = «»-4*-(<?-3)»=:SW*(* + l)fi. 

(3) {c—a) + («— 1) == * ; («— «) ^ =■ *" ; (c— a) isof theformr»orr"fi»->, 
r or m being a factor of 6, therefore a— 1 contains the prime factors of 
{c-a). 

(4) (a-k-b) ^ k {c~a) makes a not prime to b, contrary to hypothesis. 
(6) 2a-l = (a + *)-(<J-a), 2*+l = (a+*) + (tf-a). 

(6) By (1) since a is > 1, 4 > a. 

(7) (* + l)»-ft- >ni»-S (A + l)*»-{(*+l)-l}*'< n(* + l)»-i. 

(9) If a Hea between («i*-»)^^* and {«(*+l)*-i}^^*, 
we have a + 1 > (nft»-i)^* + 1 > {« (* + l)«->}^^*, 
or (»*»-') + f 1 j, since 4 > n. 

Similarly, (a-1) < {« (ft + l)-»}^*-l < [«{(*+ l)-l}->]^^* 
<{«(* + l)-»}'^*-(l.l). 

Therefore a + 1, a— 1 lie outside the limits. 

(10) Each of these quantities is of the form r" or t^n^'^, 

(11) As above, see Baklow, Cap. vi. (Ba&low's mistake occurs 
further on). 

(12) I do not understand this. If e contains n as a factor, a-^b con* 

tains ft as a factor and is of the form r"w*->. For ^ "^^ is prime to n 

a + b ^ 

if + 6 is prime to ft, and, if a + d contains n, contains ft. It has 

a-^o 

been assumed all along that a, 6, are prime to one another. 

(13) (« + yy»-«"-jr= («+y) {(a?+y)»-*-(«*-i + ««*V + ...+y"*^)} 

= *y(«+y){[(«-i) + i]^-' 

^^(ft-l )(n-2) ^^j^,.^^_^^^^_^^^^jy,,3^ 



Digitized by VjOOQIC 



58 

-A^ (' -^-J^)* -^~S^* ^^l^ci^ * is <>^ ^^ fonis 6i:L 1, ifl made aero by piittii^ 
tpy for X, w betng an imaginarj cube root ol unitr. If s is of the form 
4M;-I-1, tibe derired fimctioa n 'x-t-y *-'— ai*-** is ande aaro by the 
aame aobstitiitioa. The fimctioa -x-i-jr)*— x*— jr* TirntufTtft therefore 
(*— wy) (x— ii^y) or ** + j^f +y2, and in the Lut case twice orer. 

(14), (15) lliefle are dn«ct conseqaences of the fovegoing. 

(16) («-«) + {«-^) > 0, «• = «- + 4^ < (• + *)-. 



8810. (Ber. T. R. Tkut, M^.) — Fmd tiie equation to the 
straig^ linies throu^ the origin and the intenection of the conies 
j?»-3xy-4x + y-l«0, 2x9-4-^-i-4f>-l-Sjr<i> ISy^^S = 0. 



^2k<umi 3y H. FoBTBT, M Jl ; <A#PBOPoaK; mmiothen. 
In the most general case, the equations to the oomos are of the form 

•!» + »i + «*« =0>» 
whence u^^—vm^ = — {u^v^— ri«t}» ^ ^i "* ^i J 

therefore the equation to the four lines required is 

In the particular case this reduces to {x—yY (2x +y}' = 0, t^., the four 
lines coincide two and two with the lines x- y = and 2x -i-y = 0. 



8843. (Kev. T. P. KntKMAN, F.R.S.) — The angles fia*c.../« of 
any convex plane M-gon N are joined to a point P in its plane, within or 
without it. In Pa, P^, ... Pit are taken points a^biCi ... m^n^^ such that 
ffOi^i ... miMi is a broken line L beginning and ending on Vn. L is thus 
formed: first, f}a|>aj<i; and next, if dye^^ be any three consecutive 
points of it, the distances in the directions /]«i and e^d^ of «i from the lines 
/e and «^ in N are equal. The line ftjti»i meets the edge nm of N in r. 
Prove that, if from « in an produced we draw Mi =r rn^^ «fij is parallel to 
the first portion nai of L. 

Solution by the Pboposer. 

The described fig^e is the orthogonal projection of a pyramid whose 
vertex P is anywhere over the base N, and on which the line over 
nai ... »t|»i crosses every face about P, in a path equigradient from n to nj. 
For, as the lines drawn from ^i are equal, they are projected sides of an 
isosceles triangle drawn from tf, in P^ to edges of N, in the faces about P 
whose intersection is Ve ; and t^e sides of this triangle make equal angles 
with the base, or are equigradient. Hence «w, and nax are equigradients 
directed upwards from s and n in the same plane aPm, and are therefore 
parallels. 



Digitized by V^OOQIC 



59 



Thus we can find (Quest. 8325) with the ruler the fly's upward path 
from n through aj to a point «i in P». A few trials with the ruler in 
piano, using a,, 03, &c. instead of aj, will hring us to n^, n^y &c., one of 
which is sure to he near enough for engineering purposes to the starting 
point E. 

As the solution of Mr. Biddle's interesting Quest. 8325 is not likely to 
be completely worked out, this easy approximation to it by plane geometry 
may be worth having. 



8814. (The Editor.} — Prove that the triangle formed by joining 
the mid-points of the altitudes of a triangle is one -fourth of the pedal 
triangle, and that the theorem holds good for any other three concurrent 
lines drawn through the vertices of the triangle. 




Solution by Professor Db Lonochamps ; D, Biddle ; and others, 

1. Qeometrieally : — Soient A', B', 
C les milieux des c6t^s du triangle 
consid^, PQR le triangle p^al 
correspondant k M. Les droites AP, 
BQ, CE rencontrent les cdt^ de 
A'B'C respectivement en des points 
P', Q', R'. Consid^rons aussi les 
points P", Q", R" isotomiquea de 
r\ Of, R' ; c'est k dire des points 
tels que B'F'=P'C', etc.... La droite 
AT" est parall^e k AP, et d'une 
fa9on plus g^n^rale, on pent dire que la figure A'B'CT"Q"R" est 
homoth6tique k la figure ABCPQR ; le centre de gravite de ABC 6tant 
le centre de I'homothetie et le rapport d'homoth^tie ^tant 6gal k i. De 
Ik, nous concluons d'abord que I'aire du triangle P"Q"R" est le i de celle 
du triangle PQR. D'autre part, on sait (Journal de Mathhftatiques 614" 
mentairea, 1877, p. 224), par une propriety que nous avons indiqu^e autre- 
fois, que deux triangles isotomiquea P'Q'R', P"Q"R" sont equivalents, 

Ainsi, en resume, le triangle FQ'R' est le \ de PQR. 

2. Analytically : — Si nous consid6rons trois points quelconques Mq, M^, 
H], dont les coordonn^es barycentriques sont, respectivement, 

la surface S du triangle MqMiM^ est donnee, comme Ton sait, par la 



formule ±825- 



oo A) 7o 
oi ^1 7i 
02 ^ 7^ 



S d^signant Taire du triangle de reference 



ABC; oq, /So, 70 ••• repr^ntant lea eoordonniea ahaoluea des points con- 
siddr^s. Prenons un point 11 dont les coordonn6es soient (a', /8'> 70 ; les 
droites A/u, B/u, C/u rencontrent les cotes du triangle ABC en des points 



Digitized by VjOOQIC 



60 



Mq, Mj, Ms ayant pour coordonnees 



0, 



S3' 



sy 



/3' + 7' i3' + 



., etl'ona: ±S^Z = 



0, 



B»' sy 



/3'+y' i3'+y 
-?^ 

;' + y' ' o' + 7' 

Sa' Sis' 



2 d^signant maintenant Taire du triangle p6dal Mq Mi Mj. Cette fonniile 



a'3V 



.(1). 



donne, finalement, ±S = '-.-, , ^,. ,^, ... , ... 
(o' + i3')(i8' + y)(y + a') 

Consid^ons maintenant les points /io* Mi> A4 milieux des segments AMq, 
AM|, AM]. Les coordonnees barycentriques absolues de ces points sont 

s. s fi' 8_ y 

2' 2 fi' + Y' 2 /8' + 7'' 
La surface :;' da triangle juq Mi Ms est donn6e par la formnle 

^ s y 



±S«2' = 



1- «'' 
2 o' + 7'' 
S a' 



2 a' + i3'' 2 o' + /8" 



2 /S' + y' 2 i3' + y 

2* 2 ^3^ + 7' 

8 iS' S 



2 



done ±2' = 



S^ 1 

8 (a' + i3')(/8' + 7')(y + «0 



/s'+y, is', y 
«', «'+y, y 

/3', a' + /8' 



Le determinant qui entre dans cette formule ^tant d^velopp^, ou trouve 
facilement (mais ce r^sultat pent dtre vu aussi par des voies plus directes et 
plus elegantes) qu'il est ^gal k 4a'/3'7'- 

S g'iSV 



Ecrivons done 



±2' = 



2 (a' + i3')(i8'+y)(y + aO 
La comparaison des formules (1) et (2) prouve que 2 = 42'. 



(2). 



8858. (Professor Cochez.) — B^soudre les Equations 

{xt/ + l) {x + y) « axi/y (xY+ 1) (x^ + y^) = b^^Y- 
Montrer que ce syst^me est quadratiqtte, Appliquer les formules au cas 
particulier suivant : a = J^, ^ = ijf. 



Solution by Professor Stbggall ; R. F. Davis, M.A. ; and others. 

Let ic + 1/a; = I, y + 1/y « ?7 ; then the equations become ^ + ?; ■= «, 
|2 + ^ = ^2 ^ 4^ whence |, ti are easily found. In the particular case 
I = xo or 2 ; ij = 2 or ^f- ; and we have a; = 1, y = 3 or i, y -« 1, 
a; = 3 or ^. 



Digitized by VjOOQIC 



61 

8498. (AsOtosh MukhopAdhyAy, M.A., F.R.A.S.) — The equation 

\dx) dfi dx'dt'dxdt \dt / dx'^ \dx I 

is inte^ble when (1) Q—O, (2) Q » dx/dt. Hence, obtain the complete 
primitive. 

Solution by Professor Lloyd Tanner, M.A. 

The proposed equation is a transformation of — - + Q » 0, where x, y, 

dt^ 

are independent variables. The solutions required are, therefore, 

X = <f>y + t}l/ff when Q = ; and x = <f>y + e-*^ when Q ■» dx/d(, 

[See Messenger of Mathematics, New Scrips, Vol. v., p. 71.] 



Note on Anallagmatic Curves. By Prof cssok Wolstbnholme, Sc.D.. 



In a curve which is its own inverse with respect to a point 0, if any 
straight line through meet the curve in two points P, Q, so that the 
rectaugle OP . OQ is constant, a circle can be drawn touching the curve 
in P, Q. Hence, when we suppose Q to coincide with P, the bitangent 
circle will in the limit have four-point contact with the curve at P, 
and will therefore be a circle of curvature of maximum or minimum 
radius. The points of greatest or least curvature (ottier than the vertices) 
in any such curve will therefore be the pointa of contact of tangent? 
drawn from a ceutre of inversion of the curve. 

I do not remember ever to have seen this simple deduction drawn. I 
ought to have myself noticed it years ago, as I have two particular cases 
of it in my book of problems ; but in both I obtained the result in a mudi 
' more laborious way. No doubt many interestiHg particular cases might 
be considered. I have not much hope of doing anything at it myself, so 
should be glad to have the above note published, pro bono publico. 



8786. (R- Tuckeb, M.A.) — P;o, Qy, Rr are focal chords of a 
parabola ; if P, Q, R have a conormal point, then the centres of curvature 
for p, Qy r are collioear. (This is another way of putting the first part.of 
Question 8693.) Find the equation to this Ime. Prove that this central 
line envelopes (1) a hyperbola when tangents atjp, q meet on the directrix, 
(2) an ellipse when the tangents meet on the latus rectum. 



Solution by R. Enowles, B.A. ; Prof. Beyens ; and others. 

In the solution of Quest. 8693 it is shown that the centres of curvature 
of three points on a parabola are collinear when 2y ~^ == ; if a;|, ^i, &c., 

VOL. XLVH. H 



Digitized by 



Google 



62 

be the coordinates of PQR, those otp, q, rare 

— , , &c., and 5y-i = -4a^ {y^ + yj + ^3) « 0, 

^1 yi 
«ince F, Q, E have a conormal point. 

The coordinates of the centres of curvature aip, q, r are 
da^ + 2ax. 16a* « 

and if the tangents at p, q meet in the directrix ^1^3 *— 4a'. The 
equation to the central line 

16a* ^ ^aW + yit/2 + yJ^ /^_ 3a2 + 2a£3\ 

3yiy2(yi+y^ V x^ I 

becomes yi + ya = -^8 (1)> 

-2ayj«+ 3ay,«-48a6 = (ys» + 4a») (yj»a:-12a8^12ay32) (2), 

if the tangents at^^ meet in the latus rectum yjyj = 4a2, and (1) becomes 

3ay3y-48a6 = -(y32-4a2)(y3«a?-12a8-2ay32) (3), 

and the envelopes of (2) and (3) are respectively the hyperbola 

9y2-16:rS+ 112aa;~ leOa-^ » 0, 
and the ellipse 9y> + 1 6^^ — 1 6aa; + 32a2 » 0. 



"- „.» 



8207. 0^- J- C. Sharp, M.A.) — If A be the angle contained in 
the half of a small Circle of a sphere, angular radius a, by arcs of lengths 
b and ; i.e., if ABC be a spherical triangle having the angle A = £ + C ; 
show that (1) cos d + cos « = 1 + cos a ; (2) cos A = —tan ^b tan ic ; and (3) 
hence deduce Euclid I. 47 and III. 31. 



Solution by Rev. T. Gallibrs, M.A. ; Prof. Matz, M.A. ; and others. 
Let AO = OB = OC == R ; then, from triangles AOC, AOB, 
cos b = cos AO . cos OC + sin AO . sinOC cos AGO 

= cos2R + sin2RcosAOC (1), 

cos e = cos AO . cos OB + sin AO sin OB cos AOB 

= cos2R + sin2RcosAOB (2). 

From (1) and (2) by addition, and since AOC + AOB = 180°, 

cos* + cos<?= 2cos?R = 1 + C0S2R = 1+cosa (3) ; 

also, from triangle ABC, 

A cosa— cosJcosc cos3 + cos<?— 1 — cosicosc 

cos A = r— r-: = :— T": 

2 Binm 2 A^Hc taniJtaai. (4). 

4 Sin ^6 COS ^d smi(; COS |0 
Now, the formulsp in Plane Trigonometry corresponding to (3) and (4) 
are found by writing a — ajr, b — fi/r, c — y/r and making r infinite, 
when, a, iS, y become the sides of a plane triangle. 



Digitized by VjOOQIC 



63 



Thus (3) becomes cos— + cos-^= 1+ cos— (6). 

But 

cos-^sl^ — . ■^-+— - . j5 .... cos— «1 — - .-«--+— .-3^ — ,.., 

cos^«l-±.^+JL.i5l- 

Substituting in (6) , we have, after reduction, jS^ + T^^o^ + A — , 

where A is a quantity which remains finite when r is made infinite. 

Thus, when r is infinite, 0^ + y^ ^ aK a result which agrees with • 

Euc.1.47. 

Also, when * - ^, c » ^21, in (4), we have cos A = _^}I^[ ^r).in(yf2p 

2r 2r 3!V2ry ' 2r 2r SlVir/ * 

also 

cosA^l-J-(AV + J_(J.V-..., cosX.i-lf^V + J-fJLV 
2r 2 V2r y ^4 ! V2r j ' 2r 2 U^ / 4 ! \2r I 

Therefore cos A » ^ ' /rL » where \' and u remain finite when r 

l+/*.(l/r2)' 

is made infinite ; therefore, when r is infinite, cos A — 0, or A = 90**, a 

result which agrees with first part of Euc. III. 31. 



8571. (As<^T0SH MukhopIdhtIt, M.A., F.R.A.S.) — Show that 
the reciprocal polar of the e volute of the reciprocal polar of the evolute of 

the conic ^ + ^ ^ 1, with respect to the circle described on the line 

joining the foci as diameter, is the curve 

{{•.HmiiniY-dniYHf-i)'- 

Solution by Profs. Sibcom , Nilkantha Sarkab ; and others. 
The reciprocal polar of the evolute of ^ + ^ =h ^^ respect to 
s3+y« a o'-^s is the locus of a;y, where xx^ +yyi = a?—**, -^ « "p^ 

*^^ "^ + ^ •= ^» whence ^ + -^ =1. The reciprocal polar of the 

a* ir X* y* 

evolute of the last locus is the locus of xy, where 



Digitized by VjOOQIC 



64 



and, from (2), _-.*-(-) y, -_-y.^-j ^. 
Substituting in (3) and reducing, we obtain the required result. 



whencoi from (1), 



8311. (R. SwAMiNATHA AiYAB, B. A.)— -Find how many numbers 
can be formed, having n for the sum of the digits (zero not being 
used as a digit). 



Solution by G. G. Stobr, M.A. ; the Proposbb. 
Let Pm represent the tiumber of numbers that have n for the sum of the 
digits. Of these P», those that have 1 for their first digit are P«-i, those 
that have 2 for their first digit are Pn.2, those that have 3 for their first 
digit are P»-s, and so on. Therefore 

PH-P»-l + Pn-2 + PH-8 +P»-». 

ThiiB Pn is the coefficient of a;** in the expansion in powers of s of 

, r, that is, of - — "^ -^ . 



r(l-»){l-aj(2-«»)}- 



l-2a;+a;W 

= (!-«) {l+«(2-a^)+a^(2-«»)«+ «»(2-a;»)* + }. 

The coefficient of a:" is 2"-(»- 3) 2*-^^ + (n-18)(n-19) ^n-ao^ ^^ 

_ j2»-«-(«-io) 2-"+ ("-laHn-ao) 2— >-&c.} . 

It will be noticed that, if n be not greater than 9, or if the radix of the 
scale of notation be not less than n + 1, Pm = 2**~ \ 



7356. (Professor Wolbtenholmb, M.A., Sc.D. Suggested by 
Quest. 7286.) — ^At each point P of a given curve is drawn a s^aight line 
U, makinga gven angle with the ttrngent at P, and a straight line Y, 
such that U , v are equally inclined to the ordinate at P ; prove that the 
point of contact of XT with its envelope is the projection upon U of the 
centre of curvature at P, and that the point of contact of Y with its en- 
velope is the projection upon Y of the image with respect to the tangent 
at P of the centre of curvature at P. [That is, if O be the centre of cur- 
vature at P, and OPO' be a straight line bisected in P, then, if OL, OIVI 
be let fall perpendicular to XT, Y respectively, L, M will be the points of 
contact of U, V with their envelopes.] 



Digitized by VjOOQIC 



65 




Solution by Profs. Nash, M. A. ; Nilkanta Sarkar, M.A. ; and oth^t, 

1. Let P, P' be two adjacent points on 
the curve, PR, P'R the positions of the 
line U, and O the centre of curvature at P, 
then ultimately P, P' may be considered as 
points lying on the circle of curvature. 
Now, since Z OPR = Z OFR, 
therefore PR'OR are concyclic, therefore 

ZPRO«ir-ZPFO. 

But in the limit PP' is a tangent, and PFO 
a right angle, therefore OR is at right 
angles to PR. 

2. The line Y makes with the normal an 
angle iir~ a — 2^, where <p is the inclination 
of the tangent to the axis of x. 

In the figure, 

POP'- 9<p, QPR - iir-o-24», 
QT'R = iir-a-24> + 25^, 
PEP' = RFQ'-RPQ-POF = 5^, 
RP _ sinRFP 
PP 
and PF 



therefore 




sin84> 
OP5^==OPsin8<^, 
therefore RP - OP cos RPQ ; therefore, if PO' - OP, R is projection of 
upon V. 



8548. (Asparagus.) — Two straight lines turn about two fixed 
points 0, O' with angular velocities which are as I : 3, both straight lines 
coinciding with OO' initially ; if P be their point of intersection, prove that 
the envelope of a straight line drawn through P at right angles to OP 
will be a parabola whose directrix passes through 0' and is at right angles 
to 00' and whose latus rectum is twice 00'. 



Solution hy R. F. Davis, M.A. ; G. G. Stobb, B.A. ; and othen. 

Produce 00' to S, so that OO'-O'S ; 
and let zPOS-0, 

Z PO'S - 36, Z OPO' - 20. 
Draw PZ, O'Z perpendicular to OP, 
00' respectively. Then, since O, O', 
P, Z are concyclic, 

ZPZO' = POO'-e, 
and zSZ0'-0Z0'-0P0'=2a. 

Hence PZ bisects the z SZO', and therefore envelopes a parabola with 
focus S and directrix O'Z. 




Digitized by 



Google 



66 

8607. i^- Curtis, M.A.) — Between two curves, /(a:, y) — and 
^ (Xf y) s 0, of degrees m and n, show that there can be drawn mn straight 
lines of a given length /, parallel to a g^yen straight line, from the curve 
f{xy) « to the curve ^ {xy) — 0, and mn others from ^ {xy) = to/(a;y) 
= 0. [The lines must be drawn in a given direction, say from left to 
right.] 



Solution by Rev. T. B. Te&ry, M.A. ; Professor Bbtbns ; and otken. 

As the degree of the curves is not altered by changing the axes, we may 
take the axis of x parallel to the given straight line. 

Since /is of the m^^ degree, and of the it^, there are mn points of in- 
tersection of the curves f(x, y) = and ^(2:—/, y) ■» 0, therefore mn 
straight lines can be drawn from f to (p parallel to axis of x and frx>m 
left to right. Similarly, the other result. 



8405. (F. 0. Wage, M. A.)— If aj + flj + Oj + .. . + a^ « «, prove that 

(t-')"(T-'r {i-)"<<"->-- 



Solution by R. F. Davis, M. A. ; Rev. T. R. Terbt, M.A. ; and othert. 
If a|, Oj, a„ be positive integers, take 01,03, an quantities each 

equal respectively to - — 1, ~ — 1, — — 1 , 

€ti 02 an 

in all « quantities whose sum = tw— » = (»— 1) ». The inequality of the 
question is the expression of the fact that the Arithmetical mean of these 
quantities is greater than the Geometrical mean. 



8557. (Professor Mathews, M.A.)— If 

^+i^ = l and ^+C -1 

are two oonfocal ellipses, such that polygons of r sides can be simul- 
taneously inscribed in the second ellipse and circumscribed to the first, 

prove that a - g^sn ^^"^)^ , i - *' en — . 



r 



where the modulus of the elliptic functions is equal to the eccentricity el 
the inner ellipse. Verify the above when r » 3, 4, 6 respectively. 



Digitized by VjOOQIC 



Solution by John Griffiths, M.A. 

The solution of this question is contained in my paper "On the 
Derivation of Elliptic Function FormulsB from Confocal Conies," given in 
the Proceedings of the London Mathematical Society, Vol. xiv., p. 47. 



8420. (Emily Perbin.) — Prove that the axis of perspective of a 
triangle and its pedal triangle is the common radical axis of the ciroum- 
circle, nine-point circle, and self-conjugate circle. 



Solution by H. 0. S. Davibs, M.A. ; W. J. Babton, M.A. ; and othen. 

Let ABC be the triangle, H its 
orthocentre, DEF the pedal tri- 
angle ; and let EF meet BO pro- 
duced in P. Then, since BFEO 
is a cyclic quadrilateral, 

PB.PO-PE.PF 
and PA2-PB.PC + AE.AC. 
Hence 

PB.P0=PA9-AH.AD=PH2-HD2 + AD2-AH.AD-PH2 + AH.HD. 

Thus P is a radical centre of the three circles of the question. Similarly, 
if FD, OA meet in Q, and DE, AB in R ; then Q, R are also radical 
centres and the axis of perspective PQR is the common radical axis of the 
three circles. 




(R. Curtis, S.J., M.A.) — Find (1) the locus of a point in a 
material lamina, such that if the lamina were moving without rotation in 
its own plane, and that point were suddenly fixed in space, the ensuing 
rotation of the lamina would be given ; (2) where the point should be if 
the rotation produced is a maximum ; and (3) where the point should be 
in order that the loss of kinetic energy would be given. 



Solution by Professors Steggall and Betens. 

Taking the axes of x, y through the centre of gravity, and that of x 
parallel to the direction of motion, u being the initial velocity, tv the 
final velocity about the point a:, y, arrested ; then uy^w {k^ + ar^ + y^), 
whence locus (1) is a circle ; point (2) is given by a; = 0, y = k, as may 
be seen geometrically ; locus (3) is given by «*— (Jk^ + aj'^ + yv^ m;2 « con- 
stant, or -TJ"* la^ ~ ^» where b is arbitrary. 



Digitized by VjOOQIC 



68 



7707. (Profeasor 8. Malbt, F.B.8.}— Prove that 



rlog Bin e jlog tan e-log A (9)} 

J^iogoos e {log cot ^-log A (fl)} 
AW de^i^K, 

rlOgA(e) {log A («) -log SID e cose} 
AW ^*>-iKlog*log*' + i»K'log*', 

wliereA(a)s(l-Jt«8in«a)*, 1«+*^=1, and K and K' are complete 
elliptic integrala of the first kind with moduli lespeetiYely k and i/. 



Solution by D. Edwabdbs ; Sabah Mabks, B.Sc. ; and oihert. 
Let « « amu, A = ~^, B = 2ife^4-» C = it^; we have 

log-£-mM--i::^coe^-il^co8?^-4i=2?cos?^-&c.. 

IK 1 

log an f* A* « 4K log — — i»K', 

logcn»-log*B.l±fcoBf-i;-^,co.f.i;-±Jcoa«^-&c.. 

logdnit = logC+4 / j£_coe^ +^_^coe^+&o. j. 
Hence I [(logsn<()'-log8ni(logcn«dni(](ft( >-— E I log ^] 
-21og^j^logsni«<i« + KlogtBClog^+K(l + -l. + l- + &c.), 

rK 

or, substituting for A, B, 0, and I log sn m <^ their values, this reduces to 
the stated result. In the same way we find 

i>K 

I [(log en «)'— log en u log sn m dn m] <2m — ^ifi K. 
Jo 

And putting K~ m f or m in this, and subtracting from the first integral, 
we get 

-log;fc»riog ^"^''" tfm- r[aogdnf»)«-logdnf#logsnf#cnf#]«ff#. 
Jo cn« Jo 

But ^\og''^^^^^du^[^\og-^^du(K^uiotu)^(i, 
Jo CUM Jo sntidntf 

and therefore, &c. 



Digitized by VjOOQIC 



69 

8977. (Professor Haughton, F.R.S.)— The form of the Terrestrial 
Radiation Function has been proved to oe A (B—Bq)'* = a, where A, e©, 
n are unknown parameters, and e, a are g^ven by observation. The mean 
monthly observations at Greenwich, ext^ding over thirty-six years, g^ve 

January, A (38-9-eo)" = 21-4 ; February, A (40-4-eo)*» = 36-6 ; 

March, A(42-8-eo)»» - 66-9. Find e^,, A, and n. 



Solution by A. B. Johnson, M.A. 

log4«lr:®o /iogl?l=e. . log ^±^ /w^, 

"38-9-eo/ *40-4-e„ *21-4/ ^3o-6' 

1*5 / 2*4 21981 

or, as a tentative approximation, — -— / — -- — -— = — - — , 

' *^*^ ' 79-3-2eo/ 83-2-2eo 19718* 

or 162-6-400 - ^|f|] x 3-9 ; therefore %^ - 3716. 

23177 

Substituting this value in the equation 

^log g'^^e^ - 1^8^ i^l' ^« ^^« « = ^'^^^ - [l"-91341]. 
Lastly, find A from the equation 

log A+n log 6*66 = log 66*9, whence A = 13'63 = [1-13129]. 

On using these values for A, %y ft, there result the values 21*40, 36*63, 
66*90 in the left-hand sides of the observation equations, so that, to judge 
by the standard of accuracy taken, the results are sufficiently near the true 
ones to dispense with further approximation ; summarily, 

A = 13*63, n - 0-819, % = 37*16. 

[The Pkoposbr remarks that this question involves, for the first time, 
60 as an unknown quantity equal to the control temperature that guides 
radiation.] 



5955. (Professor Sylvester, F.R.S.) — By a Cartesian Oval in space 
let us understand a curve the distances of whose points from three fixed 
points are linear functions of each other, or, wluch comes to the same 
thing, is the intersection of two surfaces of revolution, described by two 
plane Cartesians having a focus in common. Conversely, when two points 
can be found whose distances from any point in a space-Cartesian are 
linear functions of one another, let them be termed foci. Required to 
prove, that the locus of such foci is a plane curve of the 3rd deg^e. It 
will be observed that this curve for Cartesians of double curvature is the 
exact analogue of the three foci in a straight line for plane Cartesians. 



Solution by W. J. C. Sharp, M.A. 

Let the Cartesian in space be the intersection of the surfaces generated 
by the revolution of the plane Cartesians tnr + r^ ^n and mV + /' ■» n' 
about the lines joining their foci ; let the axes be taken so that the quanti- 

VOL. XLVIl. I 



Digitized by VjOOQIC 



70 

ties r, r', r" are measnred from the points (0, 0, 0), (A, k, 0), and (A', 0, 0) 
respectively. Then, if (a?, y, z) be any point on the curve, 

r"2«a:S + ya + «2.2yar + A'2, 
and therefore 2cc ^ ^^^^^/^^ ^ r»(l-m-»)4-2mVr.^A-2-n-^ 

= Ar2 + 2Br + C, say, 
where A, B, and are constants ; and 

2 _ r^-r'^ + h^-^k^-2hx {il'-m^)r^ + 2mnr + h^ + Ji^''n^^2hx] 
^ r^ " k 

— AV + 2BV + C, say, where A', B', and C are constants. 
H p8 = a;2 + y2 + g2_2|a:-2i7y + p + i7«, 

p«-r»-|(Af3 + 2Br + C)-u(AV2+2BV + C0+p + i,a 
= r»(l-A{-A'i7)-.2r(B{ + B'i,) + p + i,2-C|-C'u 
■■ (Hr + K)2 (where H and K are functions |, 17 and constants), 
if (l-A^-A'„)(? + ,r'-C{-C'i,) = (B{+B'i,)», 

and therefore if ({, ij) lie upon this plane circular cubic, p = Hr + K, and 
if (^', ij')be any other point upon the plane curve, o* = H'r + K', where 
H' and k' are functions of (I', V)* ^^d constants, ana therefore 

H'p-Hp' = H'K-HK', 

and therefore, &c. The three given foci lie upon the circular cubic. 



8924. (Captain Bbocabd.) — Former I'^quation des paraboles 
tangentes aux deux bissectrices de chaque angle du triangle et aux per- 
pendiculaires aux cdt^s de cet angle en leurs milieux. Ces coniques 
admettent, comme on sait, pour directrices les medianes et pour foyers les 
sonmiets A", B", C" du second triangle de Brocard. 



Solution hy Professor Boubals ; B. F. Davis, M.A. ; andothert. 

Prenons pour axes les c8t6s A Car, ABy du triangle. L' Equation 
g^n^rale des coniques tangentes aux deux bissectrices AE', AE'' est 

X (y2-a;2) + (y + fxxi-y)^ = 0. 

Elle repr^ntera une parabole si x = /i-— 1, ce qui donne 

(jii/ + xy+2v(fAx + t/) + p^^ 0. 

Les perpendiculaires aux milieiix des c6t^s AC, AB (ou mediatrices) ont 

Sour Equations ar + ycosA— i* = 0, arcosA+y— ^u = 0. La parabole 
evant Itre tangente h ces deux droites, on a les conditions 

v8in»A + d(/i-cosA) « 0, i^ sin^ A + c (1 — /i cos A) = 0, 

d'oii Ton tire ^ « ^±A£2L4, ^ « _J^*£_. 

b + c cos A b + c cos a 



Digitized by VjOOQIC 



71 

L'^quAtion de la parabole est done 

[(tf + * cos A) y + (* + c? cos A) a>]* 

— 2bc[{c + i cos A) a? + (b + tfcos A) y] + b^e^ = 0. 
Si maintenant Ton prend le triangle ABC pour triangle de r^f^rence, 
les f ormules de transformation 

^ ^7_ M_ 

aa + bfi + ey aa + bfi + ey 

donneront pour 1' Equation en coordonn^es trilineaires de la parabole 

L' equation aa— (efi + by) cos A = repr^sente la corde de contact des 
deux tangentes rectangulaires AE', AE'^ D'oti Tidentit^ 

^ + c2— flS « ^i,c + be) cos A « 2be cos A, 
qui exprime que cette corde passe par le point A". 

Autre verification : — La symediane AA" doit dtre perpendiculaire k la 
corde de contact. Or, la symediane a pour equation efi—by ^ 0. On 
doit done avoir identiquement (Casey, A Treat, on Conie Sect,, p. 63), 

(^2- c2) cos A- cos A (be- *«) - cos B (— oi) — cos C {ae) — 0, 
ou (J2— (j2)cosA + a(ico8B— <?cosC) — 0, 

ou (i2-(jS)(*2 + ^_^2 + «2j + ^_J4«0, 

ce qui est bien une identity. 



8658. (W. J. C. Shabp, M.A.) — BooLB obtains the result 

^(n-l)(|--2)^_3j3.^^ 

by a symbolical method ; show that it may also be obtained by ordinary 
methods. 

Solution by Professors Bbyens, Stone, and othen, 

Cette formule est certain pour » =» 2, n — 3, etc., et nous aliens k 
demontrer que si elle est verifi^e pour le cas (n) elle sera pour le suivant. 
Supposons-nous que pour (n— 1) integrales on aie 



III 



,..Xdx^-^ 



nous aurons f f f f ... Xdx « [ ^* f (( **~ — X^^**~* " 

[^3? / ix*'-^{xdx-{n''2)x''-^{Xxdx...±{Xx»'^dxl 



Digitized by VjOOQIC 



72 

Mais, appliquant la m^thode d'int^g^tion par parties, nous aurons 

-(«-2) (dx.z^'^iXxdx:^ -x'^-^lXxOx^iXaf^'^dx, 

(n-2)(n-3) C^^^n-i [Xx^dx ^*^ s^-^{Xx^dx^^ [Xx^^-'dx, 
1.2 J J 1.2 J I.2J ' 

et ainsi de suite nous aurons finalement, 

:h{dx{Xx*"^dx '»±x{Xa^'^dxT{Xa^-^dx. 

Bemplafant ces valeurs en (1), le coefficient de | Xi:**'^^^^ sera f apr^ 

avoir s^par^ le facteur commun -— ; — — J , 

1.2. 3... (rt— 1)/ 

-l.(n-l)- ("-;)(-^) ...=F(n-l); 

mais, d'apr^s une propri^te bien connue, 

_H.(„_1)_ <JLrlK^ ... =P(„_1)±1 = 0, 

1 • ^ 

suivant (n — l) pair ou impair, et par suite la formule (1), deviendra le 
th^r^me propose. 

fLet T^''^' therefore f f ...Xdx** = y, 

and, by integration by parts, 
{XxPdx={xP^dx^xP'^^^pxP''^J^^p(p-l)xP'^^-&c. 

Now, if all the values 0, 1, ... « — 1 be given to p, the result of eliminating 
the successive differential coefficients of y will be the given formula. I 



Note on a Probability- Qubstion. By the Editor. 

The problem in Question 7624 having excited much interest, we give 
space to the subjoined additional remarks and developments in regard 
to the solutions and notes that have been published in our previous 
volumes. 

Mr. Simmons' solution seems to Mr. Putnam to be ** only applicable when 
A is the first man to enter the train, and B the second. In that case there 
are g—l vacancies in the car A has entered, and pg—l vacancies in the 



Digitized by 



Google 



73 

train. The chance is then {g-l)l ( W— 1) . It is impossihle, however, to 
find any other condition in which this will prove true. Instead of throw- 
ing out tn, Mr. Simmons' solution assumes m ^ pq—l. By the conditions 
of the prohlem, the chance under this assumption is the least possible. 
Although Mr. Biddls's solution is intricate, I do not see how it can be 
avoided, if the solution is to follow mathematical principles. 

" If we may eliminate any of the data, let q be thrown out. Then, if 
there are vacant seats anywhere, the carriage in which A is seated will 
have an equal chance with the rest, and the number of vacancies belong- 
ing to it will be w/p, and the chance of B*s taking that carriage will be 
l/nt X m/p = l/pf or the same chance there will be that B will enter the 
same carriage A is seated in without reference to vacancies.'* 

In further reply to Mr. Simmons, who, in Vol. xlvi., p. 37, has g^ven 
what he considers to be test examples to prove his case, — viz., that m, in 
the question, is an immaterial quantity, — Mr. Biddlb wishes to say tiiat 
he is equally convinced that ^e subject cannot be so summarily dis- 
missed." A very similar (not to say identical) question occurs in Mr. 
Whitwobth's work on Choice and Chance (No. 229, p. 234) ; but, although 
answers are g^ven to most of the questions, none is given to this, which 
would scarcely have been the case if the author had considered the matter 
so simple as Mr. Simmons does. 

'* One thing is clear, the particular passenger. A, must not be re- 
garded as being selected at the time the new-comer, B, enters the train ; 
nor, on the other hand, must he be regarded as invariably the first pas- 
senger who takes a seat. So far as our information goes, he is simply 
one oipq—m passengers, who, before the train began to be filled, had an 
equal chance of occupying any carriage and any seat therein. We must 
therefore consider the number of ways in which pq^m passengers can be 
distributed in p carriages, each containing q seats, and the probability of 
each kind of arrangement. We must next take the total number of 
occupied carriages given in the full tale of these arrangements, and find 
what proportion of them have one vacant seat, what two, what three, and 
so on. These wiU be the chances, as regards vacant seats, for any par- 
ticular occupied carriage, such as that which A will occupy, because the 
a priori chances are the same for all. And here it may be observed, that 
it is only a posteriori that we can say that A has twice as much chance 
of being where there are two passengers as where there is only one. 

The rest is easy, and the examples given by Mr. Simmons will equally 
serve the purpose now in view. " Let us take the particular instance of 
3 compartments, each of 3 seats, and test the numerical results." 

Here, taking to represent a vacant seat, and X a passenger, the pos- 
sible contents of any one carriage will be 

(a) {b) (c) {d) 

XXX XXO XOO 000 

(i.) *' Suppose that, besides A, two other passengers, P and Q, are 
already seated." 

Then the essentially different ways in which the train may be made up 
can be represented as follows, the probability of each being given at the 
end: — 

{a + 2d) XXX 000 000 

(b + c + d) XXO XOO 000 



ti 



(3c) XOO XOO XOO ^ff. 



Digitized by VjOOQIC 



74 

*' In ihefirst arrangement^ the first passenger entering decides the carriage, 

2 1 1 
then — ^ s — ^ the probability that the other two join him. In the 

8.7 28 *^ "^ ^ 

second arrangement, the two first passengers may enter as they list ; if 
they go together, of which the probability is f , the chance that the third 
enters another carriage is f ; if, on the contrary, they enter distinct 
carriages, of which the probability is f , the chance that the third joins 

one of them is 4. Therefore r-^ + ^ = -^ — the probability of the 

^ 8.7 8.7 28 *^ *^ 

second arrangement. In the third arrangement, the chance that the 
second passenger does not join the first is f , and that the third does not 
join either isjf. Therefore f . f =« -^ = the probability of the third ar- 
rangement. We now see that, where the first arrangement occurs once, 
the second occurs 18 times, and the third 9 times. Consequently, the 
state (a) of a single carriage occurs only once, whilst {b) occurs 18 times, 
(e) 45 times, and {d) 20 times. Disregarding (d), because A cannot be in 
a carriage where the seats are all vacant, and sdso (a}, except for addition 
to the denominator, because B cannot enter where no seats are vacant, 
we find that, of occupied carriages, the probability that the state shall be 
(fi) a I) ; that it shall be (e) » ff . Therefore, since there are, in all, 
6 vacant seats for B to choose irom, the probability that he joins A 
'^ H'h+H*i=^-ht instead of -^ or i, the probability given by Mr. 
Simmons." 

(ii.) ** Suppose that, besides A, there are already thre^ other passengers, 
P, Q, and R." 

Here the specific arrangements are also three in number, and their pro- 
babiHties as follows : — 

{a+e + d), XXX XOO 00 0, ^ 

(b + 2e), XXO XOO XOO, if 

(2* + (^, XXO XXO 00 0, ^• 

From which we gather that the chances of any carriage being as (a), (fi), 
(c)f (d) are as 4, 30, 40, 10. Consequently, as the vacant seats are 6 in 
number, the probability that B joins A » ^J.| + f$.f « if, instead of 
ii o' i» the probability given by Mr. Simmons. 

(iii.) " Moreover, if there had been only one passenger P besides A," 
the specific arrangements would be two in number, and their probabilities 
as follows : — 

(* + 2rf) XXO 000 000 I 
i2e + d) XOO XOO 000 f. 

" Hence we see that the chances of any carriage being as («), {b), (c), (d) 
are as 0, 2, 12, 8. Consequently, as the vacant seats are 7 in number, 
the present probability that B joins A = A*f + Tl-f — tv> instead of 
il or ^, as given by Mr. Simmons. 

"In fact, the only case in which the same probability is certainly 
arrived at by both methods, is where A is the sole passenger in the 
train when B enters. But if it was a mistake, as Mr. Putnam intimates, 
to say that the probability would be the same according to either method, 
in the case of only one vacant place remaining, the mistake arose from 
judging of the probabilities of A's position by an a posteriori process." 



Digitized by VjOOQIC 



75 



8685. (The Editor.) — Given an angle at the base of a triangle, the 
sum of the two sides, and the distance between the given angular point 
and the point of contact of the escribed circle touching the base ; con- 
struct the triangle. 

Solution by the Proposer. 

Let L and / represent the two given lines. 
Draw the Z EBD = the given Z , making EB 
= L and Bl) = I. Produce BD to F, so that 
DF = L + ;; thenBD + DF = BF = L + 2/. 

Also produce BE to H, so that BH = BP. 
Bisect BH in I : draw IF ; and from I apply 
IM = IH to meet FE produced ; also draw 
EN II MI, and through N draw KC || BH, and 
CA 11 EN. The A required is ABC. For, 
by construction, Z ABO = given Z , and BD = I, 

Moreover NK : IH=FN : FI=NE : IM=NE : IH, .-. NK«NE=«NC; 

but AC = AE; therefore AB + AC = EB « L. 

Again, CF = CK = 2CN - 2AC, .-. BO + CF - BC + 2AC = L + 2/; 

but AQ + AC = L; therefore, adding, AB + 3AC + BD + DC - 2(L + 0; 

but AB + AC -I- BD = L + / ; therefore, subtracting, 

2AC + DC = L + /= AB + AC + BD; 

therefore AB + BD » AC + DC. 

Now, it is well known that the angular point opposite the base of a 
triangle and tht point of contact of the escribed circle with the base bisect 
its perimeter. Therefore D is the point of contact of the escribed circle 
with the base BC. 




8501. (H. Knowles, B.A.) — In any triangle, show that 

4coi^Acos'B-cos(A-B)(3cosAcosB-sinAsinB) =cos«C. 



Solution by Isabel Madison, B.Sc. 

Identically, Ax^- {x + y) (3ar~y) = {x^y)\ and putting x • 
y — sin A sin B, the required theorem is proved. 



cos A cos B, 



(F. R. J. Hervet.) — Find in how many ways n lines of 
verse can be rhymed, so as to have r different rhymes, and no line un- 
rhymed ; and show that, in the case of the sonnet^ the numbers of ways 
with 2, 3, ... 7 rhymes are, respectively, 8177, 731731, 6914908, 12122110, 
4099095, and 135135. 



Digitized by VjOOQIC 



76 

Solution by the Proposeb. 

Let ^ (ft, r) denote the required number ; we shall obtain all the case 
as follows. Take aU the cases of » - 1 lines with r rhymes, and append 
to each a line rhyming with some that precede; this gives r^ (n — 1, r) 
ways. Again, take all the cases of n ~ 1 lines with r distinct endings one 
of which is unrhymed, and append to each a line rhyming with the odd 
line. The number of these is — (n — 1) 4> (n — 2, r - 1). 

Hence 4> («, r) == r4> («— 1, r) + (n— 1) ^ (n - 2, r — 1). 

Form a table, as below. Write the first row (of units), and first cypher 
of each subsequent row (forming an oblique column) ; and continue these 
rows both ways, satisfying the relation ^r (« + 1) = r4>r« +«<^r-i (»— 1) ; 
^rW denoting throughout the number of r^ row which falls under the 
number n : thus ^r^ = ^ (»> ^) so long as n > r. The values of ^r^ from 
« = r to » =s follow an obvious law, which, it is easy to show, must 
hold for each subsequent row ; we have, then, 0^0 = (-l)*""^/r!. 






1 


2 


3 


4 6 


6 


7 


8 


9 


10 


1 


1 


1 


1 


1 1 


1 


1 


1 


1 


1 


-/2 


-1 


-1 





3 10 


26 


66 


119 


246 


601 


/3! 


/2 


1 


1 





16 


106 


490 


1918 


6826 


-/4! 


./3! 


-/2 


-1 


-1 








106 


1260 


9460 



The complete solution of the equation ««♦!— rtt« == ^, ^f being a 
rational integral function of the k^^ degree, is 

ii^=:Cr«-{/(r-l) + A/(r-l)2+...+A*/(r-l)**»}if^«Cr--x«,8ay; 
whence, if an integral function be required, the only solution is Un— —x*** 
If the given equation be Un+i—run = «***^4^, we have Un/a" as before, 
giving 

Un = 0*»-a*»x'»> where x» «= « (1 + «A +...+«* A*) \pn and « = al{r^a). 
Again, if Kn+i— rw« = the sum of several terms of the form a'»*^if^ 
(which denote by 'Xa*^*^^)i we may find Mm by solving as many equations 
of the preceding form and adding the results ; giving a solution of the 
form «n = Or** — 'Za**xn, 

To find ^r-¥\n we must solve the equation «n+i — (»*+ 1) «n = n^r (♦•—I). 

Thus 4»in- 1, ^»«2'»-i-(fi + l), 2<p^n » 3'»-»-2»-»(« + 2) + »2+„+i^ 

64>4» « 4»»-i-3 . 3»»-8 (» + 3) + 3 . 2'»-» (ff2+ 3» + 4)-(n3 + 2n+ 1) ; 

generally, it will be shown that 

r ! iprn - r~-Cr,i(r-l)»-iP + 0r.2 (r-.2)»-2Q-... ± Or.r-iT, 

[otherwise, (r- 1) ! 4>r « = r«-*-Or-i,i (r- 1)»»-*P + ...], where Or,m is 
the coefficient of x^ in (1 +«;)♦■, and P, Q, ... T are functions of n with 
unity for coefficient of highest power ; these must first be determined. 

Unity, in the first term of the above expression, may be regarded as 
the starting-point of a new series of these functions, each formed from 
the preceding, which are to appear in the 2nd term of ^rfi» the 3rd of 
^r+2> &c. ; the law of formation being 

/*« = {l+rA/A:+...}{nA-i(n-l)}, 



Digitized by VjOOQIC 



17 

which, inverted, gives {l^rAl k)fkn ■» w/i.i («-l) (1). 

Thus /o» = 1, /i » = w + r, nfi (»- 1) « n'' + (r- 1) n, giving 

giving /j n = w' + (3r— 3) «*- + (df^-Zr + 2) w + r^, and so on. 

Thus written, in powers of «, the law is not evident; hut we may 
observe (and, for the moment, assume) the relation 

/k« = ^*-i(/j-l)+r/A.i« (2). 

From (1) and (2), A/**» *= ^/t-i«> whence A%« = k (A;— l)/*.2ft, and so 
on; also, from (2), /tO = r/^-iO =» ... == r* ; hence 

...+w(n-l)...(ft-A;+l). 

This is the function sought ; for it will he found to satisfy (1), which, 
/A_iWng supposed known, is an equation of differences for finding/*, 
having only one such solution. Denoting it in future by/(r, A;), assuming 
the form previously suggested for <pr, and applying the general process 
to form <t>r*h ^3 have 
rl iprn « r'»-Cr,i(r-l)«-i/(/-l, 1) + a,2(r-2)«-2/(r-2, 2)-... 

... ±Cr,^_i/(l,r-l), 

r!4»r*in = A(r + l)~-r.r-2/('', 1) + ^ C.,i(r- 1)^-8/(^.1, 2) 

-^a.2(r-2)«-V(r-2, 3) + ... T -^C.,..i/(1, r), 

(r+1)! ^r*i« - B(r+l)»-a.i,ir»-i/(r, 1) 

+ Cr+i,2(r-l)'-2/(r-l,2)-... T Cr*i.r/(1, r), 

(r + l)l0r*lO - (-l)*- «=B-Cr4l,l+aM,2-... =FCrU,r; 

whence B = 1, and the above formula is completely established ; n "being 
any integer, positive or negative. It may albo be written 

(r-1) \<t»rn^ r«-i-0r-i,i(r-l)«-2/('--l, 1) 

+ a-i,2(r-2)'»-»/(»--2,2)-.... ±/(l, r-1). 

If we collect from the last the complete coefficient of a given fac- 
torial, «(n— 1) ... (n-w+l), we shall find it (by help of the relation 
Cr.mwC«w,«« Cr,fnCr-m,t) to bc ± Or- 1, mA*-"»-i l"""*-! (derived from 
l»»-»M-i^ 2"-*"-*, ...). Hence, restricting n to be not < r, we may 
easily throw <l>rn into the form 

^r« = A(»-)0»-Cn,iA^'-»)0'»-l + On,2 A^*-*) 0«-«- ... ± On,r-l A^D O"-***!, 

where A^*") means A^jrl. From this, and from the fact that (prn 'k if 
«>r<2r, it may be inferred that A^''JO*'"^^, i? being const mt and > 0, is 
an integral function Fr, of degree 2/?, containing the factors r(r + \) 
... (r+j3), (which is otherwise known) ; and that (pr i^-^p) or ^(r+jo, r) 
is the (r+^)*^ difference of the first zero value F(— ^). [This result 
is easily connected with the meaning of A^**) 0" previously found.] 

The case n = 2r is noticeable. By the construction of the table, 
iPr{2r) = the product of odd numbers 1.3... (2r— 1). This may also 

VOL. XL VII. K 



Digitized by VjOOQIC 



78 

b^ inferred as follows. Suppose it specified that p are lines to rhyme 
together, g* otiiers to do bo, and so on ; where j3 + ^ + . . . « «. The number 
of ways will be « !/ {jp ! §' ! ...} if jp, q ... are all different ; but, if fc of them 
are equal, we must divide by A; ! ; and so on. Now, in the above case, 
the lines necessarily rhyme in pairs; hence 4> (2r, r) = (2r} !/(2''.r !). 
It follows that the coefficient of r^ in FriB I (2P.p !). 





2 8 4 6 


6 


7 


8 


9 


10 


11 


12 


IS 


14 




1111 


1 


1 


1 


1 


1 


1 


1 


1 


1 




8 10 


25 


66 


119 


216 


601 


1012 


2035 


4082 


8177 






16 


105 


490 


1918 


6826 


22935 


74316 


235092 


731731 










105 


1260 


9450 


56880 


302995 


1487200 


6914908 














915 


17325 


190575 


1636635 


12122110 


















10395 


270270 


4099095 






















135185 



8820. (Charles F. Lodge.) — ^A mirror, measuring 38 inches by 22 
iDches, is to have a frame of uniform width, whose area is to equal that 
of the glass ; show that the width of the frame is 5| inches. 



Solution by D. Biddle ; H. L. Orchard, B.Sc. ; and others. 

Lines drawn perpendicular to the sides of the mirror, from its centre, 
divide not only the mirror itself (w), but its frame (/), into quarters. 
Moreover, \m^if\ iw» = 2x3x (6^)2 ; therefore ^m + i/ = 4 x 3 x (5i)-. 
But 4 = 3 + 1, and 3 = 2 + 1. Consequently 1 x 6^ = the width of frame 
required, in inches. 



8813. (J« Gripfiths, M.A.) — If we have 
f{x) - A^ + 2A2T2 + 2h^x* +... ad inf., ^ (a:) « 1 - 2B2J^'- ^B^s^^... adinf. 

where A© - 1 + 22(-)*cn2 ?5, Bj = 2 (-)* en — -1- sn* — , 

n n n 

Aj = A;2 2 (- )• en' '^bh^'^, B4 = 2 (-)' cn^ ^ ^ sn* *-5, 
n n n n 

A4 - A;4 2 (-)• cn2 !58n4?5, B« =. 2 (-)' cn^ *-5 ^ sn«^, 
n n n n 

and a is a number from 1 to «— 1, show that /(a:) . <f> (.r) = A©. 



Digitized by VjOOQIC 



79 

Solution hy D. Edwardes ; Sarah Marks, B.A. ; and others. 

Referring to Mr. Ghifpiths' Paper {Ptoceedinga of London Math. Soc,f 
Nov. 12, 1886), we have 

Jy « 8ii(Mm, a), X = 8n(w,^'), «o= — » ^o - -g^ K, 

mod. A:, * = 1 to w-1, < »« 1 to «, 

M = nsn^*-^ + nsn^?^ K, A - k^" nsn* ?^ k] 
/» 2n 2»» j 

Ma arn-irM( (-)'cn2?-. ^ 

* (l-A'^a^y l->l«Bn»*-5:.A 

^ /» «^ 

= ^I1:=:^{A, + 2A^»+2A^+ }. 

(1-AV)* 

Changing y, a? into 1/Ay, I/Aj:, we have 

M^ ^ (1-A'V>* 3 „ ^ ^ n 

'"•"-''r -"T[-=/(-t)j: 

= 11:^^!^{1-2B^«-2B,:.^- }. 

Hence, M-A/A:* =/c.^ar, and, when a? and yare small, M'a/A;2 = Aq, 
and therefore, &o. 



8117. (Professor WoLSTENHOLMB, M.A., Sc.D.) — ^Two conicoids S, S' 
have two common plane sections, and the poles of these planes with 
respect to S are the points P, P' ; prove that (1) if S' pass through P, it 
will also pass through P' ; (2) also, in this case, the following relations 

must hold (02-.3*A) (ee'-6AA') + e2AA'= 0, 6' = 27^^ (ee'-6AA'). 

[The discriminant of XS + S' is aA^ t eA-^ + ♦A^ + e'A; + A'.] 



Solution by Professor Nash, M.A. 

Let S = ax'--^by^-¥cz^-¥du:^ - 0, and let P, P'he the points (0, 0, 0, 1), 
(X, Y, Z, W), then the equation of S' is 

flA2 + *y2+«2 + rfi^ + 2u;(a;rX + 6yY + czZ+<fM;W) = 0. 

If this passes through (0, 0, 0, 1), 1 + 2W =« 0, and S' hecomes 

ar' + V + <^«'+2u;(aarX + *yY + csZ) - 0, 

which is satisfied by the coordinates ;X, Y, Z, W). 



Digitized by VjOOQIC 



80 



The discriminant of A:S + S' ia 

(k + l)a, 0, 0, aX 

0, (k + l)b, 0, *Y 

0, 0, {k+l)Cy eZ 

flX, *Y, fZ, kd 



»0, 



or 

putting 

therefore 

therefore 
therefore 



k(k + l)^abed-(k + iyabe{aX^ + bY^ + cZ^) « 0; 

aX^ + bY^' + cZ^^dF, 
abcd{k^ + dl<^+{Z-F)k^ + {l-'2F)k^'P} =0; 

A ^ e ^ ♦ ^ e' A' 

1 "" 3 3-P 1-2P" -P' 

e2-3A* = 3Pa2, ee'-6AA'= zaK 



From these we obtain the given relations 

(e*- 3a*) (ee' - baa') + ©«aa' = o, e< « 27a2 (ee'- caaQ. 

[It would be better if the coordinates of the second point P' were written 
X . Y : Z : W ; so that X, Y, Z, W may involve an arbitrary multiplier ; 
otherwise the equation 1 + 2'W «» will seem to limit the position of P'. 
The general form of S', when S is given, is 



where (XiY^ZiWi), (X^YjZaWa) are the two points P, F.] 



X 



rfS 
dx' 



...), 



8B69. (Professor Stbgoall, M.A.) — A shot of mass m is fired in 
vacuo from an air gun of length /, with a charge of air that at normal 
pressure p would occupy a volume v ; this air initially occupies a length b 
of the barrel of the gun. Show that the time of passage along the barrel 
and the velocity with which the shot leaves the gun are gfiven by the 

equations T = {l + il^bi) ^ (S^w/m)*, V = (1 -bil2fi){opvlm)\ 

where b is small compared with I, and the ratio of the specific heats of 
air is faken as 1*4. 

Solution by D. Edwardes ; Charlotte A. Scott, B.Sc. ; and others. 

If P be the pressure at time t on the shot, the equation of motion is 
mS = P = Ox'i (7 ratio of specific heats), on the supposition that there 
is no communication of heat. By the Question C — pvy^. 
Hence, integrating between I and b, we have 

mV«.5j,e;(l-^), or V ^ ( ^ )*( 1 - ii ) approximately ; 

l^Yat=: ^— , 

V m J (I~i?/u;l)» 



Digitized by VjOOQIC 



81 



;* z5& 



■fterofore ('£!L\* T ^ & r -^ 



- Sit (rf - j!)»— "f-i! (Jl -48)! + 1 (i!_iS)l 



8931. (Emile ViOARifi.)— On donne deux points et A et on con- 
sid^re toutea les paraboles ayant O pour sommet et qui passent par A. 
Trouver geom^triquement le lieu (1) du point de concours des tangentes 
en A et O ; (2) du point d* intersection de la normals en O et de la tan- 
gente en A (ces deux lieux sont tangents en O) j (3) du point d' inter- 
section de la tangente en O et de la norraale en A ; (4) du point de con- 
cours des normales en O et A (ces deux lieux sont tangents en A). 




Solutions by (I.) Professor Schoutb ; (II.) R. F. Davis, M.A. 

I. In the diagram Pj, Qg, Rj, S4 are the 
points that describe the lour loci, when 
the axis a of the parabola through O and 
A rotates about O ; M is the centre of the 
segment OA, O of the segment A'A ; AL^ 
is perpendicular to the rotating axis. 

(1). As the locus of Lq is the circle 
described on OA as diameter and 

OPi iALo, 

the locus of Pi is the circle described on 
OM as diameter. 

(2) The locus of Q^ is the circle described 
on A'O as diameter. 

(3) and (4). The couples of points Pj and R„ Qj and 84 are so situated 
on radii vectores through 0, that the segments P1R3 aod 0.^84 appear 
from A under right angles. Therefore the loci of R3 and 84 are the curves 
that correspond to the loci of Pi and Qg in the involutive quadratic trans- 
formation of the points on radii vectores through and conjugated to one 
another with respect to the point circle A. 80 we find for these loci uni- 
cursal circular cubics with the isolated point 0, that touch in A the per- 
pendicular to OA. The nine points common to these two cubics are O 
counted four times, A counted twice, the two circular points at infinity, 
and the point at infinity common to the perpendiculars to OA. The real 
asymptotes of these curves are different ; the first intersects OA in a point 
B, so that AB = OA, the second in a point B', so that AB'= A'A. 

The particular kind of quadratic transformation made use of in this 
solution is treated analytically by Prof. G. de Longchamps ("Etude 
d'unc Transformation Reciproque," Journal de Mat/iematiques Specialea, 
1882, p. 19), and geometrically by mo (**8ur la Construction des 



Digitized by 



Google 



82 



Cubiquos Unicursales," Annuaire de V Association Frangaise pour VAvanec' 
ment des ScienceSy Congr^s de Grenoble, 1885, p. 169). 

II. Otherwise: Let A« be the 
ordinate of A to the axis of any 
parabola having () for its vertex. 
Then if, on this axis, On' be taken 
= Ow, Aw' is the tangent at A, 
and y, its middle point, lies on 
the tangent at the vertex. If 
yE be drawn parallel to the axis, 
it bisects OA in the fixed point 
E. Now (1) the locns of y is 
the fixed circle described upon 
OE as diameter. (2) if AO be 
produced to A' so that A' = O A, 
the locus of n' is the fixed circle 
described upon O A' as diameter. 
[These circles touch each other 
in O.] (3) If a fixed straight 
line through A perpendicular to 
OA meet the axis in i, then 
A»*^ = On . bn^ and bn = latus rectum. The normal at A will therefore 
bisect bn in y. Now, the locus of n is the fixed circle upon OA as diameter, 
while A* is the tangent A to the same circle. Therefore, since O, n, ^, b 
are collinear and bff «= w^, the locus of ^ is a certain curve symmetrical 
with respect to Ok. touching the circle in A. [This curve is a circular 
cubic J its polar equation is r = a (sec + cos 6) and its Cartesian equation 
y2 (x^a) = x^ (2a— ir) referred to O as origin and OAas axis of x. It has 
for an asymptote the line through E perpendicular to OA. OA =« 2/i."| 
(4) Since g bisects bfij it may be easily shown that e bisects hm. But 
O, m, Cj h are collinear, while the locus of m is the fixed circle upon OA 
as diameter, and that of c the tangent *Acto the same circle at A. There- 
fore, as before, h describes a certain curve [r=2a (2 sec — cos 0)] symme- 
trical with respect to OA. These two curves touch each other at A, and 
lie upon opposite sides of their common tangent. 




8716. (Professor Mathews, M.A.) — Prove that the real common 
tangents of the circles x^ + y^ — 2ax — 0, x' -^y^-lby = are represented 
by 2ab (x^-^y^- 2ax) = {by -ax + aby, or, which is the same thing, by 

2ab{x'^ + y^-2by) = {by-ax-ab^. 



Solution by W. J. Babton, M.A. ; Kate Gale, B.Sc. ; and others. 

By elementary geometry we have, if (xq, y^j be coordinates of the 
point of intersection of the common tangents, 

ab 



whence XQ= — yQ—- 



a-b 



Digitized by VjOOQIC 



83 

Substituting these values in the equation to the pair of tangents from 
.(a^o» Vq) to x^-¥y^-2ax = 0, viz., 

and dividing by — -, we get, for the equation to the real common tan- 

gents, 2ab (x^ + y^ - 2ax) = {by -ax + ab)\ 



8660. (By B. Hanumanta Rau, M.A.) — ^Investigate the singular 

solution of the equation ^[^"^V j-] +25fy2_a-yJ^j_o, and show 

that it is the envelope of a series of circles described on the subnormal of 
an ellipse as diameter. 

Solution by Robert Rawson, F.R-A.S. 
From the g^ven equation, we have 

^._:^i^+ 445 = 0). 

4 y y* 4 

Then, the (p) and {c) equations are readily found to be 

8yi?-17a;=F5(9a;2-16y2)i^0 (2), 

C-log{(9.»-16,^)* i 6x} T ^,^,^,,%,^,^ - (3). 

From (2), ^ = -17ar=F(9^»-16yg)* ^ JO . (4). 

dy 8y2 (9;c2- 162/2)* 

Now, (4) becomes (00 ) when y = and 4y = ± 3a;, neither of which is 
a solution of (1). 

-Again, ^[L]^ 8{^45.^17(9.^~16,^)ny ^^^^ 

d^^P I (9jr3- 16^)* {17a; ± 6 (9a;2- 16y )*} 

Now, (5) becomes (00 ) when 4y = ± Sa; and 4iX^ + 2by^ = 0, neither of 
which satisfies (1). 

The condition of equal roots of (2) and (3) fails to give a solution. 
It appears, therefore, that neither of the above recognised methods 
avails in this case in finding the singular solution. The present theory of 
singular solutions of differential equations is not, I fear, free from the 
reproach of not being in such a satiafactory state as it might be, consider- 
ing the attention which has been given to it by every mathematician of 
eminence since the days of Dr. Bkookb Taylor. Indeed it appears, from 
the researches of Sir James Cockle and others, that 

^ = 0, -^ = 0| derived from the C-cquation, 

dC d\j 



= 00, -^ — ) = 00, derived from the jt7-equation. 
P I 



dp d 

dy dp 



Digitized by VjOOQIC 



84 

The condition of equal roots of the C -equation and j7-equation can have 
but a limited application. Another solution besides (3; can ie obtained 
by the following consideration : — 

Let C + R i/CS") - (6) 

be the C-equation, where R, S are functions of x, y. 
Thej^-equation, from (6), becomes 

P-^ —TTl^ 1 ^ ^^^• 

figs " ±/(S)'"^s 

Now (6) is, therefore, the complete primitive of (7), which is rendered 
tt-i 

exact by the multiplier 1/«S ** . But (7) is also satisfied by 

S = (8). 

Hence (6) and (8) are both solutions of (7), and if (6) does not include 
(8) by giving a constant value to iC), then (8; is a singular solution by the 
usual definition. The solution (8) may, or may not, be of the envelope 
species ; this depends upon the functions R, S. 

Compare (6) with (3), then 

when S = 0, then {9x'^'-l6if^^ ± 6x ^ ./9). 

From (9), a;2 + y2 = q (10). 

Now (10) satisfies (l),andis not included in (3) as a particular integral. 
It must, therefore, be a singular solution by the definition. 

Professor Cayley, a very great authority in this matter, would I 
apprehend reject this solution as being singular. At all events, whether 
it is singular or otherwise, it is a solution of (1), and is not included as a 
particular case of (:i) by giving a constant value to (C). 

The locus of (10) is two straight lines, viz., y = ± tr, where t = v^— 1, 
but the relation of (10) to the complete primitive is not readily observed. 
It is not difficult to show that (10) is the envelope of a series of circles 
described upon the subnormal, as diameter, of an ellipse or hyperbola 
whose excentricities satisfy er+l = 0. 



8727. (B. Hanumanta Rau, B.A.)— The sides BC, CA, AB of a 
triangle ABO are produced to a, b, c, such that Cfl, A.b, Be are respectively 
equal to BC, CA, AB. Prove that the centroid of the triangle abc coin- 
cides with that of ABC. 

Solution by Rev. D. Thomas, M.A. ; C. E. Williams, M.A. ; and others. 

Let o, /3, 7 be coinitial vectors of A, B, C respectively and o', /3', y' of 
a, bj e respectively ; then a'= jS + 2 (7 — /8) and 5a' = 2a, therefore, &c. 



Digitized by VjOOQIC 



85 

8135* (I^v. T. C. Simmons, M.A.)— If Ot be the centroid, I the in- 
centre, of a plane triangle, prove that 

IGP « fR2 (1 +C08 Acos B coBC)-iRr + lr\ 



Solution by Kev. T. Gallibrs, M.A. ; G. G. Storr, M.A. ; and others, 

Ji r ^ distance between two points whose trilinear coordinates are 
(«i, Pm 7i)> («2» ^i 72)> we know that 

»« = ^ {« cos A (01-02)' + * cosB (i3i-j82y» + « cos C (7,-7a)«}, 

^ _ sin 2A (oi -fla)2 + sin 2B (3i - jSg)' + sin 2C (yi— 72)' 
2 sin A sin B sin 

For 1 we have o, = 2j3i - 71 = y** and for G, oj = ^p,, jSj-i/J^ 78 = fc»i» 
where P|, ;D], j?, are the lengths of the perpendiculars from A, B, on 
opposite sides. 
Also jOi « c sin B = 2R sin B sin C, and so for p^ and p^. 
Therefore 2 sin A sin B sin C . IG^ « (sin 2A+ sin 2B + sin 20) r* 

— fRrsinAsinBsin (cos A + cos B + cos 0) 
+ 4lt«(8in2Asin2Bsin2C + sin2Bsin2Csin«A + sin20 8in«A8in»B); 
but sin2A + sin2B + sin2C a» 4sinAsinBsinC, 

therefore IG' = 2rJ- |Rr (cos A + cos B + cos 0) 

+iB? (cos A sin B sin + sin A cos B sin C + sin A sin B cos 0). 
Now, cosA + cosB + cosC = 1 + 4 sin ^A sin ^B sin iO «= 1+r/R, 
and the trigonometrical multiplier of 

iB? asinGsin(A + B) + sinAsinBco8Cal-i-cosAco8Boo80; 
therefore, finally, 

IG2 = 2r»- Jra-|Rr +iR2 (i + cos A cos B cos 0)» 

or « JR» (1 + cos A cos B cos C) -^Rr + |r», 

the required result. 



2935. (Professor Sylvester, F.R.S.) — If f(x) be a rational integral 
fimction of a; of a higher order than the second, prove that it is impossible 
for (fx)^^ {fxY to be a perfect square unless /(a;) contains at Iwwt two 
groups of equal factors. 

Solution by W. J. C. Sharp, M.A. 

If /(a;) be of order «, and F (x) = {f{x)Y + {/' {x)}^ F {x) is of order 
2w. Now, if F(a;) be a perfect square, it measures its own Hessian 
(Quest. 8296), which only differs from 2«F . F"-(2m- 1) F2 by a positive 
factor (Quest. 5762) ; therefore F must measure F'^ and F' = 2/' (/+/"), 
therefore /a+Z's measures /'2t/+/")2 and/'2 (/+/'7 = {P-^fW'^'^^U 

TOL. :LLyu, L 



Digitized by VjOOQIC 



86 



where uisa. function of x of an order not higher than 2n— 4 (and hcncoi 
all that follows implies n > 2) ; therefore 

f^ {2fr+f^-f^~u} =/% (1), 

and either/ and/' have a common measure, and /has one group, at least, 
of equal factors, or m is a multiple of /'*^ ; but/'* is of order 2/i— 2, and u 
at most of 2»>— 4, and therefore there is at least one group of equal 
lactors. 

Now, let f{x)^{x^a)^<p{x) and f {i) == {x-a)'^'^4>i{x), 
and ^ /'W = (•!?- «)•"-* ^2 W, 

then 0, ^1, and ^^ ^^^ ^^^h of order n—m, and, if ^ {x) does not contain 
x—a as a factor, neither do ^j {x) or <p.2 {x). From (1;, 

or (^i* {2 (flj-a)*"*-^^^^^ (a; _a)2"«-4 <^^a _(a;«fl)2m-2^ja_t<} = (ar-a)2<^«M, 

and, therefore, u must be divisible by {x^a)'^*'*-K Let « s (a?— a)**~* v, 
then V is at most of order 2 (n—m), and 

4>i2 {2 (a?-«)2<^4)3 + 4>32- (j;-a)- 0i2-r} b (j?-a)2<^, 

ai^ either ^j and ^ have a common measure, or v s <p^u?, where tc is in- 
dependent of Xf and 

2{x-ay i^<p.2 + 4>22 - (x - ff)2 4>i2 _ ,^^^2 s (a; _ a)2 (^ V, 

an identity which requires that the two leading coeflBcionts of <p should 
vanish. Therefore ^j and ^ have a common measure, and therefore ^ and 
fp' for ^1 = m^-\-{x—a) <i>\ and there must be a second group of equal 
factors. 



7459. (Professor Wolstenholme, M.A., Sc.D.)— Prove that if, in a 
tetrahedron, any one or two of the eqnutioiis a :kx = b ^p = e ±. z be 
true, then will also the corresponding one or two equations of the set 
A±X«B±Y = C±Zalsobe true. 



Solution by the Proposer. 

In the tetrahedron OABC, OA = a, OB = b, 
O^ ^ e; BO = ar, CA = y, AB = « ; suppose a 
«phere can be drawn to touch the four edges 
tf, b, X, y (none produced), then, if t^, ^j, /g, t^ be 
the lengths of tangents to this sphere from O, A, 
B, C, a=<i + ^2, X=t^-ltt^\ b^t^Art^, y=^t^-\-t^, 
whence a^rx =^b+y = ^, + ^-^3 + ^4. Thus 
such a sphere can be drawn ita + x = b + y. Let 
I be the centre of the sphere ; ;;„ p., p^y p^ the 
perpendiculars 10', lA', IB', IC on the laces ; 
a'y h\ x\ y* the points of contact with the corresponding edges ; then the 
dihedral angle A (opposite to the edge a) is the supplement of the 
'angle O'lA' ; and if 2o, 2/8, 27, 25 be the angles subtended at I by 




Digitized by VjOOQIC 



87 



diameters of the circles in which the sphere is met hy the faces, the angle 

0'IA'= o + i8, or A = IT— o— /3; so X « ir— 7-8; 
or A + X « 2ir-o-i8-7-5 = B + Y. 

The proof that, when «-«:= *-y, then will also A— X*B— Y, is 
exactly similar, the sphere then touching the produced edges. « 



8783. (Captain H. Brocard.) — Les droites qui joig^ent I'orthocentre 
aux mili< ux des segments interceptes par les hauteurs du triangle sur les 
rayons du cercle circonscrit aboutissant aux sommets du triangle sont 
perpendiculaires aux syniedianes correspondantes. 



Solution by Professor Ionacio Bbtens, M.A. 

Soit O le centre du cercle circon- 
scrit, H Torthocentre. OA' p'-rpen- 
diculaire sur BtJ coupe les hauteurs 
BH. Cn en M et. Q; les triangles 
AB(J, HMQ sont semblables, leurs 
cotcci etant perpendiculaires ; et 
ranj?le MH(i = A. 

Soient M', Q' les points de ren- 
contre de BH, CH avec le rayon OA, 
nous avons HM'Q' = M'OB'— B, et par suite M'Q' est antiparall^le par 
rapport au c6te QM du trianj<le HQM, c'est-a-dire, que la droite qui joint 
U au milieu de M'Q,' sera la symediane qui part du sonmiet H du 
trianjfle HMQ, et par suite elle sera perpendiculaire h. la symediane de 
Tanj^le A du triangle propose, car ces lignes sont homologues en figures 
semblables qui out leurs c6tes perpendiculaires. 




8837. (W. J. Grebnstreet, B.A.) — In the "ambiguous case" of 
triangles, given «, A, and A; r, r' boiug the radii of the in-circles, p, p' 
of the escribed circles to a ; prove that 

r/ (* + a) «« pp' (A—a), rp (A— a) = r^p'e. 



Solution by H. L. Orchard, B.Sc.,M.A. ; Sarah Marks, B.Sc, ; and othert. 
Let c and c' be the two values of <?. Then a/ = A'— a^, and 
r _ e is'—ff) c r^ _ c^ b — a , 



a + 6' 



a + A 



whence r/(A + a) = pp'(6-a), and rp(b'^—a^)^r'p'L^f&Q. 



Digitized by VjOOQIC 



88 

8715. (Professor Hudson, M.A.) — Prove that 
tan37i° « V6+^/^- v'2-2. 



Solution by G. G. Stobr, M.A. ; Rev. T. R. Tbeby, M.A. ; and others. 

If a; = tan 37i°, then 2aj / (1 -a^) = tan 76° « 2 + -^3 ; whence, solving 
a? = ±(-v/6--v/2)-(2-v'3), the upper sign gives tan 37i°, the lower 
sign gives tan 127^. 



8929. (R. Tucker, M.A.)— Find (I) the equation of the circle through 
the images of the centroid of a triangle with respect to the bisectors of 
angles ; and show (2) that the sum of the squares of tangents to it frum 
the angles (taken once) is \ {0^ + 61^ + c'). 



Solution by the Paopobbr. 

1. Since the median and symmedian lines of a triangle make equal 
angles with the bisectors of angles, therefore the images in question lie on 
the symmedians and the circle is concentric with the in-circle. Assume 
its equation to be 

•^(oiBy +... + ...) - (aa+. .. + ...) [a («-a)*a+ ... + ...] +\(aa + ... + ...)» 

(i.); 

then, since the centroid is on (i.), we have 

a2 + ^ + <^ « 3 [(«-a)2+ ... + ...] +9x, 
therefore 9 A = - 2 (a^ + *« + c^j + 3,2^ and 36a =* 65a* - b^\ 
The circlethen can be put under the form 
abo(afiy¥..,-\-...) 

= (ao+ ... + ...) \aa («-a)2+ ... + ... +,^ (62ai-63a2)(aa+ ... + ...)]. 

2. If ^«, tbi tc are the tangents from the vertices, we have (see Mather 
maticafrom Educational Times, Vol. 46, Appendix n., p. 138) 

^3 = («-a)2 + ^V(62a^-52«2) = ^^^ [9 {b + c-ay + 61ab'b:$a^] 

= ^ [4Xa^ + 2Ue-'l2ab~12ac] = ^ [^a^ + ebe-Ub-Sac], 

therefore <«* + ^6^ + te^ « \:ia^ = ^ (»« + ^ + e-). 



8765. (L. J. Rogers, M.A.)— If a^i + asj + ^^3 + oj + oj + oj = 0, prove 



that(l) 



tan (xi + oi) , tan (arj + oj) , tan (x^ + oi) 
tan {xi + 02), tan (x^ + 03), tan {x^ + 03) 
tan(:ri + 03), tan(a:2 + a3), taniajg + oj 



0; 



and (2) the same is true if tangents are replaced by sines. 



Digitized by VjOOQ IC 



89 

Solution by F. R. J. Hervey ; Belle Easton, B.A. ; and others. 

1. Each element is a product of three tangents which may be replaced 
by their sum ; moreover, each tangent appears in two elements with con- 
traiy signs : hence the whole vanishes. 

2. Any element represented by sin a sin b sin c may bo replaced by 
Bin o cos ^ cos <?+... + ... ; which being done thr«»ughout, the complete 
coefficient of a given cosine =« ; f or instance, that of cos (Xi + oj is 

fiin {x2 + oa) cos (xg + 03) + cos (^2 + 02) sin (ar^ + 03) 

— {sin (x^ + oj) cos (X2 + a^ + cos (x^ + a.2) sin (0:2 + og) } = ; 

and the same for any other, by interchange of suffixes. Hence, if each 
term sin a cos b cos e be taken twice, the whole can be arranged in nine 
groups of four mutually destructive terms, and vanishes identically. 



8797. (Professor SiRAnlaANJAN RIy, M.A.)— OAP and OBQ are 
two fixed straight lines intersecting at O, and G is a circle touching them 
both at P and Q ; prove that the perimeter of the triangle OAB, circum- 
scribed by any circle passing through O and touching C externally, is 
constant. 

Solution by Professor Wolbtenholme, M.A., Sc.D« 

One of the trigonometrical questions in my book of problems is : '' The 
radius of a circle touching two sides AB, AC of a triangle ABC and the 
circumscribed circle is r sec? |A or rjSec^^A, according as the contact of 
the two circles is internal or external." Hence, with the notation of the 
questipn, the perimeter of the triangle OAB will be 

20P cos» iPOQ / sin iPOQ ; 

and similarly, if the contact were internal, the excess of the two sides 
OA, OB over AB would be 20P cos^ ^POQ / sin ^POQ. 



8473. (Professor Booth, M.A.)— If a, J, c be the sides of a plane 
triangle, prove that the diameter of the circumscribing circle is a root of 
the equation 



Solution by Isabel Maddison ; Rosa H. W. Whafham ; and othert. 

If a; be the diameter of the circum -circle, we have x^—d^ — «'cot?A, 
&c. , and the equation becomes cot B cot C 4- cot C cot A + cot A cot B » 1 , 
which is correct. 



Digitized by VjOOQIC 



90 

2832. (Professor Sylvester, F.R.S.)— Prove that the curve of in- 
tersection of two right cones with parallel axes is a spherical curve, and 
that a third right cone may he drawn through it. Prove also that a plane 
circular cuhic may he found such that the distances of any two fixed points 
on it from every point in the curve of douhle curvature ahove-mentioned 
shall he in a fixed linear relation. 



Solutions by (1) Professor Sircom, M.A. ; (2) W. J. C. Sharp, M.A. 

1. The right cones x' + y^ ^ mz^, a:3 + (y — 6)* « h (z-c)' have their 
axes parallel, and the sphere 

(«+l)(a;2 + y=2-//iz2) - (m+1) {.c2+ (y -*)»_« (e-c)^} 
passes through their intersection. Also the surface 

a;2 + y2-,ns2 = X {a^ + (y-*)«-n («-r)2} 

is a right cone if its discriminant vanishes, that is, if x = — . -^ — -. 

ft O^—MC^ 

The circular cuhic in the plane a; — passing through the vertices of 
the three cones, and having for foci the four points where the spherical 
curve meets :r = 0, satisfies the given condition. 

For, taking the origin and x, y for the two fixed points, and Xi yi «i a 
point on the spherical curve, we miist have, hy the question, 

{*i»+(y.-y)»+(»i-*)'}*+^('i'+yi»+V)*-M 

for every point Xiyiti, Eliminating x^yi by means of the equations of 
ihe curve, and rationalizing, we obtain 

{(l-A^(/w+l)* + (w-m)y}«j2-2 {bz + ney + K/i (m + 1)^ b} Si 

identically; hence, equating the coefficients to 0, and eliminating \, 
fL, we obtain 

{(n-in)y + (in + l)*}{*(y> + «')-.(*2-iic2)y} « (*» + '^y)'. 
the required circular cuhic. 

2. OtJherwite : — If the vertex of one of the cones be taken as origin of 
rectangular coordinates, its axis as axis of «, and the plane of the axes as 
plane of yz ; then, the other vertex being (0, Ar, /), the equations to the 
cones are a:' + y« + 2- = m^^, d?=»+ ijy—k)^ + («-0* — «' («— 0' I 
therefore, at intersection, 

(n^- m2)(a?<» + y2 + ^2) + 2m^ky + 2m^h^m?k^--w?P - mW (2k - P), 
and the curve of intersection lies on the sphere 

a;2 + y3 + «2 + 2ay + 2fo « c, 
if w-^ . ^ mn{\-n^ ^ ^ m^{k^^{\^n^l^] _ ^ 

and k and / are determined by the equations 

(d: + bl=^c and tnrkl ** {l^m^l-bk, 
80 that a third cone passes through the curve. If (0, k^, f) be the vertex 
■ of this cone, and «' the corresponding value of w, at every point of the 
curve of intersection, r = mz, r' = n{z-l), and r" = n' {z—f)^ where 
r, r', r" are the distances of (Xy y, z) from the vertices (0, 0, 0), (0, k, /)» 



Digitized by VjOOQIC 



91 

and (0, A;', 1% and nr-mr' ^ nl and n'r—mr" « nT, bo that the curve is 
a Cartesian oval in space (see Quest. 5955). 
Now, if p be the distance of (z, y, z) from the point (0, ty, f), 

(if the point is on the curve) 

and p«A3 + B, 

hence, if any two points be taken on this curve, and p and p' be the radii 
Tectores from them, p = Az-^B, p' = A'« + B', therefore 

A>-Ap'«A'B-AB'. 
The circular cubic passes through each of the vertices. 



8873. (Professor Edmttnd Bordaoe.) — Given two relations, 

(2S = a + A+c), 
g + y cos g + g cos ^ _^ y + z cos g -f 3- cos g __ z + a: cos ^ + y cos a « 
cos(8-a) ™ C08(S-^) C08(S-<r) * *"' 

deduce therefrom -^ =. _iL « _!_ =. J?L-. 
sin a sin 6 smg smS 



Solution by "W. J. C. Sharp, M. A. ; Professor Matz, M.A. ; and others. 
2 ^ (a: +.V cos g + zcos^) A + (y -h zcosa -h a; cos g) B ->- (z -f a; cos^ +y cos^i) 

A cos (S — «) + B cos (iS — A) + C cos (8 —c) 
If Aco8g + B4 Ccosa = 0, and Acos^-t-Bcosa + O — 0, 

A : B : C = — sin'a ; cosg— cosacosA : cos &— cos a cose, 
and the numerator = — z {l — cos^a-cos^i— cos*^g + 2cos acos* cosg}. 
The denominator 
e { — 2sin2aco8(S-a) sin S + 2 (cos g— cos a cos*) cos(S— *)8inS 

+ 2 (cos * — cos « cos g) cos (S — g) sin S} -•• 2 sin 8 
■a { — sin'a [sin (* •(- g) + sin a] + [cos g- cos a cos*] [sin (a + g) + sin*] 

+ [cos *— cos a cos g] [sin (a + *) + sin g] } -»- 2 sin 5 
Bs { —sin a (1 — cos' a— cos^*— cos^g + 2 cos a cos b cos e) 

+ Bmb (— sin'acosg + cosg-cosa cos* + cos<icos*— cos'acoBg) 
+ sin g ( — sin- a cos * -i- cos * — cos a cos g + cos a cos g — cos' a coe b) J 
-^2sinS 
— {— 8ina(l— C08'«— cos'*— co8'g + 2co8«cos*cosg} -h2BinS, 

2x 
therefore 2m »- -: — x sin S, 

sina 



Digitized by VjOOQIC 



92 



or -7^ *= -^ « -7^ « -7^, by symmetry, 

sin a siiiS Bin^ siik; 

[Obserying that 2 sin S cos (S— a) == sin a + sin (^ + c) and for brevity^s 
sake writing A; for m/ sin S, we see that the given relation 

{x + t/coae + z cos b) sec (S— a) = 2m 
is equivalent to 

{x^k sin a) + (y — A: sin b) cos e + («— A; sin <;) cos ^ = ; 
and similarly, from the other given relations, we have 

(y — A: sin ^) + (z— A; sin c) cos a + (a:— A: sin a) cos c = 0, 
(«— fc sin c) + (a? — A; sin a) cos 5 + (y — A; sin ft) cos « = 0, 
whence a?— A:sina = 0, y — Arsinft = 0, 2— A: sine =0. 

Hence, if a, ft, <; be the sides of a spherical triangle, any quantities 
Xt y, z which satisfy the given relations are proportional to the sines 
of tiie angles, and conversely.] 



8980. (Professor Stegoall, M.A.)— In a rectangular hyperbola OY 
is drawn at right angles to the tangent at P ; prove that, if YX produced 
cuts SP in R, and Y be joined to X', then XB = YX', where X, X' are 
the feet of the directrices. 

Solution fty R. Knowles, B.A. ; Sarah Marks, B.Sc. ; and others* 

If a;'— y^ =• a* be the equation to the hyperbola, that of the tangent at 
P {xy) is x'x^y'y = a*, that of OY x'y + y'2; = ; therefore the coordi- 
nates of Y are aV/(a;'2 +3^2)^ -ay/(a;'2 + y'2); those of X,X', S are (fl/2*,0), 
{'-al2^f 0), (2*a, 0), respectively; whence the equations to SP and XY are 

y'a;+(2*a-a;0y = ^W ...(1), 

2V^-(a;'=* + /^-2W')y = ay (2). 

From (1), (2), we have 

ax'{2h-Za) ^ aY 

(2*a;-«)2 ' {2^x'-af 

the coordinates of R, whence we find XR and YX' each 
=:a(2*a;' + «)V2*. 



9010. (J. J- Walker, F.R.S.)— Prove that the area contained by two 
tangents to a central conic and the semi -diameters to points of contact is 
equal to ftV + aV- aH^. [See Question 3099, Vol. xiv., pp. 74, 76.] 



Digitized by VjOOQIC 



93 

Solution hy C. E. Williams, M. A. ; B. Knowlsb, B.A. ; and others^ 
Projecting the ellipse into the auxiliary circle, the (area)' would have 

to be multiplied by -^ , and as X -■ a;, T « 4~^> ^^ expression becomes 
0^ b 

(area)2 - a«X«+a«Y»-«* « a« (X» + T«-a«), 
or area « (rad. x tang.)* which is evidently true for the oirde. 



8903. (Professor G-enese, M.A.)— If /(r, 0) be the equation to a conic 
referred to any pole O, the value of {df)l(dr) at any point P of the plane 
varies as (PQ + PR) / OQ . OR, Q, R being the intersections of OP with the 
conic ; thus, for a point Q of the conic, {df)l(dr) oc QR/OQ . OR ; and 
in particular, if be a focus, (df)l{dr) is constant over the curve. [In 
thi« case /= (/-^ cos 6)2- f«.] 

Solution hy R. F. Davis, M.A. ; Professor Chakravabti, M.A. ; and others. 
The polar equation of the conic referred to O is of the form 
/(r, e)=Ara + Br + C = 0, 
where A, B are functions of $ alone, and is constant. For any definite 
value of d, we have ri + r^^^— 2B/A, rir^ «= C/A. 

Now Wl{dr) =- 2 (Ar + B) = 2A(r + B/A) 

a {2r-(ri+r2)}/rir3 0c (PQ + PR)/OQ.OR. 

If P coincide with Q, PQ = 0, &c. If be a focus, (<y) /(rfr) -constant, 
which is a priori evident, since the semi-latus-rectum is a hannonic mean 
between OQ, OR. 



9003 & 9013. (9003.)— (R. F. Davis, M.A.)— If upon each side of 
a triangle a pair of points be taken so that the pairs on any two sides are 
concyclic, prove that all three pairs are concyclic. 

9013. (Emilb Vioaki^.) — Les projections orthogponales de deux points 
inverses Mj, M2 (a/, /, z^ sur les trois cAt^s d'un triangle ABO, sont six 
points d'une mdme circonference dontl'^quation en coordonn^es normales 

est: (y«sinA + «ar8inB + a?ysinC) (a:'sinA+y'sinB+«'sinO) 
X (y'r'sinA+^VsinB + ic'y'sinC) 

^sinAsinBsinC(^sinA + ysinB + mnC)[^^Cv^^^T^y-'^^"^) 

i sinA 

y/(a;" + g^co8B)(g' + a;'cosB) ^ gg" (a/ + y" cos 0) (</ + a;" cos 0) *) 

sin B sin C ) 

[M. ViOARrf remarks that " on appelle points inverses en France ce 
que M. Caset appelle isogonal co^ugate points {Sequel to Euclid, 1886, 
p. 166)."] 

vol. XLVn. >L 



Digitized by VjOOQIC 



94 

Solution by A. B. Johnson, M.A. ; C. E. Williams, M.A. ; and others. 

It seems best to take these together, as they are both «Eisy deductions 
from the form of the equation in trilinears to any circle, viz. : 

4R2 |y«8in A + »rsinB + arysinC} sin AsinBsinC 

«= {a? sin A -f y sin B -I- 2 sin G} {p"iFsin A + fy sin B + 1^« sin 0} (I), 

where p, q, r are the tangents from the vertices to the circle. 

Let D, D' ; E, E' ; F, F be three pairs of points on the sides, and con- 
oyclio in pairs. Then, if we take i?» = AE . AE' - AF . AF ; 

(?«-BF . BF'-BD . BD'; r> = CD . CD'= CE . CE', 

the circle represented by (1) must cut the sides in D, D', E, E', F, F ; 
80 that all six points are coney clic. 

Now let D, B, F ; D', E', F' be the projections on the sides of the 
points x'f y*, af; k/x', A//, k/z', where 

^ JsinA _^ sinB ^ sJnCJ « ^^nA + y'sinB + .'sinC (2). 

Then AE. AE'= AF .AF -*{/ + «' cos A} {«' + y'cosA} /y'«'sin«A, 

so that the projections of the points are concyclic in pairs, and therefore 
all lie on the circle for which jp* = Ar (/ + z' cos A) («' + y' cos A) /yVsin- A, 
and q^ and r* have analogous values. Substituting in (1) lor p-y g*, r*, k, 
there results the equation sought. 



8835. (Professor db Lonochamps.) — On consid^re un triangle 
ABC et le cercle circonscrit A ; soit fi une transversale renconfrant les 
c6tes de ABC aux points A', B'» C Par A' on m^ne k A une taiigente 
que Ton rabat sur BC de telle sorte que le point de contHct vienne occuper 
sur BC une certaine position A". Demontrer que la droite AA" et les 
deux droites analogues BB", CC vont passer par un m§me point M. 



Solution by Professor Ionacio Bbtbns, M.A. 
Soit A'M' la tangente men^e de A' ; nous aorons 
A'B.A'C- (A'M7= (A'A'O^ C'A . C'B = C'C"*, B'A . B'C = B'B' «, 
etaussi BA- _ A^A^-BA- ^ (A^B . A^C)*- BA- 

A"C A'C-A'A" A'C-(A'B.A'C)*' 
ou ?^' » (A^ . A^O* . A^C- A ^B . A^C + A^B . AC- BA^ ( A^B . A^Q* 
^"^ (A'C)2-A'B.AC ' 

BA^' ^ (A^B.A^C)^(AC-A^B) ^ (A^B.A^C)^ ^ (A^B)* 
A"0 A'O A'C-A'B) A'C (A'C/* 

Delamdmemani^Cona 5?^ ^ (B:C)_» ^^ AC^' ^ (ACO^. 
AB" (B'A)* BC- ^BC')* 



Digitized by VjOOQIC 



95 



Multipliant ces proportions, 

BA^' CB;;^ AC^' ( ArB.BV.AC 'U , 
A"0 * AB" • BO" " \ A'C . B'A|. BC' / ' 
/ A'B B'O AC ,\- 

done les trois droites AA", BB", 00" se renoontrent au mime point. 



8385. (J. Brill, M. A.) — Three parabolas are drawn baying a common 
focus ; from a point T, external to all three, tangents TP and Tp are 
drawn to the first parabola, TQ and Tq to the second, and TR and Tr to 
the third ; prove that Qr . Ep . Pg = yR . rP .pQ. 



Solution by the Proposer ; Belle Easton, B.A. ; and others. 
Let S be the common focns of the three parabolas, then we have 
(SP) . (Sp) == (SQ) . (S^) = (SR) . (Br) = (ST)» ; 
therefore (Qr) . (Rp) . (Pq) + (qR) . (rP) . (^Q) 

= {(Sr)-(SQ)} {(Sj.)-(SR)} {(Sj)-(SP)} 

+ {(SR)-(Sj)} {(SP)-(Sr)} {(SQ)-(Sp)} 
- {(SP) + (Sp)} {(SR) . (Sr)-(SQ) . {8q)} 
+ {(SQ) + (Sy)} {(SP) . (S/.)-(8R) . (Sr)} 
+ {^SR) + ,Sr)} {(SQ) . (8|/)-(SP) . (8p)} = 0. 
Hence it follows that Qr . Rp . Pg = gR . rP .j^Q. 



8915. (Rev. T. 0. Simmons, M.A.)— A point P being given in the 
plane of a triangle ABC, it is known that, with P as focus, five conies 
can be drawn, four of them circumscribing the triangle, and one inscribed 
in it. Show that for certain positions of P a sixth conic can be drawn, 
also having P for focus, and touching the other five ; and find the locus 
of P when this is possible. [The Proposer suggests that conies with one 
fotius common should be called eo-foeal, reserving the term confocal for 
conies having both foci common.] 

Solution by R. F. Davis, M.A, 

^ If , the system consisting of a triangle abc^ its one circumscribed and four 
inscribed circles be reciprocated with respect to any point P, we obtain a 
corre«ponding system ( oasisting of a triangle ABO and five conies each 
having P as focus, four of them circumscribing the triangle ABC and one 
inscribed in it. 



Digitized by 



Google 



96 

The nine-pomt circle of the triangle abe tonchM each of the four in- 
acribed circles; and in Uie particular case when one of the angles is a 
fight angle it also touches the circmnsoribed circle. 

Hence, if one of the sides of ABO subtend a right angle at F, the 
reciprocal of this nine-point circle is a conic (focns P) touching each of 
the above five conies. P must therefore lie on one of the^thiee cirdea 
described upon the sides of ABO as diameters. 



8688. (Professor Iomacio Bbtbns, M. A.) — Si dans un triangle la pro- 
jection du cAt^ BO sur BA qui est BH, et la projection du BA sur AO qui 
est AH', sent egales ; la bissectrice du A, la hauteur du et la m^diane 
de B se rencontrent en le mdme point. 



Solution by B. F. Datis, MJL ; B. Enowlbs, BJl, ; imd othen. 

Since BE - AW, acos B » <; cos A. Let 
the bisector of the angle A meet the per- 
pendicular OH iu O, so that 

OH : 00 - AH : AO - cos A : 1. 
Then may be regarded as the centroid of 
the masses a cos B, ^ cos A, ^ cos A, placed 
at the angular points A, B, respectively. ^^^ 
For, since 

<; — tfcosB + ^cosA »BH + AH, 
the masses at A and B may be replaced by a 
mass at H ; which with e cos A at is finally equivalent to a mass 
0(1+ cos A) atO. Hence BO divides AO in the ratio ^cosA : acosB 
■i 1 : 1, i.e.f it bisects it. 




8700. (R. W. D. Ohbistib, M.A.) — Given the sum of the ex- 
pression l»' + 2'' + 3'' + 4'' +»»•, to find a metljod of writing down at 

once the sum of l*'** + 2*'+* + 3*'** + 4*'*i +»•'♦*, where ria even or 

vice verad. 

Solution by Prof. Gbnbsb, M. A. ; Prof. Nash, M. A. ; and others. 
l' + 2r + dr + + ^r «^ +An^ + &c, = <>(») Say, 

therefore (l+^i^Y + {2 ^xy + ... {n + xy ^ 4>(n + x)^4>(x), 

This equality holding for all integral values of x (and therefore for more 

than r values), is an identity. Equating coefficients of x, 

r (l'-» + 2»-»+ +«»--i) „ ^/ (n)-ooeffioient of « in <>(«) 

-^'(ff)-.^'(0). 



Digitized by VjOOQIC 



97 



Hence, to find 23^*^ from 2«^, multiply by (r+ 1), and integrate with 
respect to n. The constant of integration can always be determined by 
the condition that the expression equals unity when n is unity, and is in 
fact nBnt when Bn represents Bemouilli's numbers. 

To find 2^ from 2s^*^ reverse the process and omit the term with- 
out n. 

[Prof. Gbnbsb adds that the above occurred to him last year as the 
proper method of dealing with the series (a + d)^ + (a + 2d)^ + (a + 3d)r + &c. , 
obtained by putting x above equal to d/a]. 



Note on Quest. 8376 (Vol. xlv., p. 



£t/ Professor Nash, M.A. 



Most of the theorems enumerated in this Question are true, not only in 
the particular case considered, but also when the lines wD, wT), &c. 
make any constant angle with the sides. Let the angles GDw, AEm, 




BF», BDV, CEV, AFV, measured in the direction of the letters, be 
all » 9 + w, where w is the Brocard angle of the triangle ABC. There- 
fore «EC « ir-(a + ») «ir— »DC; therefore wDCE are concyclic, and 
similarly «EAF, «FBD, wD'CE', w'E'AF', »'F'BD' are concyclic ; there- 
fore «DE - »CA « fitf, »EF = «AB = <», &c. ; therefore u is positive 
Brocard-point of DEF, and u' is negative Brocard-point of D'E'F'. 

Also «DF := »BA = B-«, therefore FDE =r B. 

Similarly DEF = C, EFD = A, F'D'E'^ C, D'E'F= A, E'FD'- B, 



Digitized by VjOOQIC 



98 

therefore DEF, D'E'P' are similar to one another and to ABC, and w is 
centre of similitude of ABC, DEF, «' that of ABC, D'E'F'. 

Because B, D are corresponding points, the ratio of similitude is 
o>D / »B = sin « / sin (w + 0), 
similarly for D'ET', ABC the ratio is w'D'/w'C = sin a» / sin (a + $), 
therefore the two triangles are equal to one anotiiier. Again, the augle 

CD'S'- ir-C-CE'D'== ir-C-e = ir-C-CDE = CED. 

Therefore D' lies in the circumcircle of DEF, and similarly for E'F', so 
that the two triangles have a common circumcircle a Tucker's circle . 

Let Q he the centre of this circle, then Q corresponds to O, whether 
considered as helonging to DEF or D'E'F', therefore 

«Q/ «0 « «'Q/ «'0 » sinctf / sin (« + e), 

therefore Q lies on line OP which hisects the angle «0a>'. If rK he the 
radii of the circles DEF, ABC, therefore 

r / R — sin « / sin (« + e) = «Q / »0, 

therefore the radius of a Tucker's circle is proportional to the distance of 
its centre from the Brocard-points. AIro, since OwQ is similar to BcvD, 
therefore 0«Q = e. If a>i be negative Brocard-point of DEF, wi corre- 
sponds to u'f therefore »'w«i = &, ««'»i = » = »«T ; therefore »i lies on 
the fixed line wT, and similarly the positive Brocard point of D'E'F' lies 
upon a»P. If p, ff be the symmedian points of DEF, D'E'F', then P«p 
is similar to OwQ. therefore taVp = to. 

Therefore Fp is parallel to a>«', and Vp' of course coincides with "Pp, 
Since the Brocard-points are equidistant from the circumceutre, there- 
fore Q is the centre of the circle wu'a>i»'i, therefore 

wQ»i ■■ 2fi0w'»i =s 2c» =s wPfitfj, 
therefore P lies on the Brocard circles of DEF and D'E'F'. 

Again, from the figure, E'EF = ir-E'D'F'= ir-C, therefore EF' is 
parallel to BC ; and because AE . AE'=: AF . AF', thereft>re E'F is anti- 
parallel to BC. Also E'F subtends ut the circumference an angle E'F'F 
= IT — e, therefore E'F = 2r8ind; therefore the three antipurallels 
E'F, F'D, D'E are all equal, and this common length is proportional to 

OQ » --- sin « . OQ. DD' subtends at the circumference an angle 

o« 

DE'D' = CE'D' - CE'D = e - A, therefore DD' = 2r sin (» - A) , there- 
fore the intercepts DD', EE', FF' are proportional to sin (a— A), 
8in(e-B), sin(e-C). 

If 0= iir, the intercepts are proportional to cos A, cos B, cos C, and tho 
circle DEF is the cosine circle. 

In this case the sides of the triangle DEF, D'E'F' are perpendicular to 
the homologous sides of ABC, and, since OwQ =^ ^t ^ OwP, therefore the 
centre of the cosine circle is the symmedian point. 

The radius of the circle is It tan a>. 

In the case considered by Mr. Simmons, d « ^ir- «, .*. OwQ = Oaw', 
therefore Q is the mid-point of ««', and the radius of the circle is R sin w. 

The lines EF', FD', DE' form a new triangle A'B'C similar to ABC, 
and DEF, D'E'F are Tucker's triangles with respect to A'B'C. There- 
fore the symmedian point of A'B'C is the bisection oipp' as in the case of 
ABC, i.e.f the symmedian points of ABC, A'B'C coincide, therefore P is 



Digitized by 



Google 



99 

the centre of similitude of ABC, A'B'O' and also the centre of perspectiYe. 
If B' he the radius of the circle A'B'C, it is evident that 

r_ ^ 8m«__ ^ sine therefore K ^^{^'^) 

R' sin(ir— + ») 8in(# — a) R an{e + u) 

« ratio of similitude of A'B'C to ABC. 
But P»i / P» = sin Pw»i / sin P«i» = sin (6—ul)/ sin (0 + »), 

therefore P«, / P«'= Pw// P« = R/R' ; 

therefore «/ and «i are positive and negative Brocard-points of A'B'C. 
The circumcentre of A'B'C is a point 0' on PO such that QajO'— ir— 0. 
Therefore QO'/ sin e = Q»i / sin « = Qa> / sin » = QO / sin «, 
therefore 0', O are equidistant from Q. 

If the circle DEF is the T. R. circle, the lines EF, FD', DE' pass 
through P, 80 that the triangle A'B'C reduces to a point, therefore 
sin (0— w) = and 6 «= », and ti^e centre of the T. R. circle is the mid- 
point of OP. Its radius is ^^^^L^ = -^— . 
sin 2(u 2 cos u 

If EP, F'P meet BC in d\ d, FP, D'P meet CA in /, «, DP, E'P 
meet AB in f*,f, then, since E, t^'arecoirespondingpointsof the triangles 
A'B'C, ABC, therefore PE / Prf'= R'/ R, and similarly for the others. 

Therefore def, d'tff form another pair of Tucker's triangles, the sides of 
which make, with the homologous sides of ABC, an angle ir— 0. If ^ he 
the centre and r' the radius of the corresponding Tucker's circle, 

Oo^ + 0»Q = IT or ^»P =■ Q«P, 
therefore ^, Q are conjugate points with respect to the Brocard-circle of 
ABC. 

Also //r = ^»/Qa> = gP/QP = ^/Q0-8in (0-») /sin (0 + »). 

If two Tucker-circles he taken whose centres are equidistant from the 
Broctird- points, their radii will he equal, and therefore also the corespond- 
ing Tucker's triangles. As a particular case of this, we see that two tri- 
angles can he inscrihed in ABO which are equal to it in all respects, and 
that the sides of these triangles make an angle ir— 2» with those of ABO. 



8784. (By R. W. D. Christib, MA.)— Prove that, if 
»= 1-I-2 + 34- ...+», S = r- + 22 + 32 + ...+«2, S'= 13 + 2«<-3»+... + »», 
5- l^+24 + 3H...+»S <r = 18 + 26+3* + ... + f|6, 

then ?^+2^'^S^ 

52 S 



Solution by H. L. Orchard, M.A., B.Sc. ; Professor Betbnb ; and others. 

By summing the series hy the method of Undetermined Coefficients, or 
otherwise, we find 65S'S-i-2«3 = 4»*(fi + l)''L*'^»(n + l)- I]; hence, 

we have to show that n^ (n + 1)2 \2n (» + 1)- 1] = 12 (I* + 2* + ... + »«). 



Digitized by VjOOQIC 



.100 

In the left-hand memher put n^tn and m + 1 successiyely, and suhtract* 
We thus obtain 2 (m + l)«[(irt + 2)»-m«]-(m + l)S[(w+2)«-m»], which 
reduces to 12 (m -t- 1)*. Hence, if the theorem holds for « ■■ m, it holds 
for n B m + 1, and so on, universally. Bat it evidently holds for n » 1, 
and for fi — 2 ; therefore, &c. 



8763, (By H. Stbwart, M.C.P.>--Prove that 

(l)ii «(l-^2)» + ^(l-««)».a», then p + il^^O; 

ax (l-j«)* 

(2) if (l-«»)* + (l-fi«)*- a («-«), then ^ = ILz!f-I*. 

«» (I-a;*)* 

Solution by F. K. J. Hbrtbt ; W. J. G&bbnbtbbbt, B.A. ; and others. 
Putting, for brevity, (1— a?')* - *, (1— w')* = t, du/dx - «', we have 

* < x-u ViF— w </ a?— w «' 

but < (»+ -f* («— w) « «^ + 1 -«u = « (« + + * (*— «')i ••• «' = */*• 



8745 ft 8807. (W. W. Taylor, M.A. Sugjirested by Quest. 
7938.) — Prove that (1) the areas of the TATLOR-circles of the four triangles 
ABO, PBC, PGA, PAB are together equal to the area of the circum- 
scribed circle, P being the orthocentre of the triangle ABC ; also (2) the 
lines joining their centres are the SiMSON-lines of the middle points of the 
sides or of AP, BP, OP with respect to the pedal triangle. 



Solution by R. F. Davis, M.A. ; Professor Bbybns ; and others. 

The four triangles of the question have all the same circum-radius ; 
while their angles are (A, B, 0), (»— A, ^ir— B, ^ir— 0), &c. Hence, 
employing the known form for the radius of the TAYLuR-cirde 

« R {sin2 A sin* B sin« + co8« A COS* B cos* 0}*, 

we get sum of squares of the TAYLOR-radii 

« R2 (sin^A + cos^A) (sin* B + cos* B) (sin* + cos* 0) « R*. 

The second part follows at once from the fact the SiMsoN-line of the 
middle poiut of any side with respect to the pedal triangle passes through 
the TAYLOu-centre. In this case the four triangles have all the same 
peda triangle. 



Digitized by VjOOQ IC 



101 

2934. (Professor Stlybstbr, P.R.S.) — If «„ »2, »„ «<, Sj, «« repre- 
sent the sum of xyztuv, and of their binary, ternary, quaternary, 
quinary, and sextic combinations respectiyely, and if E stands for the 

symbolof Emanation a — +*—-+«?-- +(i-^ +tf-^ +/4-, prove that 

dx dy dz dt du dv 
the resultant of «i, E«2, *i» ^41 Ht ^'6 ^ ^e product of 

and of the similarly formed powers of products of the analogous linear 
functions of abcdef. 

Solution by Professor Sibcom, M.A. 

The equations «i — 0, s, = 0, S5 = are equivalent to ^ = 0, Xr^ = 0, 
:Sa^ s 0, whence we shall have 15 systems of solutiouB, such as d; + 1 — 0, 
y + M—O, 2 + 9 = 0. Now, E«3 ^ 0, E«4 »> 0, E«c = are equivalent to 
2aa;=0, S^kc^^O, 2aa;*»0, whence, substituting for tuv and writing /, m, 
» for a— rf, J— tf, tf — /, we have 

te + wy + fi« = 0, ir* + my' + fi«* = 0, ii:» + my* + fi«» = 0...(1, 2, 8), 

the resultant of which is found by multiplying together the three factors 
foj* + wyi* + n«i*, &c., where yi : x^ «i : Xy^ &c., are roots of the cubics, 
obtained from (1) and (2), 

w(m«-f|S)y3+3/»|SyJaf + 3/«mya^+/(^-fi?)a:'«0 (4), 

»(ii«-m2)23+3^%SiF + 3/%Msr»+/(/2-mS)«»=»0 (6). 

Substituting from (1) and (2), we shall find 

ir,i + myj* + n«i»-iri»(|l!-l)(^-l); 

from (4), (yiV*i'-i)(y»V%'-i)(y8V^'-i) 

= (m'— /2)(/ + m + «)(m + «— Q(/— m+n)(/+m-n) / m»(m'— n*) ; 
80 for «i : 01, &c. Whence, observing that the resultant is of the degree 
3.5 + 1.3+1.6 » 23 in the coefficients, its expression in terms of 
a, by Oy &c., is 

(a-rf)'(*-*)'(<^-/)M« + *-<'-0(« + ^-*-«) . ..(« + *+«-<'-«-/)*•••. 
There are 15 expressions of this kind corresponding to the 15 systems 
of solutions, and as there are 15 factors 0^6, the degree of each is 9 in 
the final resultant. Also the resultant is of the degree 6.15 in the foe- 
tors a + b-e-d, and 6 . 15 terms of this kind can be formed, but half of 
these differ only in sign from the rest, hence each is of the degree 2. 
Similarly, the resultant is of the degree 8 . 15 » 120 in the fiictort 
a-^b-Ye—d—e—fy of which 10 only are distinct, hence each is of the 
degree 12 in the resultant. Hence the theorem. 



9026. (Professor Catalan.]— Soit 

«(«+ l)(a+ 2) ...(a + (?) ±*(* + l)(* + 2) ...(* + (?) = (a + * + (j)^(tf, *). 
(Le signe +, si est pair). (1) ^ (a, b) est un polyn6me entier, k coeffi- 
cients entiers ; (2) si a, b sent remplac^ par des nombres entiers, ^ (a, b) 

VOL. XLVU. N 



Digitized by 



Google 



102 

devient on nombre entier ; (3) pour ces valeurs de a, b, 

(« + A + c) <^ («, *) = 3R [1 . 2 . 3 ... (c+ 1)] ; 
(4) si, en outre, a + * + c est premier, <^ (a, i) = 9)i [l . 2 . 3 ... (<?+!)]. 



Solution by W, J. C. Sharp, M.A. ; Rev. J. G. Bibch, M.A. ; and others. 

Ita + b + e = 0y we have A = — a-r, and 

a{a+l)...{a + e)±b(b + l)...{b + c)=^a{a + l)...{a + e){l±{^iy*^}, 
which vanishes for the + or — sign as e is even or odd ; therefore 
a{a+l),..{a + c)^(--iyb{b+l).,.{b+c)s{a-i-b + c)<t>(ay b), 
where ^ (a, b) is an integral function of a and b. Also, since {a + b + c)<i> (a, b) 
has integral coefficient for all values of a and b, the coefficient of if> (a, b) 
are integral, which proves (1) and (2). 
Again, where a and b are whole numbers, 

a(a+l),..{a + e) + (-iyb{b+l)...{b + c) 

s((?+l)! {«*'a^i + (-ir***^Cc*i} 

(where "Cr denotes the number of combinations which can be formed out 
of n things taken r together), which proves (3) ; and (4) follows at once. 



8981. (Professor ScHOUTB.)^Given two conies ; find (1) the locus of 
the vertex of a right angle circumscribed to these curves ; and (2) con- 
sider the particuhu* case of two homo-focal conies. [The problem in its 
general form may be otherwise stated thus : — Find the locus of points 
such that one of the tangents from it to a conic (2), together with one of 
the tangents to a second conic (3), form with the two tangents to a third 
conic (1) a harmonic pencil.] 

Solution by A. R. Johnson, M.A. 

The condition that the two lines (1'), together with one of the lines (2^), 
and one of the lines (3^), 

a^x^ + 2bixy + <riy8 « 0, a^^ + Ib^y + e^'^ = 0, a^'^ + 2b^ + ^^sS^ = 0, 

(1', 2', 30, 

may form a harmonic pencil, is 

where ^n = ^i^^i — V» An = fliCj + OjCi— ^Ajig; ©tc. 

Now the equations to the tangents from a point to (1), (2), (3) may be 
thrown into the forms (1'), (2'), (30 respectively, and from the significa- 
tions of Aji, A|2, etc. it is seen at once that, in terms of the coordinates of 
the point, A^ = »n, hy^ = »i3, etc. ; where »ij denotes (BjOi— Fj') a^ + &c. ; 
«i2 = (BjCa + BjCi — 2FjF2) x' + &c. in Salmon's notation ; the equation to 

the locus is therefore = »„ {«^-4»3j«a3}-2»,i»,2«i3«23 + »J2*is W» 

representing an octavic curve. It is seen, frt>m (A), that the conies Sy^ ~ 0, 
*ii = 0, as is geometrically evident, touch the curve where they meet (1), 



Digitized by VjOOQIC 



103 

and intersect it also where they meet the common tangents to (2) and (3), 
the equation to these last being »^— 4*22*33 *= ^« The same is true of 

the quartic '12*13 ~2'ii*23 ~ 0* 

Also, from the form of (A), it is seen that (1) touches the curve where 
it meets «i2 =° ^» *i3 ~ ^' Hence it is seen that the octavic has eight 
double points all lying on (1), which also touches a tangent at each double 
point. The other tangents at 'the double points form two groups which 
touch, respectively, «,2 = 0, s^^ = at the corresponding double points. 

Again, we may throw the equation (A) into the form 

(*ll»2S-«12«13)'-4»* «22*33 = 0, 

showing that the conies (2) and (3) touch the octavic at their eight points 
of intersection with the quartic »ii«23— «i2*i8 = 0. 

If (1), (2), and (3) have common tangents, these must form part of the 
locus ; for at each point of them three out of the six tangents to the 
conies coincide. Dividing out by the equation to these common tangents, 
the degree of the curve locus becomes correspondingly depressed. 

Suppose, now, that (1) is inscribed to the circumscribing quadrilateral 
of (2) and (3), so that (2 = denoting a tangential equation) 

2i « k^'^ + k^^s, therefore «u = k^s^-k-kJe-^^-^h^s^f 
and the equation (A) becomes 

« {aJi-2^23^'3523«22-2^2^s3a23«33-^2'V(4 + *'22»83)} {hz'^^Zi}* 
i.e., « {V*22-^3^*33}^ {4-'**2?*83}» 

representing the four common tangents and the twice repeated conic 

k2's^2-ks^8s3 = (B) 

passing through the intersections of (2) and (3), as is otherwise geo- 
metrically evident, since the tangents to two conies at a point of intersec- 
tion are conjugate with respect to any conic inscribed to the four 
common tangents. 

1 k 

Since ^3 = -r- ^1 ~ "E^ ^i> therefore V*33 " *ii~*a*i3+ V*22f 
A^3 ^3 

and (B) may be written k^i^ — ^u — ^f ^' ^3*13— *ii~0| so that (B) 
cuts (1) in the four points of contact of (1) with the four common 
tangents. 

Now, let 1i consist of two points, then »u = is the twice 
repeated joining line, and Sjo = ^» *i3 — ^ ^^^^ ^^''^^ double contact 
with one another at the points. If the points be the circular points at 
infinity, then «j2 = 0, s^^ = are the director circles of (2) and (3), and 
»ii is the square of the line at infinity ; thus leading to the well-known 
result for the orthoptic locus of confocal conies. 

* Hence 2«,i = ^2^*12 + ^3*13 5 «-^-> if (1)» (2), (3) be any conies inscribed to 
the same quadrilateral, then the director conies of (1), (2) ; (1), (3) inter- 
sect in four points on (1). It seems natural to call Sjj = the director 
conic of (1) and (2). The four points are the points of contact of (1) with 
the four common tangents. 



Digitized by VjOOQIC 



104 



90S0. (F* MoBLST, B.A.)— ABDG is a parallelomm ; O is any 
point on the line biseothig the angle A ; CO, BO meet BD, CD in E, F ; 
proye that BE » CF. 



Solution hy Rev. D. Thomas, M.A. ; Prof. Bbtbkb ; tmd •thir$. 

Let a, jS be nnit-Tectors along AC, g g_ 

AB respectiTely, and AC — a, AB «« b. 

So that AO»w(a + jS), / ^/^-^ 

AE — bfi-k-ma — (l^y) fl« + my (a + jS); 

therefore b « my, 

ji;»a(l-y)+my 

- ft + a ( 1 -i-^ - a + ft- ^ - BE. A 
\ m / m 

Similarly, 




CF-a + ft-^. 



[If we draw EEL parallel to AB, and join EF, we have, from the 
rhombus HL, BE = AL = AH ; but, clearly, LF is a parallelogram, 
hence CF » AH = BE.] 



8985. (Professor Btomakbsa Chakeavarti, M.A.) — A cylinder, 
weight W, radius r, is placed on a rough horizontal plane ; a uniform 
plaoJc, weight P, length 2Q, is inclined at an angle B to the horizon, and 
rests with one end on the ground, the other on the cylinder (the plank 
being at right angles to the axis of the cylinder) ; if ^^ be the angle made 
with the yertical by the re-action of the ground on the cylinder, prove 



that 



cot4r-cot4-+- .—.secO. 
^ 2 a P 



SolutUm by D. Biddlb. 

Let A be the point of 
contact with the ground 
of a section-area of the 
cyclinder, in the same 
plane as D the mid-point 
of the plank, as B the 
point of contact of the 
plank with the ground, 
and as C the point of 
contact of the plcmk with 
the cylinder. Also, let 
AE - W, and DF = P. 
O being tiie centre of the section-area of the cylinder, join CO and CA, 
and draw DG, GF, DH parallel to CO, CB, CA respectively, and cut off 
AI s iDH ; draw IE psurallel and equal to AE, and join EE (parallel to 
AC and equal to AI), also join AE. Then L OAE = ^, 




Digitized by VjOOQIC 



105 



Now, in the diagram, the plank is tangential to the cylinder. In this 
case, cot 4^ = (EKcosiO + W)/(EKBinie) « ooti«+W/(EK8iniO) 

-cotie + W/(iDHsinia)=coti« + W/(iPcos«8ecie8inie) 
- cot ie + W sec e/(iP tan ^$), 

which, by taking ^r tan i$ » a, reduces to cot i9 + — . ^ . sec 9. 

But the plank can rest (on the cylinder) lower than 0, say C. Let 
Z OAC - ^. Then 

cot^- (EK'coe^ + W)/(EK'8in4>) - cot4> + W/(EK'8in4>) 
= cot ^ + W/ { iP cose sec (0-^) sin ^} 

=:cot4>+^ (cot^ + tane). 

But, in this case, 

a = rseca/} 2(cot^ + tane) + :^(cot^— cot^O) i 

/Co • ii o iiPA Qsina \» P f / QsinO \* 
/ (, LVr-Qsine/ 2W(. \r-QsinO/ 

l+CO80\ TJ 

which fits neither \r tan OAC, nor \r tan OAC, nor ir tan i«. 



8847. fW. Mills, B. A.) —Given the focus of a parabola, one tangent, 
and a point on it through which another tangent passes ; proTC that the 
locus of the point of intersection of the variable tangent with a diameter 
through the point of contact of the fixed tangent is a circle which touches 
the fixed tangent at the g^ven point on it. 



Solution by C. E. Williams, M. A. ; Prof. Sarkar, M. A. ; and other$. 

Let S be the focus, PT the fixed 
tangent touching at P and T the 
fixed point in it, through which 
passes the variable tangent OTQ 
touching the parabola at Q and meet- 
ing the diameter through P at O. 
Let TK, QK at right angles to PT, 
TQ meet at E; then STP » SQT, 
therefore STK - SQE ; therefore 
S, T, Q, E are ooncydic and TSE 
a right angle ; but, ST being constant in position and magnitude, therefore 
TE is, and the locus of Q is a circle touching PT at T. And since 
TO = TQ, therefore the locus of is another equal circle touching PT at 
T on the other side. 




Digitized by VjOOQIC 



106 

9011. (R- liACHLAN, B.A.) — Show that the product o the three 
normals drawn from any point on a conic is equal to the product of the 
perpendiculars from the point on the asymptotes and the diameter of 
curvature at the point. 



Solution by A. R. Johnson, M. A. ; Prof. Matz, M.A. ; and others. 

This is one of a numerous class of deductions that may he drawn from 
the theorem that the product of the m + n normals drawn to a curve of 
order m and class n is equal to the product of the n tangents and m per- 
pendiculars on the asymptotes from the point. 

Let the point approach the curve ; two tangents and one normal he- 
come evanescent, and the product of the tangents is equal in the limit to 
the product of the normal and the diameter of curvature ; thus, then, the 
product of the m-k-n—l normals to a curve from any point on it is equal 
to the continued product of the «— 2 tangents, the m perpendiculars on 
the asymptotes, and the diameter of curvature at the point. 

Since the product of the perpendiculars on the asymptotes of any conic 
from a point on it is constimt, the result might be stated, — The product of 
the three normals to a central conic from any point on it varies inversely 
as the curvature at the point. Analogous is the theorem, — The product of 
the four normals to a central conic from any point on a similar and 
similarly situated conic varies as the product of the tangents from the 
same point. 

In the cubic the product of the perpendiculars on the asymptotes from 
any point on the curve varies as the perpendicular on the satellite of the 
line at infinity ; and genef ally the product of the perpendiculars varies as 
the quotient of the normals to the (w — 2)-ic through the m (w— 2) points 
of intersection of the asymptotes with the curve, divided by the product 
of the distances of the point from the real foci of the curve. 

[By the aid of these considerations many new theorems may be obtained.] 



8729. (R. W. D. Christie, M.A.)— Of the series 

1. 1 + 3.5 + 5.13 + 7.25 + &c. 

to n terms, show that, (1) the sum is a square; (2) the square root is 
= 1 + 3 + 5 + 7 + &c. to n terms ; (3) each term is the product of two num- 
bers which may be taken to represent the shorter side and hypothenuse of 
a right-angled triangle ; (4) if unity be added to the nth term, it is 
divisible by 2w, and if unity be subtracted it is divisible by « — 1 ; and (5) 
all terms are odd numbers. 



Solution by G. G. Storr, M.A. ; Rev. G. H. Hopkins, M.A. ; and otJtera, 

The wth term is (2n— l)(2w2_2«+ 1), whence obviously (4) and (5). 
This term = »*—(»- 1)S whence (1) and (2). Also (3) is the interpreta- 
tion of the fact that (2«2_2// + 1)-- (2n- 1)2 = {in {n- 1)}" = perfect 
square. 



Digitized by VjOOQIC 



107 

8380. (P. C. Ward, M.A.)— Prove that (1), 

a-k-b^Cy 4«, 6«, 4a 

4i, a + A— tf, 4a, 6a 

6*, 4*, « + *—«, 4a 

4*, 6*, 4A, -a + A-(J 

s (a + A + (?)< - 8 (a + ft + c)2 (*tf + (?a + a*) - 1 28 a^<? (a + * + c) + 16 (be + ca + aft)' 

= result of rationalizing a* + ft* + o* = ; and hence (2) the above deter- 
minant is symmetrical with respect to a, ft, c. 



Solution by R. F. Davis, M.A. ; Satis Chandra Basu ; and others. 

Let ft=A*.a; then <? = a (1 + a)*, or a + ft--tf + 4a\ + 6a\2 + 4aA3 = q. 
Multiplying successively by A, A^, A^, and putting for A its value ft/fa, four 
linear equations in the variables A, A', A* are obtained. Eliminating 
these variables, determinant required = 0. 

Or thus; if Sj = a + b + e, 23 « be + ea + ab, 23 = aftc, 
2, ==« + ft + (j==:2ftM +... + ..., 2i2= 4 {ft<j + (ja + aft + 2aW{a* + ft* + <?*)}, 
(2r-423)'=642s(a + ft + (? + 2ftM+. .. + ...) =6423(2, + 2i) - 1282i2s. 



8714. (Professor Genese, M.A.) — S is a focus of a conic, PN a 
fixed ordinate to the diameter through S, PQP' a circle with centre S ; 
a variable radius SQ meets FN at L and the conic at B. Prove that the 
cross ratio {8LQR} is constant. 



Solution by 0. E. Williams, M.A. ; Sarah Marks, B.Sc. ; and others. 

Let SP = K = SQ, QSC - <p, 

then SL = (K- /) / [e cos <^), SR = ^ / (1 - « cos <^), 

and the ratio SL . QR : SQ . LB reduces by substitution to (K— /) : E. 



8676. (Professor Bordage.) — The three sides of a triangle forming an 
arithmetical progression, (a) being the shortest, {of) the longest ; if the 
distance of the centres of the inscribed and circumscribed circles is desig- 
nated by i, and the diameter of the nine-point circle by D, prove tluit 
oa'- 3 (D2-t2). 

Solution by K. F. Davis, M.A. ; E. Enowles, B.A. ; and others. 

Let ft be the mean side, so that 2ft » a + a', 2s ^ 3ft, 2aa'« = 3afta'. 
Then 2Rr = 2 (afta74S) (S/») = abaJ2s = «a'/3 ; 

but i2 » R2- 2Rr = D'-aa'/S ; therefore, &c. 



Digitized by VjOOQIC 



108 



8876 ft 9032. (Professor Hudson, M.A.)~ProTe that the normal 
chord which subtends a right angle at the focus of a parabola is divided 
by the axis in the ratio 2:3. 



Solution by G. G. Storr, M.A. ; D. E. 
Shorto, M.A. ; and others. 

Draw PQ a normal to a parabola at P, 
and QL perpendicular to the diameter 
through P ; then 

SP = PL == iNL = iSQ ; 
therefore, since SG = SP, 
we have 

QG . QP = SQ«-SP2 « 3SP», 

QP2 = SQ2 + SP- = 6SP«; 
therefore QG : QP = 3 : 6, 
and PG : GQ = 2 : 3. 




8737. (W. J. Greenstrbbt, B.A.) — ^ABC is a spherical triangle, the 
mid-points of its sides are the angular points of a triangle DEF ; prove 
that (1) co8EF/cosi« = cosFD/cosi* - cosDE/cosic ; 
and (2) if cos^ ia » cos i (6 •«- c) cos ^ ip^^)* ^^ angle D is a right angle. 



Solution by Rev. T. B. Tbrrt, M.A. ; A. Gordon ; and others. 

1. cosEF « co8idcos^0 + sin|&sin|tf[coB0— co8aco8 6]coBecdco8ec0 

« (cos^ \a -¥ cos^ \b -I- co^ i^— 1) / (2 cos \b cos \e) 
whence the first result. 

2. Also D is a right angle if cosEF — cos ED cos DF ; that is, if 

cos' \a — COS* \b + cos* ^c— 1 = cos' \b cos* \e — sin' \b sin' \c 
= cos i (* + c) cos i (i— tf). 



8998. (Rev. T. B. Terry, M.A.)— Solve the equation 
^ 6ar^+(12««-6)-f^4(2««-3)ary = 

rf'rJ /MM air. 



dx* dx^ 



Solution by D. Edwardes ; G. G. Storr, M.A. ; and others. 

If ir= - — 2x, the equation becomes lA/ =z 0, 
ax 



And ir->0 

Therefore, finally, 



= (>«", ir-iC^«^(Car + C2). 



Digitized by 



Google 



109 



9002., (B. M. Lakolbt, M.A.)— If a oirde and a Simson-line of one 
of its points be both inverted with regard to that point, the two inverses 
w^ have the same relation to each and to the given point that the 
originals have. 

Solution hy the Pbofosbb. 

Let be the point, OD, OE, OP 
the perpendiculars from to the 
sides BO, OA, AB of a triangle in- 
scribed in the circle ; and let A', B', 

C, D', E', F be inverses of A, B, 
0, D, E, F. Then, since E, F, A 
lie on a circle through O, therefore 
E', F', A' lie in a straight line ; and 
since A, A', P, F' are concyclic, 
therefore Z OA'F' = OFA, there- 
fore OA' is perpendicular to E'F'. 
Similarly OB' is perpendicular to 
FD', and OC to D'E'. But, since 

D, E, F are in a straight line, there- 
fore jy, E', F' lie on a circle 
through 0, therefore A', B', C lie 
on a Simson-line of circle D'E'F'. 




8912. (Professor Bordaob.) — A square ABCD and a straight line A 
in its plane are gfiven, and through the points A, B, 0, D perpendiculars 
A A', BB', OO', DD' are drawn on A, A and being two opposite sum- 
mits; prove that (1) (BB')2+(DD')*-2AA'.BB' is a constant quantity 
for every position of A ; and (2) deduce therefrom an application for the 
envelope of the straight lines, such that the sum of the squares of the dis- 
tances of one of them from two given points is constant. 



Solution byCE. Williams, M.A. ; Prof. Byomakbsa Ohakbavaeti, M.A. ; 

and others, 

1. This is proved in Casby's Sequel to Euelidf Book II., Prop. 8, the 
constant being the area of the square. 

2. If (BB')'+(DD')> is constant, then AA'.CC is constant, and the 
line envelopes a conic whose foci are A and C. 



8979. (Professor Brunbl.) — Soient ABO un triangle, AiBiCi un 
autre triangle dMuit du premier en menant par les sommets A, B, des 
droites f aisant avec les cot^ du triangle et dans le mdme sens un angle ^ . 
Du triangle AiBiOi Ton deduit, de memo, un triangle AgBsOs et ainsi de 



VOL. XLVII. 



Digitized by VjOOQIC 



110 

smtOi ioujours avec le mdme anffle tp. Demontrer que lea points 
A, Ap Aj ... Bont BUT trois groupes de spirales logarithmiques ayant pour 
p61e8 les points de Krocard. Four quelles voleurs de ^ le triangle denve 
AiBjCi est-il 6g&l au triangle propose ? 



Solution by Profs. Nash, Schoute, and others. 

The construction shows that ABC is a Tucker triangle of AiBiCi, 
AiBjOi a Tucker triangle of the same species of Afi^^^j and so on. Hence 
the triangles have one or other of the Brocard points as a common centre 
of similitude, and mA/mAi » o^Ai / wAj ^ &c. » sin (» + ^) / sin a>, where » 
is the Brocard angle. Also the angles Aa^Ai, AiwAj, &c. are all equal 
to ^. Therefore A, Ai, A2) &c. all lie upon an equiangular spiral whose 
pole is <a. to will be the positive or negative Brocard point according as 
the line through A makes an angle ^ with AB or AG. The triangles 
ABC, AiB^Ci, ^\\G2i &c. will all be equal if sin (» + ^) « sin «, 
f.tf., if = X — 2ft>. 



8830. (Professor Bordaoe.) — Given the equation 

sin2a.a;^ + 2 (sin a + cos a) 2; •»- 2 * 0, 

find) and solve, the equation of the second degree that has as roots the 
squares of the roots of the given equation. 



Solution by W. J. Barton, M.A. ; W. J. Grbenstreet, B.A. ; and others. 

TM a AX. i. . « 2 (sin a + cos a) « 2 

If a, /8 are the roots o + ^ = ^ — :— ', a$ = t— r-i 

sin 2a Bin 2a 

whence a^ + 02^—L^^ a^02 * 



Bin2 2a' sin^ 2a' 

and equation required is sin^ 2ax^—4x-¥i ^ 0; 

of which the rooto are ^^^^f^^H or -1-, -1-; 
Bin2 2a sm^a cos'a 

as might have been expected, since the roots of original equation are 
— l/sina, - 1/cosa. 



8927. (S. Tebay, B.A.) — If Ai, Aj, Aj, A4 be the areas of the 
faces of a tetrahedron, E the radius of the circumscribing sphere, and 
Bi, Rs' ^8* ^4 ^^ ^^^^ of spheres passing through the centre of B and 
the angles of Aj, A3, As* A4 ; the volume 



^=*^'(l;+|*fc^f> 



Digitized by VjOOQIC 



ni 

Solution by W. J. C. Sharp, M.A. ; Professor Beybns, M.A. ; and others. 

If 0, Oj, O2, Oj, O4 be the centres of the spheres whose radii are R, Rj, 
R2, R3, R4, OOi cuts the plane of the triangle BCD, area Aj,- in the 
centre of the circum-circle Ki ; hence 

OKi + OiKi = Ri, OKi^ + CKi3 = R', 0, K^s + CKi« « Ri«, 
therefore (OKj + OiKi)(OKi - OiKO = il' - R^s, 

therefore OKi-OiK, = ^-Ri, therefore OKi = ^, 
R} 2Rj 

.-. the tetrahedron OBCD - iOKi. A, - JR- ^, OCDA = ^Rs ^, &c., 

R^ Rj 

and the volume = jRsf |; + ^' + |3 + 44] . 

(.Ri R3 R3 ^a) 

[A corresponding formula holds for all simplicissima. (See Quest. 
8242.) Thus, if Rj, R2, Rj be the radii of the circles through two vertices 
of a triangle, and the centre of the circumcircle (of radius R), we have 
Area = iR2 (aR; » + *R - » + cR- 1) ; 

and generally, if Y. be the content of a simplicissima in space of n dimen- 
sions, Ri, Rj, &c. the radii of the hyper-spheres through all the vertices 
but one and through the centre of the circumscribed hyper-sphere (of 
radius R), and Vj, Vj,, &c„ the contents of the simplicissima (in space 
of »— 1 (^mensions) whose vertices coincide with all but one of those of V, 

we shall have y = — 2 ^.1 

«'. Ri ■* 



8543. (R. Curtis, M.A.) — Prove (1) the following formula of trans- 
formation for the equation of a conic to a triangle of reference 
the sides of which are the polars of the vertices of the former 
triangle («, A, c,f, g, h) (r, y, ^)* « A-» (A, B, C, F, G, H) (X, Y, Z)*^; 
and hence (2) show ttiat the equations of a conic referred to an inscribed 
triangle and to one circumscribed at the points of contact will be 

ftfz-^-gzx + hxy « and (/X)* ± (^Y)* db (AZ)* « 0. 



Solution by the Proposer ; Professor Beyexs ; and others, 
ax -i- hy -i- yz = Xf hx + by+fz^Y, gx+fy + ez = Zf 
therefore Aar = AX + HY + GZ, Ay « HX + BY + FZ, 

A«=-GX + FY + CZ; 
therefore xX + yY + rZ = (a, b^ c^fj g, A)(;r, y, «)2, 

and A {jcX + y Y -r zZ) = (A, B, C, F, G, H) (X, Y, Z)' 

is («, ^ e, /, y, h) (X, y, 5/ = ^ (A, B, C, F, G, H) (X, Y, Zf. 

A 



Digitized by VjOOQIC 



112 



Again, if « - 0, * = 0, « =- 0, then A = - /», B = -g*, - - A«, F = ^A, 
&o. ; tiierefoTe, by above, 2/ys + 2gzx + 2Ad;y » becomes 

Of (/X)*+(5rY)» + (AZ) =0. 



8994. (Ohablottb A. Scott, B.So.) — ^A rectangular aheet of stiff 
pimer, whose length is to its breadth as V2 is to 1, lies on a horizontal 
table with its longer sides perpendicular to the edge and projecting over 
it. The comers on the table are then doubled over symmetrioallj so that 
the creases pass through the middle point of the side joining the comers, 
and make angles of 45^ with it. The paper is then on the point of falling 
over ; show &at it had originally f | of its length on the table. 



Solution ^G. E. Williams, M.A. ; W. J. Green 
STREET, B.A. ; and others. 

Let BD-.AD»DE-a, AG»2V2a. B 

Taking moments about A, we have 
4v'2a^3;-2a'.fa + 2«(2^/2-l)a.[a+(^/2-i)a] 

3 ' 12 V2 48 



A 



8856. (B. Hantjmanta Bau, B.A.)— AB is the diameter of a semi- 
circle; AC and BF tangents. GF cuts the semicircle in D. DG is 
drawn at right angles to GF, meeting AB in G. Prove that 
AG.BF-AG.GB. 



Solution by A. W. Gave, M. A. ; W. J. Greenstreet, B A. ; and others. 

Gircles can be drawn round the iig^es 
AGDG, DGBF, 
therefore Z ACG- Z ADG - 90°- Z GDB 

= 90°-ZGFB = ZFGB, 
therefore triangles GAG and BFG are 
equiangular; therefore 

AG : AG « GB : BF ; 
therefore AG.BF = AG.GB. 




Digitized by VjOOQ IC 



113 



9060. (Maurice D'Ocaonb.) — ^AB et MN 6tant deux diam^tres d'un 
memo cercle, si une parall^le quelconque k AB coupe la corde NA en B', 
la corde NB en A', et que les droites MA', MB' rencontrent le cercle 
respectivement aux points A" et B", les droites AA", BB" se coupent au 
pied H de la perpendiculaire abaiss^e du point N sur la droite A'B'. 



Solution by Rev. D. Thomas, M.A. ; 
J. H. Stoops, M.A. ; and others. 

Let the figure be drawn, and 
join AH, HA", 

NHA" = NAA" 

= i(NA" + MB) 

= i(NA" + AM'), 

ANH = i (AM'), 

therefore 

NHA"-ANH - i (NA"), 

•whichshowsthat AHA"are coUinear, 
similarly, BHB" are collinear. 




8905. (Professor Neubbro.)— D6montrer qu'Ji tout point /de la dis- 
tance des foyers F, F' d'une ellipse E correspond une droite D exterieure 
au plan de E et parall^le au petit axe, telle qu*il existe le rapport con- 
stant c : a entre les distances d*un point quelconque de E k f et D. 
Lorsque/ occupe toutes les positions sur FF', D se d^place sur un cylindre 
qui a pour base une ellipse semblable k £. 



Solution by E. F. Davis, M.A. ; C. E. Williams, M.A. ; and others. 

Let PNP' be a double ordinate of the 
ellipse, the normals at whose extremities 
pass through f;x,y the coordinates of 
P. With centre /aud radius /P, describe 
a circle having double contact with E. 
Then, if Q be any other point on E, the 
tangent from Q to the cirde — ^ x perpen- 
dicular QL on PP', or 
C^-P/2«^.QL«, Q/T»=^(QL2 + P/2/^). 

If, therefore, the line D be parallel to PP' 

(or BB') at a height z above the plane of E » "Pfje, which is of course 

independent of Q, we shall have Of ^ ex perpendicular from Q on D. 

Since z^ - P/»/«!2 = (W«2^2) (a3-*2a;S), we have x^a^ + z^/b^ = 1/^ as the 
equation of the trace oi the cylinder (swept out by D) upon the plane 
tlurough the major axis of E perpendicular to E. This is obviously a 
similar elli^iso passing through the feet of the directrices of E. 




Digitized by VjOOQIC 



114 

The above semi-geometrical method is only applicable when / lies 
between the limits x ^ -^ m^ and the circle is iW ; analysis woold, 
however, not require any distinction to be made between the cases when 
the circle was real and when imaginary. 

When/ lies between x^ae^ and j:=tf«, although the circle is imaginary,, 
the chord of contact is nevertheless real, lying outriide the ellipse. 

The focus F itself may be regarded as a point- circle having imaginary 
double contact with the ellipse, the chord of contact being the directrix. 
In this limiting case, when /reaches F the line D lies in the plane of £, 
and in fact coincides with the F directrix. 

[Let (j-, y) be the point P, {k, 0) the point/, and (a; = X, « = Z) the line 
D : then P/^ = (x ^ky + i/^ = a-'^c^x^^2xk + k^ ^ ^ and (distance from P 
to D)-= {x — X)2 + Z^ ; hence, comparing coefficients, Ff : distance from P 
to D « c : a, iiX^a^c-H; X"^ + Z-^ = a^e-'^ {k^ + b^) ; and, eliminating k, 
we get, for the base of the cylinder described by D, a" =*X^ + ^-=^Z*=a2c- '-*.] 



8521. (Professor Wolstenholmb, M.A., Sc.D.) — The circle of 
curvature is drawn at a point P (aw^, 2mn) of the parabola y- = ^ax, RR' 
is the conmiun tangent of the parabola and circle, and meets PQ their 
common chord in T ; prove that 

(1) TR : TR' = 1 + 4m2 : 1 ; (2) RR' » l^arn^ ( 1 + m^)* = PQS/ 32a2 ; 
(3) TQ : TR'= TR' : TP = m^ : 1 +m^i 
and (4) the locus of T is a sextic having no real rectilinear asymptotes. 



Solution by R. Knowles, B.A. ; Rev. T. Galliers, M.A. ; and others. 
The equation to the circle of curvature, centre O, is 

a^i + yi _ 2« (2 + 3w2) X + ^aniMf — Zahn* s= ; 
putting X = y^/iay this reduces to {y—2am)^(y + 6am) — ; hence the 

coordinates of Q are 9am^, —6am; therefore PQ = Sam (I +m^* (1), 

and its equation is x-^my—Zam'^ = (2). 

The equation to the tangent to the parabola at R'(a/y') is 

y'y^2a{x + x') = 0, 
and, since this is also a tangent to the circle, we have 

a (2 + 3w2) ^ ,;,3y _ (i + ^^jS) ^iax" + 4a2)i = ; 
putting a/= y'^/4a, this reduces to (y'—2am)^ [y' + 2am (4m2 + 3)] = ; 
therefore the coordinates of R' are a//*2 (4m2 + 3)2, -2a»» (4m2 + 3), hence 

RR' == (0R''-.0R2)^ =. I6am^{l+m^)^ (3), 

and its equation is x + m (im^ + Z) y + anfi {im^ + Z)- = (4). 

From (2), (4) wc obtain x = am^ (2w3 + 3) (4^2 + 3) / (2«i2 + 1), 
y = — 2am (hn* -t 6m' + 3) / (2w*2 + i)^ 



Digitized by VjOOQIC 



116 

coordinates of T. Whence we have 

TP = 8aw(l+m2)$/(2m2+l), TQ « 8«w»(l +w2)i/(2m»+ 1)...(6,6). 

TR'«8am3(l+m3)l/(2m2+l), (7). 

TR = RR'-TR'=8am»(l+m2)t(l+4m2)/(2m2-Hl) (8), 

From (1), (3), (6), (6), (7), (8) we obtain the results in parts (1), (2), (3) 
of the question. 

(4) The locus of T is obtained by eliminating m between equations (2) 
and (4), and we get, after some labour, the sextic 

8i^« + 162«V + 729aV + 72a»y* + lexY 

+ 108a3d:V+ 324a3ay« + 48air*y3-2916a»a; 

-3888a4a;2-432aV-576aV-64aa:» « 0, 

whose rectilinear asymptotes are given by the equations y s oo , 
y = ± (- 2)* a? + B, and are therefore not real. 



8718. (Professor Neubbro.)— L'angle BAC d'un triangle est fixe, 
et (1) le centre de gravity, (2) Torthocentre, (3) le centre du cercle cir- 
conscrit parcourt une droite donn^. Trouver I'enveloppe du c6t6 BC. 



Solution by the Fboposbr. 

V Lorsque le centre de gravity G parcourt la 
droite EF, le sommet du parall^logramme GABD 
d^rit une droite E'F' parallMe k EF. Par con- 
sequent, BC enveloppe une parabole tangente h 
AB, AO aux points de rencontre de oes droites 
avec E'F'. 

2° Lorsque Torthocentre H parcourt EF les ^^-^^ \ / 

hauteurs BH, CH marquent sur AB et AC des ^^*n^ \ / 

ponctuelles semblables. Done I'enveloppe deBC est ^'^ ^, / 

encore une parabole. ^^ 

3® Les projections du centre O du cercle ABC, 
lorsque se meut sur EF, marquent sur AB et AC des ponctuelleB 
semblables. Done BC enveloppe une parabole. 




8920. (B. Enowles, B.A.) — From a point O (x, y) tangents ste 
drawn to meet the ellipse a^y^ + h^x^ = aH'^ in P and Q, a tangent parallel 
to the chord PQ meets OP, OQ in p and q respectively ; prove that 
j3^ : PQ = (aV + ^^a;2)* : «*+ (ay + *«a:2)4. 



Digitized by VjOOQIC 



116 



8oluti<m by Gbbtrudb Poole, B.A 
Let ON, On be the perpendiculars 
from (4?, y) on PQ, pq ; then PQ 
Xa; Yv , 

ON - <*V-»-^^*-«'^^ . 
{a*x^ + b*y^^ ' 

^.isy + g^.^C^s^ + aVV, 

and PQ : /^g- = ON : 0«, therefore 

PQ : i»? = (aV + If^x^-a^l^ : {aV + *'«» + «* (**«* + «V)*} 
» stated result. 




8236. (Rev* T. R. Terry, M.A.)— A uniform lamina bounded by 
the arc of a parabola and its latus rectum is revolviog with angfular Telocity 
M about the latus rectum : suddenly the latus rectum becomes free and the 
vertex becomes fixed. Show that the angular velocity about the tangent 
to the vertex is )«. 



Solution hff Rev. T. Gallibrs, M.A. ; 0. Morgan, M.A. ; and others. 

Let the equation of the parabola which forms the 1/ 
curved boundary of the laxnina be ^ = iax, and let 
PMP' be a double ordinate of the curve at a distance 
AM s a; from the vertex, yAi/ the tangent at the vertex. 

The only dynamical condition is that the moment of 
momentum of the lamina about yA/ before the vertex 
becomes fixed is equal to the moment of momentum of 
the lamina about yAt/ after the vertex becomes fixed. 
Let f»' » angular velocity about tangent at vertex after 
the vertex becomes fixed. 

Now, in the first case, the velocity ot every point ^ 
in PMP' = («— a?) » perpendicular to ^kne of lamina, 
therefore moment of momentum of lamma about yAy' before A becomes 

fixed = A: r a?(a— a?)».y«iir, 

where A; Is a constant depending on the nature of the lamina. Also moment 
of momentum of the lamina about yAy^ after A becomes fixed 

= A; r a?.a?.«'. yeto, hence a> I [ax-^^^y d» — vf \ a^ydx^ 
or, by (1), w f* («P*-«8) <te - «' [* x^dxy .-. (f-f) « = ?«'> ^^ «*' = !•• 

Jo JQ 



Digitized by VjOOQIC 



117 



8302. (A. Gordon.) — Prove that the surface it' + y* + e' « c* will 
represent a surface of revolution if ooB^xy » cos^yz » oos^zx, where xy 
d^otes the angle between the axes of x and y. 



Solution by Professor Ignacio Bbyens ; the Peofosbr; and others^ 
Let x^+y^ + «2 become A (X' + Y^ + CZ' by change to rectangular axes, 
therefore a^ + y' + z^—\ {x^ +y^ -^ z^ + 2xy cos xy + 2yz cos y« + 2m? cos a* ) 
becomes A (X« + Y^) + CZ^- \ (X' + Y^ + Z^) ; 

but the latter equated to 0, and hence the former, represents two coind* 
dent planes when A » A, viz., 

(0-A)Z2=«0 or (te + »iy + «a)' = 0, 
therefore 1 - a = /^ - m* = »*, —A cos"^ = Im, 

— Acosyas = mn, — Acosac = nl, 
from which coa^xy = cos^yz — cos'za;. 



8336. (Asparagus.) — From a point are drawn two chords OPP', 
OQQ' of a given circle, and two tangents OA, OB ; a conic is drawn 
touching the straight lines PQ, PQ', P'Q, P'Q' and passing through A 
or B ; prove that this conic will touch the circle in A or B. ' 



Solution by H. London, M.A.; R. Knowles, B.A. ; and others. 
Given a conic touching PQ, PQ', FQ, FQ', and passing through B* 





Let K, L, M, N be points on the reciprocal conic whose polars are P'Q', 
PQ , FQ, and PQ. Then T, R, S, O are the poles of DC, PP', QQ', RS. 
But, if SR meet the conic at V, TV is tangent by the properties of a 
quadrilateral ; therefore T is the pole of SV, therefore DO is the polar 
of O. Hence OB is the tangent to the given conic at B. 



VOL. XLVII. 



Digitized by VjOOQIC 



118 

8656* (W* J* Gbbbnbtrbbt, B.A.)'In a spherical triangle, prove 
that im^B » (co8a-co8>R}' + 8m*a8iiL3Bsm3(S-A). 



Solution hy Rev. T. Gallib&s, M.A. ; Sarah Marks, B.Sc. ; and others. 

We have tan R cos (8 — A) = tan ^a, 

therefore sin' (S - A) = 1 - tan' \a cot« B, 

therefore (cos a - cos' B)' + sin' a sin' R sin' (S - A) 

= oos'a— 2cosaco8'R-hcos<R + 8in'asin'R— sin'asin'Rtan'^cot'B 
= C08'a + 1-2 sin' R + sin^R+ sin' R-cos'a sin'R-1 -Hsin'R 
— co8'« + cos' a sin' R = sin** R. 



8661. (A. Gordon.) — Any curve of the 4th degree intersects 
a conic in 8 points. These are joined, forming an octagon A1A3 ... A«. 
Show that the 8 intersections of Ai with A4 and A«, A3 with A« and Ag, 
&c., &c., will lie on a conic. 



Solution by H. G. Dawson, M.A. ; Professor Nash, M.A. ; and others. 
Dr. Salmon {Higher Flane Cttrves, 3rd ed., p. 19) proves that, if a 
polygon of 2f> sides be inscribed in a conic, the n{n—2) points where each 
odd side meets the non-adjacent even sides will lie on a curve of the «— 2^ 
degree ; therefore put » = 4, and we have the required theorem. There 
does not appear, then, to be any necessity for the 8 points to lie on a 
quartic. 



863L (Professor Sylvester, F.R.S.)—Find the discriminant of 
a^ + y» + gS + Se^h/ + 6exyz. 



Solution by Professor Mathews, M.A. 
Let ttSa;«+y»-H«» + 3«2«'y + 6«cy«s«(a; + y-H «)»-(«- l)(«»-Hy» + »»). 
Then, if the curve m «> have a double point, 

e(j; + y + «)'-(<?-l)«'= 0, tf (a; + y + «)»-(tf-l)y' » 0, 
«(a; + y + «)'-(«-l)a'=0, 

therefore ^ (^) ^ {^) ^ (J-f^ x, 

and the discriminant is found by rationalising this equation. 
Now, the rationalised form of «;* = ± x^±y^±z^ is 

(w' + X''\-y' + «'- 2i0X'- . . . - 2y2- . . .)'- 64xy»io - 0, 
and, putting w — e^l, x '^ y ^ z ^ e, the left-hand side becomes 
\[(tf-l)i-«tf(<j-l)-3i,8}2-64«3((.-l)=-(l + 4e-84j')'-64«»(#-l)-l + 8r. 



Digitized by 



Google 



119 



(B. Hanumanta Rau, M.A.)— If a body describe an ellipse 

about a centre of force in the focus, the time of describing the arc between 
the extremities of the latera recta on the same side of the major-axis is to 
the periodic time as sin~^ (e) : «-. 



Solution by Professor Stonb, M.A. ; 
C. E. Williams, M.A.; and othert. 

The times are as the areas described 
round the focus; but area LSL'= LCL' 
and LCL' : area of ellipse » KCK' : area 
of aux. circle 

« angle K'CB : ir ■■ sin-* {e) : ». 




8298. (R- Knowles, B.A.) — I£pr denote the coefficient of aT, in the 
expansion of (1 + x)**, where n is a positive integer, prove that 

Solution by Profesaor Sikcom, M.A. ; Professor Hatz, H.A. ; and othert. 

--" '-^*»--*'^-i:ic<-'>*'- ■ 

or, writing 1— « for a?, we have the integral ^ 

n+l Jo l-x » + I V 2 »+l/ 



8423. (I>. Edwardbs.)— Prove that 

(1) (*" sin X (log sin a;)3 £te = 2 + (log 2)«- 2 log 2 - -^ir*, 
Jo 

(2) l*'sin«a;(logsinar)«(to = Jir{2(log2)2-21og2 + Jir»-l}. 
Jo 

Solution by Professor Sebastian Siucom, M.A. 

1. f^'sina; {logamx)^dx = i T {log (l^x^Ydx 
Jo Jo 

= if[{log(l-j)}-i.2log(l-a;)log(l+») + {log(l+a:)}']rf(», 
Jo 



Digitized by VjOOQIC 



120 

f*{log(l+a:)}2(««-2(log2)«-41og2 + 2 (by Quest. 7797)... («), 

£ {log(l^x)Ydx - £ (logy^^y - 2 (3). 

Alflo log(l+a?)log(l-«) -«-(a; + iar«+ijr3 + ^a;4+...)log(l+ar), 
and f x'«log(l + fl>)(;^a: 

--i-.log2 L^f J---i+...(-l)"-i + (-l)«log2], 

hence f log(l+a?)log(l-a?)(ia? --21og2 ^ J— + _L^+...\ 

-21og2aog2-l)-2(^+i- + -i- + ...)-l 



+ 



1 



- + -^ + ...-22Lii— («>m). 



1.2 2. 

therefore 2 [^ log {l-i-x) log (l--x) dx^ 2 (log 2)^ -A log 2 + i-\ii' ...(c), 
Jo 

aince (log 2)2 - Jir« + 22 ^— ^^ (» > m). 

m ft 

Adding (a), {b), (c), we obtain the required result. 

2. We hare (log cos x)^ = {log (2 cos a') - log 2}' 
■= {cos 2a)— i (cos 4aj) + ^ (cos 6x) — .. }* 

-2 log 2 {cos 2aj-i (cos ix) + J (cos 6a;)- ...} + (log 2)». 
Expanding, and patting 

2 cos 2miB cos 2 (m + 1) a; s= cos 2x + cos 2 (2m + 1) jt, 

vehave'(logcoSfl>)2 « ^^ir« + (log2)3-co8 2a; f21og2 + -i--H -i- +...'\ 

\ 1.22,3 / 

+ terms in cosines of multiples of 2x 

= ,.Vx«+(log2)2-co8 2x(21og2 + l)+ (a). 

Integrating between ^t and 0, we obtain the known result 

[*' (log cos xy dx - ^ir8 + Jit (log 2)2. 
Jo 
Multiplying [a) by cos 2x and integrating, 

r cos 2x (log cos a;)2 rfa; « — iir log 2 - ^ir, 

then r* sin2 x (log sin x)^ <f « « I ' co82 x (log cos a;)2 dx 
Jo Jo 

= J I * (1 + cos 2jr)(log cos xydx 
.^ir{2(log2)2-21og2 + iir2-l}. 



Digitized by VjOOQIC 



121 

[Referring to the solution of Question 8466 (Vol. xlvi., p. 76), we have 
1 1 sin a (log sin 6 cos <^)' ded<f> = 1 1 sin m (log «)2 dto (f/i » ^ir [ (log z)^ dz = ir. 

The limits are and ^ir throughout. Also, as is proved in the solution 
referred to, I sin 6 log sin 6rf6 = log 2 — 1 ; 

and we have the known results 

f logcos<^rf<^ == -Jirlog2, f (logco8<|))2(f^ « i'r{(log2)« + ^ir'}, 
and hence the first result immediately follows. 

Again, j | sin a cos a (log sin e cos ^)^d0d<t> = j j sin^ tosmfx (log cos«)'^« dfi. 

And [ sin a cos 6 (log sin 6)^ dd— iz (log z^dz -« i, 

f sin a cos e (log sin 6) rfa — — i, [ sin a cos Ode = i, 

80 that 

^ir- J |logcos<^e/^ + i r(logcos<^)V<^ « (logcosa»)Va»- fcos^(logcos«)'rf», 

giving f cos2 n (log cos w) 2 rfw = Jir { 2 (log 2)' — 2 log 2 - Jir^) } ] . 



8728. (Captain H. Brocard.) — De chaque sommet d'un triangle 
on m^ne des perpendiculaires aux cStes adjacents, jusqu'^ leurs rencontres 
avec le c6te oppos^. Les centres A', B', C des cercles circonscrits aux 
trois triangles ainsi formes sent les sommets d'un second triangle homo- 
logique avec le propose. Le centre d*homologie est le centre O du cercle 
circonscrit au triangle donn6. En d'autres termes, les nouveaux cercles 
sent tangents au cercle circonscrit, aux points A, B, C. 



Solutions by (1) C. E. Williams, M.A. ; (2) Professor db Lonqchamps. 

1. As BAoj, CA^Tj are 
right angles, we have the 
angles BA^„ CAaj equal. 

If TA be the tangent at 
A to the circumscribed circle , 

TAB = ACB, 
therefore 

TA^i = ACB + CA«2 = Aoafli, 
therefore TA is also the tangent to the circle circumscribed to A«i«2i 
therefore the circles touch and their centres are in a line with A. 

2. Otherwise : — Soit ABC le triangle propose. Les perpendiculaires 
elev^es au point A aux c6tes AC, AB rencontrent BC en des points a^ 
a2i et il faut demon trer que les cercles Aa^a^t ABO sont tangents. 




Digitized by VjOOQIC 



122 



En coordozmees barycentriques, et d'apr^s la notation que nous avons 
propos6e (M6moire **Sur un nouveau Cercle remarquable du plan d'un 
triangle, '^ Journal de Mathimatiquei Spieialei, 1886), le cercle AayO^ peut- 
dtre repr^sente par I'^uation 

(a + /8 + 7)(Ma + t>i8 + w;7)-f = (A). 

Dans cette ^galit^ u, v, w repr^sentent, en grandeur et en signe, lea 
puissances des sommets A, B, G par rapport au cercle Aaja,, et ( le 
premier membre de F ^nation du cercle ABC. 

Nous avons, dans le cas pr^nt, 

w«0, v = - Ba, . Bflf- == -A; ( « ^Ii 

' * ^ cosBV cosCr 

On a, de mdme, w « -^ ^^ ^ , 

' cos B cos C 

et, finalement, P equation de AaiO^ est 



ou r«»— c? 



cos A 
cos B cos C* 



cos A 



,(a + ^ + 7)(c«/8 + ftV) + f-0. 



cos B cos C 

L*axe radical A des cercles ABC, Aa^a^ est done repr^sent^ par T^qua- 
tion c^fi + b^^O ; c'est-^-dire que A est, pr6cis^ment, la tangente en A 
au cercle ABC. 



pr^c6dente, 

immSdiatement, et presque sans effort, k ces nombreuses questions de la 
g^m^trie modeme dans lesquelles, consid^rant diffi^ents cercles deriv^ 
du triangle, on demande d' observer quelques unes de leurs propriety. 



8946. (W. J. 0. Sharp, M. A.)— Show that 



X, -a, -a, 
b, X, -a, 
by b, X, 



b, b. 



-a, -a 
—a, —a 
-a, -a 



b, ... X, -a 
b, b, ... b, b 
where the determinant has n+l rows. 



= b{x + a)'*, 



Solution by Isabel Maddison. 
Multipljring the last row by ab-'^, and adding to each of the other rows, 



the determinant becomes 



. 

* + a, b + a^ x + a . 

b + Uf b + ttf b + a . 
bs b. b, 



0, 





0, 





0, 





a; + fl, 





*, 


b 



= b (» + »)». 



Digitized by VjOOQIC 



123 

8948. (W. J. C. Shabp, M.A.)— Show that 

n— l!Bm'*6\ dej 

and co8«a = — — : (sin^tf— \ (sin 9 cos 0). 

«— llsin^tfV de) 

Solution by D. Edwabdbs. 

/ rf.»-i j_ (w-U! Put <=.- cote, ^^--1, then the 

\dt] t-^j ^ ' {t+jY ' ^ 

left side becomes — ( sin* ^-j-] ^^i ^ (cos ^ -»-/ sin 0), and the right side 

becomes — (fi — 1) ! sin** (cos n0 +/ sin n0) . Equating real and imaginary 
parts, we have titie formulsB in question. 



8754. (Professor Mahbndra Nath Ray, M.A., LL.B.)— Prove that 

f^xws^xdx ,, ,» r*2;sin'0e£r / -v 



Solution by A. Gordon ; W. J. Grbenstrest, B.A. ; and others. 
Galling Z the first' integral, Y the second, we have 

Jol + cos2a; Jol+cos^a; 

Jo \l + cos'iC l+sm^xj 

therefore Z = ir (- 1 + ^x). Similarly, Y = »(-3 + J + ir)r=ir (ir-|). 



8671. (P' Edwardes.) — Prove that the latua rectum of a parabola 
is half the harmonic mean between any two focal chords at right angles 
to one another. 

Solution by the Rev. T. Gallibrs, M.A. ; W. P. GU>udie, BAl. ; and others. 
From the equation 

r=:— ^^ weget f ^ —^ and/=-~^, 
1 + costf * "^ 1-cos'tf "^ l-sin«0* 

therefore -r^ + -=- = —• 

f f \a 



Digitized by VjOOQIC 



] 



124 

9017 & 2810. (A. Russell, B.A.)-(9017.) If «i, x^ ..., Xn - a,, 
Os> •••> <<») prove that 

ai(ai-«2)...(«i-a») Oj (oj-aj) ... (oj-Oj) 



Solution by Thomas Mui&, LL.D. 

9017. For convenience in writing, and not for simplification of the 
proof, let ft *B 4. Then the common denominator of the fractions is 

and the total numerator 

+ «i%»4 {* (<h«3«4) [«3* — '^^ ^1 + V 2^i*a - «i Xci%a?8 + XiX^e^il 

— + 

This, when the coefficients of 2^i, Xt^i^s, &o. are collected, becomes 

|«i'«s'«8'«4*l - ^ilV«3'«s'«4'| + 2afia?3|aiia,2a8»a4»| 

The first determinant here =^ a^a^i^^ ^ iflx^^^i the last» — ^ (<>i<W4)9 
and the others vanish ; hence Mr. Russell's expression is eqiuJ to 

2 XxX^X^X^ ^ 

2810. A generalisation including Professor Stlvbsteb's theorem in 
Quest. 2810 (see Vol. xlv., p. 129) may, in like manner, be readily 
obtained (see Froc, Roy, Soe,, Edinburgh, 1886-87). 



7661. (Rev. T. C. Simmons, M.A.)-If 

sin(a + /3) + sin(/3 + 7)+sin(7 + a) - 0, 
show that cosi (a— /3) cos^ (/3 -7) cos i (7— a) cannot be greater than |^. 



Solution by the P&oposbb. 

Let PQR be a triangle touching an ellipse in points A, B, C whose 
eccentric angles are a, /S, 7 ; then, by Quest. 7663 (Vol. xm., p. 73.) 

aABO : aPQR « 2 cos i (0-/8) cos i {fi-y) cos i (7-a) : 1. 

Now, let a, i8, 7 be such that sin (a + i8) +sin (i8 + 7) + sin (7 + o) = 0, 

then the normals at A, B, C will be concurrent, and in this case 
aABO : aPQR cannot be >i ; hence, subject to the above condition, 

2 cos i (0-/8) cos i (/3— 7) cos i (7 -a) cannot be > i. 

[A solution is desired by a direct method.] 



Digitized by VjOOQIC 



APPENDIX L 



SOLUTION OP QUESTIONS 8886 and 9009. 
By Rev. T. P. KIRKMAN, M.A., F.R.J 



8886. (By Rev. T. P. Kibkman, M.A., F.R.S.)— Two 28-edra have 
the following triangles, the summits being marked abc ,.. npq. Required, 
all the symmetry of both. 

abg, abe, acd, ade, aef, afff, hje, bihy bfh, bigi q/l, elk, ckd, dhm^ dfm, def, 
fmyfgQy 9Hy 9^*y ^Pj\ ^PiJlf^J^Pj f^nq, mnp, mpk, pqn, 

abffj afff, afe, aed, abc, acd, bhff, bhi, bci, cj'i, co{j\ djk, dkl, dfl, def, fkl, 
/km, fgrtiy ghm, yq, ihp, ipn, inq, jkn, jnq, kmn, hmn, hpn, 

9009. (Rev. T. P. Kirkman, M.A., F.R.S.)— P and Q are a regular 
20-edron and 8-edron. EILMN are 4-edra, each on a base that covers a 
face of P or of Q. Required the number of polyedra,t)f which none is 
either the repetition or the reflected image of another, that can be made 
by laying one or more of KLMN on as many faces of P or of Q, with an 
account of the summits and synmietry of the constructed solids. 



Solution, 

(9009.) The required solids are, one 7-acron, three 8-acra, three 9-acra ; 
six 10-acra ; one 13-acron, six 14-acra, thirteen 15-acra, and forty- two 
16-acra : in all 76 solids. Their summits and synmietry are as follows, 
where the terms preceding the colons are the summit-signatures ; e.g,, 
the 7-acron has the signature bH^'A^ showing three pentaces, three 
tessaraces, and one triace ; the first 15-acron has 87*6^5^3', showing one 
octace, two heptaces, two hexaces, seven pentaces, and three triaces. 
One 7-acron : 
1 ; 5^433 : 3-zoned monaxine heteroid. 
Three 8-acra: 
1, 2; 63534333, 65H33: both 2 -zoned monaxine heteroids. 
3 ; 5*33 : 3-zoned monarchaxine homozone, which has one janal 3-zoned 
axis, with repeating zones, and three like contrajanal 2-ple zoneless axes. 
Three 9-acra: 
1, 2; 7635*433, m^Z^i two monozones. 
3 ; 6^5^33 : 3-zoned monaxine heteroid. 

Six 10-acra : 
1 ; 86H3^ : 4-zoned monaxine heteroid. 
2 ; 7^5'3^ : 3-zoned monaxine heteroid. 
3 ; 7363533* : 2-ple zoneless monaxine heteroid. 
4; 76*53*: monozone. 

VOL. XLVn. Q 



Digitized by VjOOQIC 



126 

6 ; 6*3^ : zoned tnazine, haying three unlike janal 2-zoned axes. 
6; 6*3^: homozone triaxine, having one janal 2-zoned axis, 
with like zones, and two like janal 2 -pie zoneless axes. 

One 13-acron : 
1 ; 6S5'3 : 3 -zoned monaxine heteroid. 

Six 14-acra : 
1, 2 ; 7'6*5«3^ 6«5«32 : both 2-zoned monaxine heteroids. 
8, 4 ; 76«6732, 6«5«32: both monozones. 
5 ; 6*5*3' : 2-ple zoneless monaxine heteroid. 
6 ; 6*5^3' : 3 -zoned monarchaxine homozone. 

Thirteen Id-acra : 
1, 2, 3, 4, 5, 6, 7 ; 8726-5733, 876^5«33, 7"6«6«33, 726»5«38, 7^75438, 

7-67 '*33, 6'5'3' : seven monozones. 
8 ; 73635*33 : 3 -zoned monaxine heteroid. 
9 ; 6'5333 : 3-ple zoneless monaxine heteroid. 
10, 11, 12, 13; 7'635633, 72655«3S, 7675*3», 7675*3': four 

asymmetricals. 

Forty-two 16-acra : 
1, 2, 3, 4, 5, 6, 7, 8, 9 ; 973625»3'», 87563553^ 8726«5*3<, 87«6«543S 

8767533S 736«533S 736«533S 736*5334, 736*533^: nine monozones. 
10, 11 ; 83335*3^ 736*533-* : two 3-zoned monaxine heteroids. 
12, 13, 14, 15; 827W3S 7^6*6^3^ 726*523^ 7^^6^Z*i four 

2-ple zoneless monaxine heteroids. 
16, 17 ; 826*5^3S 726^523^ : two 2-zoned monaxine heteroids. 
18; 7*6^543*: zoned triaxine. 
19 ; 736*533^* : 3-ple zoneless monaxine heteroid. 
20; 726^5-3'*: 2-ple monaxine monozone, having a contra janal 

2-ple zoneless axis. 
21, 22, 23, 24, 25, 26, 27; 8276^563^, 873685*3'*, 873635*3*, 

87-^6*5*3*, 8726*5*3*, 8726*5*3*, 8726*5*3* ; 
28, 29, 30, 31, 32, 33, 34; 8767533*, 8767533*, 7*6*5*3*, 736*533*, 

7*6*533*, 736*533*, 736*533* ; 
35, 36, 37, 38, 39, 40, 41 ; 736*533*, 736*5*3*, 736*5*3*, 736*533*, 

726*523*, 726«523*, 76>*53* : twenty-one asymmetricals. 
42; 6*234. zoneless tetrarchaxine, which has four principal 

3-ple heteroid, and three secondary 2-ple janal, axes, all 

seven zoneless. 
There is nothing tentative in the enumeration or construction of these 
solids on the bases P and Q, to one familiar with the resulting changes of 
symmetry. 

(8886. ) — The answers are the 1 5«acra, 1 6-acra 42 and 1 3 above given. In 
first 24-edron, i, e, I, n occur thrice only, and in the second elqp 
thrice only. These summits are the vertices of the imposed 4-edra. 
IBither figure is easily drawn on a triangle whose sunmiits all occur more 
than three times. 

If we construct the first on dfmy we have a zoneless tetrarchaxine whose 



Digitized by VjOOQIC 



127 

four like 3 -pie principal axes hare for polar summits e^ i, ly n ; and for 
opposite 3 -pie polar faces, the triangles hpjf dkm^ fgq, and abe. The four 
janal secondary 2 -pie faces bisect the pairs of edges ag, kp ; bj, fm ; 
edf hq. If we efface the twelve bases of the triaLgles whose vertices are 
«, », /, w, we obtain another zoneless tetrarchaxine 16-edral l6-acron, with 
signature 4^ 3'*, autopolar ; and if we remove next the four triaces ^, i, /, n, 
we get a zoned tetrarchaxine 8-edron, which is obtained by cutting 
away the summits of a 4-edron. The reciprocals of all these are readily 
made from the hexarchaxine 12-edron. By cutting away four of its 
summits so as to make twelve 6-gons, we construct the reciprocal of 
No. 42, the first in Quest. 8886. 

If we construct the second in Quest. 8886 on its face abg, we have a 
2 -pie zoneless monaxioe heteroid, whose axis bisects the edges bg smdj'k, 

I have only recently learned, in tumiug over the leaves of Vol. lxviii. 
of Crellb'8 Journal f that in this aud Vol. lxvi. there is a treatise on the 
symmetry of Polyedra. Above twenty years ago, its French author did 
me the honour to converse with me in my house, in fluent and good 
English, on Groups and Polyedra. I was more amused than surprised to 
find that he seems to write, in Ckelle*s Journal, quite unaware that his 
subject had been previously and thoroughly discussed in Fkil, Tran. 
1862. My complete familiarity with all the aspects of polyedral 
symmetry lacks the appetite required for a real study of M. Job.dan'8 
Memoirs upon it ; but they ought to be useful to the student who wishes 
to verify the above solution of Quest. 9009. I may here, in mentioning 
Crblle's Journal, be permitted to request the student's help in the solution 
of a puzzle that beats me — how did the Cambridge and Dublin Mathematical 
Journal, Vol. ii., N. S. (vi. O. S.), p. 191, contrive to steal so much from 
a later paper in Crellb*s Journal^ Vol. lyi., p. 326, on exactly the same 
problem in combinations ? 

The following letter to me is before me, gummed into my copy of the 
learned and noble writer's life : — 

** 41, Chalcot Villas, Adelaide Road, N.W. ; 
'' April \%th, 1862. 

" My dear Sir, — I have to thank you for several papers, which I have 
looked at from time to time. Your excessively complicated subject ¥rill 
take shape in your hands I see. You have ascertained that the French 
Institute is mainly founded for the purpose of advancing the claims of 
Frenchmen — either by this means or that. But why go and buy your 
experience, when you might have had it for nothing from history? 
By printing in England you may save yourself from having your 
theorems given to a Frenchman. Yours truly, 

A. Db Morgan.*' 

The ** printing" is that of my Theory of the Polyedra, which is also 
the ** subject *' spoken of. The subject I have long, long ago exhausted, 
at least so far that I know how to answer any question that I know how 
to ask in it. The printing has been a distinguished failure. Yet I am 
satisfied that enough has seen the light in London and Liverpool to pre- 
vent the giving either of my theorems or of my methods to another. 

The Theory of Knots, which Professor Tait and I have pretty nearly 
wound up in the Phil. Trans, and Froc. Edin., is a brief supplementary 
chapter of the Polyedra. Every knot is a polyedron, or a polyedral 
reticulation, which has only tessarace summits, and which has or has not 
2-gonal faces. 



Digitized by VjOOQIC 



APPENDIX II. 



MISCELLANEOUS SOLUTIONS AND NOTES. 
By R. F. DAVIS, M.A. 



8559. (Professor Wolstenholme, M.A., Sc.D.) — From a fixed point 
O are drawn OP, OQ tangents to one of a system of confocal conies (foci 
S, 8', centre C), and from are let fall perpendiculars on the normals 
at P, Q ; prove that the envelope of the straight line joining the feet of 
these perpendiculars is the conic (parabola) 

■[X (a: cos a + y sin a) + Y (a: sin a— y cos a) — <^ cos a}' 

— 4XY (x cos a + y sin a) {x sin a~y cos a), 
where is origin, OS axis of x, (X, Y) the point O, SS'= 2c, and a is 
the sum of the angles which SO, S'O make with the axis of x. 

[If X2- Y2 - c^, the straight line is fixed.] 



Solution.* 

Let 0, tphe the eccentric angles of the points P, Q for any conic of the 
system whose semi-axes are a, *, where a^—b^ = e^. To find the co- 
ordinates of L, the foot of the perpendicular from the centre on the 
normal at P, we have 

da; cos 9 + 0y sin 9 a 0, 

fla: sin 6— *y cos 6 — c» sin 9 cos 9, 
therefore (a' sin^ 9 + *3 cos^ e)x — cH sin^ B cos 9, 

( )y « —c^^ sin 6 cos' 9. 

Similarly for M, the foot of the perpendicular from the centre on the 
normal at Q. 
The equation, therefore, to LM (whose envelope is required) is 

^ y 1 

c^aBin^ecose, — c^A sin e cos* , flSsin^e + J^cogSe 

<?2a sin' ^ cos ^, — c'^sin^cos'^, a'sin'^ + ^cos-^ 

or Aa? + By + C= (2), 

where, after reduction, it is found 

A« — c2i(sine— sin^)) {^-co8''«ecos'</>— a'sinasin^ (1 +sinesin4))}, 

B « ^a (cos ip - cos ff) {a2 sin' a sin' </>-*' cos a cos ^ (1 + cos % cos ^) } , 

Co* C^adsin0sin^cos0cos^sin(^— 0). 

* For this Solution I am indebted to the Editor. 



= ...(1), 



Digitized by VjOOQIC 



129 

The relations (useful generally) connecting 0, <p with X, Tare found to be 

(sin^— 8in6) : (cos©— cos^) ; sin (^—9) « AX : aY : ab (3), 

andcosacos</> : sinesin^ : I=a2(*2_y2) . ^(a2-X2) : (*2X2 + a2Y«)...(4), 
where (3) follows from the consideration that the tangents at $, f pass 
through X, Y ; and (4) from the cousideration that 6, ^ are the points of 
intersection of the polar of X, Y with the ellipse. 

By means of (3) and (4) we are enabled to substitute completely for 
$, <f> in (2) ; thus, 

{(i3.y2j2« (a»-X2) (63 + Y«)}Xc- {(a2-X2)3- (68- Y«) («« +X«)}Yy 
+ c2(a2-X2)(62-Y2) = (6). 

If we now put a'— X* « x— jc, 6-— Y* — X, where \ is indeterminate 
and X2-Yi-c2«n, then a^ + X^^ x-ic + 2X2, ^2 + Y^ = X2+2Y2 ; and 
(5) reduces to 

^x3+{(ic-2Y«)Xc+(2X2 + jc) Yy-<j2jc} \ + 2kY^Xx-'K^i/ = ...(6). 
Now, with the notation of the question, 

a « cot-» -^^ +cot-i 5±f « cot-i -^, 

therefore ic = 2XY cot a. 

Then equation (6) becomes 
<?2x' + 2Xr{(Xcoto-Y}ir+(X + Ycoto)y-<J«cota}A. 

+ 4X2Y8coto(jr-ycota) = (7), 

and the envelope required is 

{(Xcoto-Y)a;+(Ycota + X)y-c«coto}2 = 4c2Ycota (a;-ycoto)...(8), 

or {X (a;co8a + y sino) — Y(j;sino— ycoso) — c2cosa}3 

= 4c3Ycosa(arsino— ycosa)...(9), 
which can be easily shown to agree with Professor "Wolstbwholmb's 
result. 

In a former Note (see Vol. xlv., Appendix in., pp. 169 — 172) I 
have shown that, if O be a fixed point from which tangents OP, OQ are 
drawn to any one of a system of confocal conies, then ^1) the normals at 
P, Q touch the same parabola, which is the envelope of PQ, whose focus 
F is found by making the / PCS = Z OCS and taking CF . CO = OS', and 
whose directrix is OG; (2) these same normals intersect on a fixed 
straight line FH perpendicular to OF. I therefore re-proposed Professor 
Wolstenholmb's question in the form given below, the geomeCrical 
solution of which can be reconciled with the equation to the envelope 
given above (8). (See Appendix, Section 17.) 



8692. (R. P. Davis, M.A. Suggested by Question 8659.)— If, from 
any point in a given fixed straight Ime passing through the focus of a ^- 
rabola, tangents be drawn to the curve, prove that the envelope of the line 
joining the feet of the perpendiculars on these tangents from a given 
fixed point on the directrix is another parabola. 



Digitized by VjOOQIC 



130 



Solution by the PfioposBR. 

From any point T in the given fixed straight line ST, let there be 
drawn two tangents TP, TP' to the parabola focus S. Let B be a fixed 
point on the directrix ; draw BQ, BP, BQ' perpendicular to TP, TS, 
TP' respectively, and let the diameter through T meet the directrix in Ii. 
Then F is fixed point ; and Q, F, L, Q' all lie on the circle described 
upon BT as diameter. 

Let fall the perpendicular FK on QQ' ; then we shall show that the 
locus of R is a fixed straight line, and consequently the envelope of Q,Qf 
is another parabola having its focus at F. 




Since from the point T two tangents are drawn, / PTS = / FTL or 
/ QTF = Z LTQS therefore FQ = LQ' and FL is parallel to QQ' and 
perpendicular to FR. Now FR =- FQ sin STF = BT sin STP sin STP' 
= BT (SY / ST) (SY / SP) = BT (SA / ST). Produce FR to O so that 
FO « 2FR, and draw SM perpendicular to BT. Then FO. ST = BT.SX 
and FO/SX = BT/ST « BF/SM, hence FO/BF - SX/SM. 

Again, Z BFO = complement of ZBFL, or of ZBTL= zTBL 
= Z MSX, since S, X, M, B are concyclic. Thus the triangles BFO, 



Digitized by 



Google 



131 

MSX are similar, and / OBP « L XMS = L XBS. Hence the locus of 
O is a straight line through B, making with BF (the perpendictdar from 
B on the given fixed straight line ST) an angle OBF eqiml to the angle 
XBS between BX and BS. Also, since F is a fixed point and FR = JFO, 
the locus of B is a fixed straight line through the middle point of BF. 
Hence the envelope of QQ' is a parabola focus F. 



8768. (B. F. Davis, M. A.)— Required an Analytical Proof of Fbuee- 
BACu's Theorem. 

[An analytical proof is given by the Rev. J. J. Milne, M.A., in the 
Introduction to his Weekly Problem Papers, but this proof may be modi- 
fied and improved.] 

Solution, 

Let ABO be a triangle inscribed in a circle S, and t^i ^ ^3 be the 
lengths of the tangents from its angular points to another circle S'. Then, 
if at^ ± ht^ ± ct^ « 0, 

these circles will touch one another. 

For if Pn p^f i?3 be the perpendiculars from A, B, C on the radical axis 
(L) of ttie two circles, ^1*, ^j^ ^3' are proportional to Pi, p^, p^ respec- 
tively. Hence aVpi + b Vp^ + e ^p^ = 0, and this is the biown condition 
that L should touch S, or that the points of intersection of S and S' 
should coincide. Now the tangents from the middle points of the sides of 
any triangle to the inscribed circle may easily be shown to satisfy the 
above relation ; hence the circle through these middle points touches the 
inscribed circle. 



8875. (Professor Nash, M. A.)— Professor Casey's Quest. 7839 may 
be enunciated as follows : — DD', EE', FF' are the intersections of the 
sides of a triangle ABC with the cosine circle, the order of the letters being 
such that E'F, FD, IKE are diameters. The circle round AE'F cuts the 
circles on AB, AC as diameters in the points a, a' ; and d, h\ Cy (f are 
similarly determined upon the circles BF'D, CD'S. To show that the 
circles round AC0, BA^, QY^c pass through a> the positive Brocard-point 
of ABC, and the circles ABa', BC^', CAc' through the negative Brocard- 
point. Prove the following additional properties : — (1) The circles AOa, 
ABa' touch at A the sides AB, AC, and intersect again in a vertex of 
Brocard's second triangle ; (2) the taugents to tbese circles at aci bisect 
AB, AC ; (3) the points B, C, a, a' are concyclic, and the circle through 
them touches OB, DC ; (4) AaD and Ka'Ti' are coUinear ; (5) C«, Ba' in- 
tersect upon the symmedian AP, and Ca', Ba upon the perpendicular AL; 
(6) aE', a'Y meet BC at the foot of the symmedian AP ; (7) aa\ E'F, 
BO, and the radical axis of the circles ABC, AE'F, are concurrent; 



Digitized by VjOOQIC 



132 



(8) the three points of concQirence of the three sets of lines in (7) lie upon 
a line paralld to w»\ and therefore the triangle formed by the lines aaf^ 
hh'y ctf is in perspective with ABO ; (9) the pole of aa' with respect to the 
circle AE'F lies upon the median of ADD' ; (10) the pairs of points ac^^ 
ba\ cV are isogonaL, and the inscribed conic whose foci are aef touches 
CA at ^'d foot of the perpendicular from B ; (11) one directrix of this 
conic is the line joining to the intersection of FD' and DE', and the 
other the line joining A to the intersection of EF, FD'. 



Solution, 



Let ABC be a triangle, K its symmedian point, DD', EE', FF the 
intersections of the cosine circle (centre K) wiUi the sides of the triangle 
so that E'KF, F'KD, D'EE are diameters. 




Upon AD, AJy take points a, (^ such that 

Aa.AD =- Aa' . AD'== AF . AB = AE'. AC. 
Then, since F, a\ a, E' are the inverses (with respect to A) of the four 
colUnear points B, D', D, C, the circumcircle of AE'F passes through 
a, a'. Also, since B, F, a, D are concyclic, the Z BaD = / BFD = ^w, 
by a known property of the cosine circle. Hence a lies on the circle 
described on AB as diameter ; similarly a' lies on the circle described 
on AC. 

Since D, a, E', C are concyclic, and DE' is parallel to AB, Z AaC = ir 
- Z CaD = IT- Z CE'D = w - A. Similarly Z Aa'B = ir- A. Hence 
the circumcircles of ACa, ABaf pass one tlu-ough the positive Brocard- 
point of ABC and one through the negative Brocard-point ; while 



Digitized by VjOOQIC 



133 

(1) They intersect again in a vertex of Bbocard^s second triangle. 
Moreover, since Z BAa = / aDE'= Z aCA, AB is a tangent to the first 
circle. 

(2) The circles ACa, ABa may be considered as the inverses (with re- 
spect to A) of the perpendicular lines DB', DF. They therefore intersect 
orthogonally, and the tangent at a to the first circle passes through the 
centre of the second circle which is the middle point of AB. 

(3) Since F, D', D, E' are concyclic, their inverses B, a\ a, C are also 
concyclic. Now Z BaC = Z BaD + Z OaD = iw + A ; and if be the 
circum-centre Z OBC = Jjt— A, hence OB touches the circle Ba'aO. 

(4) The points a, of were assumed to lie on AD, AB' respectively. 

(6) Since DE', FD' are parallel to AB, AC respectively, they intersect 
in a point A' on the symmedian KA produced such that KA'=KA. 
Then, since AE'. AC = AF . AB and A'D . A'E'- A'D'. A'F, hence AKA 
is the radical axis of the circles CE'aD, BFa'D'. 

In the remainder of this solution frequent use will be made of the well- 
known theorem that the radical axes of any three circles (taken two at a 
time) are concurrent. 

From the circles CE'aD, BFa'D', Ba'aC we infer that Ca, Ba' meet on 
the synmiedian AKA' ; and from the circles ACa', ABa, Ba'aC, that Ca', 
Ba meet on the perpendicular AL. 

(6) From the circles CE'aD, BFaiy, AE'F we infer that dE', a'F meet 
on the symmedian AKA'. 

Moreover, since Z aBD - iir- Z oDB = ^ir- Z aE'C « Z D'E'tf, the 
four points B, D', a, E' are concyclic. Then, from the circles BD'aE', 
BFa'D', AE'F, we infer that cE', a'F meet on BC. But they have already 
been shown to meet on AKA' ; they therefore meet at the foot P of the 
symmedian ; and Ca, CA' divide the angle at C harmonically. 

(7) From the circles BFE'C, Ba'aC, AE'F we infer that aa% E'F, BC 
meet in a point a ; which, since aB . aC = oE'. aF, is also a point on the 
radical axis of the circles ABC, AE'F. If /3, y be similarly determined, 
a, /3, 7 lie on a straight line which is the Pascal line of the hexagon 
DD'EE'FF'. 

(8) This line afiy is obviously the radical axis of the cosine- and circum- 
circles, whose direction must be perpendicular to KO or parallel to «»'. 

(9) The pole of aa* with respect to the circle AE'F is obviously the 
symmedian line through A of the triangle Aaa' ; or the median line of 
the triangle ADD', since aa\ DD' are antiparallel. 

(10) Draw Bi/ perpendicular to CF', and BM to AC. Then, since 
ZBaC+ ZBi/C » iir + A + Jir = ir + A ; and similarly ZAaB+zA^B 
= ir+C; hence a, c' are isopfonal conjugates. Moreover, since ZaMB 
= Z aAB = Z f/CB = Z (j'MB, M is the point of contact of the side CA. 

(11) Since Z BMC is a right angle, and Z BMa = Z BAa == Z aDE' 
— Z aCM ; therefore Z MaC is a right angle and C a point on the direc- 
trix. Since CA, CB are tangents and a the focus, CA' (the fourth har- 
monic to CA, Ca, CB) is the directrix. 

[For information respecting the ** cosine- circle*' see the forthcoming 
volume, entitled ** Companion to the Weekly Problem Papers," by the 
Rev. J. J. Milne, M.A. (Macmillans), which has an admirable section 
devoted to the new geometry of the triangle, contributed by our 
correspondent the Rev. T. C. Simmons, M.A.] 

VOL. XLVn. R 



Digitized by VjOOQIC 



134 



Kon o9 THX Bbocabdai. EixmK. 




1. If a point Q be taken on 
tibe tangent at any point P of 
an ellipse (Fig. 1% sach that the 
angle SQP is constant (= fi\ 
the locos of Q will be a drde 
whose centre O lies on the 
minor axis of the ellipse, and 
which has doable contact with 
the ellipse. 

For, drawing SY perpendi- 
cnlar on the tangent, since 
ZQSY«i»-3, and SQ = 
SY coeec^, the locus of Q is a 
circle of radios a cosec $. Let 
O be a point on the minoraxis, 
soch that Z SCO = $9, then the 
triangles CSY, OSQ are simi- 
lar and in the linear propor- 
tion of 1 : cosec fi. Hence the 

locos of Q is a circle whose Fii^l. 

centre is O, and whose radius « SO/* =»the normal to the ellipse 
passing throogh O, and conseqoentlj has dooble contact (real or 
imaginary) with the ellipse. 

It is easQy seen from the above proof that, when SY revolves in the 
positive direction throogh an angle i*-3 into the position SQ, the locus 
of Q has its centre O on the minor axis at a distance aecotfi below the 
centre ; for the negative direction of rotation, O is at the same distance 
o^ofv the centre. There is no restriction as to the value of fi, and by 
varying it we get a series of circles having double contact with the 
ellipse. 

2. If the tangent to 
the ellipse at P (Fig. 2) 
meet a particular circle 
in Qi, and the other 
tangent from Qj to 
the ellipse meet the cir- 
cle again in Q^ then 
Z SQ2Q1 = fi ; aud so on. 
Starting from any point 
P on the ellipse in this 
way, we obtain an un- 
closed polygon PQ1Q2Q3 Fig. 2. 
...Qn of aoy number of sides we please, whose vertices He on the circle, 
and whose sides touch the ellipse, such that 

ZSQiP = ZSQ,Q, « ZSQ3Q2 = ... =: zSQ„Qn.i - fi; 
and con8equently,ifS' be the other focus, 

Z S'QiQ, = z S'QjQa -...==/ S'Q^.iQn = fi. 
'4. ^il ^^Z^ ^® determine jS, by means of the invariant condition, that 
It should be possible to inscribe figures of 3, 4, 6, 6.. .sides in the circle 
which at the same time circumscribe the elHpse, QiQc—Qn (of the pre- 




Digitized by VjOOQIC 



135 

yious article) is no longer an unclosed figure. The two foci become in 
this case the two Brocard points, and /S becomes the Brocard angle c». 
Taking the equation to the two curves in the forms 

a^/«2 + y«/*2= 1 (2% 

and »' + (y + a«j cot w)^ = «' cosec^ o», 

a;2 + y2 + 2a<?ycot«-(a2 + ^cot««) - (2). 

The equation determining #c, which causes 2 + k2' to break up into 
straight lines, is (after reduction) 

(ic + fl2)2(K + ^cosec««) - 0. 

Hence A', e', e, A are proportional to 1, 2a^ + b^ cosec' », a* + 2aH^coaec?a^ 
a*^co8ec2 w, or as 1, 2 + 2, 1 + 2z, s, where 2 = A* cosec^w / a^. 

Now, the condition that it shotdd be possible to inscribe triangles to 2 
that are circumscribed to 5' is e'-— 4A'e = 0. 
Hence (2 + 2)2-4(1 + 2a) =0, or z^-iz = 0, « - 4. 

Thus ; —i; sm«= — ; ^«1 — 1— 4sm'i». 

o^ 2a d^ 

Quadrilateral : e'3-4ee'A' + Sa^^a « 0. 

(2 + 2)a-4(2 + 2)(l + 2z) + 82-0, aj8-2«2- 0. « » 2. 

^^ ^co8e^^2 or sin., --A.. 

fl2 v^2a 

Hexagon : 

{e'2-4A'e}3 = 4 {e'8-4ee'A' + 8A'2A} {e'8-4ee'A'+i6A'«A}, 

whence « = ^. 

4. Summarizing results : — If B be the circum-radius and m the Brocard 
angle of a 

(i.) Triangle: the semi -major axis of the Brocard ellipse is Bsinw, 
the semi-minor axis 2 ft sin- w, and the eccentricity = v^ (1 — 4 sin^ w). 

(ii.) Quadrilateral: Rsinw, A/2Rsin2«, V(cos2«). 

2 
(iii.) Hexagon : R sin «, — — R sin^w, V (1 — 4 *^' »)• 

The above note was suggested by, and is an amplification of, No. 1056 
in Professor Wolstenholme's larger collection of Problems. Published 
so far back as 1878, it contains (^as was first pointed out to me by the 
Rev. T. C. Simmons) a precise and complete statement of the existence of 
the Brocard ellipse. 



Geometrical Construction for the Brocard- Angle, &c. 



I. Let ABC be a triangle. Describe a circle touching AB in A and 
passing through C. Let AP be the chord of this circle parallel to 
BC. Join BP, meeting the circle in a, which will be the positive 
Brocard -point. 



Digitized by VjOOQIC 



136 



For, siDce AB touches the circle in A, l AAB » I fiCA, and also 
« L APA b I ABC. An analogous construction gives the negative 
Brocard-point A'. 

II. Since L ACB - A^ 
L CAP and Z BAC » 
Z APC, the triangle PCA 

is inversely similar to 
ABC. Hence a simple 
ruler-construction can be 
stated for the graphical 
determination of the Bro- 
card-'ingle of a triangle. 

This construction (on 
the authority of the Rev. 
T. C. Simmons, M.A.) is 
already known to conti- 
nental mathematicians. 

III. Similarly, if onBC 
a triangle CQB be de- 
scribed inversely similar 
to ABC, ZAQB=ZAPB 
SB to, the Brccird-angle ; 
and, since Z ACP = B and Z BCQ = A, the points P, 0, Q are collinear ; 
also PC : AC « AB : BC and QC : BC = AB : AC, 
therefore PC . QC = AB^. 

Now P, Q both lie on the circle 2, which is the locus of points at which 
AB subtends a constant angle », supposing AB and w only given. If 
AB = a, the centre of 5 lies on the line bisecting AB at right angles at a 
distance therefrom = ^t^coto); while radius = ^acosecw. Moreover, 
since PC . QC = a^ = constant, the locus of C is a circle concentric with 
2, whose radius p is given by the equation 

W C08ec2 « - p' = a' or 4p2 = a^ (cot^ « - 3) . 
The locus of C under these circumstances is termed Neuberg*s circle ; and 
its equation, referred to AB and the perpendicular bisector of AB as axes, 
is «' + (y — ia cot w)' = ia^ (cot' «— 3) or x^ + xf^—ay cot « + J a' « 0. 




Digitized by VjOOQIC 



APPENDIX III. 



SOLUTIONS OF SOME OLD QUESTIONS. 
By W. J. CURRAN SHARP, M.A. 



2391. (Professor Sylvester, P.R.S.) — Let /u points be given on a 
cubic curve. Through them draw any curve (simple or compound) of 
degree v; the remaining 3v— m (say ^Q points may be termed a first 
residuum to the given ones. Through these /u' points draw any curve of 
degree / ; the remaining Zif' — fi' points may be termed a residuum of the 
second order to the given ones ; and in this way we may form at pleasure 
a series of residua of the third, fourth, and of any higher order. If fi is 
of the form 3«- 1, a residuum of the first or any odd order, and if /x is 
of the form 3i+ 1, a residuum of the second or any even order in such 
series, may be made to consist of a single point, which I call the residual 
of the original fi points. Prove that any such residual is dependent 
wholly and solely on the original fi points, being independent of the 
number, degrees, and forms of the successive auxiliary curves employed 
to arrive at it. 

3651. (Professor Sylvester, F.R.S.)— If through 3fi + I given points 
on a cubic curve a curve of the order N -»- « be drawn, and through the 
remaining 3N— 1 intersections of the two curves a third one be drawn of 
the order N ; prove that this will intersect the cubic at a fixed point. 

[This point may be called the opposite of the 3n + 1 given points ; it 
becomes Dr. Salmon's opposite, us defined by him in the Fhilosophical 
Transactions for 1858, when » = 1, N « 1, and is independent of the 
value of N.] 

Solution, 

(1) If /» =s 3t— 1. Since every «-ic curve through 3«— 1 points in a 
cnbic cuts it in the same additional point (Salmon's H. P. C, p. 133), 
any t-ic through the /u points will intersect the cubic again in the same 
point A (the residual). Let this be V = 0. 

Now if U a be a v-ic through the /u points and fti others, 
Ui =0 „ vi-ic „ Ml >t "f ii 

Us =0 „ va-ic „ /«2 i» A*« i> 

U2r =0 „ V2r-ic „ M2r „ 1 „ 

this one point will bo A. For evidently Zvr ^t*r + f^-nt 



Digitized by VjOOQIC 



138 

and if * + »'i + "s ••• + ''2r-i ~ "*» 
3m « 3» + 3vi + 3v8+ + 3i'2r-i 

= l + 3v + 3v2+ + 3^2^-2 +^''2r"'^ 

«» 3(^ + 1^3+ +vj 

and /tt + /ui + fi2+ ... +;*2^ — 3m— 1 ; 

therefore V.Ui . U3 ... U2r.i = and U .Ua... XJ2r - 
are m-ic curves througli the same 3m — 1 points on the cubic, and there- 
fore through the same additional point, which is A, the residual on the 
first, and the remaining intersection of Ujjr = on the second ; which proves 
the principle for this case. 

(2) In the other case it is advisable, first, to show that every «-ic curve 
through 3n — 2 points on a cubic cuts it again in two, such that the line 
joining them passes through a fixed point. 

For, if V — be one «-ic through the 3«— 2 points and A and B 
W — be another ,, „ „ and and D 

find a = be the line AB and /S = the line CD. 

i8V «» and aW « are two (« + l)-ics through the same 3« + 2 points 
(viz., the given ones and A, B, C, and D), and therefore their remaining 
intersections with the curve must coincide, i.e., a » and /S ^ meet the 
cubic in a fixed point. 

If /* = 3»+ 1, and V be (i + l)-ic through the fi points and A and B, 
and a = be the line AB, the third intersection of o = is fixed. 

Now let U s be a v-ic through the /x points and fti others 
Ui =0 „ vi-ic „ fAi „ fii „ 

then, if m = 1 +if + v2+ ... +^2^.2, 

3m = 3 + 3v+3y2+-- + 3i'2r.2 = 3 + /« + /Ui + ^ + jU3... +A*2r.i 

= 3l + 4+3Vi + 3>'3... + 3l'2r-l-l = 3 (l + » + lfi+V8... +V2,._,), 

therefore a U . Uj ... U2r-2 = and V . Ui . U3 ... U2r-i = are m-ic 
curves through A, B, and the /i + /*!••• +H-2r-\ poii^ts on the cubic, i.e., 

through the same 3»»— 1 points, and therefore the same additional point, 
which is the third intersection of a = (AB). 

This important theorem is thus fully proved. In Question 3651, the 
single point is the one co-residual to the 3»+ 1, and is therefore fixed by 
(2) above. 



3535. (Professor Sylvester, F.R.S.) — 1. Eltler has shown that the 
number of modes of composing n with i distinct numbers is equal to the 



Digitized by VjOOQIC 



139 

denumerant (that is, the number of solutions in positive integers, zeros 
included) of the equation 

a?+2y + 3a5+ ... +»» = »-i (fS+i). 

(1) Show more generally that the number of modes of composing n 
with i numbers, all distinct except the largest, which is to be always 
takeny times, is the denumerant of the equation 

Jit + {j+L)p+ ... +«« = «- J (f-y+l)(»+y). 

(2) Show also that, if all the partitions of n into t parts are distinct ex- 
cept the least J which is to be taken y times, then the number of such par- 
titions is the denumerant of the equation 

ay + 2y+ ... +{i-j)<f^ + iu ^ n-i {(i-jy + Zi^j}, 



Solution, 



1. The required number of ways of composing n with t numbers all dis- 
tinct except the largest, which is to be taken j times, is the coefficient 
of x**zi^ in 

a?'*' + (1 + xz) J^^ + (1 + xz){\ -H xH) x^s^ + &c. 

= a// { Ao + Ax2 + AggS + &c. } suppose. 

That is, it is the coefficient of x'^-iz^-i in 

l + (l+a;2)ar'+(l +X2)(l + a;'2)i;^ + &c. = A<, + AjZ + Aja* + &c., 

where Aq « .. 

Let xz = y, therefore "" 

l + (l+y)a;^+(l+y)(l + ay)a:^ + &c.«Ao+'^y+^y2 + &c.; 

X x* 

but l + (l + y)a^'+(l+y)(l+iry)ir^ + &c. 

= l + (l+y);t^{l+(l+xy)a^+(l + a:y)(l+fljV)»^ + &c.} 

« l + (l+y)a/{Ao + Aiy + Ajy2 + &c.}, 
and, equating coefficients, 

^-;r^(Ao + Ai), ^ = a^CAx + Aj), and generally^ - ar'CAn-i+AJ ; 

therefore Aj — ~ Aq, A^ =* r^^ Aj, &c., 

1—* 1— » 

A„ 



(lW;(l-ar'*')(l-a?'^^..(l-a;^*")' 
and therefore the number required, the coefficient s^"^ in A»_,-, is the 

coefficient of ;p*-*('->*i)«*» in 1 , 

(l-ic')(l-a^*')(l-;r^^*)...(l-aO 
which is the denumerant of 

ya; + (y+I)y + (y+2)«+ ... +t«=.n-i(t-y+l)(f+». 
2. If all the partitions of n into t parts are distinct, except the least, 
which is always to be taken/ times, the number of these is the coefficient 
of df"«< in a?'(»0(l+ir«2)(l + a^)... + ic*V(l + a:S2)(l+;r^2)... + &c., 



Digitized by VjOOQIC 



140 

or that of aj^'V"-' in 

(l+a?»«)(l + »»«)... +a?^ (I + aj%)(l +a;*«)... + &c. = Ao+Ai« + A2«*+&c., 
suppose. 

Uxz^y this becomes 

(1 ■¥xy){\ ¥x^y){l + ajSy)... + of {l-¥a^y){\ +«3y)... + &c. 

= Aq + Ai-^ +A2^ +&C., 

X X* 

but the first member = (I + xy) (1 + x^y){y + ic^y) . . . 

+ a;^' {(1 +«V)(1 + «'y)-.- +^(1 +^y)(l + «'V).- + &c.} 

= 1+ r^-y+ -. ^. ^y3 + &c. + ar'{Ao + A^ + A2y» + &c.}, 

1— a? (1— flj)(l — a:-*; 

A ar***^""^^ 
and therefore -f == — r- — + A„ar, 



An = ^ 



a; 



in(n-t-3} 



'(l-a:)(l-a;S)...(l-a:")(l-a/*")' 
and therefore the number required, the coefGlcieDt of xl^'i in A<.^, is the 

coemcientoi a; in (i_^)(i_a;3)...(i_a;.-i)(i_a:0' 

which is the denumerant of 

a? + 2y+... + (t-^') <► + «« =n-i{(t-»2 + 3t-y}. 



3427. (Professor Sylvester, F.R.S.) — ^If ^, ^ are quantics in x^ y, 
each of degree /a ; F a quantic in ^, if^ of degree m, and. consequently in 
X, y of degree m/i ; and if J denote the Jacobian of 4>, ^^ that is, 

dtp d^ d<f> d}j/^ 
dx dy dy dx* 

D^,^ F the discriminant of F treated as a quantic m ^, if^ ; IDx, yt F the dis- 
criminant of F, treated as a quantic in a;, y ; and if E be used as the 
symbol of " resultant in regard of a?, y ** ; prove that 

D,,y F = 2 [R «►, ,^)]«--2m E (F, J) (J>^,^¥y, 

As a particular case of the foregoing theorem, show that the discri- 
minant of F, any symmetrical quantic of an even degree in ar, y, is of the 
form F (1, 1) F (1, — 1) Q.^, where XI is a rational integral function of the 
coefficients in F. 

Find also what the general formula becomes when ^, 4^ are taken linear 
fiinctions of a;, y. 

Solution, 
By the Question, F s (^ — w^) (^ _. jSi^) (^ — y^^) to m factors, 
therefore I)x,y F » the product of the discriminants of ^— on^, ^p—fi^t &c. 
X the squares of the resultants of ^— aif^, ^— i3>|#, &c. taken two and two. 



Digitized by VjOOQIC 



141 

Now the resultant of <f>-a\lf and 0— jSif' is (o— j8)'*R (^, tf^), 
(Salmon's Higher Algeh'ay Arts. 180, &c., whence this solution, due to 
Prof. Sylvester, Comptes JRenduSf Lvni., 1078, is mainly derived.) 
Therefore D,,y F = the product of the discriminants of ^— otf^, ^— i8«f^, &c. 

x(a-i8) (a~7)^'* [R(<|», tf^) ]»»("- D, 

= the above product x (D^,^ F)'* [R (<^, Tf^)J"»('"-i). 
Again, the discriminant of ^— aif' is the resultant of 

^-.^ = and ^-.ff:-[0 (I.) 

ax ax dy ay 

or, by Euler's theorem, of <>-oi^ = 0, # . ^ - ^ ^ s J = 0, 

dx dy dy dx 

unless ^ — 0, if' = 0, or the x or y introduced vanish, which last 
suppositions only give numerical factors, and the discriminant can only 
differ by such a iaetor from R (<|>— oif^, J) -f- R (<t>, ^), and therefore the 
product of the discriminants = R (F, J) + [R (^, if^)]*" ; 
and therefore D^, y F = [R {<t>y if^)]*"' -«'« R (F, J) (D^, ^ P)'*. 

If F be a symmetrical quantic of even degree, let ^ = a;2+y2^ i|> __ ^y . 
therefore J = 2 {x^-y^ and R (F, J) = F (1, 1) F (1, - 1), 

R {<!>, rp) - I and (P^,^¥y = (o-i8)2'* (0-7)2- = Xi*, 

where XI is a rational integral function of the coefficients of F. 

If ^ = Xaj + /*y, if' = \'x + f/y, R (4), i|») = x'/*— ^/*'> 

J = A'/*- A/*', and R (F, J) = (a'/*-a/)»» ; 
therefore, by substitution, Dx,y (F) « (a'/*- A/)"* ("»-i) (D^,^ F), 
which is the evidently correct result, since the discriminant is an in- 
variant, and aV— A/i' the modulus of the linear transformation. 



5271. (Professor Caylby, F.R.S.) — If « be an imaginary cube root 
of unity, show that, if 

(a>-ft>2)a; + ft,V , dy ^ {u^w^dx , 

and explain the general theory. 

Solution. 

II ^ ^ mdx ,j. 

{(l-y2)(I-AV)}* {(l^x^){l^i^x^)y ^" 

y must be a function of x. Let y = ^ (:r). 

Now if, in (1), —y and —a: be written for y and x, the equation is 
unaltered; therefore —y^<p(—x), and therefore xy and y/x are even 

functions of :r. JjQi y j x = y^ {x^) (2). 

(This form is chosen in order that y and x may vanish together.) 

VOL. XLVII. 8 



Digitized by VjOOQ IC 



142 

Again, if, in (1), — and — be written fory and x, the equation is 
unaltered: therefore — sii'f-— K 

andtherefore — =^M^(-^) (3). 

It therefore appears that y^ {x^ is fractional. Now, assuming that the 
transformation is rational, let 

and, from (3), Kfi{x^fi (-i-^) ^Aa(^s)« ( -^), 

or 

kB.(^-.») ft-,')...(»«-««) X ( ^--±,) ( *,- -1-,) ... (».- -1.) 

^AA»(«.-.»)(«,-.^...K-.V («.-;^)(«,-^) •••{''--;^). 

an identity which involves m = n (unless some of the quantities 
aia2...rtm» bib^.-.bn he zero), and a series of conditions of the form 
br-x^ = 0, equivalent to a^ — ^'^ = 0, or ar — [ 1 / (ic^a?-)] = 0. Each equiva- 
lence of the first class serves to reduce the order of the transformation, 
while each of the second class gives *, = 1 /(<»rK^)f 

andtherefore tL = A h-x^) («,-^«) ■■■ (a,-:t^ 



■B (1-«2kV;(1-4Vj;2)...(1-PkV) 

il a, = o', ffj = i», &c. &c. ; 

and, from (3), -1 - ^ (a* ... /)!! k2» ; therefore ~ \ , 

'^ li' B (K'»-'A)»(«4...i)-' 

« V A y (1-<j-A')(1-«2k%2)... (1-Pic2^s/ 

while, hy substitution in (1) and putting x = 0, it appears thatj 

?_) («i...;r = m»,&c., 
therefore y = ,„:. . / " V ^ ^i—l LI. 



Digitized by CjOOQIC 



143 

which agrees with the form given by Prof. Caylby (EUiptie'Functumi^ 
p. 169), and the quantities a, ^, ... ^ might be foond from the identity 

(1 _a;') (1 - k2^) {dyY = (1 -y2) (1 - xVO m« {dxy. 

This would, however, be a very long process, and an equivalent result may 
be obtained as follows. 

Since (l-y^(l-AV) must be equal to (l-ap2) (1 -«'«') multiplied 
by a square factor, assume either 

. ^_ {/W}'(i-^) „ {/Mp(i^») , 

"^^^'^ ■ -^[h) ""'^' -Mi^O 






"'(Hi) 



of which either set fulfils the condition, while the equation assumed 

gives just enough conditions to determine a, 6, ... /, and g^ve a relation, 
the modular equation, between k and X. 

In the particular case where n = 1 — the cubic transformation (assuming 
the second form that the quantities may be real), 

and let ic = «*, X = t;*. 

Therefore 2A-1 « f — V »* - ^ «», 2A-A2 - aV = w8a?, 

therefore ?^^* - (2A-1) ««, or 2A (l-«V) - A^-fA;* ; 

therefore ± 2 ~ (l-iA>«) - ^ -wV, or ± 2uv (1-«V) = «*-!;<, 
the known form of the modular equation (Caylby's FunetUms, p. 188). 



Digitized by VjOOQIC 



144 



Returning to the form y = ( — ya^x -^^» 

and comparing it with the given equation y = \"~w ;^ + «-g 
It appears that m = f -^V a^ = «-««, -i = ?^ , 



and aV « «» («- w^j j therefore «-=-«* = -«, and 



(f)'-- -^ 



therefore — = w* = — k^, and k = -x, and x' = ic^ = — « ; 

and therefore ^ ^ ^"-T"'^^^ . 

(1 -y2)* (1 + «y2)* (1 -*2)* (1 + 0,2:2)* 

a result which may be obtained from the equations which lead to the 
modular equation, viz., 

In these let k = - X ; 

therefore 2A- 1 = ic (-l)*a2^ 2h^h'^^K^a^, A««ic(-1)*, 

therefore {2h - 1) A^ + 2A-A2 = 0, 

and, since h is not zero, A^- A + 1 = 0, therefore A = - w, 

therefore ir^ « — A* = — w* — — « = X^, 

ind ic2«3= l-(l_^)2== -2«-<»«- l-« = «2(„.j^^ 

and • a2 - -ft, (0,-0,2) = - «Il5^, 

«2 

which give the same transformation. 



5420. (Professor Sylvester, F.R.S.) — Prom the expansion of 
{log (1 +a:)}*» , in a series according to powers of », prove that S,-, j [the 

coefficient oiO is the developed product of (l + ^)(l + 2^) (! + •<)] 

is divisible by every prime number greater than /+ 1 contained in any 
term in the series »+ 1, i, »— 1, , «— y+1. 



Digitized by VjOOQIC 



145 

Solution. 
Let z « log, (1 + x), therefore 1 + a; » <?% and 
*v « (1 + x)yl^ 

1 a 1.2 \z J 1.2.3 \ z I 

therefore {log(l+.)r = «- = ^{l-f^^. + ^-;^^5^j.' + &c.], 

a. 

and hence ^^ r (—1)^ is the coefficient of a:*** in 

(•->+2)...(i + l)^ ' 

{log(l+^)}W.i = :^-.>iJl-| + ^-|. + &c.]*-'*\ 

which proves the proposition, as the coefficient of a!**^ in this last expan- 
sion cannot involve any prime factor (in the denominator) greater 
thany+ 1, and will involve i—j'-i- 1 (in the numerator), if that number be 

a prime, unless n (t-y+ 1) =• + 1 when t + 1 — -^ and • -^ + 1 =* -^ 
and i—j'+l is not greater thany+ 1. 

iProf. Sylvester observes that his theorem ought to be stated as 
ows:— Sf,y contains every divisor of the product (*+ 1)»(»— 1) ... 
(f —j'+l) which has no prime factor less than J +2. He also observes that 
the well-known theorem ordinarily associated with Wilson's, viz., that 
Sj»-i,j when^ is a prime number and/<p— 1 contains^, is an immediate 
consequence of his theorem, which teaches this, but much more besides ; 
that is to say, his theorem is at once a generalization and an extension of 
the theorem associated with "Wilson's theorem. 

In general the coefficient ofa^'in (l + -^ + -«-+---+ ,• — r + &c j 

consists of two groups of fractions. In one group each fraction (»— /+ 1) 
appears as a factor in its numerator, in the other group the factor (i -/ + 1) 
will not so appear. 

But this latter group has no existence unless y contains i—j'+l as a 
factor, which cannot be the case when i —J + 1 is greater than j nor d fortiori 
when i— y+ 1 is greater thany+ 1. In every fraction of either group the 
denominator contains only powers of 2, 3, ... {j+ 1). 

Hence, striking out from the product of 

(» + l)f.(i-l)...(i-y+2) (.-->+ 1) 
all powers which it contains of the numbers 2, 3, ... (y+ I), or, which is 
the same thing, of all the prime numbers not inferior toy + 2, the quantity 
which remains must be a divisor of S,- ,. 



Digitized by VjOOQIC 



146 

5640. (W. J. CuRRAN Shabp, M.A.)— Salmon says THiffher Plane 
Curvet, p. 98] that the curve parallel to a given curve may be obtained (1) 
as the envelope of a circle of given radius whose centre moves on the given 
curve ; (2) as the envelope of the parallel to the tangent to the given curve 
drawn at a constant distance. Prove that these processes are equivalent. 



Solution. 

On the first supposition, we have to find the envelope of 

U-xy+{n-y)^^k^' where <p{xy)^0 or L + M^ = (1,2). 

ax 

Now, from (1), ({-a:) + (ij-y) ^ = 0, therefore ^ = ^Zllf, 
ax Jj M 

and ntp {xy) =La; + My + N=»0 and {i—xY + {-n—yf ■» ** ; 

. t:f _ n^zV ^ ^ . L(^-a:)+M(tT~y) ^ k 

*' L M ""a/L2 + M^' *' L^+M2 \/La + M*' 

therefore L| + Mij + N = k>/'i?-¥U\ 

the equation to the line enveloped upon the second supposition. 



5755. (W. J. C. Shakp, M.A.) — If, through any point of inflection 
O on an n-ic, there be drawn three straight lines meeting the curve in 
Ai, A,...An-i ; Bi, B2...B„-i ; Ci, Cg-.C^-i, respectively ; prove that every 
curve of the n^ degree through the 3« — 2 points O, Aj, A2...An-i; 
Bi...B»-i ; Ci...C„_i will have O for a point of inflexion. 



Solution. 



If we call the second line, which, with the inflexional tangent at 0, 
makes up the polar conic* a sapra -harmonic polar, and if the three lines 
meet this in A, B, and C, these points are also common loci of supra- 
harmonic means of the point O with regard to all curves through the 
3» — 2 points. This locus, then, which in general is a conic, must, since 
these three points of it are in a right line, be for all those curves the 
same right line ; and, therefore, the point O must be a point of inflexion 
on each of them. 

In the above I have called OE the supra-harmonic mean between 

ORi, OE,...OR»>if ll^l == J-- + -1- + ... + 1 



OK OKi OEj, 0R„_ 



5809. (D. Edwardes.) — If O be any point within a triangle ABC, 
prove that OA* sin 2 A + OB^ sin 2B + 00^ sin 20 is least when O is the 
centre of the circumscribed circle. 



Digitized by VjOOQIC 



147 



7050. (D. Edwaedes.) — If P be any point in the plane of a triangle 
ABC, and d its distance from the circumscribed centre, show that 
PA2 sin 2A + PB^ sin 2B + PC- sin 2C = 4 (RS + ^) sin A sin B sin C. 

7069. (^' Edwardes.) — If X, y, z be the distances of a point P from 
the angular points of a triangle, prove that the mean value of x^ sin 2A 
+ y2 sin 2B + z^ sin 2C, as P ranges over the circle about ABC, is three 
times the area of the triangle. 



Solutions. 

(7050.) If the centre of the,circum-circle be taken as origin of rect- 
angular Cartesian coordinates, and (x, y), (arj, yj, (a-j, yj), {x^, y^ denote P, 
A, B, C. 

PB2 = (aj-a?2)^+ (y-yz)^ = R2 + ^^-2 (^arg + yya)* 
PC3 - {x-x^f+iy-y^-^ - R2 + rf2-2(:rd;3 + yy8), 

therefore 



PA2, X,, y, 




^1, Vly 1 


PB^ x^, y. 


=:(Ra + rf2) 


^2» y2» 1 


PC2, 0^3, ^3 




^3» ya» 1 



or 2PA2. A BOC + 2PB2. A CO A + 2PC*. A AOB = 2 (Rs + rf2) . a ABC. . . ( A), 
and 2AB0C « R2sin2A, 2aC0A = R=^8in 2B, 2aA0B - R2sin2C; 
and therefore 

2AABC =s R2(sin2A + 8in2B + 8in2C) = 4R« sin A sin B sin C, 
therefore PA2 sin 2A + PB^sin 2B + PC2 sin 2C= 4(R2 + d^) sin A sin B sinC. 

(6809.) And PA^ sin 2 A + PB^ sin 2B + PC^ sin 2C is a minimum when 
d =0, i.e.y when P coincides with O the circum- centre. 

In space of n dimensions, if V be the content of a simplicissimum ABC, 
&c. (Quest. 8242), and Yj be the content of the simplicissimum OBC..., 
Va of OAC..., &c., the formula 

PA2.V, + PB2.V2 + PC«. V5+...= (R2 + rfS)V 

may be established in the same way as the formula (A), and it follows 
that the sinister is a minimum when P coincides with O, the centre of the 
circumscribed spheric (hyper- sphere). 

(7069.) From the above, we have 

a;2sin2A + y2 8in2B + 22sin2C= 4 (R^ + rf^) gin jj^ gi^ b gjn C, 
and the mean value 

= 4sinAsinB8inCx4f f^'"'^V + y2 + R)2rf^rfy^ 4 pP"*''^*^^ 
Jo Jo Jo Jo 

= 4R2sin AsinBsinC ^ (x'^-vy'^^-l)dxdy -ir \ dxdy 

jqJo JoJo 

= 4R2sinAsinBsinCx ?f + — « 3 x 2R2 sin A sin B sin C 
8 4 

= 3 A ABC. [This integration may be effected thus : — 

0;'-""(-%y'+i)<^<^y = j;(f% i) (!-.=)» .i^ 



Digitized by VjOOQIC 



148 

2 f 1 



« 1 1^ ar2 (1 -a;2)* ^^+jf (1 -^)*^* 



24 3 "* 8 * 



We may, however, more readily obtain the mean value as 

= 4 sin AsinB sin C f^ C'{t^ + 'R')rdrde + T C'rdrde = &o. 
Jo Jo Jo Jo 

From Question 7050 it appears that, if • 

PA2. ABOC + PB2.ACOA + PC2. AAOB 
is constant, P lies on a circle whose centre is O and &c. in higher space.] 



6686. (W. J. C. Sharp, M.A.) — If a bar naturally curved be strained, 
the bending moment at a point, whose natural curvature is t"^ and 
strained curvature f>-^, isE(p-^— r-^) {l + Ar-*+Br-*+d;}. 

Solution. 
Using the notation of Minchin's Statics, p. 510, if PP' be the portion 
of any fibre intercepted between two near normal sections, PP" the same 
fibre in its unstrained state, nn' the portion of the mean fibre intercepted 
between these sections, and r and p the initial and strained radii of cur- 
vature, we have 

«« p nn' r PF' \p r J r-¥y 

and consequently the longitudinal stress on the prism standing on a small 

area8«r ^^( L -l.\r JL. z^ 

\p r/ r±y 

and therefore the bending moment = E ( — j I -^ — dff 

r 
r=E (— - J-)I, ifthebarbeof small section I 

^E^i- i^ 5"!+ l^d(r+ {^dir + &c,\ in any case 



5828. (Professor Darboux.) — On coupe une pyramide triangulaire 
SABC par un plan parall^lek la base ; ce plan rencontre les aretes lat^rales 
SA, SB, SC en A', B', C ; on m^ne ensuite les plans CA'B', AB'C, BC'A'. 
Soit P leur point commun. Determiner le lieu d6crit par le point P lors- 
que le plan A'B'C se deplace en demeurant parallele k la base. 



Digitized by VjOOQ IC 



149 



Solution. 

Taking the given tetrahedron as tetrahedron of reference, and the 
faces opposite to S, A, B, C, respectively, as X = 0, ft * 0, y — 0, ir =* 0, 
the coordinates being tetrahedral, so that x+/A + y + iraOis the equation 
to infinity ; let x— a (a + /* + y + tt) = be the equation to A'B'C, 
then at A', if = 0, ir = 0, \-a (\ + fi) = 0, 
B', IT = 0, /x « 0, \-a{\ + v) = 0, 
C, /i = 0, </ = 0, x-a(x + ir) « 0; 
then the equations to CA'B', AB'C, BC'A' are 

a{\ + ft + y) =X, a{y + ir + \) «» A, and a(ir + A + fi) = A, 
and therefore at P the common point of these planes, /x = y = », and the 
locus of P is the line drawn from S to the centroid of the face ABC. 

A similar resuU may be obtained in exactly the same way in space of 
n dimensions. It may be enunciated as follows : — If SABO... be a sim- 
plicissimum (Quest. 8242) in space of n dimensions, and a linear locus 
A'B'C... parallel to the face ABC... meet the edges SA, SB, SC... in A', 
B', C... &c., respectively; the locus of the point P in which the linear 
loci AB'C'..., BA'C'..., CA'B'..., &c., meet, wiU be the line joining S to 
the centroid of the opposite face ABC... 



6827. (Prof. Catlet, p. R.S.)— Consider a triangle ABC, and a point 
P ; and let AP meet BC in M, and BP meet AC in N (if, to fix the ideas, 
P is within the triangle, then M, N are in the sides BC, AC, respectively, 
and the triaugles APN, BPM are regarded as positive) ; find (1) the locus 
of the point P, such that the ratio (AAPN+ A BPM) : A ABC may have 
a given value ; (2) drawing from each point P, at right angles to the 
plane of the triangle, an ordinate PQ of a length proportional to the fore- 
going ratio ( A APN + A BPM) : A ABC, trace the surface which is the locus 
of the point Q, a surface which has the loci in (1) for its contour lines ; 
(3) find the volume of the portion standing on the triangle ABC as base ; 
and (4) deduce the solution of the following case of the four-point prob- 
lem, viz., taking the points P, P' at random within the triangle ABC, 
what is the chance that A, B, P, P' may form a convex quadrangle P 



Solution, 
Take A as origin of rectangular Cartesian 

coordinates, and AB as axis of x, and let 

4, ij be the coordinates of P. 
Then N is the intersection of ^ = tana .^ 

(AC)andy=-^ (a:-c), (BP), 
^— (J 

therefore A APN - A ABN- aAPB 

^ g C grytana ") 

** 2 lij + (g-Otano '') 

_eri( -ty + ^tang 7 

"* 2 (.ii + (c-.|)tanay 

VOL. XLVII. T 




3 CD* 



Digitized by VjOOQIC 



150 

Similarly, M is the intersection of 

y = -(a;-c)tani8... (BC) and y - (ij/fla?... (AP), 
therefore aBPM = aABM- aAPB 

« £ f ^tan/8 7 s_ ^ ( (/?— tan g-t? 7. 

** 2 iij + |tan/3 " > 2t ij+^taniS )* 

hence, if AAPN+ aBPM : A ABC :: r : 1 and;? = isina {\ and 17 being 
the current coordinates), the equation to the locus of P is 

tan a ^ (g-|) fan g- ty > _ ^^ 



(.17 — 4tano + 



f c tan a 17 + ( tan g 

or (2ij + fy)(ij-4tano + ictano)(ij + |tan j8-ictana) 

+ \c {ij [t^p (tan a + tan jS) + c tan o tan j8] + i rpc tan a tan jS} — s M, 
a cubic through A and B, the asymptotes of which are the parallels to 
AC and BC through the middle point of AB and the line 2t7 + r/7 » 0, a 
parallel to AB at a distance depending on r ; and therefore, as the satellite 
of infinity is also a parallel to AB at a distance depending upon r, the 
asymptote 2iy + rp « touches at an inflexion at infinity. 

Any node there may be will lie upon dM/d^ = 0, 
or (2rf + rp) {(tanjS— tana) ly-(l-ic) tan o tan iS} = 0, 

and there is no node on 2ri-\-rp; therefore, if there is one, it lies upon 

(tanjS— tan 0)17- (^-^c) tan o tan /3 = 0; 
but, where this cuts the curve, 

{2ii + rp)rr^ + icn[rp (tan a + tan jS) + <? tan a tan /9] + i^rj^c^ tana tan /9 =■ 0, 
so that the cubic is non -singular. With the same origin and axes of :r and 
y, and the line at right angles to the plane of the tri^gle at A as axis of 
z, the equation to the locus of Q is 

(•2y f nz){y—x tan a + ^c tan o)(y + x tan jS— Jtf tan j8) 
+ i<^ {y [m2 (tana + tani8) + tftanatan3] + ^ r/tB tan o tan jS} = 0; 
or, denoting 2y-v^z by L, y— ^tana by M, and y+a;tani8 by N, this 
may be written 

LN (M + «tan .)_, (Mtang + Nt.!..^' _ 
tana + tan)3 
90 that the planes Ls2y + /t2 ^ 0, M + ctano = y— (a?-(j) tano = 0, 
N 5 y + :r tan j3 = U all meet the surface where it meets the plane 
Mtan)3 + N tano=j».y = 0, and the lines thus determined lie wholly 
on the surface, and any planes through these lines meet the surface on 
the three systems of qnadrics, 

LN = A(j (M tan j8 + N tan o), L (M + c tan o) = Ac (M tan 3 + N tan o), 
N (M + c tan o) =» Ac (M tan jS + N tan a), 
which are hyperbolic paraboloids. The curves (1) are the sections by 
planes parallel to 2 = 0, the plane of the triangle and the plane 2y + /u2 « 
will meet the surface at inflexions in these sections, and the planes 
y~x tan a + ^6 tan a = and y -1- tan j8 — ^(; tan j3 = will always con- 
tain two of the asymptotes to these sections ; as all these sections are 
non-singular, the surface will not have any multiple point or multiple 
line. If the equation be written in the form 

( ~ y 4- a; tan a g tang— a? tang — y \ 

y — a;taua + ctana y + a:tang >* 



Digitized by VjOOQIC 



151 

it appears that there is only one value of z for any values of x and y, and 
that z cannot become infinite for any finite values of x and y unless they 
make y + x tan 6 ^ 0, ot y-x tan a + e tan o = ; and the traces of these 
planes are the parallels to BC through A and to AO through B, and thesd 
are always external to the triangle, and therefore z does not become 
infinite within the limits of the integration required for the solution of (3). 

The volume required in. (3) is the integral of 

__ y r — y-^-rtan a gtanj8~a;tan j8—y •) 

"" /* Xy—xta.na + ct8LD.a y + xtamfi ) 

_J.f-2y + c(tano + tani8) + <?tan2o — r^^^fr etan^fi X 

fii y-{x-c)tajia y + xtaufi) 

with respect to x and y over the whole area of the triangle ABC, 

rbcos* cxtauA re r(c— x) tan /? 



C rbcos* fxtauA ce ("{c— x)tan/J ") 

and therefore — ] \ + I I f <* * 

(.Jo Jo J6C06«JO J 

and \udy = — { — y^ + cy {ta.n a + ta.nfi) + c{x— c)tan*o log [y — (a? — «) tan a] 
—ex tan* $ log {y + x tan )3)}, 

1 rb COS m r 

therefore the volume = — i —x^ tan' a + cx (tan a + tan 0) tan a 

M Jo C 

+ c (a?- c) tan^a log -^ - (ja; tan2 iS log ^^5^±^?£i ] ete 
^ ' e—x taniS J 

+ — f } -{c-xytaji*fi'¥e{e''X){ta.jia + taii0)t&a$ 

M JiCOB* C 

+ (? (or-c) tan^alog ^''^^^^^^ «^-,; tan'iSlogi- ] do? 
tan a « ) 

(or writing a? for c— a? in the second integral) 

■s J_ r *3 —ajstan^o + <?« (tan o + tan i8)tano + (?(«— (?) tan- o log— ^ 
M Jo C , <^-* 

- „ « , tan a + tan /9 7 , 

— <?a?tan2^ log — ^ 1: l ^x 

taniS ) 

+ - [''^""'y — a:2tan*j8 + c2;(tano + tani8)tani8 + tf(a;-c)tan2i8log — 
M Jo C ^-^ 

. ., ^ tan a + tan 37 ^^ . 

— (jartan-^alog — y ax; 

tana ) 

and the first integral 

__ b^ cos a sin^ a ^ sin 7 sin o a^tf cos* /3 sin* a , sin 7 

^ 3 ~i ' cos iS 2 ' cos* a sin a cos /9 

g*gcos*i8 8in*a __ ^ cos* a siV* Q |^ sin 7 _ ^ si n*a . 
4cos*o 2 * C08-ia sin/3coso 4 'cos^o' 

and the second is derived from it by interchanging a and b and a and /9. 

Therefore the volume = — 5 -z — - * cos a 

be An a • b . a*c C082j88in2o 1^ sin 7 

+ — : — s»n 7 « + -IT ' 5 log ■: — 

2 cos 3 2 COS* a Bin o cos /3 



Digitized by VjOOQIC 



152 



_£wn2« ,^^ , ^c corf 

4co^ft^ ^ 2 



xBin*3 



C08?/3 



log 



a'am^/8 



aco6)3 + 



«<? sin j8 



siny 
sin /3 cos a 

sin' 3 



a **<? cos- 

sin 7 + -— . r— 

coso 2 cos^^ 



log 



amy 
sin /3 cos a 



4co8'/3 
/*(. 3 



((J«-d3coB«o)-^'' 



cos' j3 sin* 
C08*^ a 



-log 



sm'y ( 
sin a cos /3 J 



2 cos a cos 3 4 



pe 



pg' sin 7 
4 cos a cos /3 



4 cos a cos 3 



(a sin o cos' 3 + A sin /9 cos' o) I 



4 cos o cos i8 sin 7 

[sin'(a + i3) — 2sinasini3co8aco8i3] r 
= ^ { —4 + il = ~ (where /? = a sin jS or i sin a). 

If, in the figure, APE'S is to be a convex quadrilateral, P' must lie in 
one of the triangles APN or BPM ; and, therefore, for any position of P, 
the chance for a convex quadrilateral is (AAPN+ aBPM)/ AABC = r 
and fjiz =^ rp, and the chance is fiz/p ; and consequently the chance for 

any position of P in the triangle = | — diccfy + I Xdzdy^ both integrals 
being taken over the triangle, and by what precedes this 

pxbfi 

aaid the chance for a re-entrant quadrilateral may be at once deduced as 
1 — ^ = f, or it might be obtained, in a similar way, by finding the locus 
of P where the ratio A APB + fig. CNPM : a ABC is fixed, and describing 
a surface of which the ordinates should be proportional to the ratio for 
each point, and then integrating as above. This surface, if described on 
the opposite face of ABC, would coincide with the surface in the question 
if the latter were removed a distance unity towards ABC. 

[The above result may be confirmed as follows : — The mean values of 
AAPB, aANB, AAMBare^AABC, iAABC, iAABC; andthemean 
valueof (AANP+ aBMP) = mean value of (A ANB+ AAMB-2AAPB) 
s= -^AABC, which gives the same result as above (see §66 of Prof. 
Cbopton's article on FrohabUity^ in the Encyclopaedia Britanniea.)'] 



P£ ^ 
2 



7270. (W. J. C. Sharp, M. A.)— Eliminate a, *, e, a\ b\ e' from 





a-^b + c 


^d 


ab 


+ be + ca 


=s S 




abc 


^ 8 



a'b' + h'c' + cfa' =■ *' 
a'b'c' = 8' 



a'bc + ab'c + oM = 6, 
ab'e^a'btf^-a'h'c^^. 



Digitized by VjOOQIC 



153 

Solution, 

(a, by e) are the respective roots of a:*— ffjr^+wr— 8 = (1), 

K*i, O »» ' » »» a^-d,x^ + 8,x-9,^0 (2), 

(a*„*ri, Ml) „ „ „ a^-sx^ + dSx-n' --0 (3), 

(fli*i, Vi»^i«i) »» M a:8_«^;c2 + ^^8jar-8,2^0 (4). 

Let G, A, Gri, Ai have their usual meanings for (1) and (2), and ff, S'A, 
ffi, Ji^^i for (3; and (4) ; then (Burnsidb's Theori/ of Equations y p. 113) 

a«l + Wl + CCl-tf=i<f<fl+3[i(-G+^/A).i(-C^l-^/Al)]* 

+ 3 [i (-G- a/A) . i (-Gi+ a/Ai)]*, 

a,J(? + «*!<? + «*<?! = d - i efi« + 3 [i (-^ + 8a/A) . i (-Gx- a/Ax)]* 

+ 3 [i (-ir-8A/A) . i (-Gi+ a/Ai)]*, 

with a similar equation for Oj. These three, being rationalised, give the 
results of the elimination. 

[If ax^ + by^ + cz^ + 2fyz + 2^3® + 2hxy = m, 

a'a;2 + b'y^ + ^j'z^ + 2/^5? + 2g'zx + 2A'icy = m* 
be two concentric quadrics referred to rectangular axes, 
« + * + <? = <?, a' + V tc'=.d\ 

ab + bc + ea^P^p^-^h^^8, a'*' + *V + cV-2/2-2^'S-2A'2 = ,', 
Qbc + 2fyh^aP'-bff^-ch^=ny a'bV + 2fyh' -a'f^--by^--c'h'^sZ', 

aa' + **' -\-cc'-2ff'-2gg'- 2hh' = ^ , 
a'^c + aA'c + ab</ + 2/5^^ + 2fg'h + 2fgh' --a'P-b'g'^- (fh^ 

^2aff^2bgg'^2chh'^e, 
ah'(/ + a'b</ -S-a'b'c-^^fg'h' + 2fgh' ■¥2fg'h^aP'-bg'^-cU^ 

- 2«'/'/- 2b'g'g-'2c'h'h = a', 

are their invariants and co-variants for rectangular transformations ; and 
the given values of dy 8, 8, d\ »', 8', e, $, and 6' are those which they 
assume when the quadrics are coaxal {Proceedings of the London Mathe^ 
matical Society ^ Vol. xiii., Nos. 193 — 4), and are referred to their principal 
axes, and hence the resultants so elegantly obtained by Prof. Sircom are 
the invariant conditions that the quadrics should be coaxal.] 



7301. (Professor Cayley, F.R.S.) — If the function ^^^^5 = ^pu 

7W+8 
is periodic of the third order {<ph4 = u) : given that the cubic equation 

(a, by Ot d'Qxy 1)3 = has two roots u, v such that v « °" , find u as 
^ "^ ' 7« + 8 

a rational function of a, b, e, d, a, jS, 7, 8 ; and examine the case in which 
u is not thus expressible. 

Solution. 
7«t5 ^^^' ^^' (o + 5)7W + (8-T)37) 



Digitized by VjOOQIC 



154 



and 






and therefore, in order that ^^ (u) = m, 

a? + 20/87 + /B7« = 83 + 2^878 + 0/87, ai^d o' + «*+tt8 + /87= 0, 
which are hoth satisfied if (^ + 8'+oJ+ ^87 « 

(for j3=0, 7 = 0, = 8 the other conditions make ^ (i#) = t#) , and this is 
the necessary and sufficient condition that ^ should he a periodic function 
of the third order. 

Again, if aa^-^-Zba^-k-Zexy^-k-d^ — have two roots m and v such 

that « - 5?i±|, 

7« + 8 
the equation 

a(ar + /B)3+3A(oa; + i3)S(7iF + a) + 3c(o4? + i3)(7» + 8)« + rf(7* + 8)'=«0, 
or {aa^ + 3*0^7 + 3(?a7* + drf) a^ 

+ 3 lao^jS + A (asa + 2a)87) + <J (2a78 + /Sy*) + (i7««} a;" 

+ 3 {aai82 + A (i827 + 2oi38) + <J (2)875 + a82) + £^5*1 a: 

+ («/83 + 3Ai8«8 + 3tfi352 + <?«8) = 0. 

Say a'a:' + 3A'a;'+3<?'a? + rf' = 6 has the root v in common with the 
orig^inal equation (a, b^ e, d^x, 1)^ = 0, and therefore the resultant 
+ (ad'-afd)^-9 {ad'^a'df (io'-A'c) + 27 {ad -afey{cdf-dd) • 
27 {pd'^b'd)'{ab'''a'b)^%\ {aV-a'b) {bd-Vc) {ed'-ifS) 
-27 {ad'^€^d) {ab'-afb) (ed'-e'd) = 

must he satisfied hy the ahove values of a', b', tf^ and d*. The root may 
he found hy finding the Gr. C. M. of 

(«, b, e, rfjaj, 1)3 and (a', A', <?', rf'Ja:, 1)3, 
which will he found to he 

{9c (a*'- a'A)2- 9* {fib' - a'A) («y - o'c) + 3a (o^- <?'<?)« 

-a (aA'-a'i) (flkf -a'<f)} ar, 
+ {3d {ob'-afbY-Zb {ab'-a'b) {ad'-a'd)'-a{a(f -ale) {ad'-a'd)\, 
and therefore v — 

^{Zd{ab'^a'b)^^%b[a}/-a'b){ad' ---a'd)-a{ae -'a'e){ad'--ald)\ 
9c {ab' ^a'b)^-^9b{ab' ^a'b) {ad ^a'c) + 3a (a<?'-a'c)2-a(a*'-.a'^) {ad'-a'd)^ 

and « « ^, so that hoth u and v are expressed rationally in terms 

of a, A, c, d and a, /3, 7, 8. 

If, however, the two quantics (a, A, c, ifja;, 1)3 and (a', *', c', rf' Ja?, 1)3 
have a common quadratic factor, v as found ahove = 0/0. 

The G. C. M. is 3 {aV -a'b)x^-\-Z{ac ^a'c)x+{ad'-a'd), 
and the common roots ohtained from a quadratic equation are not 
expressed rationally in terms of the quantities a, b, c,d; a', b\ </, d\ 

In this case, if u, v, to are the roots of (a, b, e, d^x, 1)3, 

r> r» are the roots of (a, 3', (t, a'Oa?, 1)', 

7M + 5 7t; + 8 7m; f 8 ^ ' * ' .x. » / > 



Digitized by VjOOQIC 



156 



andweliavenotonlyv= ^i±^, but also either w = ?l±^orelaef«-?^^; 

. au + B yi*+ 9 yv + B 7W + 8 

and, if ^3^(11), the roots are either w, <t>{u), and ^(w), or 

w, 4>(tr), and ^^'Ctt?). 

And hence, if the function he periodic of the third order, the trans- 
formed equation will be identical with the original one ; so that, if 

a» + 82+a5 + 37 = 0, 

and ao3 + 3Aa?7 + 3ro>« + ci>3: aa^fi + b (a^Z + 2afiy) -^ e (2ai^+ By^+d'^H 

: Ba0^ + b($^ + 2a$9) + e[2fiyZ + a^)-\-dyP : afi^ + Zbfi^i + Zefii^ + dh^ 

:: a : b : e : df 
the equation («, *, r, d'^x^ 1)' = has its roots u, ^(w), and ^(«), 
where ^ is a periodic function of the third order, and is unchanged by 



the transformation 



y = 



ax-i-B 



7« + 8 

(where a: $ : y :Z may be found from the above rdatioos, and also the 
condition which must be fulfilled.) 



8204. (Alice (Gordon.) — ^If A, B, C, D, E are five points on a sphere, 
Y]s ^e volume ACBE, Y,) the volume ABDE, &c. : prove that, with a 
certain convention as to sign, 

Vu (AB)2 + V,5 (AC)2 + Vi4 (AD)2 + Vjs (AE)^ - 0. 

Solution, * 

K (xj, vi, «i), (a^a, y^ z^ , (jTg, ys, «g) be the rectangular Cartesian 

coordinates of A, £, G, D, and E, 

^i' + yi'+«i', a^, yi. «i» 1 
^s'+ya' + ^a'i «8» y* «2> 1 
^3^ + y5^ + «8^ arj, ys, 23, 1 =0; 

^4' + y4' + «4'. *4, y4, «4» 1 
a?6' + y«^ + «6^ ^5, y6» «6, 1 

therefore, multiplying the second, third, and fourth columns by 

^11 2yi> ^) and subtracting the sum from the first, 

-a-i^-yi'-V, ari, yj, «i, 1 

x^-¥yf^'Z^-2XyX^-2yiy^-2z^z^ arj, yi, «j, 1 

ar,2 + y3« + 23«_22rja:,-2yiy3-2«,«3, x^, y^, 23, 1 

a^4^ + y4*+«4^*2af,a?4-2yiy4-2zia4, x^, y^, £4, 1 

a^5* + y5* + «6^-2a?ia?5-2yiy5-2ziZ5, x^, y^, 25, 1 

then, subtracting the top line from each of the others, 

(«i-ar2)«-|.(yi-y,)2+(si-2-:)«, x^-x^ y,-yi, «,-«i 

(ici-a:3^;2+(yi-y3)'+(^i-^)^ a^-^i» yj-yi* «s-«i 

(a;i-aJ4)2+(yi-y4)2+ (21-24)2, af4-jri, y4-yi, S4-«i 
(Xi-ar5)2 + (yi-y5)2 + («i-r5), a-j-ari, y^-yi, r^-Si 
which proves the property. 



-0; 



^Qf, 



Digitized by VjOOQIC 



156 

This method of proof applies equally in space of any dimensions, and 
the general property may also be proved by means of simplicissimum 
content coordinates, as I have done in a paper on the ** ftoperties of 
SimpHcissima," &c., read before the London Mathematical Society in 
April, 1887. 



8237. fW"- J- C. Sharp, M.A.) — If nCr denote the number of combi- 
nationa r together which can be formed out of n things ; show, from 
a priori considerations, that 

nOr " nCn-r> nPr =■ — n-lCr-l, n^r = n-lCr + n-lCr-1 (1, 2, 3), 

or, more generally, nCr — n-pOr +pCi n-pOr-i +1.C2 »-pCr-2 + &c. 



Solution. 



The given equations, which easily follow from the known formula 
for "Cr, may also be proved from d priori considerations. 

Since every set of r formed out of n things implicitly forms a set of «— r 
(those not taken), and that every change in the one set involves a corres- 
ponding change in the other, "Cr = "Cn-r- If one of the things be re- 
moved, **"^Cr-i sets of r— 1 things, and "-^Cr sets of rthings,may be formed 
out of the remaining « — 1 ; and, since the missing thing may be prefixed to 
each of the "-^Cr-i sets of r — 1 things, there are "-^Cr-i sets of r things 
in which a particiilar one appears, and **"^0r in which it does not ; hence 
*»Cr - **~^Cr + "~^Cr-i; also. Considering each of the n things in turn, 
« *»~^Cr-i includes all the sets of r things, so that each occurs r times, once 
in virtue of each of its components ; therefore r »*Cr = «"~*Cr-i. If i? of 
the things be removed, the number of sets of r will be made up of those 
which contain none of the p, one of the p, two of the^, &c., or of ""^Cr, 
"-^Cr-i, and ^Cj, since each of p may be taken with each set of r- 1 of 
the »— jP, **"'Cr_s, and^Cj, &c. ; therefore 



8631. (Professor Sylvester, F.E.S.) — Find the discriminant of 
«3 + y8 + «8 + 3tf ScV + 6^a;y«. 



Solution. 
The discriminant is the resultant of the equations, 

a:2 + 2^ar(y + a) + «(y + 2)2 = 0, y^ + 2ey (z + x)-¥e {g + x)^ ^ 0, 
22 + 2tj5 (a; + y) +c (a: + y)' =s 0, 
which are equivalent to the system 

i?(» + y + 2)2-(«-l)a;2 = (c-l)y2«(^-l)22 (A), 



Digitized by VjOOQIC 



157 

and from the first equation 

{(«« + tfir' + <f«^'-4tf«(a:V^ + y8«8 + «V)}« - 64tf^%* (ar + y + »)« 
from the equations (A), 

therefore {<f2 (l + 2tf)2-4<f2x 3tf2}s - 64^7 {e- 1), 

therefore {(1 -*)* (1 + 4tf-8d2)}«+ 64<f^ (1 -c)» « 0, 

and this expression is the discriminant ; and the invariants T and S are 

(1-^)4(1+4^-8^ and e{l-e)\ 
results easily confirmed by substitution in the general values g^ven in 
Salhon's Higher Flans Curvet, pp. 184 — 6. 



8928. (Rev. T. R. Tbk»y, M. A.)— Find the value of the definite 
— smpx (fl cos' x + b sin' x), when p > 2. 

X 



Solution, 
Denoting the integral by I, 

dl f* 
4 -:- ~ 4 I cos 17^ (a cos' x + b eia^x) dx 
dp Jo 

= (a-b) I {com {p + 2) X + C06 {p~2) x] +2 {a+b) C cos pxdx 

= Of unless p *= ± 2 or 0, in which case it is infinite. 
Therefore, if j? is not ± 2 or 0, I is independent of p. Let i' « 1, there- 
fore 21= [ ^sinar {<» + * + (a-i) cos 2a;} 
Jo * 

= (a + *) fsin^i:^ + i!?:i^r(sin3a?-sinar)^ 
' ' Jo X 2 Jo ' 'a? 

«(«+*) iir + i (a-i)(itr-iir)(aaBG0RY'8 JSxampleSf p. 480), 
therefore I « (« + *) i*. 



8978, (Professor Sylvester, F.R.S.)— Show algebraically that, if 
a, b, e are the three sides of a triangle of reference, and A, B, C, the three 
perpendiculars on a variable line from the angles of that triangle, are 
re^irded as its inverse coordinates, then the equation to the two circular 
points at infinity is 

««(A-B) (A-.C) + 3'(B-A)(B-C)+c'(C-A) (C-B) - 0. 

VOL. XLVn. V 



Digitized by VjOOQIC 



158 



Solution. 
In areal coordinates the circular points at infinity are represented by 

(fifUf + UhX + t^XffO, and X+ju + f-0 (1> 2), 

and 'if l\ + mfi + ny ^ represent any straight line, the square of the per- 
pendicular upon it from [\V/= Q (/\' + ntf/ + wi/)]*; as I have shown in a 
paper read before the London Mathematical Society^ m April last ; therefore* 
A:B:G::/:m:f>, and the equation to the line may be written 

Ax+BAt + Cr = (3), 

therefore from (2) and (3) x : m .' v :: B— C : C— A : A— B, and therefore 
from (1), a2(A-B) (A-C) +^(B-C) (B-A) + e2 (C-B) (C-A) - 0, 



8996. (Mahbnora Nath Rat, M.A., LL.B.) — If x^ y, z denote the 
respective distances of any point in the plane of a given triangle ABC 
from the angular points ; show that the following relation subsists among 
them, (a:3 + y«+«5 + a2 + j2 + c2) {a^x^-\-h-y^-^<?z^) = 2aH'^{€fi + a?) 



Solution. 



If the volume of the tetrahedron ABCD, whose edges are DA = x, 
DB = y, DO = », BC = a, OA = A, AB = c, is (Quest. 8242) V, 

V2 - -JL- o» 1» 1» 1> 1 
8 X 36 1, 0, x\ y^, «» 

1, x\ 0, e\ h^ 
1, y^ c2, 0, a2 
1, z^ h^, a\ 
and therefore, if D lie in the plane of ABC, 



0, 1, 1, 1, 1 

1, 0, x\ y\ «2 
1, a?2. 0, ^, ^2 
1, y\ c\ 0, a2 



= 0, 



1, z\ ^, cr, 

which, when expanded, gives the result required. The same result is ob- 
tained in a different way in Salmon's Gemnetry of Three Dimensions, p. 32. 



9003. (R. F. Davis, M.A.)--If upon each side of a triangle a pair of 
points be taken so that the pairs on any two sides are coney clic, prove 
that all three pairs.are concyclic. 



Digitized by VjOOQIC 



159 




•(1), 



Solution, 

If Aj, Aa ; B^, Bj ; Ci, Cj be the points in 
which BC, CA, and AB are divided, 
ABi . ABo = ACj . ACj, 
CA, . CaJ = CBi . CBa, 
and BAi . BAj = BC, . BCj. 

Now, let the circle through Ci, Cj, B,, B^ cut 
BC in Aj', A3' ; then 

BA/. BAj' = BCi . BCj « BAj . B Aj . 
CA/. CA2' = CBi . CB2 = CAj . CAj, 
or (BC-BA/) (BC-BA2') = (BC-BAi) (BC-BA^), 

and therefore BAi' + BAj'= BA1 + BA3 (2), 

therefore, from (1) and (2), BA/ and BAj' are the roots of the same quad- 
ratic as BAj and BA2, and A,' and Aj' coincide with Aj and Aj. 

If «, b, c be the middle points of the sides of the triangle, and D, E, P 
the feet of the perpendiculars from the vertices upon the opposite sides, 

A3 . AE = ^bc cos A = A(j . AF 
CA . CE « ia* cos C = Ca . CD 
Ba.BD ^iaecosB » Be . BF 

and therefore the points a, A, e, D, E, F are concyclic, or the circle about 
a, by c passes through D, E, and F. And if O be the intersection of AD, 
BE, and CF, the circle through D, E, and F, the feet of the perpendiculars 
from the vertices upon the opposite sides of the triangle AOB (viz., D, E, 
and F) will bisect the sides, and so the circle about DEF passes through 
a, bj c and bisects AG, BO, CO. (I think this is, perhaps, the simplest 
proof of the existence of the nine-point circle.) 

The equations (A), may be proved by elementary Geometry as follows : 
The circle on BC as diameter will pass through F and K (Euclid iii. 31), 
therefore AE . AC = AF . AB (iii. 36), or 2AE . Ab = 2AF . Ac, and 
similarly for the others. 



.(A); 



9008. (S. Roberts, M.A.) — Given three circles Cj, Cj, C3, determine 
a circle cutting C^ orthogonally, bisecting Cj, and bisected by C^ ; and 
show that in general there are two such circles which may coincide or be- 
come imaginary. 

Solution. 
Let the axes of rectangular Cartesian coordinates be taken so that 
ar2 + y2_^3=0sCi, (ar-A)2 + y8-A2=0=C2, (a:-.A')2+ (y-A-')2-c««0sC5, 
and (a:— {)2+ (y— iy)2_R2_-o = S, the required circle. Then, since Ci and 

S cut orthogonally, ^2 + ^2 =, ^2 + ^2 (1) ; 

since S bisects Cj, (|-/0"2 + >?' = R^-*^ (2); 

and, since C3 bisects S, U-'h')^ + {'n-k')' « c^-R* (3) ; 

from (1) and (2), { = — , 



Digitized by VjOOQIC 



160 

and therefore, from (3), ilfji - A''+*^-«s+a»+2R'- 4^(<.»+is + A») 

A 

sA + 2R«- ^B, say, 
h 

and 2A'| = ~ B ; 

.-. 4R< + 4R2^A-^B-;fc^| + (A^~By+|^B2-4a2A;'s = 0, 

and the yalues of R^ are real, equal, or imaginary, 

according as I A- ^B-k'^Y> - < /a- ^b]V |!B«-4a2Ar^, 

„ „ _2(a~^b)+A^> -< ^-4«», 

A A* 

When the values of R^ are real, those of n are so, and those of { are 
always so, and therefore, &c. 



9024. (Professor Stlybsteb, F.R.S.) — For g^reater distinctness, the 
name of Hyper -cartesian (not to be confounded with a ht/per-cartesic) being 
giyen to that particular form of the bicircular quartic in which four con- 
cyclic foci become collinear; prove that, if four points are given in a 
plane, the locus of the curve in space whose distances from any three of 
them are subject to a given homogeneous linear relation is a curve of the 
4th order. ('JChis space curve may be termed a Hyper-cartesic.) 



Solution, 

If the plane of the foci be taken for that oixy, and the axes be so chosen 
that they are (0, 0, 0), (A, 0, 0), (h\ k\ 0), and (A", A;", 0), and if ^, q, r, $ 
be the four focal distwices of the point {x, y, z) on the curve, 

«2 « ^a_ 2h"x - 2k" y + h"^ + k"^, 
and Ip^mq + nr = and rp-^m'q + n^t « 0, 

or 2^y}2 + 2;n2»Vr2 + 2«2/2,.2p2_^^_^y_^V* = (1), 

2l'^m'^P'q^ + 27n'^n'^q^s^ + 2n'^ns^p^-l'*p*~m'*q*-n'*f^ « 0...(2), 
or ap*-j-2ljpl^ + S = and ay + 2Ly + S'= 0, 

where L and L' are linear and S and S' quadratic functions of x and y ; and 



Digitized by VjOOQIC 



161 

their resultant, which is the trace of the curve on the plane of xi/, is 

(aS'-a'S)2+4 (a'L-aL') (LS'-L'S) - 0, 
a quartic, to every point of which two points, one above and another below 
the plane, correspond, so that these lie on two quartics, each of which is 
the reflexion of the other. 



9063. (Mahbxdba Nath Ray, M.A., LL.B.)— If aj, a^ a, ... <i2f.-i be 
2n-l positive numbers connected by the relation aya^^„,a2n i °» 1 ; 
show, by elementary algebra only, that the minimum value of 
(I +ai) (1 +03) (1 + aa) (1 + aj ... (1 + aa,..i) is 22»»-i. 



Solution. 

If a? be any positive quantity x+l/x > ^ < 2, 
according as {x—l)^> — < 0, 

but (^—1)' is <t 0, therefore x+ljx <}: 2, 

i.e.f 2 is the minimum value oi x + I/0. 

Now, if a^a^... «n *= 1> 

(1 +«,) (1 +<0 ... (! + «„) = (a.-T,...a.)» (a{ + «;«) (,» + «;•) ... {«» + «;») 

= («! + «:') ("J + «;*)-(«i +«;'), 

and the minimum value of each factor is 2 ; therefore the minimum value 
of product is 2»». Of course the quantities aj, a*, &c. are the positive or 
signless square roots. 



8637. (Professor Hudson, M.A.) — The tangent and normal to 
y/c = i (€'/<'+ *-'!<=) cut the axis of a? in T, G, respectively ; find the mini- 



mum value of GT 



Solution, 
Let the tangent be inclined to the axis of x at an angle <t> ; then 
GT »« tf sec 4> (tan (p + cot4>) = <j/ (sin^^— 8in'4>) ; 
this is least when sin4» « 1 / >/3, and therefore GT » ie^/Z, 



8888. (S. Tbbat, B.A.)--Di, I>2» I>8 are the shortest distances of 
opposite edges {a, a; b, b; c, e) of a tetrahedron, V the volume, and A 
the area of one of the equal faces ; show that 

V = J (DiD.Da), A' - i (Di^aH D./ J*- + D3V), 
and Di^ + D-Z + Da*- i(a2 + ^ + cj2) - D^HaS^ &c. 



Digitized by VjOOQIC 



162 



Solution, 

Let ABCD be the tetrahedron, and let BO = DA = a, CA = DB = b, 
AB = DC = c bisect BC in E' and AD in E ; then, since DB = AC and 
BE'=CE',andDBE'=DBO =BCA=ACE', 
DE'« E'A, and therefore EE' is at right 
angles to AD ; similarly, it is at right angles 
to BC, and it is therefore the shortest dis- 
tance between them. Therefore 

DE2 + EE'i = DE'S 
and DE's = i (^ + c--^a^, 

therefore 

Di2 = EE'2 = DE'2-DE2 « i {b^ + c*^a^, 
and, similarly, 

D22=i(c2 + a8-*2), D32=i(a2 + A»-c2)» 
therefore 

Di^ + DjS + Da^ = i (a2+i2 + ^2) ^ Di« + a3 = &c. 

= Tff {«^ (*^ + c2. fl2) + ^ (c3 + ejj_i2) + c3 (oS + ^S- C^)} 




Sx'i 



also (Quest. 8242), V^ - ^-^ 0, 1, 1, 1, 1 

1, 0, c^, *2, a2 

1, ^S 0, a2, ^2 

1, *2, «S 0, c3 

1, a2, ^2, (^, 

or (replacing the 2nd column by the sum of the 2nd and 5th less that of 
the 3rd and 4th, replacing the 3rd column by the sum of the 3rd and 2nd 
less that of the 4th and 6th, replacing the 4th column by the sum of the 
4th and 2nd less that of the 3rd and 5th, which gives 4 times the deter- 
minant). 



1 



4 X 8 X 36 



^Di^D^Da^ 
144 



0, 0, 0, 0, 1 

1, «2-i2-(j2, ^^ifi^a\ l^-<^^a\ a2 

1, J2 + ^_^2^ c2-a2_^^ rt2^.c2_i2^ ^ 
1, d2+c2_fl,2^ fl2+^_^2^ J2_a2_^2^ ^2 

1, a^^jfi^c"-^ a2^.32_c2, a2 + c2_j2^ o 



1, -1, -1, -1 

1, 1, -1, 1 

1, 1, 1, -1 

I, -1, 1, 1 



_ D^2Dom,2 p^p^T)^ 

9 9 • 



^ D,2D22D,2 



36 



1, 


-1, 


1 


1, 


1, 


-1 


-1. 


1, 


1 



I 



Digitized by VjOOQIC 



163 



1856. (Professor Sylvbstbb, F.R.S.) — Prove that the Jacobian of the 
time of a planet's describing any arc, the chord of the arc, and the sum of 
the two extreme distances from the sun, in respect to the eccentricity and 
two extreme eccentric anomalies, is zero ; and hence deduce the time for 
a planet or comet in terms of the said chord and sum. 



Solution. 
"With the usual notation (see Tait and Steele's Bynamies), we have 
CQSU—e ^, J 1 . „ /, 1 — ^ 



COS0 : 



1— tfCOSW 



, therefore 1 + «*co8 tf = 



1— ^COSM* 



"^^"Vl \l-.cos«j j- l-.cosu ' 

= iSyzfL^ail^ecoBu), 
1 + c cos d 

Now, if <i, <2 be the times from perihelion to the ends of the arc, T the 
time of describing it, «i, u^ the eccentric anomalies of the extremities, r,, 
rj, Biy 02 the corresponding radii vectores and true anomalies, S the sum 
of ri and r^, and C the chord, P being the periodic time. 



T= t^-ti =i-{«8-«sint<2--(wi-tfsin«i)} 



.(1), 



S =ri + r2 = a (l^e coaui + l-e COB u^) (2), 

03 = ri2 + r32-2rir2COS {di-O^ = ri^ + r^^-^Tir^ {cos a, cos 63 + sin tfj sin a.^} 
« a»r(l-*^s«i)*+(l-(?cos«2)'-2(l-^cos«i)(l-^cos«2) 

L f cos«i-g co8«2— g ^ (l-g2)sin«i8inM2 ^"1 

^ 1 1 - g cos Ml 1 - g cos Wa (1 - tf cos Mi)(1 -e cos Wg) ) J 

= a2 [2— 2cos«iCOS«3-2sin«iSinw2-^'(8ui«i-8iii«^)*] (3)- 

Again, the Jacobian of 02, T, S is merely that of 0, T, S multi- 
plied by; 20, and therefore they will vanish together unless « 0, 
a case which may be disregarded. Now, T (0*, T, S) 



'' 2ir 



-2c(sinwi-sinw2), 2sinwiCOS«2-2cosMi8in«#2 . ^ 

— 2^2 (jQg f^^ (sin ^ — sin «J, 

sin Wi - sin u^, - 1 + ^ cos Wj, 

-(costti + coswg), esinwi, 

-2sinMiCOS«2 + 2cos«*isin«/2 + 2c2cos«,(8in>i-sintt2) 

1 — CC0S«2, 

gsinM2 
(adding 2e times the second row to the first) 

0, sin Ui cos «3 - cos u^ sin 1*2— « (sin Wj - sin eij), 

sin«i-sin «*2» - 1 + c cos «i, 

- (cos Ui + cos M3) , * sm «i , 

- sin Ml cos wj + cos Mj sinM2 + * (sin ^i - sin Mg) 

I — gC08M2 



p^ 



Digitized by VjOOQIC 



164 

K (adding the third column to the second) 
— {— 8intfiCOB«2 + co8WisinMs I sini#i— siniia, ^(cosi#i— cobkJIsO; 



+ 1 (sin Wj — sin fij) } | — (cosf#i + cosmj, « (sinnj — sin Wj) | 
and therefore T (C, T, S) = 0, 

and T is a function of e^ M|, and u^ only as it is a function of S and C 
(Boole's Biff. Equa.y Sup. Vol., p. 56), and therefore it is possihle to 
eliminate e^ Mj, and t<s from the equations (1), (2), (3). 

From (2), e t , ^^"^ -, 

a (cos Ml + cos U3) 

therefore, from (3), 

C-a»f2-2coB(«.-.>.)-(gg:^V(''""l-"""'V] 
I ^ ' \ a I \co8«, + cosa,y j 

(. ^^ ^ \ a I l + cos(Wi-f*Jj 

a quadratic in cos (tij— t«i), whence 

"•<-^ - ('-^)o-^°) wn'-'^°)'{'-'-^r 

C-2g«-t-(2a-S)« '> 
2a« j 

-('-=-^)(-5£2)^V![-('-S^)"] 

« C0S2 cosz^'f sine sins' » cos(z— c^, 

if 1-8^:2^^^ ^^ l-S + 0^cos«'. 

2a 2a 

So that z and s^ are functions of S and C. Therefore 

tan r^i=^\ - tan '-=^ - Bin,-gin»' 
\ 2 / 2 cos» + cosr 

Now, from (1), (2), 

T- ^ (tf,-ti,- ^~^ sinwj-sinjwj^ 
2ir (. a cos Uj + cos M3 J 

2irC « 2 >' 

and — Zl5 = coB«+co9«' and tan "87**^ = tan^^, 

a 2 2 

therefore ^ *" ^ {«—«'— sin r + sin e'} 

a formula given by Lambebt, as an extension of one of Eulbb's for para- 
bolic motion, viz., T - -i- ((S + 0)*-(S-C)*}, 
6v^ju *■ 

which is proved in Tait and Steele. See also Pontecoulant, Sf/sthne du 
Monde, Vol. i., pp. 277 and 287—291. 



Digitized by VjOOQIC 



165 



5305. (Professor Syltbsteb, F.R.S.J — Definition, — Six right lines 
along which six forces can be made to equilibrate are said to be in Statical 
Involution. 

Prove that any six right lines lying on a ruled cubic surface are in 
statical involution ; and, vice verady if six right lines are in statical 
involution, a ruled cubic surface can be made to pass through them. 



-0. 



Solution, 

If aj, ^1, Ci, ai, jSi, 7i ; a^t ^3> ^ 03, $^9 72* ^c* ^ ^® direction oosines, 
and the coordinates of fi^ed points upon the six lines, and P|, P„ &c. the 
forces acting along them ; the conditions of equilibrium are 
2Pa = 0, 2Pi = 0, 2Ptf = 0, 
2 P (tfjB- J7) « 0, 2 P {ay-ea) = 0, and 2 P (ba-afi) = 0, 
and the necessary and sufficient condition for statical involution is 

«1> <»2> ^ ^4f ^i ^ 

bi, ^2* hi *4» hf *« 

Ci, Cj, C^t Ut ^61 ^6 

eiPi-biji, e^^b^y^, <?8^8-%a> <^434-*474» <?636-*676» ^^ei^e-^cTc 

«m — <^l«l> ''272— <V«2> <'878-<?808i ''474-^4«4» ^^^n/b — HHj «m- <?6«« 
Vi-Oi^i, b^t^-a^ ^503-03^8, *4«4-M4» V8-«6^5> *6««-««i86 

Now, if a\ + b n-k-ev -¥{€$— by) p-k- {ay^ea) <r + {ba^a0) t = 
be satisfied by any point in the line 

a?-a y—B g-y 
a " 6 " tf ' 
it is satisfied by every point in the line, t.^., 

aA + A/i + ttr + (cy — fo) p + (az^ex) v + (Aa?— «y) t = ; 
and, if there be six sach Hues, the determinant above will be transformed 
into the equation to a ruled cubic surface, upon which each of the lines 
lies, by writing x for each of the a's, y for each of the jS's, and z for each 
of the 7*s. Also any ruled cubic surface may be reduced to this form, 
the components of each column being the coordinates of any six lines upon 
it, and so the proposition is proved. 



TOL. XLTtt. 



Digitized by VjOOQIC 



APPENDIX IV. 



NOTE ON THE USE OF COMMON LOGARITHMS IN THE 
NUMERICAL SOLUTION OF EQUATIONS OF THE HIGHER 
ORDERS. By Major-Gbnbral P. O'CONNELL. 



The problem to be solved may be stated as follows : 

Given f{x) «■ aa?**» + ^a?'» + ca^+ ... + Z = 0, 

where a, J, c, ... Z; m, w, p, &c. are constants ; w, », jp, &c. being larg^ 
numbers consisting eiliier of whole numbers, whole numbers and fractions, 
or whole numbers and a few places of decimals arranged in descending 
order of magnitude, i.e., m is the largest and every other is smaller than 
any of those preceding it. 

To find a value of x which, when substituted in /(a:), shall reduce that 
function to as small a value as the extent of the tables of logarithms used 
and the true value of x permit it. 

When m, n, p, &c. are small whole numbers, I know no method of 
solution better than Hobneb's, which has been very fully explained by the 
late Professor Db Morgan, by Todhuntbb, and others. 

When #», «, Pt &c., are large whole numbers, the solution may be ob- 
tained in the manner shown by that able mathematician and computer, 
the late Mr. Thomas Weddle, in a pamphlet published in 1842, by 
Hamilton, Adams, & Co., Paternoster Row. Mr. Weddle does not use any 
table of logarithms, and his computations are therefore very long. 

The method here used is nearly that of Mr. Oliver Byrne, as explained 
in his ** Dual Arithmetic," published by Bell & Dalby, 186, Fleet Street, 
1864. It differs therefrom, however, in the use of common logarithms, and 
in presenting the calculations in a more condensed form, a single page be- 
ing sufficient for the solution of an equation of a very high order, as may 
be seen by reference to my Solution of Question 9185 [which will appear 
hereafter in Vol. 48], where an equation of the 622nd degree is solved, 
and a value of x is found containing six places of decimals or seven 
figures in all. 

In Mr. Weddle' s, Mr. Byrne's, and in the present method, x is con- 
ceived to be resolved into factors. In Mr. Weddle' s method, 

«=R(l+ri)(l + r2)(l + r3)..., • 

where R is the first significant figure of x multiplied by some power of 10, 
and rj, rg, rg, &c. form a series of decimals each consisting of one signi- 
ficant figure preceded by a decimal point, and 0, 1, or more zeros, the 
number of zeros increasing as we pass along successive values of r from 



Digitized by VjOOQIC 



167 

left to right. In Mr. Btbne's method 

where M contams one or more significant fig^es of x multiplied by some 
power of 10, and m^f m^ imj, &c. may have as values any suitable whole 
numbers or zero. In the present method; 

ar=ai(l+a2)(l + a3)..., 

where ai is the first approximation to the value of x^ and may contain only 
a whole number of one, two, or three figures, or a whole number and a 
decimal, a^ (1 + 03) is the second approximate value of x, a^ (1 +03) (1 + a^) 
is the third approximate value of x, and so on, 03 being less than unity, 
as less than 03 ... , and a^ less than ^n-i ... , &c. ; one advantage of usini? 
a table of logarithms being, that each approximate value will generally 
be found to contain nearly twice as many correct figures as the one 
preceding it. 

In all these methods, the values of the factors into which x is conceived 
to be resolved are discovered successively, commencing with the left-hand 
one, then the one next to it on the right, and so on. 

ITie first factor is usually obtained, in using Wbddlb's or Bybne's method, 
by a process of guessing and trial, or by some simple transformation or 
other algebraic artifice. 

In using the method about to be described, it will often be found that 
a value of the first factor containing three figures may be obtained by the 
use of logarithmic slide rules. This facilitates its application and shortens 
the whole process, and is therefore recommended to all who are accustomed 
to use these instruments. It may, however, be conveniently applied by 
those who do not use such aids. 

The method of passing from the first to the second approximation in 
the value of x, due to Sir Isaac Newton, and, as far as I know, generally 
followed until it was slightly altered by Mr. Oliver Byrne, is thus de- 
scribed in Toi)HUNTER*s Theory of Equations (4th Edition, 1880), page 
142:— 

** Let f{x) — be an equation which has a root between certain limits 
a and jS, the difference of which is a small fraction ; let c be a quantity 
between a and jS which is assumed as a first approximation to the required 
root ; and let c-k-h denote the exact value of the root, so that A is a small 
fraction which is to be determined. 

" Thus, f(c + h) = 0, that is, by article 10, 

f{c) + hf (c) + Y^r(c) + ... |^yw(,) ^ 0. 

Now, since h is supposed to be a small fraction, A*, h^y &c. will be small 
compared to A ; if we neglect the squares and higher powers of h in the 
above equation, we obtain f{c) + h ./' (c) = 0, thus 

/'W 

Supposing, then, that we thus obtain an approximation to the value of A, we 
have c^'ryfl as a new approximation to the root of the proposed equa- 
tion." 



Digitized by VjOOQIC 



168 

The simple alteration introduced into the above process by Mr. Btbnb 
is, to express the second approximation to the root, not ase— "^^t but as 
ell — ^^- I . This slight variation suffices to change the method to one 

suited to logarithmic calculation. 

The first step in the process is to find the logarithmic values of the 
several terms of the given equation when a.^ (the first approximation to 
its value) is substituted for 0. When this has been done, we have the 
set of tenns, 

log {« (ai)-}, log {h {a^^}, log {c (aj)"} ... log /, 

to which, adding 

^{^-i^'^- '-{-^r -{'-.^r' 

we obtain 

log [« {ai (1 +0,)}-], log \h {qi (1 +«4)"], log \c {«, (I +«2)}p] ... log/. 

Before we can add the second line of logarithms, shown above, we must 

first obtain the value of o^ »> — )^, v ' ^^^^ can be conveniently 

effected by arranging the calculation in the form shown in the following 
example, where t^e three operations of finding (I) the logarithmic values 
of the several terms of the given equation, (2) the natural numbers cor- 
responding thereto, and (3) the value of the next factor in the value of 
the unknown quantity, are recorded in different parts of the same central 
columns, a separate column on the left hand being reserved for the 
logarithms of the required powers of the successive factors of the value of 
the root and for the final value of the root, the several portions being 
numbered so as to facilitate reference, as well as correction if necessary. 

In this way the computation circulates from the left-hand column down 
the central one, and back again to the left-hand column, and so on again 
until it is complete. 

The example worked out is (Weddle, No.7, page 27) an equation having 
four real roots, of which two are positive. One of the positive roots is 
given by Mr. Weddle, and the other is here found. 

This equation in x has first been transformed to one in y J «■ f ^ j i, 

in which a first approximation to the value of y is easily guessed. 

It will be seen that only three factors are obtained, and that 

y - 10-» X 1-5 (1-0372)(1-00064574), 

an ordinary seven-figure logarithm table having been used. If more 
factors be required, Vegas' ten-figure table, or Peter Gray's twenty-four- 
fig^e table, may be used. When the latter table is used, a good deal of 
room and calculation are needed for passing from a large natural number 
to its logarithm and back from a large logarithm to its natural number, 
so that it is better to keep this portion of the computation separate and 
transfer the results alone to the form here used. 



London : Printed by 0. F. Hodgson ft Son, Gough Square, Fleet Street, E.G. 



Digitized by VjOOQIC 



K'ow ready, Boyal 8yo, d67 pp., with 285 Diagrams, price 340. 

A SYNOPSIS 

OF ELEMENTARY RESULTS IN 

PUEE MATHEMATICS, 

CotUiUning Six Thousand Propositions, Formula, and methods of Analysis, with abridged 
Demonstrations; supplemented by an Index to the articles on Pure Mathematics which 
have appeared in thirty-two Bbitish and Foreign Jousnals and Transactions 
during the present century, containing about 14,000 references. 

By G. S. OARR, M.A., 

Late Prizeman and Scholar of Gonville and Caius College, Cambridge. 



The work may also be had in Sections separately, as follows : — 

Section I. — Mathematical Tables ; including 0. G. S. units, s. d. 
Least Factors from 1 to 99,000. The 

Gamma-function, &c Price 2 

„ n. — ^Algebra 2 6 

., III. — Theobt op Equations and Detebminants 2 

IV. — Plane Tbigonometby C together ; with ") o a 

„ v.— Sphebical do. c 22 Diagrams. > 

„ VI. — Elementaby Geometbt ; with 47 Diagrams 2 6 

„ VII. — Geometbical OoNics ; with 35 Diagrams 2 

„ VIII. — DiPPEBENTIAL OaLCULUS 2 

„ rX. — Integral Calculus 3 6 

„ X. — Calculus op Variations -j 

„ XI.— DiPPEBENTIAL Equations C 3 6 

„ XII. — Calculus op Finite Dippebences ) 

„ XIII. — Plane Coobdinate Geometby, with 182 Diagrams 6 

„ XIV. — Solid Coobdinate Geometby, with 26 Diagrams 3 6 

N.B.— The Index will not be sold separately, but possessors of previous sections may complete the 
volume by returning these to the publisher to be bound up with the remaining sections and the Index. 



The work is in part designed for the use of University and other Candidates who may be reading for 
examination. It forms a digest of the contents of ordinary treatises, and is arranged so as to enable the 
student rapidly to revise his subjects. To this end all the important propositions in each branch of 
Mathematics are presented withm the compass of a few pages. This has been accomplished, firstly, by 
the omission of all extraneous matter and redundant explanations, and secondly, oy carefully com- 
pressing the demonstrations in such a manner as to place only the leading steps of each prommently 
before the reader. The whole is intended to form a permanent work of reference for mathematicians. 



" The book before us is a first section of a work which, when complete, is to be a Synopsis of the whole 
range of Mathematics. It comprises a short bnt well-chosen collection of Physical Constants, a table of 
fkctors up to 99,000, fh>m Borckhardt, &c. &c .... We may signalise the chapter on Geometrical Conies as 
a model of compressed brevity .... The book will be valuable to a student in revision for examination 
papers, and the completeness of the collection of theorems will make it a useful book of reference to the 
mathematician. The publishers merit commendation for the appearance of the book. The paper is good, 
the type large and excellent."— Journal of EdiuxAion. 

•• For the most part a carefully edited print .... a well selected synopsis, well arranged too for its 
chief purpose as a book of repetition, and containing a good number of useful theorems, especially in the 
Algebra, which are not usually to be found in the books of elementary reading .... It is clearly printed, 
and there is a good table of contents. On the whole, if the work, when complete, is supplemented by a 
good index, it will occupy a place not filled up at present." — Popular Science Review. 

"Having carefully read the whole of the text, we can say that Mr. Carr has embodied in his book 
he most useful propositions in the subjects treated of, and besides, has given many others, which do not 
o frequently turn up in the course of study. The work is printed in a good bold tyi)e on good paper, and 
he figures are admirably drawn." — Nature. 

** Mr. Carr has made a very judicious selection, so that it would be hard to find anything in the ordin- 
ary text-books which he has not labelled and put in its own place in his collection. The Qeometrical por- 

on, on account of the dear figures and compressed proofs, calls for a special word of praise. Tlte type 
exceedingly clear, and the printing well done."— Educationai Times. 

"The compilation will prove very useftil to advanced students."— 27k« Jowmal c/Seknee, 



Digitized by VjOOQIC 



New and Revised Edition, small crown 8vo, price 28. 

AN INTEODUCTION TO GEOMETEY. 
FOR TEJE USE OF BEGINNERS, 

CONSISTING OP 

EUCLID'S ELEMENTS, BOOK L 

ACCOMTANIBD BY NuMBROUS EXPLANATIONS, QUBSTIONS, AND ExBRCIflBS. 

Bt JOHN WALMSLEY, B.A. 

This work is characterised hy its abundant materials suitable for the tnun- 
ing of pupils in the performance of original work. These materials are so 
graduated and arranged as to be specially suited for class- work. They fur- 
nish a copious store of useful examples from which the teacher may readily 
draw more or less, according to the special needs of his class, and so as to 
help his own method of instruction. 

OPINIONS OF THE PRESS. 

"We cordially recommend this book. The plan adopted is founded upon a proper 
appreciation of the soundest principles of teacning. We have not space to give it in 
detail, but Mr. Walmsley is fully justified in saving that itjarovidea for 'a nafural and 
continuous training to pupils taken in classes.' ^'—At7ien<Buin. 

*• The book has been carefully written, and will be cordially welcomed by all those 
who are interested in the best methods of teaching Geometry.*'— iSc^oi Guardian. 

" Mr. Walmsley has made an addition of a novel kind to the many recent works intended 
to simplify the teaching of the elements of Geometry. . . . The system will undoubtedly 
help the pupil to a thorough comprehension of his subject." — School Board Chronicle. 

* When we consider how many teachers of Euclid teach it without mtelligence, and 
then lay the blame on the stupidity of the pupils, we could wish that every young 
.... . <• .. . 'Itake"*"" 



teacher of Euclid, however high he may have been among the Wranglers, would take the 
trouble to read Mr. Walmsley*s book through before he begins to teach the First Book to 

g boys."— Journal qf education. 

Ve have used the book to the manifest pleasure and interest, as well as progress, of 



young boys."— Journal qf education. 

" We have used the book to the manifest pleasure and interest, as well as pre. 
our own students in mathematics ever since it was published, and we have the greatest 
pleasure in recommending its use to other teachers. The Questions and Exercises are 
of incalculable value to the te&cher.**— Educational Chronicle. 



KEY to the above, price 3s. 



E 



Thirteenth Edition, 12mo, cloth, price Id. 

XAMPLES IN ALGEBRA FOR JUNIOR CLASSES. 

Adapted to all Text-Books ; and arranged to assist both the Tutor 
and the Pupil. 

Third Edition, cloth, lettered, 12mo, price 3s. 

EXAMPLES IN ALGEBRA FOR SENIOR CLASSES. 

Containing Examples in Fractions, Surds, Equations, Progressions, &c., 
and Problems of a higher range. 

THE KEY ; containing complete Solutions to the Questions in 
the " Examples in Algebra for Senior Classes," to Quadratics inclusiye. 
12mo, cloth, price 3s. 6d. 



T 



Demy 8vo, price 6s. each. 
IRAOTS relating to the MODERN HIGHER MATHE- 

MATICS. By the Rev. W. J. Wright, M.A. 
Teact No. 1.— DETERMINANTS. 
„ No. 2.— TRILINEAR COORDINATES. 
„ No. 3.— INVARIANTS. 

The object of this series is to afford to the young student an easy introduc- 
tion to the study of the higher branches of modem Mathematics. It is pro- 
posed to follow the above with Tracts on Theory of Surfeuses, Elliptic In- 
tegrals and Quaternions. 



Digitized by VjOOQIC 



MATHEMATICAL WORKS 



PUBLISHBV BY 



FRANCIS HODGSON, 

89 FARRINQDON STREET, E.C. 



In 8yo, doth, lettered. 

PROCEEDINGS of the LONDON MATHEMATICAL 
SOCIETY. 
Vol. I., from January 1865 to Noyember 1866, price 10s. 
Vol. II., from Noyember 1866 to November 1869, price 168. 
Vol. III., from November 1869 to November 1871, price 208. 
Vol. IV., from November 1871 to November 1873, price 3l8. 6d. 
Vol. v., from November 1873 to November 1874, price 168. 
Vol. VI., from November 1874 to November 1876, price 21b. 
Vol. VII., from November 1876 to November 1876, price 2l8. 
Vol. VIII., from November 1876 to November 1877, price 2l8. 
Vol. IX., from November 1877 to November 1878, price 2l8. 
Vol. X., from November 1878 to November 1879, price 188. 
Vol. XI., fix)m November 1879 to November 1880, price 128. 6d. 
Vol. XII., from November 1880 to November 1881, price 168. 
Vol. XIIT. from November 1881 to November 1882. price 188. 
Vol. XIV., from November 1882 to November 1883, price 208. 
Vol. XV., from November 1883 to November 1884, price 208. 
Vol. XVI., from November 1884 to November 1886, price 208. 
Vol. XVn., from November 1886 to November 1886, price 268. 

In half-yearly Volumes, 8vo, price 68. 6d. each. 
(To Subscribers, price 68.) 

MATHEMATICAL QUESTIONS, with their SOLU- 
TIONS, Reprinted from the EDUCATIONAL TIMES. Edited by 
W. J. C. MiLLBB, B.A., Registrar of the General Medical Council. 

Of this series forty-seven volumes have now been published, each volume 
containing, in addition to the papers and solutions that have appeared in the 
Educational Times, about the same quantity of new articles, and comprising 
contributions, in all branches of Mathematics, frt>m most of the leading 
Mathematicians in this and other countries. 
New Subscribers may have any of these Volumes at Subscription price. 

Demy 8vo, price 78. 6d. Second Edition. 
{ITsed 08 the Text-book in the Royal Military Academy, Woolwich,) 

LECTUEES on the ELEMENTS of APPLIED ME- 
CHANICS. ComTOn8ing—(l) stability of Structures; (2)Strangthof 
Materials. By Mobgam W. C&ofton, F.R.S., late Professor of Mathematics 
and Mechanics at the Royal Military Academy. 

Demy 8vo. Price 7b. 6d. Second Edition. 
iJTeed as the Text-book in the Royal Military Academy, Woohoich,) 

TRACTS ON MECHANICS.. In Three Parts— Parts 1 
and 2, On the Theoij of Work, and Graphical Solution of Statical 
Problems ; by Morgan W. Crofton, F.R.S., late Professor of Mathematics 
and Mechanics at the Royal Military Academy. Part 3, Artillery Machines ; 
by Major Edoar Kensington, R.A., late Professor of Mathematics and 
Artillery at the Royal Military College of Canada. 

Third Edition. Extra fcap. 8vo, price 48. 6d. 
(Used as the Text-hook in the Royal Military Academy, Woolwich,) 

ELEMENTARY MANUAL of COORDINATE GEO^ 
METRY and CONIC SECTIONS. By Rev. J. White, M.A., Head 
Master of the Royal Naval School, New Cross. 



Digitized by 



Google 



T>i 



Kinth fidition, small crown 8vo, cloth letteredi price 28. 6d. 
AN INTRODUCTORY COURSE OP 

PLANE TEIGONOMETRY AND LOGARITHMS. 
By JOHN WALMSLEY, B.A. 

" This book is caareftilly done ; has full extent of matter, and good store of examples."— 
Aihsnaum. 

** This is a carefully worked out treatise* with a very large collection of well-chosen 
and well-arranged examples."— Papw*/or the Schoolmaater, 

"This is an excellent work. The proofs of the several propositions are distinct, the 
explanations clear and concise, and the (ceneral plan of arrangement accurate and 
methodicaL"— 2%tf Mu9«um and English Journal (f Education, 

** The e^lanations of logarithms are remarkably mil and clear. . . . The several parts 
of the subject are, throughout the work, treated according to the most recent and 
approved methods. ... It is, in fact, a book for beainnerSt and by far the simplest and 
most satisfiActory work of the kind we have met witlL"-~Education4U Times, 

Price Five Shillings, 

Af$d will he supplied to Teachers and Frivate Students only, on application to the 

Publishers, enclosing the txruL price; 

A KEY 

to the above, containing Solutions of all the Examples therein. These num- 
ber seven hundred and thirty , or, taking into account that many of them are 
double, triple, &c., about nine hundred; a large proportion of which are taken 
from recent public examination papers. 

By the same Author, 
New Edition, fcap. 8vo, doth, price 5s. 

»LANE TRIGONOMETRY AND LOGARITHMS. 

JL FOR SCHOOLS AND COLLEGES, Comprising the higher branches 
of the subject not treated in the elementary work. 

'* This is an expansion of Mr. Walmsley's ' Introductory Course of Plane Trigo- 
nometry,' which has been already noticed with commendation in our columns, but so 
greatly extended as to justify its being regEirded as a new work .... It was natural that 
teachers, who had found the elementary parts well done, should have desired a com- 
pleted treatise on the same lines, and Mr. Walmsley hiEbs now put the finishing touches 
to his conception of how Trigonometry should be taught. There is no perfunctory work 
manifest in this later growth, and some of the chapters— notably those on the imaginary 
expressAon ^^^, and general proofs of the fundamental formulae— are especially good. 
These last deal with a portion of the recent literature connected with the prpora for 
sin {A+Bhjkc., and are supplemented by one or two generalized proofs by Mr. Walmsley 
himself. We need only further say that the new chapters are quite up to the level ra 
the previous work, and not only evidence great love for the subject, but considerable 
power in assimilating what has been done, and in representing the results to his readers." 
—Educational Times, 

By the same Author. 

Preparing for Publication. 

Suitable for Students preparing for the University Local and similar 

Examinations, 

AN INTEODUOTION TO MECHANICS. With 
numerous Examples, many of whidi are fully worked out in illustra- 
tion of the text. 

Demy 8vo, price 6s. 

ALGEBRA IDENTIFIED WITH GEOMETRY, in 
Five Tracts. By Alexander J. Ellis, F.R.S., F.S.A. 

1. Euclid's Conception of Batio and Proportion. 

2. " Camot's Principle'* for Limits. 

3. Laws of Tensors, or the Algebra of Proportion. 

4. Laws of Clinants, or the Algebra of Similar Triangles lying on the 
Same Plane. 

0. Stigmatic Geometry, or the Correspondence of Points in a Plane. 
With one photo-lithographed Table of Figures. 



Demy 8vo, sewed. Is. 4d. 

"iVrOTES ON THE FIRST PRINCIPLES OF DY- /^ 

-L^ NAMICS. By W. H. H. Hudson, M.A., Professor of MathematicjA 
in King's College, London. q/^ 



Digitized by 



Google 



Digitized by VjOOQIC 



Digitized by VjOOQIC 



4 



Digitized 



.Google 







''1 

1 1 H M II M t 


1 




I 2044 102 915 998